F.A.Q. LATEX

tex-lion.gif
meta-lioness.gif

DE FR.COMP.TEXT.TEX
Foire Aux Questions classée et mise en page













Auteur : Marie-Paule Kluth,
Mainteneur : Benjamin Bayart,
Mise en Page : Yann Morère

Version 2.28 16/10/2001

`: `; `! `?

Introduction

Le forum fr.comp.text.tex est un forum français de discussion concernant TEX et LATEX. comp.text.tex est son équivalent anglophone. Cette FAQ (recueil des questions les plus fréquentes) est essentiellement composée à partir de messages postés dans ces forums et de mon expérience personnelle.

Je tiens à souligner que cette FAQ LATEX ne prétend pas être correcte sur tous les points qu'elle énonce (j'essaie toutefois de tester un maximum de réponses mais je ne dispose pas toujours de tous les packages), ni même complète. C'est pourquoi, je vous invite tous à y contribuer par vos remarques. En outre, des exemples concis susceptibles d'illustrer l'une ou l'autre des réponses seront les bienvenus.

Concernant l'évolution de LATEX2.09 en , sachant que certains sites utilisent encore l'ancienne version (notamment pour travailler sur d'anciens documents utilisant des styles ou macros non compatibles avec ), cette FAQ présente encore des aides pour LATEX2.09. Cependant, il est entendu que ces dernières seront amenées à disparaître.

Note (destinée aux puristes) : Afin de ne pas tomber dans l'excès qui consiste à traduire toutes les expressions anglo-saxonnes utilisées dans LATEX, en français, on parlera, entre autres, dans ce qui suit de package et non d'extension, de viewer, du WEB, etc.

Suite à de nombreuses demandes, les accents ont été ajoutés. Le programme `recode' de GNU est capable d'en générer une version sans les accents (recode latin1:flat faq_latex). Ceux qui ne possèdent pas un

tel programme peuvent me demander une version sans accent par e-mail.

Pour toutes remarques, commentaires ou ajouts, contactez le mainteneur par e-mail : bayartb@edgard.fdn.fr

Si vous désirez, distribuer ce document par FTP ou sur le WEB, ou placer un pointeur vers ce dernier, merci de m'en informer par e-mail et de me communiquer l'adresse correspondante.

Contents

1  Présentation
    1.1  Quelle est l'histoire de (La) TEX ?
    1.2  Quels sont les principes de base de TEX ?
    1.3  Quels sont les principes de base de LATEX ?
    1.4  Qu'est ce que le préambule du fichier source ?
    1.5  Comment faire ses premiers pas ?
    1.6  Comment sont gérées les options de package ?
    1.7  Quelle est la structure d'une page LATEX ?
    1.8  Quelles sont les commandes de compilation LATEX ?
    1.9  Quels sont les fichiers utilisés par LATEX ?
    1.10  A quoi correspondent les messages Overfull ?
2  Documentation
    2.1  Que puis-je lire sur TEX ?
    2.2  Que puis-je lire sur LATEX2.09 ?
    2.3  Que puis-je lire sur ?
    2.4  Que puis-je lire sur AMS- LATEX ?
    2.5  Que puis-je lire sur la typographie ?
    2.6  Où trouver une bibliographie complète ?
    2.7  Que puis-je lire sur les fontes ?
3  Sites Web
    3.1  Où trouver des infos sur le WEB ?
    3.2  Où trouver cette FAQ ?
    3.3  Existe t'il des listes de discussion francophones ?
    3.4  Où trouver d'autres FAQs LATEX ?
4  Sources TEX et LATEX
    4.1  Où trouver les sources pour Unix ?
    4.2  Où trouver les sources pour VMS ?
    4.3  Où trouver les sources pour DOS, OS/2, Windows3.x/95/NT ?
    4.4  Où trouver les sources pour MacOS ?
    4.5  Où trouver d'autres sources ?
        4.5.1  Atari
        4.5.2  Amiga
        4.5.3  Tandy 6000
        4.5.4  TOPS-20
5  Éditer LATEX
    5.1  Alpha
    5.2  Cicero
    5.3  Eddi4 TEX
    5.4  (X-)Emacs
    5.5  Funtek
    5.6  GNU emacs
    5.7  JED
    5.8  LYX
    5.9  MicroEmacs
    5.10  NEdit
    5.11  Scientific Word
    5.12  STEAD
    5.13  TEX nicCenter
    5.14  TEX Shell
    5.15  ViM
    5.16  WINEDT95
    5.17  wintex95
    5.18  Xcoral
    5.19  X-Window Shell pour TEX
    5.20  xtem
    5.21  Autres
6  Gestion de la mise en page
    6.1  Comment modifier l'interligne d'un document ?
    6.2  Comment gérer un document recto-verso ?
    6.3  Comment modifier le style des titres ?
    6.4  Comment obtenir un document multicolonnes ?
    6.5  Comment composer une brochure ?
    6.6  Comment définir un format de document ?
    6.7  Comment redéfinir les marges d'un document ?
    6.8  Comment changer l'orientation d'un document ?
    6.9  Comment justifier verticalement un paragraphe ?
    6.10  Comment modifier la commande \ caption ?
    6.11  Comment modifier un changement de page ?
    6.12  Comment obtenir des colonnes parallèles ?
    6.13  Comment définir l'espace de début de paragraphe ?
    6.14  Comment supprimer la date sur une page de titre ?
    6.15  Comment mettre en valeur la première lettre d'un chapitre ?
    6.16  Comment préserver les tabulations en mode verbatim ?
    6.17  Comment modifier les entêtes de chapitre ?
    6.18  Comment définir des tabulations ?
    6.19  Comment obtenir des lettres accentuées dans tabbing ?
    6.20  Comment encadrer du texte ?
    6.21  Comment gérer des URL WWW ?
    6.22  Comment mettre en page des exercices dont les solutions sont reportées dans un autre paragraphe ?
    6.23  Comment positionner un objet dans une page ?
    6.24  Comment lier le placement des flottants aux sections ?
    6.25  Comment griser le fond (background) d'un paragraphe ?
    6.26  Comment modifier l'espace inter-colonnes ?
    6.27  Comment modifier les environnements de liste ?
    6.28  Comment souligner plusieurs lignes ?
    6.29  Comment réaliser des onglets ?
    6.30  Comment réaliser des QCM ?
    6.31  Comment modifier l'orientation des flottants ?
    6.32  Comment faire référence plusieurs fois à la même note de bas de page ?
    6.33  Comment éviter les orphelins en début ou fin de page ?
    6.34  Comment définir de nouveaux flottants ?
    6.35  Comment utiliser la commande \ caption hors d'un environnement flottant ?
    6.36  Comment encadrer un objet flottant ?
    6.37  Comment changer l'orientation d'un caption ?
    6.38  Comment mettre en page un programme ?
    6.39  Comment inclure le source d'un programme ?
    6.40  Comment obtenir un espace insécable ?
    6.41  Comment insérer une page blanche ?
    6.42  Comment supprimer l'indentation des paragraphes ?
    6.43  Comment modifier l'espacement entre caractères ?
    6.44  Comment supprimer certaines coupures de mots ?
    6.45  Comment mettre en forme des équations chimiques ?
    6.46  Comment mettre en page un calendrier ?
    6.47  Comment forcer un caption sur plusieurs lignes ?
    6.48  Comment générer des barres de modification dans un document ?
    6.49  Comment encadrer du texte verbatim ?
    6.50  Comment écrire du texte en forme de losange ou autre ?
    6.51  Comment isoler les flottants par un trait horizontal ?
    6.52  Comment insérer un espace vertical dans une page ?
    6.53  Comment insérer un espace horizontal dans un texte ?
    6.54  Comment définir un style de paragraphe ?
    6.55  À quoi sert la commande \ stretch ?
    6.56  Comment justifier un paragraphe à gauche ou à droite ?
    6.57  Comment mettre en page des citations ?
    6.58  Comment insérer un texte sans que LATEX le mette en forme ?
    6.59  Comment visualiser des espaces ?
    6.60  Comment insérer du code LATEX dans un document LATEX ?
    6.61  Comment écrire du texte en couleurs ?
    6.62  Comment construire un arbre ?
    6.63  Comment mettre en page un poème ?
    6.64  Comment aligner des paragraphes ?
    6.65  Comment construire une liste d'éléments ?
    6.66  Comment faire une note dans une marge ?
    6.67  Comment faire une page de garde ?
    6.68  Comment utiliser \ thanks dans chaque titre de chapitre ?
    6.69  Comment mettre un résumé et un abstract dans un document ?
    6.70  Comment définir de nouvelles couleurs ?
    6.71  Comment placer des figures face à face en recto-verso ?
    6.72  Quelles sont les différences entre a4paper et letterpaper ?
    6.73  Comment couper une cha îne de caractères ?
    6.74  Comment modifier l'espace entre une figure et sa légende ?
7  Gestion des tableaux
    7.1  Comment faire un tableau ?
    7.2  Comment gérer les tableaux de plus d'une page ?
    7.3  Comment modifier l'orientation d'un tableau ?
    7.4  Comment nommer un tableau ?
    7.5  Comment modifier l'épaisseur des lignes d'un tableau ?
    7.6  Comment griser des cellules d'un tableau ?
    7.7  Comment changer la fonte d'une colonne ?
    7.8  Comment créer des notes de bas de page dans un tableau ?
    7.9  Comment écrire un texte sur plusieurs colonnes ?
    7.10  Comment passer certaines cellules en reverse vidéo ?
    7.11  Comment fixer la largeur d'une colonne ?
    7.12  Comment écrire un texte sur plusieurs lignes ?
    7.13  Comment diviser une cellule par une diagonale ?
    7.14  Comment définir une colonne en mode mathématique dans un tableau ?
    7.15  Comment modifier le nombre de tableaux par page ?
    7.16  Comment mettre deux tableaux côte à côte ?
    7.17  Comment définir un séparateur de colonne ?
    7.18  Comment obtenir des lignes partielles dans un tableau ?
    7.19  Comment éviter que du texte de grande taille n'atteigne le cadre des cellules ?
    7.20  Comment fixer la largeur d'un tableau ?
    7.21  Comment tracer des traits discontinus ?
    7.22  Comment fixer la taille et justifier une colonne ?
    7.23  Comment faire une liste de tableaux ?
8  Gestion des figures
    8.1  Comment inclure une figure ?
    8.2  Comment nommer une figure ?
    8.3  Comment placer des figures côte à côte ?
    8.4  Comment modifier le nombre de figures par page ?
    8.5  Comment superposer du texte sur des figures ?
    8.6  Comment réaliser des captures d'écran ?
    8.7  Comment tracer une courbe ?
    8.8  Comment est géré le positionnement des figures ?
    8.9  Comment placer une légende à côté d'une figure ?
    8.10  Comment insérer des figures dans multicol ?
    8.11  Comment faire appara ître toutes les figures en fin de document ?
    8.12  Comment insérer des images Mathematica ?
    8.13  Comment modifier la taille d'une bounding box ?
    8.14  Comment obtenir une figure avec un titre non numéroté ?
    8.15  Comment redéfinir le style de caption ?
    8.16  Comment numéroter les figures en fonction des sections ?
    8.17  Comment fondre une image dans du texte ?
    8.18  Comment réaliser des diagrammes en bâtons ?
    8.19  Comment faire un organigramme ?
    8.20  Comment centrer une figure très large ?
    8.21  Comment passer de ps à eps ?
    8.22  Comment changer l'orientation d'une figure ?
    8.23  Comment gérer des sous-figures sur plusieurs pages ?
    8.24  Comment générer une liste des figures d'un document ?
    8.25  Comment faire une figure sous LATEX ?
    8.26  Comment mettre un commentaire à côté d'une figure ?
9  Inclusion de fichiers
    9.1  Comment inclure des fichiers en mode verbatim ?
    9.2  Comment gérer un document par parties ?
    9.3  Comment isoler une partie d'un fichier ps ou dvi ?
    9.4  Comment inclure un fichier PICT ?
    9.5  Comment spécifier un chemin pour les fichiers à inclure ?
10  Hauts et bas de pages
    10.1  Comment définir les hauts et bas de page ?
    10.2  Comment obtenir une note de bas de page ?
    10.3  Comment mettre les notes de bas de page en fin de document ?
    10.4  Comment réduire les rappels de titres dans un haut ou bas de page ?
    10.5  Comment référencer une note de bas de page ?
    10.6  Comment supprimer la numérotation des pages ?
    10.7  Comment numéroter les pages par rapport à la dernière ?
    10.8  Comment supprimer le trait de séparation des notes de bas de page ?
    10.9  Comment modifier la numérotation des pages ?
    10.10  Comment supprimer les entêtes et bas de page de pages vierges ?
    10.11  Comment gérer des en-têtes avec des environnements verbatim multi-pages ?
    10.12  Comment utiliser \ footnote dans un titre ?
    10.13  Comment placer les notes de bas de page les unes à côté des autres ?
    10.14  Comment réinitialiser le compteur de note de bas de page à chaque page ?
    10.15  Comment modifier le style des notes de bas de page ?
    10.16  Comment utiliser le mode verbatim dans une note de bas de page ?
    10.17  Comment éviter qu'une note de bas de page s'étale sur plusieurs pages ?
11  LATEX, le français et les autres langues
    11.1  Comment franciser un document LATEX ?
    11.2  Comment corriger les coupures de mots accentués ?
    11.3  Comment utiliser les lettres accentuées ?
    11.4  Comment composer du texte en grec moderne ou classique ?
    11.5  Comment composer du texte en hébreu moderne ou classique ?
    11.6  Comment composer du texte en arabe ?
    11.7  Comment définir les césures de groupes de mots conjugués avec un trait d'union ?
12  Mathématiques
    12.1  Comment passer en mode mathématique ?
    12.2  Qu'est que AMS- LATEX ?
    12.3  Comment écrire les symboles d'ensembles ?
    12.4  Comment numéroter les équations ?
    12.5  Comment aligner des équations ?
    12.6  Comment générer des vecteurs ?
    12.7  Comment écrire du texte en mode mathématique ?
    12.8  Comment ajuster la taille de délimiteurs ?
    12.9  Comment changer de fonte en mode mathématique ?
    12.10  Comment obtenir le L de la transformée de Laplace ?
    12.11  Comment réaliser un tableau en mode mathématique ?
    12.12  Comment obtenir d'autres symboles mathématiques ?
    12.13  Comment définir une fonction ?
    12.14  Comment définir une matrice ?
    12.15  Comment encadrer des formules ?
    12.16  Comment ajuster la longueur d'une flèche par rapport à un texte ?
    12.17  Comment obtenir des indices ou exposants à gauche ?
    12.18  Comment tracer des diagrammes commutatifs ?
    12.19  Comment ajuster la taille de certains opérateurs ?
    12.20  Comment mettre en page des algorithmes ?
    12.21  Comment mettre en page des formules longues ?
    12.22  Comment créer des unités de mesure ?
    12.23  Comment écrire proprement a/b ?
    12.24  Comment créer des notes de bas de page dans une formule mathématique ?
    12.25  Comment forcer le style d'un tableau à displaystyle ?
    12.26  Comment réduire les espaces gérés par eqnarray ?
    12.27  Comment réduire la taille des indices ?
    12.28  Comment aligner des indices ?
    12.29  Comment mettre en page des théorèmes ?
    12.30  Comment changer la taille de la fonte ?
    12.31  A quoi sert la commande \ displaystyle ?
    12.32  Comment aligner des données sur le point décimal ?
    12.33  Comment gérer les espaces en mode mathématique ?
    12.34  Comment ecrire un interieur proprement ?
    12.35  Comment obtenir des accolades horizontales ?
    12.36  Comment obtenir des points de suspensions ?
    12.37  Comment mettre en page un système d'équations ?
    12.38  Comment mettre en page des diagrammes de Feynman ?
    12.39  Comment obtenir une fraction ?
    12.40  Comment obtenir une racine ?
    12.41  Quels sont les délimiteurs disponibles ?
    12.42  Comment appeler les lettres grecques ?
    12.43  Quels sont les accents mathématique standards ?
    12.44  Comment superposer deux symboles ?
    12.45  Comment définir un nouvel opérateur ou symbole ?
    12.46  Comment obtenir des lettres grecques en gras ?
    12.47  Comment tracer un tableau de variations ?
13  Références Croisées
    13.1  Quelles sont les commandes de base ?
    13.2  Comment obtenir un renvoi à une page ?
    13.3  Comment obtenir des références croisées à partir de plusieurs sources ?
    13.4  Comment définir des liens hypertexte sous LATEX ?
    13.5  Comment faire référence à ses propres compteurs ?
    13.6  Comment faire référence au début d'un flottant et pas au titre?
14  Table des matières
    14.1  Comment générer une table des matières ?
    14.2  Comment générer plusieurs tables des matières ?
    14.3  Comment ajouter une entrée dans la table des matières ?
    14.4  Comment changer le titre de la table des matières ?
    14.5  Comment changer la profondeur de la table des matières ?
    14.6  Comment gérer des chapitres de préface, d'introduction, et de conclusion non numérotés ?
    14.7  Comment enlever le numérotation des pages de tdm ?
15  Bibliographie sous LATEX
    15.1  Comment construire une bibliographie ?
    15.2  Comment gérer plusieurs bibliographies ?
    15.3  Comment changer de langue dans une bibliographie ?
    15.4  Comment renvoyer une référence en note de bas de page ?
    15.5  Comment faire référence à un document ?
    15.6  Comment grouper des références multiples ?
    15.7  Comment changer le titre de la bibliographie ?
    15.8  Comment changer le style de la bibliographie ?
    15.9  Comment construire une bibliographie à partir de plusieurs fichiers .bib ?
    15.10  Comment utiliser la commande \ cite dans un \ caption ?
    15.11  Comment référencer une thèse française ou un mémoire ?
    15.12  Comment supprimer la virgule supplémentaire dans une liste d'auteurs ?
    15.13  Comment configurer la commande \ cite ?
    15.14  Comment construire une liste d'auteurs ?
    15.15  Comment spécifier un tri dans une bibliographie ?
    15.16  Comment référencer les pages contenant des citations ?
    15.17  Où trouver des styles de bibliographie ?
    15.18  Comment faire des références croisées ?
    15.19  Comment citer une URL ?
    15.20  Comment définir des initiales à deux lettres ?
    15.21  Comment conserver les majuscules dans les titres ?
    15.22  Comment changer l'espace entre les item ?
    15.23  Comment réaliser des fiches de lecture ?
    15.24  Comment utiliser la commande \ cite dans un item ?
    15.25  Comment générer l'expression et al automatiquement ?
16  Index
    16.1  Quelles sont les commandes de base ?
    16.2  Comment construire un index hiérarchique ?
    16.3  Quels sont les générateurs d'index ?
    16.4  Comment changer le style de certains mots indexés ?
    16.5  Comment changer le style des pages de référence ?
    16.6  Comment rappeler certains mots dans un haut de page ?
    16.7  Comment générer plusieurs index ?
    16.8  Qu'est ce que IdX TEX ?
    16.9  Qu'est ce que xindy ?
17  Glossaire
    17.1  Quelles sont les commandes de base ?
    17.2  Quels sont les générateurs de glossaire ?
18  Styles prédéfinis
    18.1  Où trouver un style de thèse ?
    18.2  Comment faire son CV en LATEX ?
    18.3  Où trouver un format de publication ?
    18.4  Où trouver un style de manuel de référence ?
    18.5  Où trouver un style de poster ?
    18.6  Comment mettre en forme une lettre interne ?
    18.7  Comment créer son propre style ?
19  Création de transparents
    19.1  Quels sont les packages et styles existants ?
    19.2  Comment définir un contour pour des transparents ?
    19.3  Comment inclure des commentaires dans les transparents ?
    19.4  Comment modifier l'interligne sous seminar ?
    19.5  Comment définir des en-têtes et pieds de pages ?
    19.6  Comment modifier la taille du cadre d'un transparent ?
    19.7  Comment empêcher les figures de flotter ?
    19.8  Comment gérer la couleur avec seminar ?
    19.9  Comment imprimer des transparents en miroir ?
    19.10  Comment inclure une image de fond ?
    19.11  Comment imprimer plusieurs transparents par page ?
20  Lettres, mailing et faxs
    20.1  Comment structurer une lettre ?
    20.2  Comment préparer un mailing ?
    20.3  Comment faire des références dans une lettre ?
    20.4  Comment mettre en page un fax ?
    20.5  Comment positionner une adresse pour une enveloppe à fenêtre ?
    20.6  Comment suppprimer la date sur une lettre ?
    20.7  Comment inclure une figure dans une lettre ?
21  Symboles et logos
    21.1  Où trouver des symboles électroniques ?
    21.2  Comment dessiner des circuits électroniques ?
    21.3  Quelles sont les polices de symboles sous LATEX ?
    21.4  Comment obtenir les symboles mâle et femelle ?
    21.5  Comment obtenir le symbole degré ?
    21.6  Où trouver des symboles astronomiques ?
    21.7  Où trouver une police de symboles phonétiques ?
    21.8  Où trouver des opérateurs de logique floue ?
    21.9  Comment obtenir le symbole de paragraphe ?
    21.10  Comment obtenir le caractère 'registered' ?
    21.11  Où trouver le symbole trade-mark ?
    21.12  Comment obtenir un underscore ?
    21.13  Où trouver le symbole radioactif ?
    21.14  Comment obtenir le logo LATEX ?
    21.15  Comment obtenir le logo ?
    21.16  Où trouver des chiffres entourés ?
    21.17  Comment obtenir le symbole numéro ?
    21.18  Comment obtenir les symboles pourcent et pourmille ?
    21.19  Comment obtenir un e dans l'o ?
    21.20  Quels sont les symboles réservés dans LATEX ?
    21.21  Comment obtenir des lettres cursives ?
    21.22  Comment obtenir le logo AMS-(La) TEX ?
    21.23  Quels sont les symboles par défaut de LATEX ?
    21.24  Quels sont les différents tirets ?
    21.25  Comment obtenir le symbole arobasse ?
    21.26  Comment obtenir un backslash ?
    21.27  Comment faire un carré plain de fin de démonstration ?
22  Musique
    22.1  Comment écrire de la musique sous LATEX ?
    22.2  Comment convertir du midifile en Music TEX   ?
    22.3  Existe-t'il une liste de discussion de musique ?
    22.4  Comment éditer un livre de chants ?
    22.5  Comment mettre en page un programme de concert ?
23  Conversion de fichiers
    23.1  Comment générer un fichier .ps à partir d'un .dvi ?
    23.2  Qu'est ce que le ``Literate Programming'' ?
    23.3  Comment convertir du LATEX en word ?
    23.4  Comment convertir du word en LATEX ?
    23.5  Comment convertir du scribe en LATEX ?
    23.6  Comment convertir du WordPerfect en LATEX ?
    23.7  Comment convertir du LATEX en RTF ?
    23.8  Comment convertir du RTF en (La) TEX ?
    23.9  Comment convertir du Excel en LATEX ?
    23.10  Comment convertir du HTML en LATEX ?
    23.11  Comment convertir du LATEX en HTML ?
    23.12  Comment convertir un fichier dvi en ascii ?
    23.13  Comment convertir du WEB en LATEX ?
    23.14  Comment convertir du TEX en Framemaker ?
    23.15  Comment enlever les balises LATEX d'un document ?
    23.16  Comment convertir du SGML en (La) TEX ?
    23.17  Comment convertir du WinWord en LATEX ?
    23.18  Comment convertir un fichier 8 bits en fichier 7 bits ?
    23.19  Comment convertir un fichier ChiWriter en TEX ?
    23.20  Où trouver une FAQ de convertisseurs (La) TEX /Traitement de texte ?
    23.21  Comment convertir une image en police metafont ?
    23.22  Comment convertir un fichier postscript en ascii ?
    23.23  Comment convertir un fichier pdf en ascii ?
    23.24  Comment convertir du LATEX en PDF ?
    23.25  Comment définir son propre format de sortie ?
24  Préviewers et viewers
    24.1  Où trouver un previewer ?
    24.2  Comment visualiser des fichiers non postscript sous xdvi ?
25  Logiciels de dessins
    25.1  Où trouver un logiciel de dessin ?
    25.2  Comment inclure des formules Latex dans Xfig ?
    25.3  Comment gérer différents formats de figures ?
26  Correcteur
    26.1  Où trouver un correcteur orthographique ?
    26.2  Où trouver un vérificateur de syntaxe LATEX ?
27  Assocations
    27.1  Qu'est ce que l'association GUTenberg ?
    27.2  Qu'est ce que l'association TUG ?
    27.3  Qu'est ce que l'association As TEX ?
28  Fontes
    28.1  Que signifient les sigles T1, mf, fd etc. ?
    28.2  Quels sont les attributs d'une fonte ?
    28.3  Comment utiliser une fonte ?
    28.4  Comment changer la forme d'une fonte ?
    28.5  Comment changer la taille d'une fonte ?
    28.6  Comment modifier la fonte des numéros de paragraphe ?
    28.7  Comment modifier la fonte du mode verbatim ?
    28.8  Comment réaliser des changements de fontes relatifs ?
    28.9  Comment mettre en évidence une portion de texte ?
    28.10  Où trouver des fontes ?
    28.11  Comment suivre le chargement des fontes ?
    28.12  Pourquoi Make TEX PK est lancé alors que la fonte existe ?
    28.13  Comment utiliser \ textsc dans un titre en conservant le gras ?
    28.14  Comment utiliser des fontes TrueType?
29  Divers
    29.1  A quoi sert la commande \ par ?
    29.2  Comment commenter une partie d'un source LATEX ?
    29.3  Comment utiliser LATEX sur des petites machines ?
    29.4  Comment visualiser des paramètres ?
    29.5  Comment visualiser des compteurs ?
    29.6  A quoi servent \ makeatletter et \ makeatother ?
    29.7  Comment numéroter les lignes d'un document ?
    29.8  A quoi sert la commande \ special ?
    29.9  Comment réaliser des calculs avec les variables LATEX ?
    29.10  Où trouver une fonte 9pt ?
    29.11  Comment automatiser les compilations LATEX ?
    29.12  Comment obtenir des cadres gris ?
    29.13  Comment obtenir certaines abréviations ?
    29.14  Comment gérer les espaces après une macro ?
    29.15  Où trouver les notices d'utilisation des packages ?
    29.16  Comment obtenir des listes d'objets flottants ?
    29.17  Comment conna ître les versions des fichiers utilisés lors d'une compilation ?
    29.18  Pourquoi certaines commandes sont elles indépendantes ?
    29.19  Comment installer un package ?
    29.20  Comment générer des codes barres ?
    29.21  Comment gérer des abbréviations ?
    29.22  Comment imprimer un fichier postscript sur une imprimante non postscript ?
    29.23  Comment surcharger une commande déjà existante ?
    29.24  Comment reporter l'exécution d'une commande à la fin d'une page ?
    29.25  Comment définir des scripts interactifs ?
    29.26  Comment identifier une version provisoire ?
    29.27  Comment obtenir des caractères barrés ?
    29.28  A quoi sert la commande \ ensuremath ?
    29.29  A quoi servent les commandes \ (re)newcommand ?
    29.30  Où trouver des hirondelles ?
    29.31  Comment tracer des lignes ?
    29.32  Comment imprimer le caractère ?
    29.33  Comment visualiser tous les caractères d'une fonte ?
    29.34  A quoi sert % dans les macros ?
    29.35  Comment inclure l'heure dans un document ?
    29.36  Comment compter le nombre de mots d'un fichier ?
    29.37  Comment rendre inactif un caractère ?
    29.38  Comment utiliser le mode verbatim dans une commande ?
    29.39  Comment redéfinir la commande \ year ?
    29.40  Qu'est ce que la magnification ?
    29.41  Comment sont gérés les postscripts dans LATEX ?
    29.42  Pourquoi LATEX n'accepte-t'il pas tous les formats d'image ?
    29.43  A quoi sert la commande \ (re)newenvironment ?
    29.44  Comment récupérer le nom du fichier compilé ?
    29.45  Comment gérer des conditions de traitement dans un style ?
    29.46  A quoi servent les commandes \ setlength et \ addtolength ?
    29.47  Quelles sont les unités de mesure de TEX ?
    29.48  A quoi sert la commande \ mbox ?
    29.49  Comment obtenir des points de suspension ?
    29.50  Comment désactiver une ligature ?
    29.51  Comment gérer les versions d'un document ?
    29.52  Comment changer certains titres ?
    29.53  Comment insérer un code source dans un document ?
    29.54  Comment tracer une ligne horizontale ?
    29.55  Comment générer un espace invisible de taille donnée ?
    29.56  Qu'est ce qu'une correction italique ?
    29.57  Quels sont les accents accessibles sous LATEX ?
    29.58  Comment écrire dans un fichier pendant une compilation ?
    29.59  Comment gérer des compteurs ?
    29.60  Quels sont les différents styles de compteurs ?
    29.61  Comment programmer un traitement itératif ?
    29.62  A quoi servent les commandes savebox ?
    29.63  Comment résoudre certaines incompatibilités de packages ?
    29.64  Qu'est ce que Lollipop ?
    29.65  J'ai un problème avec babel !
    29.66  Qu'est ce que cette FAQ ?
30  Remerciement et Contributions

Chapter 1
Présentation

Le logiciel TEX est un formateur de texte et non un traitement de texte. De plus il n'est pas WYSIWYG, ce qui peut rebuter plus d'un utilisateur venant des traitements de texte inclus dans les suites bureautique. Il est surtout adapté à la rédaction de document scientifique car sa gestion des mathématiques n'a aucun égal.

1.1  Quelle est l'histoire de (La)TEX ?

Le logiciel TEX (1978) est le formateur de texte de D. E. Knuth. A l'origine, Knuth a développé TEX (en WEB cf. paragraphe ) notamment pour réaliser de beaux documents et écrire des formules mathématiques.

LATEX, écrit par L. Lamport (1982), est un jeu de macros au-dessus de TEX, plus facile à utiliser que ce dernier. Il propose notamment différents styles de document auxquels correspondent des classes de document et une grande diversité de macros qui répondent à divers besoins des auteurs. LATEX a été conçu pour rédiger des articles, des rapports, des thèses ou des livres ou pour préparer des transparents. On peut insérer dans le texte, des dessins, des tableaux, des formules mathématiques et des images sans avoir à se soucier (ou presque) de leur mise en page. Les documents produits avec LATEX et TEX sont d'une excellente qualité typographique.

Plain TEX écrit également par D. E. Knuth, était le premier jeu (minimal) de macros par dessus TEX. De même Eplain, de K. Berry, est un jeu de macros intermédiaire entre TEX et LATEX.

Suite à une large diffusion de LATEX beaucoup d'extensions ont été créées par différents utilisateurs. Grâce à Murphy, ces extensions ont introduit un certain nombre d'incompatibilités et ont porté atteinte à la portabilité de LATEX. C'est de cette situation qu'est né le projet de normalisation LATEX3, sous la direction des gurus LATEX : L. Lamport, F. Mittelbach, C.

Rowley, R. Schopf et tant d'autres... Pour plus de détails, consulter : http://www.latex-project.org/

Toutefois, pour ne pas perturber les actuels utilisateurs de LATEX, la version provisoire normalisée s'appelle (1994) et elle est compatible (dans la mesure du possible) avec les anciens standards. Ainsi tous les documents écrits pour LATEX2.09 peuvent être compilés sous en mode ``LATEX2.09 compatibility mode''.

Remarque :
cette compatibilité sera amenée à disparaître au fur et à mesure des évolutions vers LATEX3.
Les membres du projet LATEX3 travaillent actuellement sur le futur LATEX3. Autrement dit, ne devrait plus beaucoup évoluer.

Il existe aussi omega, une extension 16 bits de TEX qui utilise unicode comme représentation interne et autorise ainsi la composition de textes multi-lingues dans les langues telles que l'arabe, le chinois ou les langues du continent indien. Pour plus de renseignements, vous pouvez consulter : les cahiers GUTenbeg, TUGboat, ftp://ftp.ens.fr/pub/tex/yannis/omega/, ftp://ftp.ens.fr/pub/tex/yannis/omega-babel/, ...

Signalons également NTS, un projet à très long terme qui vise d'abord à enrichir TEX mais qui à terme, n'en gardera que les concepts.

1.2  Quels sont les principes de base de TEX ?

Le principe de base de TEX est la boîte ! TEX met tout dans des boîtes et l'assemblage de ces boîtes suivant des règles données permet de mettre en page des documents.

A l'origine, TEX a été conçu aussi bien pour créer des documents d'une page de texte, que des documents de plusieurs centaines de pages contenant des formules mathématiques, des tableaux, des figures, etc. TEX travaille donc comme un imprimeur sans subir les contraintes mécaniques inhérentes aux outils de ce dernier.

Sa précision est sans limite puisque son unité de base est le point et que les calculs qu'il suscite sont effectués par les ordinateurs.

Un des avantages de TEX est qu'il possède une vision globale des choses. TEX ne se place pas uniquement au niveau du caractère que l'auteur frappe mais aussi au niveau du mot, de la ligne, du paragraphe, de la page ou du document dans son entier pour évaluer ses critères de beauté. La dimension esthétique du document est ainsi prise en compte et gérée de manière à ce qu'elle soit maximale.

Les critères de beauté utilisés dépendent le plus souvent des règles typographiques attachées à la langue ou au langage employé (mathématiques par exemple), mais il peut également, à tout moment, prendre en compte les goûts de l'auteur. TEX gère ainsi la ponctuation, les ligatures, les coupures de mots et les justifications verticales et horizontales.

Un autre avantage de TEX est la facilité avec laquelle il donne accès à toutes ces possibilités de composition. En mathématiques par exemple, il offre une quantité incroyable de symboles et connaît leurs conventions de mise en page (taille, fonte, espacement, etc).

Un dernier avantage couvre tous les problèmes de numérotation des paragraphes, de tables des matières, des figures, de références croisées, de bibliographie, d'index, etc, etc. Toutes ces aides de lecture sont gérées automatiquement par TEX. L'auteur a peu à s'en soucier, il n'a qu'à les déclarer.

En conclusion, vous n'avez qu'à penser au contenu de votre document, TEX se charge du reste.

1.3  Quels sont les principes de base de LATEX ?

LATEX peut être considéré comme un langage de programmation évolué dans le sens où il s'appuie sur TEX qui est un langage de plus bas niveau. Langage de programmation signifie également que le document que l'on veut créer doit être décrit dans un fichier source (.tex) puis doit être compilé. Ainsi, le compilateur LATEX prend en entrée un fichier source écrit en LATEX et produit en sortie un fichier .dvi (device independent). Ce fichier peut ensuite être converti en fichier postscript avant d'être imprimé. Les fichiers dvi et postscript peuvent être visualisés à l'écran à l'aide de (pre)viewers.

L'intérêt du format dvi est qu'il permet à TEX et LATEX d'être indépendants du matériel qui sera utilisé pour la visualisation ou l'impression finale du document.

Le fichier source (fichier.tex) doit comprendre un certain nombre de commandes (balises) LATEX qui vont permettre au compilateur LATEX de construire un fichier ``device independent'' (.dvi). La plupart des commandes LATEX se caractérisent par le fait qu'elles commencent par un ``backslash'', que leurs arguments obligatoires apparaissent entre accolades ({ et }) et que leurs arguments optionnels apparaissent entre crochets ([ et ]). La structure minimale d'un rapport est en est la suivante :

Exemple :
 
\documentclass{classe} 
\begin{document} 
   Votre texte... 
\end{document} 
%%%% fin exemple %%%%
Remarque :
Le passage de LATEX2.09 à se traduit dans l'en-tête des documents par :
\documentclass[options]{class} 
\usepackage{style} 
\usepackage{package}
au lieu de :

\documentstyle[options,styles,packages]{class}
ATTENTION
toutefois, certains styles LATEX2.09 ne seront pas reconnus par .
Les commandes LATEX décrivent ainsi la classe d'un document, sa structure, etc. Les principales classes de document disponibles sont : 'article', 'report', 'book', 'letter' et 'slides'. Il existe également un certain nombre d'options qui permettent de modifier le style par défaut d'une classe (le format a4, la taille de la fonte 12pt, etc). Ces classes et options permettent de disposer d'une structure de base pour un document, mais libre à vous ensuite de définir vos propres structures grâce aux styles offerts, aux packages disponibles notamment sur les sites CTAN (cf. paragraphe ) et/ou à vos propres macros TEX et LATEX.

L'intérieur d'un document de classe 'article', 'report' ou 'book' est ensuite structuré grâce aux balises disponibles de type : \part{}, \chapter{}, \(sub)*section{}, etc. Les lettres et les transparents font appel à d'autres structures particulières (cf. chapitres et ).

Les informations présentées dans ces structures peuvent être mises sous différentes formes grâce à des environnements tels que tabular ou itemize.

Exemple
:
\documentclass[12pt]{report} 
\usepackage{french} 

\title{Mon premier document {\LaTeX} \\ 
        Qu'il est beau ! !} 
\author{C'est moi l'auteur.} 
 

\begin{document} 
 

\maketitle 
\tableofcontents 
 

\part{Une partie.} 
        \chapter{Un chapitre.} 
        Texte... 
           \section{Une section.} 
              Texte... 
           \section{Une autre section.} 
              Texte... 
              \subsubsection{Avec une sous-section.} 
                 Texte... 
              \subsubsection{Plus une autre.} 
                  \begin{table}[htbp] 
                  \begin{center} 
                       \begin{tabular}{|c||c|} 
                          \hline 
                          donn\'ees & donn\'ees \\ 
                          \hline 
                       \end{tabular} 
                       \caption{Titre table. \label{table-}} 
                    \end{center} 
                 \end{table} 
\part{Une courte deuxi\`eme partie.} 
Texte... 
\appendix 
\chapter{Et une annexe pour finir.} 
Texte... 
\begin{itemize} 
   \item bla bla 1 
   \item bla bla 2 
\end{itemize} 
\end{document} 
%%%% fin exemple %%%%

1.4  Qu'est ce que le préambule du fichier source ?

Les appels à des packages ainsi que les définitions de nouvelles commandes sont placés dans le préambule du document LATEX (i.e. entre les balises \documentstyle (LATEX2.09) ou \documentclass () et la commande \begin{document}).

1.5  Comment faire ses premiers pas ?

En plus de la présentation faite ci-dessus quelques notions supplémentaires sont utiles à la compréhension de LATEX. Celles-ci concernent essentiellement la saisie d'un texte.

Lors que l'on désire travailler dans une langue comportant des caractères accentués LATEX propose des saisies un peu barbares surtout aux yeux des débutants (\'{e} pour é par exemple) mais qui permettent de conserver la portabilité du document ainsi saisi sur tous systèmes (caractères codés sur 7 bits). En revanche l'utilisation de fontes contenant des caractères accentués (codées sur 8 bits) réduit cette portabilité (pour plus de détails lire les questions et ).

Autre remarque importante du point de vue de la gestion des espaces et des retours chariot inclus dans le fichier source (.tex) d'un document. LATEX gère tout seul les espaces : il est inutile de taper plusieurs espaces de suite entre deux mots, ils seront transformés en un seul dans le fichier .dvi.

En outre, UN retour chariot est considéré comme un espace, sauf s'il est suivi d'un deuxième, il marque alors la fin d'un paragraphe et le prochain sera indenté. On peut alors sauter autant de lignes que l'on veut dans le texte, cela n'a aucun effet supplémentaire. Un passage à la ligne peut être forcé par \\ ou par \newline mais dans ce cas, la première ligne du nouveau paragraphe ne sera pas indentée. \\* empêche un saut de page après le saut de ligne demandé. La commande \par permet de commencer un nouveau paragraphe en laissant un espace vertical plus important et en indentant.

Remarque :
la commande \\[lgr]{} peut prendre comme paramètre une longueur lgr pour augmenter localement un interligne.
En LATEX, tout ce qui suit un % n'est pas lu, jusqu'au prochain retour chariot. (Au passage le caractère % peut alors être obtenu par \%.)

1.6  Comment sont gérées les options de package ?

Quand on utilise plusieurs packages :

\usepackage{package1,package2}
et que l'on veut utiliser une option du package1 qui n'existe pas pour le package2, on peut écrire :

\usepackage[option1]{package1} 
\usepackage{pckge2}
Cela évite un message du type ``unknown option1 for pckge2'' qui peut apparaître quand on écrit

\usepackage[option1]{package1,package2}
On peut également écrire :

\documentclass[option1,gnagna]{article} 
\usepackage{pckge1,pckge2}
qui permet de conserver l'ordre de chargement des packages mais pas celui dans lequel seront exécutées les options par tel package. Les options de classe sont globales et descendent à toutes les extensions chargées si elles sont définies pour ces extensions (certaines options sont par nature globales comme draft, final, french, a4paper, dvips...).

1.7  Quelle est la structure d'une page LATEX ?

La commande \layout du package 'layout' permet de visualiser la structure d'une page et ses différents paramètres. Globalement elle est composée du corps du texte, d'une entête et d'un pied de page. Des marges sont également définies de chaque côté du corps du texte.

Exemple
:
\documentclass{report} 
\usepackage{layout} 
\begin{document} 
\layout 
\end{document} 
%%%% fin exemple %%%%
A chaque classe de documents sont associées différentes valeurs aux paramètres de mise en page. L'utilisateur peut également redéfinir ces valeurs de paramètres. Cela lui permet de gérer sa propre mise en page. Cette FAQ donne un certain nombre de moyens dans ce but.

Pour les documents devant être imprimés en recto-verso, \oddsidemargin définit la marge gauche des pages impaires (recto), et \evensidemargin la marge gauche des pages paires (verso). Pour les documents simple face, la commande \oddsidemargin suffit.

Les principaux paramètres d'une page sont les suivants :

1.8  Quelles sont les commandes de compilation LATEX ?

A partir d'un fichier source fichier.tex, pour générer un fichier fichier.dvi, il faut appliquer :

latex fichier.tex
Lorsqu'une compilation échoue sur une erreur, la ligne où est située l'erreur est indiquée. L'emplacement de l'erreur dans la ligne est précisé par un retour à la ligne. Une explication succincte de l'erreur est également fournie.

Lorsque la compilation se termine normalement, elle produit un fichier fichier.dvi qui peut être visualisé par un utilitaire tel que :

(UNIX) xdvi fichier.dvi
A partir d'un fichier fichier.dvi, pour générer un fichier postscript, il faut utiliser un utilitaire tel que :

(UNIX) dvips fichier.dvi
Le fichier fichier.ps alors généré peut être imprimé. Par exemple :

(UNIX) lpr -Pimprimante fichier.ps

1.9  Quels sont les fichiers utilisés par LATEX ?

Il en existe différentes sortes :

1.10  A quoi correspondent les messages Overfull ?

Lorsque LATEX n'arrive pas à satisfaire tous ses critères de beauté, il peut avoir à en violer un. Cette entorse est alors indiquée par un message de type :

Overfull \hbox (4.02349pt too wide) in paragraph at 
lines 95-98
qui précise le type de dépassement, sa valeur et sa localisation.

Chapter 2
Documentation

Voici une liste d'ouvrage en ligne ou d'imprimeur qui vont vous permettre de devenir des pros de LATEX. Un gros investissement en terme de lecture et de travail est néanmoins nécéssaire. Il faut un peu souffrir pour avoir de beaux documents

2.1  Que puis-je lire sur TEX ?

Il est disponible sur http://www.cs.utk.edu/~eijkhout/tbt.html.

2.2  Que puis-je lire sur LATEX2.09 ?

2.3  Que puis-je lire sur ?

Remarque :
très bien pour qui cherche un guide très complet. En particulier, il met en permanence l'accent sur ce qui est différent/spécifique entre et LATEX2.09.

2.4  Que puis-je lire sur AMS-LATEX ?

2.5  Que puis-je lire sur la typographie ?

2.6  Où trouver une bibliographie complète ?

http://www.loria.fr/tex/texbib.html propose une bibliographie tenue à jour.

2.7  Que puis-je lire sur les fontes ?

Chapter 3
Sites Web

Toutes les bonnes urls, pour découvrir, apprendre, télécharger LATEX. Le système Comprehensive TEX Archive Network rassemble en un même endroit, toutes ces informations.

3.1  Où trouver des infos sur le WEB ?

CTAN ou Comprehensive TEX Archive Network Le système CTAN se propose de rassembler diverses informations concernant TEX et son environnement suivant une organisation commune. En particulier, tous les sites dits CTAN ou leurs miroirs essaient de se synchroniser pour offrir des informations cohérentes. En voici quelques uns :

Une liste complète des sites CTAN peut être trouvée sur ftp://ftp.fdn.org/pub/CTAN/usergrps/info/usergrps.tex.

ATTENTION (08/96) :
Le serveur d'archives (La)TEX aux États-Unis, ftp.shsu.edu, n'étant plus à jour par défaut de maintenance, il est fortement déconseillé désormais d'y récupérer des utilitaires (La)TEX. Il reste donc les 2 serveurs CTAN < < de base > >, l'anglais et l'allemand, ainsi que leurs fidèles copies en France dont celle du LORIA-CNRS à Nancy : ftp://ftp.loria.fr.
La plupart des sites CTAN offrent une fonctionnalité QUOTE SITE INDEX qui permet d'accéder plus rapidement à l'information que l'on recherche sous forme d'expression régulière. En France, le mieux est de consulter : http://www.loria.fr/cgi-bin/ctan-index.

Remarque
: il existe une liste de diffusion des annonces CTAN : CTAN-Ann@SHSU.edu. Pour s'y abonner, il faut envoyer un mail à listserv@SHSU.edu.
Ce catalogue présente une liste des packages disponibles avec une courte description de chacun. Il est mirroré par les sites CTAN sur ftp://ftp.fdn.org/pub/CTAN/help/Catalogue/. Une version gzip de ce catalogue est disponible sur : http://www.cdrom.com/pub/tex/ctan/help/Catalogue/catalogue.html.gz.

Voici quelques exemples de sites :

3.2  Où trouver cette FAQ ?

3.3  Existe t'il des listes de discussion francophones ?

Il existe la liste gut@ens.fr (de l'association GUTenberg) dont les archives sont stockées sur : http://www.univ-rennes1.fr/LISTES/gut@ens.fr/arc/maillist.html et sur lesquelles on peut faire des recherches par auteurs, sujets, etc.

Pour s'inscrire, envoyer un mail:

To: listserv@ens.fr 
From: votre_nom_d'utilisateur@votre.site 
Subject: 
 

subscribe gut Votre_Nom Votre_institution

Pour se désabonner :

mailto:listserv@ens.fr 
Subject: 
unsubscribe gut
Voir aussi le site Web de GUTenberg, http://www.gutenberg.eu.org/.

Il existe également la liste omega@ens.fr. Cette liste de diffusion concerne Omega, une extension de TEX développée par John Plaice et Yannis Haralambous. Pour vous abonner, envoyer un courrier électronique à listserv@ens.fr, sans sujet ni signature, dont le corps du message contient exclusivement la ligne : subscribe omega PRENOM NOM

La liste de diffusion « typographie » est consacrée aux problèmes de composition, de typographie - française ou étrangère - et de mise en pages, sans exclusive des techniques employées : du lettrage à la main à la composition numérique en passant par le plomb, du support papier à la page écran, du « bon usage » du Code typographique jusqu'aux normalisations du codage des caractères.

Ses abonnés ont en commun l'amour de la chose imprimée, y compris sous ses formes modernes. Pour s'inscrire, envoyez à listserv@irisa.fr le message suivant :

SUBSCRIBE typographie 

3.4  Où trouver d'autres FAQs LATEX ?

Chapter 4
Sources TEX et LATEX

Les distributions TEX/LATEX sont souvent libres. Les sources sont donc disponibles et il est possible de les télécharger. De plus elles sont disponibles pour la plupart des systèmes d'exploitation du monde informatique.

4.1  Où trouver les sources pour Unix ?

La distribution GUTenberg est disponible par ftp sur ftp://ftp.gutenberg.eu.org/pub/gut/. Elle est basée sur MlTEX (adapté à ), TEX, Metafont. Les binaires sont disponibles pour :

Différentes distributions source Unix de TEX sont disponibles sur ftp://ftp.fdn.org/pub/CTAN/systems/unix/.

La distribution teTEX pour Unix, Linux ou Irix est disponible sur ftp://ftp.fdn.org/pub/CTAN/systems/unix/teTeX/distrib/. A l'origine, cette distribution a été développée pour Linux. Il existe également un package 'config' qui offre des fichiers de configuration de cette distribution disponibles sur ftp://ftp.fdn.org/pub/CTAN/systems/unix/teTeX/contrib/.

4.2  Où trouver les sources pour VMS ?

TEX pour VMS est disponible sur ftp://ftp.fdn.org/pub/CTAN/systems/vms/ puis Alpha/ ou VAX/ suivant l'architecture utilisée.

La distribution OpenVMS VAX et AXP de GUTenberg est disponible par ftp sur ftp://ftp.gutenberg.eu.org/pub/gut/. Elle s'appuie sur TEX et .

4.3  Où trouver les sources pour DOS, OS/2, Windows3.x/95/NT ?

Remarque :
la distribution DOS-GUT n'est plus activement développée. Elle a été remplacée par WIN-GUT.

4.4  Où trouver les sources pour MacOS ?

Remarque :
Il semble qu'Euro-Oztex ne puisse plus être trouvé sur le réseau. Quelque spécialiste du Mac pourrait nous éclairer sur la question?
Remarque :
Euro-Oztex est une vieille version de la distribution de GUTenberg. Elle utilisait OzTEX 1.7. Elle a été remplacée récemment par Mac-GUT, qui elle utilise CMacTEX.

4.5  Où trouver d'autres sources ?

4.5.1  Atari

TEX pour Atari ST est disponible par ftp sur ftp://atari.archive.umich.edu/atari/tex/ ou sur ftp://ifi.informatik.uni-stuttgart.de/pub/atari.st/tex/ ou sur ftp://ftp.fdn.org/pub/CTAN/systems/atari/. Pour tous renseignements contacter atari@atari.archive.umich.edu par un mail ``help''.

4.5.2  Amiga

PasTEX, implémentation de TEX 3.1 et METAFONT 2.7 sont disponibles par ftp anonyme sur ftp://merlin.etsu.edu/ab20/AMIGA/ ou sur ftp://forwiss.uni-passau.de/pub/amiga/tex/ ou sur ftp://ftp.fdn.org/pub/CTAN/systems/amiga/.

On peut trouver également PasTEX 1.4 sur les miroirs FTP aminet (sunsite.cnam.fr. ftp.grolier.fr, ftp.netnet.net, ...) dans le répertoire /pub/aminet/text/tex/. PasTEX 1.4 est le portage de . La distribution comprend également dvips.

XFig est disponible dans /pub/aminet/gfx/edit/.

Ghostscript est disponible dans /pub/aminet/gfx/show/.

Une version a également été développée à l'ESIEE http://www.esiee.fr/~tex/Install/Amiga/index.html.

4.5.3  Tandy 6000

Pour tous renseignements contacter Ken Yap (ken@syd.dit.csiro.au).

4.5.4  TOPS-20

Une distribution TEX sur TOPS-20 est disponible par ftp anonyme sur ftp://ftp.math.utah.edu/pub/tex/pub/web/.

Chapter 5
Éditer LATEX

Les éditeurs ci-dessous sont classés par ordre alphabétique sur le nom, afin d'éviter toute dispute sur la place de l'un ou l'autre.

5.1  Alpha

Alpha est un éditeur Macintosh shareware assez proche d'emacs. Il est disponible entre autres par ftp sur ftp://alpha.olm.net/pub/.

Cet éditeur est hautement configurable grâce à un langage de programmation intégré, Tcl. Il possède entre autres un mode LATEX très convivial. Alpha permet aussi une interaction avec le compilateur (Texture (commercial), CMACTEX, OzTex ou Direct-Tex) en lançant la compilation d'une combinaison de touches. La dernière version d'Alpha est la 7.2. Elle comprend la version 3.2 des macros freeware Alpha LATEX de T. Scavo.

Pour plus de renseignements, voir http://alpha.olm.net/.

5.2  Cicero

Cicero est un traitement de texte sous X11R6 et Motif2.0. Le package comprend des fontes X postscript, ghostscript, TEX (dvips) et Cicero. Pour plus de renseignements, consulter : http://zeus.informatik.uni-frankfurt.de/~weis/cicero.html.

5.3  Eddi4TEX

Eddi4TEX, sous MS-DOS ou OS/2, est un éditeur spécifiquement conçu pour TEX, il offre la couleur, vérifie la syntaxe. Il est disponible sur ftp://ftp.fdn.org/pub/CTAN/systems/msdos/e4t/ ou dans ftp://ftp.fdn.org/pub/CTAN/systems/OS2/epmtex/.

5.4  (X-)Emacs

(X-)Emacs est un éditeur sous Unix qui offre en standard un mode d'édition, un peu fruste mais néanmoins pratique, facilitant la composition de documents (La)TEX. Une extension à emacs, AUC-TEX (disponible sur ftp://ftp.fdn.org/pub/CTAN/support/auctex/ ou sur http://sunsite.auc.dk/auctex/), fournit de nombreuses facilités supplémentaires (indentation automatique, messages d'erreur en anglais compréhensible, gestion des documents multi-fichiers, etc.)

Emacs reconnaît automatiquement certaines extensions (tex,sty...) dans un nom de fichier, et active le mode en question automatiquement. Si votre fichier n'est pas reconnu comme un document (La)TEX, vous pouvez spécifier sur la première ligne de votre fichier : % -*-latex-*-

Les packages 'font-lock' et 'hilit19' ('hilit319' pour Xemacs) ou plus récent 'font-latex' (basé sur font-lock), permettant de choisir les couleurs et les polices mettant en évidence la syntaxe d'un fichier, sont utilisables avec les modes (La)TEX. Pour plus de détails, voir : ftp://ftp.fdn.org/pub/CTAN/support/ultratex/.

Le package (standard) 'imenu' donne accès à un menu listant les en-têtes de section du document, et permet de retrouver celles-ci facilement dans un grand document. Une extension à ce mécanisme permet de mieux visualiser la structure du document, en indentant les sous-sections.

5.5  Funtek

Funtek, de V. Vidal, sous X-Windows system et Motif, est un éditeur texte orienté LATEX, il traite le source LATEX page à page, permet un accès aux symboles spéciaux, et une construction de tableaux automatique. La version actuelle est une bêta.

5.6  GNU emacs

GNU emacs et AUCTEX peuvent également être utilisés sous MS-DOS ou OS/2.

5.7  JED

JED est un clone multi-plateformes proche d'emacs. JED est en fait un emacs allégé qui offre des facilités dans l'édition de fichier (La)TEX. Il tourne sous Unix/VMS/Dos/Win. Il est disponible à http://space.mit.edu/~davis/.

5.8  LYX

LYX est un traitement de texte sous X11 qui offre une sortie . Il est presque WYSIWYG. LYX présente les avantages d'être petit, rapide et gratuit. LYX est encore en développement. Une version NON définitive est disponible à http://www.lyx.org/ ou par ftp sur ftp://ftp.lip6.fr/pub/linux/sunsite/X11/xapps/editors/ ou par ftp sur ftp://ftp.lyx.org/pub/lyx/ ou sur le web par http://www.lehigh.edu/~dlj0/LyriX.html. Il existe également une liste de discussion : lyx@lyx.org.

Parmi les fonctionnalités, on trouve: - éditeur d'équations, - éditeur de tables, - inclusion d'images au format EPS, - correction d'orthographe, - etc.

Les dernières docs sont disponibles par ftp sur ftp://ftp.lyx.org/pub/lyx/doc/.

5.9  MicroEmacs

MicroEmacs (inclu dans la distribution DOS-GUT), sous windows, permet d'éditer et de gérer des documents TEX.

5.10  NEdit

NEdit est un éditeur entièrement conçu sous X dès le début. Il offre des "look and feel" semblables aux éditeurs qu'on trouve sur Windows et Mac, par ex, les touches de raccourcis standards sont Ctrl+X pour couper, Ctrl+C pour copier, Ctrl+V pour coller. Ainsi, les nouveaux utilisateurs immigrés des autres système d'exploitation n'auront pas trop de mal à réadapter facilement NEdit. C'est un éditeur très configurable, par ex la police des caractères, les couleurs (enfin, parfois il faut être un peu bidouilleur aussi :) ). C'est un éditeur à usage général. Il n'est pas seulement utile pour LATEX.

L'origine de cette discussion est de trouver un remplaçant de Kedit pour Dos. Je suppose que ce programme a pas mal de fonctions qu'on ne trouve pas ailleurs, ni sous NEdit. Mais il est possible de programmer la plupart (si ce n'est pas tout) de ces fonctions sous NEdit à l'aide de macro.

(Ce message, de Seak Teng-Fong, seak.teng-fong@iname.com, a été abrégé, pour le message complet, référez vous à Message-ID: <3881E519.437B2409@iname.com>).

Pour plus de détails sur NEdit: http://www.nedit.org. NEdit est sous licence GPL.

5.11  Scientific Word

Scientific Word pour windows est un éditeur qui permet presque de visualiser un document LATEX en WYSIWYG (What You See Is What You Get). Il facilite l'édition d'un document LATEX en permettant une insertion aisée des symboles, l'édition de tableaux, ... par le "mulot" ou par raccourcis clavier. Pour l'utiliser pleinement, il vaut mieux connaître LATEX.

Remarque : ce produit est commercial et cher.

5.12  STEAD

STEAD "Sympathetic Tk-based Editor for Average Dummies" est un éditeur de texte convivial pour Unix (ressemble à Alpha sur mac). Il est simple d'utilisation, contextuel et configurable. Recherche/remplacement pouvant utiliser les expressions régulières - undo/redo multiniveau - colorisation - transformation possible de la sélection (y compris rectangulaire) par une commande Unix - ...

La particularité de cet éditeur est qu'il est entièrement interprété ! En effet, il est écrit en langage TCL et TK. Necessite l'ancienne version wish3.6 (sources et binaire fournis).

Disponible sur : http://www.ensta.fr/~diam/stead/ Contact: Maurice DIAMANTINI <diam@ensta.fr>

5.13  TEXnicCenter

TEXnicCenter, écrit par Sven Wiegand, est disponible sur ftp://ftp.fdn.org/pub/CTAN/systems/win32/TeXnicCenter/. C'est un environnement de développement intégré (IDE en anglais) LATEX pour Windows distribué sous liscence GPL. L'interface est très proche de celle que l'on peut trouver dans des outils du style de MS Visual Studio. Bien qu'encore à l'état de bêta, il offre la coloration des mots clés, une gestion aisée de larges projets, une vue structurée des fichiers, des sections du document, des flottants et plein d'autres fonctionnalités fort intéressantes. Le correcteur orthographique est prévu pour la prochaine bêta. Pour plus d'infos, vous pouvez consulter sa page web : http://www.toolscenter.org/texniccenter/index.html.

5.14  TEXShell

TEXShell, écrit par J. Schlegelmilch, est disponible sur ftp://ftp.fdn.org/pub/CTAN/systems/msdos/texshell/ts271.zip. C'est un éditeur Windows sous dos qui offre une coloration des mots clés LATEX, une aide en ligne et d'autres petites fonctionnalités telles que les compilations associées à des boutons. La distribution DOS-GUT utilise TEXshell et offre un MicroEmacs francisé.

Il existe également TEXShell pour X Window system (Tcl/Tk) disponible par ftp sur ftp://sunsite.unc.edu/pub/Linux/apps/tex/.

5.15  ViM

ViM, développé par Bram Moolenaar, dispose dans sa dernière version de la coloration syntaxique, à l'instar d'emacs; à noter cependant la légèreté de ViM par rapport à l'usine à gaz GNUienne. Vous trouverez plus d'infos sur ViM sur http://www.vim.org/ ou par ftp sur ftp://ftp.vim.org/pub/vim/ (également reflété par ftp.lip6.fr).

5.16  WINEDT95

WINEDT95,logiciel Shareware pour Windows 95, est un éditeur avec menu TEX/LATEX qui permet de repérer les commandes LATEX et de compter les délimiteurs. Il est disponible sur ftp://ftp.fdn.org/pub/CTAN/support/winedt/. Il est pourvu d'un correcteur orthographique.

5.17  wintex95

wintex95 disponible sur ftp://ftp.fdn.org/pub/CTAN/systems/win32/wtex95/ est un éditeur flexible offrant un coloriage automatique de la syntaxe LATEX. Il offre également une complétion automatique, et des palettes de symboles, un éditeur de tableaux, des touches de raccourci, le lancement de programmes externes, etc. C'est un shareware.

5.18  Xcoral

Xcoral, éditeur sous Unix offre des fonctionnalités de même type que emacs mais non interactives. Il est disponible par ftp sur ftp://ftp.inria.fr/X/contrib-R5/clients/ ou sur ftp://ftp.x.org/contrib/.

Xcoral est un éditeur multi-fenêtres pour X Window System, offrant un certain nombre de facilités notamment pour écrire des programmes perl, ada, fortran, C, C++, java ainsi que des documents LATEX ou HTML.

Cet éditeur comprend un interpréteur Ansi C 'built-in' qui permet aux utilisateurs d'étendre ses fonctionnalités facilement. Un manuel d'aide complet indexé est disponible on-line.

5.19  X-Window Shell pour TEX

X-Window Shell pour TEX (OpenLook ou Xaw/Xaw3d) disponible sur http://www.tm.bi.ruhr-uni-bochum.de/personal/marc/xtexshell.html.

5.20  xtem

xtem: une interface graphique offrant des fonctionnalités TEX/LATEX disponible sur http://ftp.lrw.uni-bremen.de/xtem/xtem_texmenu.html.

5.21  Autres

D'autres éditeurs sous dos, Mac et windows95 sont présentés dans http://www.jumbo.com/.

Chapter 6
Gestion de la mise en page

Mettre en page un document sous LATEX est assez simple si l'on est capable d'attendre la compilation pour visualiser le document final. Il est vrai que cela s'apparente plus à de la description de page que du traitement de texte.

6.1  Comment modifier l'interligne d'un document ?

Pour modifier l'espace interligne d'un document on peut utiliser la commande \linespread (solution non recommandée). Par exemple, \linespread{1.6} permet de doubler l'intervalle par défaut.

\renewcommand{\baselinestretch}{1.2} 
placé dans le préambule permet d'obtenir le même résultat pour tout le document.

Il existe également les packages 'doublespace' (pour LATEX2.09) et 'setspace' (pour ) disponibles sur ftp://ftp.fdn.org/pub/CTAN/macros/latex2.09/contrib/misc/ pour l'un et ftp://ftp.fdn.org/pub/CTAN/macros/latex/contrib/other/misc/ ou ftp://ftp.fdn.org/pub/CTAN/macros/latex/contrib/supported/setspace/ pour l'autre. setspace.sty définit les environnements singlespace, onehalfspace et doublespace. L'utilisation de ces styles est recommandée parce que plus robuste (gestion des tableaux, des notes de bas de page, ...).

Pour réduire l'interligne d'un paragraphe (à celui de small par exemple) sans modifier la taille de la fonte on peut utiliser

\small{\normalsize texte à interligne réduit}\par 
ou encore

{\advance\baselineskip -1pt le texte \par}
Localement, on peut également utiliser la commande \baselineskip de la manière suivante :

Exemple :
 
{\setlength{\baselineskip}{1.2\baselineskip} 
Texte affecté 
\par} %%% <= terminer le paragraphe 
%%%% fin exemple %%%%

6.2  Comment gérer un document recto-verso ?

prévoit directement les options de classe twoside et openright.

Exemple :
 
\documentclass[twoside,openright]{report} 
%%%% fin exemple %%%%
En LATEX 2.09 il faut passer twoside comme option de la commande documentstyle. Ensuite, pour forcer les entêtes de chapitre à commencer sur une page impaire, il faut inclure la commande \cleardoublepage avant chaque début de chapitre.

Exemple :
 
\documentstyle[twoside]{report}

 

\begin{document} 
\cleardoublepage 
\chapter{Introduction.} 
        Texte. 
 

\cleardoublepage 
\chapter{Thèse.} 
        Texte.

 

\end{document} 
%%%% fin exemple %%%%

Il existe également la macro suivante qui redéfinit la commande \cleardoublepage pour que les pages insérées soient vides (i.e. sans entête ni bas de page).

%%%% debut macro %%%% 
% whitecdp (formerly schulzrinne.sty) --provide for blank pages 
% between chapters 
% This redefinition of the \cleardoublepage command provides 
% for a special pagestyle for the "extra" pages which are generated 
% to ensure that the chapter opener is on a recto page. 
% The pagestyle is "chapterverso"; for many publishers, this should be 
% identical to "empty", so that's the default. 
\def\cleardoublepage{\clearpage 
 \if@twoside 
  \ifodd\c@page\else 
   \null\thispagestyle{chapterverso}\newpage 
   \if@twocolumn\null\newpage\fi 
   \fi 
  \fi 
 }% 
\def\ps@chapterverso{\ps@empty}% 
%%%% fin macro %%%%

6.3  Comment modifier le style des titres ?

Les définitions de \section, \sub(sub)section, etc, se trouvent dans les fichiers .cls (report.cls, article.cls, book.cls).

ATTENTION :
Il est vivement conseillé de ne pas modifier directement ces classes mais de redéfinir un fichier.sty avec les nouvelles commandes ou d'utiliser \makeatletter et \makeatother.
Exemple :
 
La syntaxe de définition d'une nouvelle section est :

\renewcommand\section{\@startsection {section}{1}{\z@}% 
        {-3.5ex \@plus -1ex \@minus -.2ex}% 
        {2.3ex \@plus.2ex}% 
        {\reset@font\Large\bfseries}}
Explication :
 
        Titre de ma section.        Nous allons parler... bla, 
        bla, bla...
%%%% fin exemple %%%%
Pour augmenter l'espace avant une section il suffit, par exemple, d'écrire, dans le préambule du document :

%%%% debut macro %%%% 
\makeatletter 
\renewcommand\section{\@startsection{section}{1}{\z@}% 
        {2cm \@plus -1ex \@minus -.2ex}% 
        {2.3ex \@plus.2ex}% 
        {\reset@font\Large\bfseries}} 
\makeatother 
%%%% fin macro %%%%
Le package 'sfheaders' de M. Loreti, ci dessous met les titres dans une fonte sans serif quelle que soit la classe de document utilisée. Pour l'utiliser, il suffit de sauvegarder les macros suivantes dans un fichier SFheaders.sty et d'appeler \usepackage{SFheaders}.

%%%% debut macro %%%%  
 % Package: SFheaders.sty  
 % Sans-Serif headers; modified from {book|report|article}.cls  
 % defaults.  
 % Last modified: MLO 1997-05-06  
 %  
 % Author: Maurizio Loreti, aka MLO or (HAM) I3NOO  
 % Work:   University of Padova - Department of Physics  
 %         Via F. Marzolo, 8 - 35131 PADOVA - Italy 
 % EMail:  loreti@padova.infn.it 
 % WWW:    http://wwwcdf.pd.infn.it/loreti/mlo.html 
 \NeedsTeXFormat{LaTeX2e} 
 \ProvidesPackage{sfheaders}[1997/05/06 Sans-Serif headers] 
 \@ifclassloaded{article}{ 
 % Here if \documentclass{article} 
   \def\@part[#1]#2{% 
       \ifnum \c@secnumdepth >\m@ne 
         \refstepcounter{part}% 
         \addcontentsline{toc}{part}{\thepart\hspace{1em}#1}% 
       \else 
         \addcontentsline{toc}{part}{#1}% 
       \fi 
       {\parindent \z@ \raggedright 
        \interlinepenalty \@M 
        \normalfont 
        \ifnum \c@secnumdepth >\m@ne 
          \Large \sffamily \bfseries \partnamethepart 
          \par\nobreak 
        \fi 
        \huge \sffamily \bfseries #2% 
        \markboth{}{}\par}% 
       \nobreak 
       \vskip 3ex 
       \@afterheading} 
   \def\@spart#1{% 
       {\parindent \z@ \raggedright 
        \interlinepenalty \@M 
        \normalfont 
        \huge \sffamily \bfseries #1\par}% 
        \nobreak 
        \vskip 3ex 
        \@afterheading}} 
 {\@ifclassloaded{book}{ 
 % Here if \documentclass{book} 
   \def\@part[#1]#2{% 
       \ifnum \c@secnumdepth >-2\relax 
         \refstepcounter{part}% 
         \addcontentsline{toc}{part}{\thepart\hspace{1em}#1}% 
       \else 
         \addcontentsline{toc}{part}{#1}% 
       \fi 
       \markboth{}{}% 
       {\centering 
        \interlinepenalty \@M 
        \normalfont 
        \ifnum \c@secnumdepth >-2\relax 
          \huge \sffamily \bfseries \partnamethepart 
          \par 
          \vskip 20\p@ 
        \fi 
        \Huge \sffamily \bfseries #2\par}% 
       \@endpart} 
   \def\@spart#1{% 
       {\centering 
        \interlinepenalty \@M 
        \normalfont 
        \Huge \sffamily \bfseries #1\par}% 
       \@endpart} 
   \def\@makechapterhead#1{% 
     \vspace*{50\p@}% 
     {\parindent \z@ \raggedright \normalfont 
       \ifnum \c@secnumdepth >\m@ne 
         \if@mainmatter 
           \huge \sffamily \bfseries \@chapapp\space \thechapter 
           \par\nobreak 
           \vskip 20\p@ 
         \fi 
       \fi 
       \interlinepenalty\@M 
       \Huge \sffamily\ bfseries #1\par\nobreak 
       \vskip 40\p@ 
     }} 
   \def\@makeschapterhead#1{% 
     \vspace*{50\p@}% 
     {\parindent \z@ \raggedright 
       \normalfont 
       \interlinepenalty\@M 
       \Huge \sffamily \bfseries #1\par\nobreak 
       \vskip 40\p@ 
   }} 
 }{ 
 % Here if none of the above (\documentclass{report} ?) 
   \def\@part[#1]#2{% 
       \ifnum \c@secnumdepth >-2\relax 
         \refstepcounter{part}% 
         \addcontentsline{toc}{part}{\thepart\hspace{1em}#1}% 
       \else 
         \addcontentsline{toc}{part}{#1}% 
       \fi 
       \markboth{}{}% 
       {\centering 
        \interlinepenalty \@M 
        \normalfont 
        \ifnum \c@secnumdepth >-2\relax 
          \huge \sffamily \bfseries \partnamethepart 
          \par 
          \vskip 20\p@ 
        \fi 
        \Huge \sffamily \bfseries #2\par}% 
       \@endpart} 
   \def\@spart#1{% 
       {\centering 
        \interlinepenalty \@M 
        \normalfont 
        \Huge \sffamily \bfseries #1\par}% 
       \@endpart} 
   \def\@makechapterhead#1{% 
     \vspace*{50\p@}% 
     {\parindent \z@ \raggedright \normalfont 
       \ifnum \c@secnumdepth >\m@ne 
           \huge \sffamily \bfseries \@chapapp\space \thechapter 
           \par\nobreak 
           \vskip 20\p@ 
       \fi 
       \interlinepenalty\@M 
       \Huge \sffamily \bfseries #1\par\nobreak 
       \vskip 40\p@ 
     }} 
   \def\@makeschapterhead#1{% 
     \vspace*{50\p@}% 
     {\parindent \z@ \raggedright 
       \normalfont 
       \interlinepenalty\@M 
       \Huge \sffamily \bfseries #1\par\nobreak 
       \vskip 40\p@ 
     }} 
 }} 
 \renewcommand{\section}{\@startsection {section}{1}{\z@}% 
              {-3.5ex \@plus -1ex \@minus -.2ex}% 
              {2.3ex \@plus.2ex}% 
              {\normalfont\Large\sffamily\bfseries}} 
 \renewcommand{\subsection}{\@startsection{subsection}{2}{\z@}% 
              {-3.25ex\@plus -1ex \@minus -.2ex}% 
              {1.5ex \@plus .2ex}% 
              {\normalfont\large\sffamily\bfseries}} 
 \renewcommand{\subsubsection}{\@startsection{subsubsection}{3}% 
              {\z@}% 
              {-3.25ex\@plus -1ex \@minus -.2ex}% 
              {1.5ex \@plus .2ex}% 
              {\normalfont\normalsize\sffamily\bfseries}} 
 \renewcommand{\paragraph}{\@startsection{paragraph}{4}{\z@}% 
              {3.25ex \@plus1ex \@minus.2ex}% 
              {-1em}% 
              {\normalfont\normalsize\sffamily\bfseries}} 
 \renewcommand{\subparagraph}{\@startsection{subparagraph}{5}% 
              {\parindent}% 
              {3.25ex \@plus1ex \@minus .2ex}% 
              {-1em}% 
              {\normalfont\normalsize\sffamily\bfseries}} 
 \endinput 
 %% 
 %% End of `SFheaders.sty'. 
 %%%% fin macro %%%%
Le package 'fncychap' disponible sur ftp://ftp.fdn.org/pub/CTAN/macros/latex/contrib/supported/fncychap/ propose un ensemble d'entêtes de chapitre prédéfinies.

6.4  Comment obtenir un document multicolonnes ?

L'option standard twocolumn permet de présenter un texte sur deux colonnes verticales.

Exemple :
 
\documentclass[twocolumn]{article} 
\usepackage{french} 
 

\begin{document} 
 

Voici un texte sur deux colonnes que \LaTeX n'équilibre pas 
par lui-même (il remplit les colonnes les unes après les 
autres). L'espace entre les colonnes peut être modifié comme 
indiqué plus loin. Une ligne de séparation des colonnes peut 
également être insérée. 
 

\end{document} 
%%%% fin exemple %%%%

Pour agir localement, on peut utiliser les commandes :

\twocolumn[texte sur une colonne]{texte sur deux colonnes}
puis

\onecolumn{Texte sur une colonne} 
ou plus généralement

\twocolumn 
et

\onecolumn.
Pour une meilleure lisibilité du source, on peut également utiliser les environnements correspondants.

Exemple :
 
\documentclass{article} 
\usepackage{french} 
 

\begin{document}

 

Dans la classe article, le texte est, par défaut, mis en page 
sur une seule colonne. Il est toutefois possible de passer 
temporairement sur deux colonnes.

 

\twocolumn[Un titre sur une colonne, un peu long pour le 
prouver.]{Et voici enfin un texte sur deux colonnes; comme 
promis! Encore une fois, \LaTeX n'équilibre pas le 
remplissage des deux colonnes si bien qu'il est obligé pour 
chaque changement de colonnage de changer de page.}  

\onecolumn 
Ceci permet de repasser sur une colonne pour la suite du 
document.

 

\begin{twocolumn} 
   Encore quelques mots sur deux colonnes. Même si le texte est 
   trop court pour voir apparaître la deuxième colonne. 
\end{twocolumn} 
\end{document} 
%%%% fin exemple %%%%

Le package 'multicol', disponible sur ftp://ftp.fdn.org/pub/CTAN/macros/latex/required/tools/, définit l'environnement multicols qui permet de redéfinir localement le nombre de colonnes désirées (10 maximum). Lorsqu'une page n'est pas complète, le texte apparaît réparti sur toutes les colonnes. Ainsi chaque changement de colonnage n'entraîne plus un changement de page.

Exemple :
 
\documentclass{article} 
\usepackage{multicol} 
\usepackage{french} 
\setlength{\columnseprule}{0.5pt} 
\begin{document} 
 

\begin{multicols}{3}[Titre sur une seule colonne.] 
   3colonnes équilibrées, 3colonnes équilibrées, 3colonnes 
   équilibrées, 3colonnes équilibrées 
\end{multicols}

 

\begin{multicols}{2}[\section{Titre numéroté.}] 
   blabla sur deux colonnes, c'est plus sérieux. C'est le 
   style qui est généralement utilisé pour écrire des 
   articles. 
\end{multicols} 
\end{document} 
%%%% fin exemple %%%%

Pour ajouter un titre numéroté qui apparaisse sur toute la largeur de la page, il faut utiliser l'option [\section{Titre.}] juste après \begin{multicols}{nb-col}.

Remarques :
 
Pour qu'une ligne de séparation apparaisse entre les colonnes, il faut utiliser :

\setlength{\columnseprule}{1pt}.
Pour redéfinir la largeur de l'espace inter-colonnes, il faut utiliser

\setlength{\columnsep}{30pt}.

6.5  Comment composer une brochure ?

Pour redéfinir un format de page, (par exemple un A4 plié en trois), il faut utiliser la commande \setlength. Il suffit de savoir quelles sont les longueurs à préciser, le meilleur moyen pour ce faire est de les visualiser avec la commande \layout (définie par le package 'layout' disponible sur ftp://ftp.fdn.org/pub/CTAN/macros/latex/required/tools/).

%%%% debut macro %%%% 
 % (Th. Bouche) 
 \ProvidesPackage{a6size} 
 % rien a voir avec la taille : ajustement du \baselineskip 
 \renewcommand\normalsize{% 
    \@setfontsize\normalsize\@xiipt{13.5}% 
    \abovedisplayskip 12\p@ \@plus3\p@ \@minus7\p@ 
    \abovedisplayshortskip \z@ \@plus3\p@ 
    \belowdisplayshortskip 6.5\p@ \@plus3.5\p@ \@minus3\p@ 
    \belowdisplayskip \abovedisplayskip 
    \let\@listi\@listI} 
 \renewcommand\small{% 
    \@setfontsize\small\@xipt{12.4}% 
    \abovedisplayskip 11\p@ \@plus3\p@ \@minus6\p@ 
    \abovedisplayshortskip \z@ \@plus3\p@ 
    \belowdisplayshortskip 6.5\p@ \@plus3.5\p@ \@minus3\p@ 
    \def\@listi{\leftmargin\leftmargini 
    \topsep 9\p@ \@plus3\p@ \@minus5\p@ 
    \parsep 4.5\p@ \@plus2\p@ \@minus\p@ 
    \itemsep \parsep}% 
    \belowdisplayskip \abovedisplayskip} 
 \normalsize 
   
 \setlength\paperheight {148mm}% 
 \setlength\paperwidth  {105mm}% 
 %\voffset-1cm 
 %\hoffset-2cm 
 \setlength{\topmargin}{-1.3cm}% 
 \setlength{\oddsidemargin}{-.5cm}% 
 \setlength{\evensidemargin}{-1cm}% 
 \setlength{\marginparsep}{0\p@}% 
 \setlength{\headsep}{0\p@}% 
 % calcule la hauteur du texte en fonction du \baselineskip, pour 
 % que les lignes soient placées au même niveau sur toutes les pages 
 \setlength{\textheight}{\topskip} 
 \addtolength{\textheight}{22\baselineskip}% 
 \setlength{\textwidth}{7cm}% 
 \setlength{\footskip}{23\p@}% (originellement : 48) 
 %\setlength{\baselineskip}{13\p@}% 
 %\setlength{\marginparwidth}{0\p@} % 
 %\addtolength{\baselineskip}{.2\baselineskip}% 
 \setlength{\parindent}{0\p@} 
 %\addtolength{\headsep}{\headsep} 
 %\setlength{\push@skip}{.2\textwidth} 
 \newenvironment{page}{\vspace*{\stretch{1}}} 
 {\vspace*{\stretch{2.5}}\newpage} 
 \pagestyle{plain} 
 %%%% fin macro %%%%
Il faut ensuite opérer ce que les imprimeurs appellent une imposition : imprimer la page tant à tel endroit de la x-ième feuille de telle sorte qu'il n'y ait plus qu'à plier la liasse pour obtenir un livre prêt à être relié. Cette étape est facile à réaliser soit à l'aide de dvidvi (mais qui n'autorise pas les rotations, ce qui peut en limiter l'intérêt pour des formats spéciaux) soit avec pstops :

pstops "2:0L@.7(21cm,0)+1L@.7(21cm,14.85cm)" un.ps deux.ps
disponible sur ftp://ftp.fdn.org/pub/CTAN/support/psutils/.

L'option a5paper de permet également de faire cela. Il faut ensuite utiliser dvidvi et dvips en -t landscape. dvips est disponible entre autres par ftp sur ftp://ftp.gutenberg.eu.org/pub/gut/sources/.

Le package 'a5booklet' est disponible sur ftp://ftp.fdn.org/pub/CTAN/dviware/a5booklet/.

Consulter également le package '2up' pour .

Le package 'poligraf' disponible sur ftp://ftp.fdn.org/pub/CTAN/macros/TeX-PS/cmyk-hax/ permet d'agir sur la mise en page d'un document avant impression.

psnup et psbook peuvent également permettre de faire de la composition mais il vaut alors mieux travailler avec des polices postscript. Ces utilitaires sont accessibles via : http://www.dcs.ed.ac.uk/home/ajcd/psutils/.

Exemple (J.A. Ferrez) :
 
#!/bin/sh 

# Turn a (clean) PS file into a booklet 

# input on stdin or file in arg 

# psbook -- reorder the pages 
#           see -s option for _very_ large files 
# psnup  -- scale and place two pages onto one 
# pstops -- reverse the odd pages for duplex printing 

# outup on stdout 

psbook $1 | psnup -2 | pstops "2:0,1U(21cm,29.7cm)" 
%%%% fin exemple %%%%
Le résultat est possible également en ne travaillant que sur le postscript :

dvips -h twoup -t landscape fichier.dvi.
Sur PC on peut utiliser dvidrv.

Pour PC twoup fait cela, mais ce n'est pas du domaine public.

6.6  Comment définir un format de document ?

La structure d'une page LATEX permet de nombreux ajustements :

Il existe également les packages 'a4' et 'a4wide' disponibles respectivement sur ftp://ftp.fdn.org/pub/CTAN/macros/latex/contrib/supported/ntgclass/ et dans ftp://ftp.fdn.org/pub/CTAN/macros/latex/contrib/other/misc/ qui permettent de redéfinir les marges et largeur de texte.

Le package 'geometry' est plus flexible que les précédents pour définir son propre format de page. Il définit de nouvelles variables de structure de la page. Il propose en outre un ensemble de formats par défaut (a0paper, b5paper, legalpaper, ...).

6.7  Comment redéfinir les marges d'un document ?

Le package 'vmargin', de V. Kuhlmann, est disponible sur ftp://ftp.fdn.org/pub/CTAN/macros/latex/contrib/other/misc/. Il permet facilement de redéfinir globalement (pour tout le document) les marges d'un document par la commande :

\setmarginsrb{1}{2}{3}{4}{5}{6}{7}{8} 
1 est la marge gauche 
2 est la marge en haut 
3 est la marge droite 
4 est la marge en bas 
5 fixe la hauteur de l'entête 
6 fixe la distance entre l'entête et le texte 
7 fixe la hauteur du pied de page 
8 fixe la distance entre le texte et le pied de page
Vous pouvez également utiliser des valeurs par défaut en rapport avec le papier utilisé, par exemple:

\setpapersize{A4}
L'environnement changemargin décrit ci-dessous permet de modifier localement les marges d'un document. Il prend deux arguments, la marge gauche et la marge droite (ces arguments peuvent prendre des valeurs négatives).

%%%% debut macro %%%% 
\newenvironment{changemargin}[2]{\begin{list}{}{% 
\setlength{\topsep}{0pt}% 
\setlength{\leftmargin}{0pt}% 
\setlength{\rightmargin}{0pt}% 
\setlength{\listparindent}{\parindent}% 
\setlength{\itemindent}{\parindent}% 
\setlength{\parsep}{0pt plus 1pt}% 
\addtolength{\leftmargin}{#1}% 
\addtolength{\rightmargin}{#2}% 
}\item }{\end{list}} 
%%%% fin macro %%%%
Exemple :
 
\begin{changemargin}{2cm}{-1cm} 
   Ceci permet d'augmenter la marge gauche de 2cm et de diminuer 
   celle de droite de 1cm. 
\end{changemargin} 
A comparer avec un texte qui occupe toute la largeur de la page, 
comme celui-ci. 
%%%% fin exemple %%%%
Le package 'geometry' permet de redéfinir les marges d'un document ou de définir le layout de la page.

Exemple 1 :
 
\geometry{margin=5pt}
équivalent à

\geometry{hmargin=5pt, vmargin=5pt} 
équivalent à 
\geometry{margin={5pt,5pt}} 
%%%% fin exemple %%%%
Exemple 2 :
 
\documentclass{report} 
\usepackage{french} 
\usepackage{geometry} 
\geometry{scale=1.0, nohead} 
\begin{document} 
Ainsi, le texte apparaît beaucoup plus haut dans une page. 
\end{document} 
%%%% fin exemple %%%%
Exemple 3 :
 
\documentclass{report} 
\usepackage{french} 
\begin{document} 
Voici une page normale pour comparer. 
\end{document} 
%%%% fin exemple %%%%
Le package 'truncate' disponible sur ftp://ftp.fdn.org/pub/CTAN/macros/latex/contrib/other/misc/ permet de fixer la largeur d'un texte.

Il existe également le package 'typearea' disponible sur ftp://ftp.fdn.org/pub/CTAN/macros/latex209/contrib/script/.

L'environnement narrow, de K. Reckdahl, ci-dessous permet d'encapsuler des paragraphes de largeurs différentes.

%%%% debut macro %%%% 
%----------------------------------------------------------------- 
\begin{narrow}{1.0in}{0.5in}   produces text which is narrowed 
%          by 1.0 on left margin and 0.5 inches on right margin 
\begin{narrow}{-1.0in}{-0.5in} produces text which is widened 
%          by 1.0 on left margin and 0.5 inches on right margin 
% Narrow environments can be nested and are ended by \end{narrow} 
%----------------------------------------------------------------- 
\newenvironment{narrow}[2]{% 
 \begin{list}{}{% 
  \setlength{\topsep}{0pt}% 
  \setlength{\leftmargin}{#1}% 
  \setlength{\rightmargin}{#2}% 
  \setlength{\listparindent}{\parindent}% 
  \setlength{\itemindent}{\parindent}% 
  \setlength{\parsep}{\parskip}% 
 }% 
\item[]}{\end{list}} 
%%%% fin macro %%%%

6.8  Comment changer l'orientation d'un document ?

Globalement, pour passer en orientation paysage, il suffit de mettre l'option landscape dans \documentclass () ou dans \documentstyle (LATEX2.09).

Exemple :
 
\documentclass[landscape]{report} 
\usepackage{french} 
\begin{document} 
Voici un document écrit dans un sens non conventionnel. 
\end{document} 
%%%% fin exemple %%%%
Le package 'lscape' de D. Carlisle (), disponible sur ftp://ftp.fdn.org/pub/CTAN/macros/latex/packages/graphics/, permet de changer localement d'orientation portrait vers paysage et vice versa. Il définit l'environnement landscape.

Exemple :
 
\documentclass[11pt]{report} 
\usepackage{lscape}

 

\begin{document}

 

\begin{landscape} 
   Un petit tour à la campagne,ldots 
\end{landscape}

 

et nous voici de retour dans la galerie, après un changement de 
page bien évidemment.

 

\end{document} 
%%%% fin exemple %%%%

Il existe également le package 'rotating' disponible sur ftp://ftp.fdn.org/pub/CTAN/macros/latex/contrib/supported/rotating/.

Exemple (tiré de la doc) :
 
\newcount\wang 
\newsavebox{\wangtext} 
\newdimen\wangspace 
\def\wheel#1{\savebox{\wangtext}{#1}% 
\wangspace\wd\wangtext 
\advance\wangspace by 1cm% 
\centerline{% 
\rule{0pt}{\wangspace}% 
\rule[-\wangspace]{0pt}{\wangspace}% 
\wang=-180\loop\ifnum\wang<180 
\rlap{\begin{rotate}{\the\wang}% 
\rule{1cm}{0pt}#1\end{rotate}}% 
\advance\wang by 10 \repeat}} 
\wheel{Save the whale} 
%%%% fin exemple %%%%
Il existe aussi le programme docstrip disponible sur ftp://ftp.fdn.org/pub/CTAN/macros/latex/unpacked/docstrip.tex.

Pour le cas où vous auriez un fichier dvi en landscape, vous pouvez obtenir un fichier postscript propre en utilisant la commande:

Exemple :
 
dvips -t a4 -t landscape -o tmp.ps toto.dvi 
%%%% fin exemple %%%%

6.9  Comment justifier verticalement un paragraphe ?

L'environnement vcenterpage ci-dessous permet de centrer verticalement un texte sur une page seule.

%%%% debut macro %%%% 
\newenvironment{vcenterpage} 
{\newpage\vspace*{\fill}} 
{\vspace*{\fill}\par\pagebreak} 
%%%% fin macro %%%%
Exemple :
 
\begin{vcenterpage} 
   Texte qui apparaît au milieu de la page. 
\end{vcenterpage} 
%%%% fin exemple %%%%

6.10  Comment modifier la commande \caption ?

Le package 'hangcaption', disponible sur ftp://ftp.fdn.org/pub/CTAN/macros/latex209/contrib/misc/, dans lequel la commande \isucaption remplace la commande \caption, permet de modifier la mise en page de cette dernière. En particulier, il permet de définir \captionwidth.

Il existe également les packages 'caption' et 'caption2' disponibles sur ftp://ftp.fdn.org/pub/CTAN/macros/latex/contrib/supported/caption/ qui permettent de modifier la commande \caption classique (largeur, style, fonte, ...).

Exemples :
 
\usepackage[small,hang]{caption2} 
\renewcommand{\captionfont}{\it \small} 
\renewcommand{\captionlabelfont}{\it \bf \small} 
\renewcommand{\captionlabeldelim}{ :} 
%%%% fin exemple %%%%
Remarque :
il est recommandé d'utiliser 'caption2' plutôt que 'caption'.
Le package 'topcapt' disponible sur ftp://ftp.fdn.org/pub/CTAN/macros/latex/contrib/misc/ définit la commande \topcaption qui permet de placer le texte de la commande caption au dessus du flottant auquel elle est rattachée.

Le package 'french' de B. Gaulle propose la commande \unnumberedcaptions qui permet de supprimer la numérotation des flottants.

Pour avoir une présentation du type :

Figure 1. Voici le texte de la légende d'une figure. Mais 
          lorsque la légende est longue, elle se présente 
          comme ceci.
au lieu de :

Figure 1. Voici le texte de la légende d'une figure. Mais 
lorsque la légende est longue, elle se présente comme 
ceci.
on peut utiliser

\caption[texte]{\protect\parbox[t]{wd}{texte}}
où wd est la largeur de la boîte (cette largeur peut être adaptée à la taille de la figure automatiquement si la figure est elle-même dans une boîte dont on récupère la largeur). \protect est absolument nécessaire pour éviter une erreur. Le texte au début [texte] est celui qui apparaîtra dans la table des figures (il peut éventuellement être plus court).

Plus simplement, le package 'caption2' permet de faire la même chose avec l'option hang.

La variable \figurename permet de redéfinir le nom du titre de la figure. De même pour \tablename et les tableaux.

Exemple :
 
\def\figurename{Croquis} 
%%%% fin exemple %%%%

6.11  Comment modifier un changement de page ?

Exemples :
 
\enlargethispage{1cm}   % ajoute un cm 
\enlargethispage{-2\baselineskip} % supprime deux lignes dans la 
                                  % page 
%%%% fin exemple %%%%
Remarque :
\enlargethispage*{lgr} donne une autorité plus grande à la commande \enlargethispage en l'obligeant à agir sur les espacements élastiques verticaux contenus dans la page courante.
J.P. Drucbert donne ci-dessous un petit papier décrivant un style block.sty bricolé à partir de macro de diverses origines (dont D. Arseneau, assez connu). Si vous passez LATEX sur ce papier, block.sty sera créé. Les macros les plus intéressantes sont \need{dim} et \lneed{N}, qui provoquent un saut de page s'il reste moins que la dimension dim (ou N fois \baselinskip) verticalement sur la page. Ce n'est pas parfait, mais cela m'a rendu des services.

%%%% debut macro %%%% 
 \begin{filecontents}{block.sty} 
 \ifx\endBlock\undefined 
 \def\block{\begingroup% 
 \def\endblock{\egroup\endgroup}% 
 \vbox\bgroup}% 
 \long\def\Block{\begingroup% 
 \def\endBlock{\unskip\egroup\endgroup}% 
 \pagebreak[2]\vspace*{\parskip}\vbox\bgroup% 
 \par\noindent\ignorespaces} 
 \long\def\IBlock{\begingroup% 
 \def\endIBlock{\unskip\egroup\endgroup}% 
 \pagebreak[2]\vspace*{\parskip}\vbox\bgroup\par\ignorespaces} 
 \def\need#1{\ifhmode\unskip\par\fi \penalty-100 \begingroup 
 % preserve \dimen@, \dimen@i 
    \ifdim\prevdepth>\maxdepth \dimen@i\maxdepth 
       \else \dimen@i\prevdepth\fi 
    \kern-\dimen@i 
    \dimen@\pagegoal \advance\dimen@-\pagetotal % space left 
    \ifdim #1>\dimen@ 
         \vfill\eject\typeout{WARNING- EJECT BY NEED} 
    \fi 
    \kern\dimen@i 
    \endgroup} 
 \def\lneed#1{\need{#1\baselineskip}} 
 % \begin{block} ... \end{block} delimite un bloc qui restera, 
 %                               si possible, sur une seule page. 
 \long\def\TBlock{\begingroup% 
 \def\endTBlock{\unskip\egroup\endgroup}% 
 \pagebreak[2]\vspace*{\parskip}\vtop\bgroup% 
 \par\noindent\ignorespaces} 
 \else 
 \typeout{block.sty already loaded} 
 \fi 
 \endinput 
   
 \def\need#1{\par \penalty-100 \begingroup 
 % preserve \dimen@, \dimen@i 
    \ifdim\prevdepth>\maxdepth \dimen@i\maxdepth 
       \else \dimen@i\prevdepth\fi 
    \kern-\dimen@i 
    \dimen@\pagegoal \advance\dimen@-\pagetotal % space left 
    \ifdim #1>\dimen@ \vfil \eject \fi 
    \kern\dimen@i 
    \endgroup} 
 \end{filecontents} 
   
 \documentclass[12pt,a4paper]{article} 
 \def\bs{\texttt{\char'134}} 
 \parskip=12pt plus1pt minus0.5pt 
 \usepackage[english,francais]{babel} 
 \usepackage[T1]{fontenc} 
 \usepackage[isolatin]{inputenc} 
 \begin{document} 
 \clearpage 
 \section{Paquetage \texttt{block}}\label{BLOCK+} 
 Ce paquetage offre divers outils permettant d'empêcher qu'une 
 rupture de page se produise à un certain endroit. 
   
 \subsection{Blocs Protégés} 
   
 Vous pouvez protéger une zone contre la rupture de page. Il 
 suffit pour cela de la placer dans un environnement 
 \texttt{Block} ou dans un environnement \texttt{IBlock}. Dans le 
 premier cas (\texttt{Block}), le premier paragraphe se trouvant 
 dans la zone ne sera pas indenté, mais le sera dans le second 
 cas (\texttt{IBlock}). Ceci est utile en particulier pour 
 éviter de séparer un texte et un exemple qui l'accompagne. Les blocs ainsi 
 protégés doivent, bien entendu, rester assez petits. La syntaxe 
 est (vous pouvez remplacer \texttt{Block} par \texttt{IBlock}): 
 \begin{quote}\tt 
 %\begin{tabular}{l} 
 \bs begin\{Block\}\\ 
 \qquad\ldots\\ 
 zone protégée\\ 
 \qquad\ldots\\ 
 \bs end\{Block\
 %\end{tabular} 
 \end{quote} 
   
 Cette méthode, très simple, a l'inconvénient de ne pas pouvoir 
 s'appliquer lorsque la zone à protéger doit contenir une 
 commande de sectionnement (c'est-à-dire du même type que 
 \verb|\section|), une note en bas de page, une note marginale ou 
 un corps mobile (figure ou table).  Dans ce cas, il faudra 
 utiliser une des commandes du paragraphe suivant. 
   
 \subsection{Réservations Verticales} 
 Vous pouvez aussi demander de changer de page (ou de colonne, si votre 
 document est sur deux colonnes) s'il ne reste pas verticalement assez 
 de place sur la page. Deux commandes de réservation verticale sont 
 disponibles: \verb|\need{|{\em dimension\/}\verb|}|, dont le paramètre est une 
 longueur, et \verb|\lneed{|{\em nombre\/}\verb|}|, dont le paramètre est 
 le nombre de lignes équivalant à l'espace vertical demandé (avec cette forme 
 le paramètre est plus facile à estimer). 
   
 \end{document} 
 %%%% fin macro %%%%
Pour éviter les coupures de page (ou les coupures de colonnes avec multicol.sty) à l'intérieur des \item dans une description (et d'ailleurs dans toute liste), solution simple sortie du TEXbook :

\begin{description}\interlinepenalty 10000

6.12  Comment obtenir des colonnes parallèles ?

Le package 'parallel' disponible sur ftp://ftp.fdn.org/pub/CTAN/macros/latex/contrib/supported/parallel/ permet d'obtenir deux colonnes dont l'une peut contenir la traduction de l'autre.

6.13  Comment définir l'espace de début de paragraphe ?

Il faut valoriser la variable \parindent.

Exemple :
 
\setlength{\parindent}{1cm} 
%%%% fin exemple %%%%
Pour agir localement, on peut utiliser \hspace*{lgr}.

Le package 'indentfirst' permet de forcer LATEX à indenter le premier paragraphe après une nouvelle section (indentation normalement non utilisée en typographie française).

6.14  Comment supprimer la date sur une page de titre ?

Il faut ajouter la commande

\date{} 
dans le préambule du document.

6.15  Comment mettre en valeur la première lettre d'un chapitre ?

Il faut utiliser le package 'dropcaps' de F. Lauwers. Il est disponible sur ftp://ftp.fdn.org/pub/CTAN/macros/latex209/contrib/dropcaps/. Ce package est utilisable avec LATEX2.09 et .

Les packages 'initial' et 'initials' pour sont disponibles sur CTAN. Le second que l'on peut trouver dans ftp://ftp.fdn.org/pub/CTAN/fonts/gothic/yinit/ fait appel à des fontes yinit particulières.

Il existe également un package 'drop' pour LATEX2.09, mais compatible , disponible sur ftp://ftp.fdn.org/pub/CTAN/macros/latex209/contrib/misc/.

Exemple :
 
\documentclass[12pt,a4paper]{article} 
\usepackage{drop} 
\font\largefont=yinitas % fontes yinit 
begin{document} 
\drop{D}OES THERE EXIST a field with 4 elements? 
\end{document} 
%%%% fin exemple %%%%
'bigdrop' accessible sur CTAN dans/CTAN/digests/ttn est une macro TEX compatible LATEX.

Le package 'bigstart' pour LATEX2.09 et permet également de faire cela.

Il existe également le package 'picinpar' pour LATEX2.09 disponible sur ftp://ftp.fdn.org/pub/CTAN/macros/latex209/contrib/picinpar/ et ftp://ftp.fdn.org/pub/CTAN/systems/msdos/4alltex/disk04/.

On peut également définir sa propre macro :

%%%% debut macro %%%% 
\font\capfont=cmbx12 at 24.87 pt % or yinit, or...? 
\newbox\capbox \newcount\capl \def\a{A} 
\def\docappar{\medbreak\noindent\setbox\capbox\hbox{% 
\capfont\a\hskip0.15em}\hangindent=\wd\capbox% 
\capl=\ht\capbox\divide\capl by\baselineskip\advance\capl by1% 
\hangafter=-\capl% 
\hbox{\vbox to8pt{\hbox to0pt{\hss\box\capbox}\vss}}} 
\def\cappar{\afterassignment\docappar\noexpand\let\a } 
%%%% fin macro %%%%
Exemple :
 
\cappar Il était une fois un petit chaperon rouge qui avait une 
grand-mère qui habitait de l'autre côté de la foret. Un jour, 
alors que sa grand-mère était malade, le petit chaperon rouge 
décida de lui rendre visiteldots 
%%%% fin exemple %%%%
Un petit dernier, le package 'dropping' qui étend le package 'dropcaps' est disponible sur ftp://ftp.fdn.org/pub/CTAN/macros/latex/contrib/other/dropping/.

Exemple :
 
\dropping{3}{\itshape{} Voici} un exemple de ce que permet de 
faire le magnifique package dropping de M. Dahlgren. La commande 
\texttt{\\dropping} peut prendre en argument un mot comme c'est le cas ici 
ou une simple lettre. 
%%%% fin exemple %%%%
Le package 'french' de B. Gaulle propose également la commande \lettrine.

Exemple :
 
\lettrine{UN jour,} mon princeldots 
%%%% fin exemple %%%%
Le paquetage 'lettrine' de Daniel Flipo propose lui aussi la commande \lettrine, mais avec une diversité d'options inconnues à french (nombre de lignes, forme, etc.) Tiré de la doc:

Exemple :
 
\lettrine[lines=4, slope=-0.5em, lhang=0.5, nindent=0pt]{V}{oici} 
%%%% fin exemple %%%%

6.16  Comment préserver les tabulations en mode verbatim ?

Le package 'moreverb' est disponible sur ftp://ftp.fdn.org/pub/CTAN/macros/latex/contrib/other/misc/. Il propose notamment un environnement verbatimtab qui permet de conserver des tabulations.

Exemple :
 
\begin{verbatimtab} 
int pattern(char *p, int n, int m) 

  int orig = current_position(); 
  int new_pos; 
 

  goto_line(n);

 

  if(p && forward_search(p) && (current_line()<m)){ 
    new_pos = current_position(); 
    goto_char(orig); 
    return(new_pos); 
  } 
  return(-1); 

\end{verbatimtab} 
%%%% fin exemple %%%%

%%%% debut macro %%%% 
\makeatletter 
{\catcode`\I=\active 
\gdef\verbatim{\catcode`\I=\active\defI{\hspace*{4em}}% 
\@verbatim \frenchspacing\@vobeyspaces \@xverbatim}} 
\makeatother 
%%%% fin macro %%%%

6.17  Comment modifier les entêtes de chapitre ?

Il faut modifier la macro makechapterhead.

Ajouter, par exemple, dans le préambule :

%%%% debut macro %%%% 
\makeatletter 
\def\@makechapterhead#1{% 
  \vspace*{50\p@}% 
  {\parindent \z@ \raggedright \normalfont 
    \interlinepenalty\@M 
    \ifnum \c@secnumdepth >\m@ne 
        \Huge\bfseries \thechapter\quad 
    \fi 
    \Huge \bfseries #1\par\nobreak 
    \vskip 40\p@ 
  }}

 

\def\@makeschapterhead#1{% 
  \vspace*{50\p@}% 
  {\parindent \z@ \raggedright 
    \normalfont 
    \interlinepenalty\@M 
    \Huge \bfseries  #1\par\nobreak 
    \vskip 40\p@ 
  }} 
\makeatother 
%%%% fin macro %%%%

6.18  Comment définir des tabulations ?

Il faut utiliser l'environnement tabbing qui permet de placer des marques d'alignement dans un texte.

Exemple :
 
\begin{tabbing} 
   Voici \= des \= marques \= de tabulation \\ 
   \> la je m'aligne sur la première \\ 
   \\\> la sur la troisième \\ 
   \hspace{3cm} \\hspace{2cm} \\kill 
   un \> autre \> exemple. 
\end{tabbing} 
%%%% fin exemple %%%%
ce qui donne :

 

Voici ¯ des ¯ marques ¯ de tabulation
 la je m'aligne sur la première
   la sur la troisième
                        ¯                 ¯ un  autre  exemple.

6.19  Comment obtenir des lettres accentuées dans tabbing ?

%%%% debut macro %%%% 
 \ProvidesPackage{Tabbing}[1996/01/16] 
 \NeedsTeXFormat{LaTeX2e}[1995/12/01] 
 \gdef\Tabbing{\lineskip \z@skip 
 %     \let\>\@rtab 
 %     \let\<\@ltab 
 %     \let\=\@settab 
 %     \let\+\@tabplus 
 %     \let\-\@tabminus 
 %     \let\`\@tabrj 
 %     \let\'\@tablab 
 \def\TAB##1{\ifx ##1>\@rtab\else 
             \ifx ##1<\@ltab\else 
             \ifx ##1=\@settab\else 
             \ifx ##1+\@tabplus\else 
             \ifx ##1-\@tabminus\else 
             \ifx ##1`\@tabrj\else 
             \ifx ##1'\@tablab\else 
                          \PackageError{Tabbing}% 
                          {Bad argument ##1 for Tabbing 
                          specification} \fi\fi\fi\fi\fi\fi\fi} 
      \let\\=\@tabcr 
      \global\@hightab\@firsttab 
      \global\@nxttabmar\@firsttab 
      \dimen\@firsttab\@totalleftmargin 
      \global\@tabpush\z@ \global\@rjfieldfalse 
      \trivlist \item\relax 
      \if@minipage\else\vskip\parskip\fi 
      \setbox\@tabfbox\hbox{\rlap{\indent\hskip\@totalleftmargin 
        \the\everypar}}\def\@itemfudge{\box\@tabfbox}% 
      \@startline\ignorespaces} 
 \gdef\endTabbing{% 
   \@stopline\ifnum\@tabpush >\z@ \@badpoptabs \fi\endtrivlist} 
 \endinput 
 %%%% fin macro %%%%

6.20  Comment encadrer du texte ?

\fbox{ 
\begin{minipage}{0.7\textwidth} 
   Texte... 
\end{minipage} 
}
 


Texte...


Exemple :
 
\newsavebox{\fmbox} 
\newenvironment{fmpage}[1] 
     {\begin{lrbox}{\fmbox}\begin{minipage}{#1}} 
     {\end{minipage}\end{lrbox}\fbox{\usebox{\fmbox}}}

 

Utilisation :
 
\begin{fmpage}{3cm} 
   Texte à encadrer dans une boîte ne dépassant pas 3 
   centimètres de large. 
\end{fmpage} 
%%%% fin exemple %%%%
Il existe également le package 'fancybox', disponible sur ftp://ftp.fdn.org/pub/CTAN/macros/latex/contrib/others/seminar/inputs/, qui définit des fonctions telles que \shadowbox, \doublebox, \ovalbox,...

Exemple :
 
\shadowbox{Texte ombré.} 
\doublebox{Texte doublement encadré.} 
\ovalbox{Texte dans un cadre aux coins arrondis.} 
%%%% fin exemple %%%%
ce qui donne :

 

%%%% debut macro %%%% 
 % eclbkbox.sty by Hideki Isozaki, 1992 
 % Date: May  28, 1993 
   
 \newbox\bk@bxb 
 \newbox\bk@bxa 
 \newif\if@bkcont 
 \newif\ifbkcount 
 \newcount\bk@lcnt 
   
 \def\breakboxskip{2pt} 
 \def\breakboxparindent{1.8em} 
   
 \def\breakbox{\vskip\breakboxskip\relax 
 \setbox\bk@bxb\vbox\bgroup 
 \advance\linewidth -2\fboxrule 
 \advance\linewidth -2\fboxsep 
 \hsize\linewidth\@parboxrestore 
 \parindent\breakboxparindent\relax} 
   
 % \@tempdimb: amount of vertical skip 
 % between the first line (\bk@bxa) and the rest (\bk@bxb) 
 \def\bk@split{% 
 \@tempdimb\ht\bk@bxb % height of original box 
 \advance\@tempdimb\dp\bk@bxb 
 \setbox\bk@bxa\vsplit\bk@bxb to\z@ % split it 
 \setbox\bk@bxa\vbox{\unvbox\bk@bxa}% recover height & depth of \bk@bxa 
 \setbox\@tempboxa\vbox{\copy\bk@bxa\copy\bk@bxb}% naive concatenation 
 \advance\@tempdimb-\ht\@tempboxa 
 \advance\@tempdimb-\dp\@tempboxa}% gap between two boxes 
   
   
 % \@tempdima: height of the first line (\bk@bxa) + fboxsep 
 \def\bk@addfsepht{% 
      \setbox\bk@bxa\vbox{\vskip\fboxsep\box\bk@bxa}} 
   
 \def\bk@addskipht{% 
      \setbox\bk@bxa\vbox{\vskip\@tempdimb\box\bk@bxa}} 
   
 % \@tempdima: depth of the first line (\bk@bxa) + fboxsep 
 \def\bk@addfsepdp{% 
      \@tempdima\dp\bk@bxa 
      \advance\@tempdima\fboxsep 
      \dp\bk@bxa\@tempdima} 
   
 % \@tempdima: depth of the first line (\bk@bxa) + vertical skip 
 \def\bk@addskipdp{% 
      \@tempdima\dp\bk@bxa 
      \advance\@tempdima\@tempdimb 
      \dp\bk@bxa\@tempdima} 
   
 \def\bk@line{% 
     \hbox to \linewidth{\ifbkcount\smash{\llap{\the\bk@lcnt\ }}\fi 
     \vrule \@width\fboxrule\hskip\fboxsep 
     \box\bk@bxa\hfil 
     \hskip\fboxsep\vrule \@width\fboxrule}} 
   
 \def\endbreakbox{\egroup 
 \ifhmode\par\fi{\noindent\bk@lcnt\@ne 
 \@bkconttrue\baselineskip\z@\lineskiplimit\z@ 
 \lineskip\z@\vfuzz\maxdimen 
 \bk@split\bk@addfsepht\bk@addskipdp 
 \ifvoid\bk@bxb      % Only one line 
 \def\bk@fstln{\bk@addfsepdp 
 \vbox{\hrule\@height\fboxrule\bk@line\hrule\@height\fboxrule}}% 
 \else               % More than one line 
 \def\bk@fstln{\vbox{\hrule\@height\fboxrule\bk@line}\hfil 
 \advance\bk@lcnt\@ne 
 \loop 
  \bk@split\bk@addskipdp\leavevmode 
 \ifvoid\bk@bxb      % The last line 
  \@bkcontfalse\bk@addfsepdp 
  \vtop{\bk@line\hrule\@height\fboxrule}% 
 \else               % 2,...,(n-1) 
  \bk@line 
 \fi 
  \hfil\advance\bk@lcnt\@ne 
 \if@bkcont\repeat}% 
 \fi 
 \leavevmode\bk@fstln\par}\vskip\breakboxskip\relax} 
   
 \bkcountfalse 
 %%%% fin macro %%%%
L'encadrement du texte est obtenu par l'environnement breakbox.

On peut emboîter des environnements breakbox.

6.21  Comment gérer des URL WWW ?

Exemple :
 
\usepackage{url} 
\urlstyle{sf} 
... 
\url{http://hostname/username} 
%%%% fin exemple %%%%
\discretionary{Avant la coupure}{après}{s'il n'y en a pas}
Remarque :
\- est défini comme \discretionary {-}{}{}

6.22  Comment mettre en page des exercices dont les solutions sont reportées dans un autre paragraphe ?

L'objectif ici est de pouvoir saisir dans le fichier source les textes des exercices suivis de leurs solutions, alors qu'au niveau de la mise en page du document, les solutions apparaîssent groupées dans un autre paragraphe/chapitre.

Le package 'answers', de M. Piff, disponible sur ftp://ftp.fdn.org/pub/CTAN/macros/latex/contrib/supported/answers/ permet également de réaliser ce genre d'exercice. Il permet entre autres :

Exemple :
 
%% 
 %% This is file `ansexam2.tex', 
 %% generated with the docstrip utility. 
 %% 
 %% The original source files were: 
 %% 
 %% answers.dtx  (with options: `ex2') 
 %% 
 \documentclass[12pt,a4paper]{article} 
 \usepackage{answers}%\usepackage[nosolutionfiles]{answers} 
 % def d'un environnement Exercise numerote 
 \newtheorem{Exc}{Exercise} 
 \newenvironment{Ex}{\begin{Exc}\normalfont}{\end{Exc}} 
 % Trois types de solutions sont proposes 
 \Newassociation{solution}{Soln}{test} 
 \Newassociation{hint}{Hint}{test} 
 \Newassociation{Solution}{sSol}{testtwo} 
 \newcommand{\prehint}{Hint]} 
 \newcommand{\presolution}{Solution]} 
 \newcommand{\preSolution}{Homework]} 
 % test 
 \newcommand{\Opentesthook}[2]% 
    {\Writetofile{#1}{\protect\section{#1: #2}}} 
 % introduction de la solution 
 \renewcommand{\Solnlabel}[1]{\emph{Solution #1}} 
 \renewcommand{\Hintlabel}[1]{\emph{Hint #1}} 
 \renewcommand{\sSollabel}[1]{\emph{Solution to #1}} 
   
 \begin{document} 
 % gestion des fichiers contenant les solutions 
    \Opensolutionfile{test}[ans2]{Solutions} 
    \Writetofile{test}{\protect\subsection{Some Solutions}} 
    \Opensolutionfile{testtwo}[ans2x] 
    \Writetofile{testtwo}{% 
       \protect\subsection{Extra Solutions}} 
   
       % Exercices 
    \section{Exercises} 
    \begin{Ex} 
       An exercise with a solution. 
       \begin{solution} 
          This is a solution. 
          \relax{} 
       \end{solution} 
    \end{Ex} 
    \begin{Ex} 
       An exercise with a hint and a secret solution. 
       \begin{hint} 
          This is a hint. 
       \end{hint} 
       \begin{Solution} 
          This is a secret solution. 
       \end{Solution} 
    \end{Ex} 
    \begin{Ex} 
       An exercise with a hint. 
       \begin{hint} 
          This is a hint. 
       \end{hint} 
    \end{Ex} 
    % gestion des fichiers contenant les solutions 
    \Closesolutionfile{test} 
    \Readsolutionfile{test} 
    % \clearpage 
    \hrule 
    \Closesolutionfile{testtwo} 
    \Readsolutionfile{testtwo} 
 \end{document} 
 %% 
 %% End of file `ansexam2.tex'. 
   
 %%%% fin exemple %%%%

6.23  Comment positionner un objet dans une page ?

Pour pouvoir positionner un objet aux coordonnées x,y par rapport au coin supérieur gauche d'une page, il suffit d'utiliser le package 'atxy' disponible par ftp sur ftp://ftp.univ-orleans.fr/pub/tex/PC/AsTeX/Paq_Base/ dans le fichier l209misc.zip.

Exemple :
 
\atxy(3cm,2.5cm) {toto adresse toto} 
\atxy(3cm,4cm) {date} 
%%%% fin exemple %%%%
Remarque :
petit défaut, si le document ne contient que des commandes \atxy il n'y a pas de dvi généré. Il faut donc ajouter n'importe quoi au début du document, par exemple, pour que ça marche.

6.24  Comment lier le placement des flottants aux sections ?

Pour lier la position des éléments flottants aux limites de sections, D. Arseneau a développé le package 'placeins' disponible sur ftp://ftp.fdn.org/pub/CTAN/macros/latex/contrib/other/misc/. Ce package définit la commande \FloatBarrier qui force le placement des flottants avant son appel.

6.25  Comment griser le fond (background) d'un paragraphe ?

Exemple :
 
\colorbox[gray]{0.5}{some words} 
%%%% fin exemple %%%%
ce qui donne :

 

some words

6.26  Comment modifier l'espace inter-colonnes ?

Il faut modifier la variable \columnsep.

Exemple :
 
\addtolength{\columnsep}{5mm} 
%%%% fin exemple %%%%

6.27  Comment modifier les environnements de liste ?

L'environnement list permet de définir son propre style de liste. Sa syntaxe est la suivante :

\begin{list}{label}{mep}\end{list}
Exemple :
 
\newenvironment{maliste}% 
\begin{list}% 
        {$\bullet$}% 
        {\setlength{\labelwidth}{30pt}% 
         \setlength{\leftmargin}{35pt}% 
         \setlength{\itemsep}{\parsep}}}% 
\end{list} }
Utilisation :
 
\begin{maliste} 
   \item premier élément 
   \item deuxième élément 
   \begin{maliste} 
      \item petit 1 
      \item petit 2 
   \end{maliste} 
\end{maliste} 
%%%% fin exemple %%%%
Le petit bout de code ci-dessous, de M. Boyer (mboyer@robot.ireq.ca), définit les commandes :

Pour l'utiliser, il suffit de le sauvegarder dans un fichier .sty et de l'inclure dans son document par une commande \usepackage.

%%%% debut macro %%%% 
 %% ---------------------------------------------------- 
 %% Copyright (c) 1993 Hydro-Quebec mboyer@robot.ireq.ca 
 %% ---------------------------------------------------- 
   
 %% Bring items closer together in list environments 
 % Prevent infinite loops 
 \let\orig@Itemize =\itemize 
 \let\orig@Enumerate =\enumerate 
 \let\orig@Description =\description 
 % Zero the vertical spacing parameters 
 \def\Nospacing{\itemsep=0pt\topsep=0pt\partopsep=0pt% 
 \parskip=0pt\parsep=0pt} 
 % Redefinition de art12.sty pour commencer a la marge de gauche 
 %\leftmargini 1.2em      % 2.5em 
   
 \def\noitemsep{ 
 % Redefine the environments in terms of the original values 
 \renewenvironment{itemize}{\orig@Itemize\Nospacing}{\endlist} 
 \renewenvironment{enumerate}{\orig@Enumerate\Nospacing}{\endlist} 
 \renewenvironment{description}{\orig@Description\Nospacing}% 
 {\endlist} 
 } 
   
 \def\doitemsep{ 
 % Redefine the environments to the original values 
 \renewenvironment{itemize}{\orig@Itemize}{\endlist} 
 \renewenvironment{enumerate}{\orig@Enumerate}{\endlist} 
 \renewenvironment{description}{\orig@Description}{\endlist} 
 } 
 %%%% fin macro %%%%
La macro ci-dessous de T. Murphy permet de remplacer les numéros de l'environnement enumerate par des caractères grecs :

%%%% debut macro %%%% 
\makeatletter 
\def\greek#1{\expandafter\@greek\csname c@#1\endcsname} 
\def\@greek#1{\ifcase#1\or$\alpha$\or$\beta$\fi}% as many as you 
% need 
\renewcommand{\theenumi}{\greek{enumi}} 
\makeatother 
%%%% fin macro %%%%
Les définitions suivantes :

\renewcommand{\labelitemi}{\textbullet} 
\renewcommand{\labelitemii}{---} 
\renewcommand{\labelitemiii}{votre-label-pour-le-niveau-iii} 
\renewcommand{\labelitemiv}{votre-label-pour-le-niveau-iv}
permettent de redéfinir les caractères utilisés par l'environnement itemize pour ces différents niveaux d'encapsulation.

Remarque :
avec certains packages comme french, il faut placer ces nouvelles définitions de commande après le \begin{document}.
De même, utilisez \descriptionlabel pour changer le style des étiquettes de l'environment description.

Exemple :
 
\renewcommand\descriptionlabel[1]{\hspace\labelsep\normalfont% 
\itshape #1:} 
produit des étiquettes en italique, avec deux points : 
\begin{description} 
   \item[Carte maîtresse] As 
   \item[Carte maîtresse à l'atout] Valet 
\end{description} 
%%%% fin exemple %%%%
Pour réduire globalement l'espace entre les items d'une liste on peut également utiliser le bout de code suivant de M. Wooding

%%%% debut macro %%%% 
\makeatletter 
\toks@\expandafter{\@listI} 
\edef\@listI{\the\toks@\setlength{\parsep}{1pt}} 
\makeatother 
%%%% fin macro %%%%
* Le package 'mdwlist', de M. Wooding, disponible sur ftp://ftp.fdn.org/pub/CTAN/macros/latex/contrib/supported/mdwtools/ permet de redéfinir certains paramètres de mise en page des listes qui ne sont pas faciles d'accès sous LATEX.

6.28  Comment souligner plusieurs lignes ?

Pour souligner un texte qui comprend des retours à la ligne, il faut utiliser le package 'ulem' disponible sur ftp://ftp.fdn.org/pub/CTAN/macros/latex/contrib/other/misc/. Ce package redéfinit en fait le mode emphasize. Les commandes \normalem et \ULforem permettent de passer du mode \emph classique au mode \emph souligné. En mode souligné, la commande devient paramétrable pour changer le style du souligné ou biffer des mots.

Exemple :
 
\documentclass{report} 
\usepackage{french} 
\usepackage[normalem]{ulem} 
\pagestyle{empty} 
\begin{document} 
 

\normalem 
Voici le mode "\emph{emphasize}" classique.

 

\ULforem 
Voici le mode "\emph{emphasize}" souligné. \emph{Il permet 
également de gérer les retours à la ligne tout en restant dans 
le style emphasize.}

 

Les autres possibilités sont les suivantes: 
\begin{itemize} 
   \item vagues: \uwave{texte} 
   \item barré: \sout{texte} 
   \item rayé: \xout{texte} 
\end{itemize}

 

\end{document} 
%%%% fin exemple %%%%

Remarque :
\underline ne permet pas de gérer les retours à la ligne du fait qu'il encapsule le texte passé en argument dans une boîte.

6.29  Comment réaliser des onglets ?

Pour insérer un carré noir, décalé vers le bas à chaque nouveau chapitre, le long de la marge des pages de droite d'un document, on peut utiliser le package 'fancyhdr' ou 'fancyheadings'. Ces packages sont disponibles sur ftp://ftp.fdn.org/pub/CTAN/macros/latex/contrib/supported/fancyhdr/ et ftp://ftp.fdn.org/pub/CTAN/macros/latex/contrib/other/.

Remarque :
On note toutefois qu'il est peu probable qu'une imprimante puisse accéder au ras de la marge. La solution consiste alors à définir un format de document plus petit et utiliser le massicot. Attention dans ce cas lors de la définition des marges.
On peut également utiliser le package 'onglet' défini ci-dessous par B. Bayart. Celui-ci nécessite le package 'everyshi' disponible sur ftp://ftp.fdn.org/pub/CTAN/macros/latex/contrib/supported/everyshi/.

%%%% debut macro %%%% 
 \ProvidesPackage{onglet}[1996/07/25 B. Bayart] 
 \RequirePackage{everyshi} 
   
 \newcounter{maxchapter} 
 \newcounter{tmpa} 
 \newlength{\basehauteur} 
 \setlength{\basehauteur}{1cm} 
 \newlength{\ajoutdroite} 
 \newlength{\htcclv} 
 \def\concatener{% 
   \setlength{\ajoutdroite}{\textheight} 
   \divide\ajoutdroite by \basehauteur 
   \setcounter{maxchapter}{\number\ajoutdroite} 
   \setcounter{tmpa}{\value{chapter}} 
   \addtocounter{tmpa}{-1} 
   \divide\value{tmpa} by\value{maxchapter} 
   \multiply\value{tmpa} by\value{maxchapter} 
   \advance\value{tmpa} by -\value{chapter} 
   \addtocounter{tmpa}{-1} 
   \setlength{\ajoutdroite}{\paperwidth} 
   \setlength{\htcclv}{\ht255} 
   \addtolength{\ajoutdroite}{-\wd255} 
   \addtolength{\ajoutdroite}{-1in} 
   \addtolength{\ajoutdroite}{-1.5cm} 
   \setbox255=\vbox{\hbox to \wd255{% 
     \box255%\relax 
     \rlap{\vbox to \htcclv{% 
       \vskip-\value{tmpa}\basehauteur 
       \hbox{% 
         \hskip\ajoutdroite\relax 
         \usebox{\laboite}% 
       }% 
       \vfill 
     }}% 
   \hfill}}% 
 } 
 \newsavebox{\laboite} 
 \def\faireboite{\sbox{\laboite}% 
 {\hbox to 1.5cm{\let\protect\relax 
 \huge\thechapter\hfill\vrule height 1em depth 0pt width 5mm}}} 
   
 \AtBeginDocument{\EveryShipout{\faireboite\concatener}} 
 \endinput 
 %%%% fin macro %%%%

6.30  Comment réaliser des QCM ?

Il existe le package 'exam' disponible sur ftp://ftp.fdn.org/pub/CTAN/macros/latex/contrib/supported/exams/. Ce package permet entre autres de paramétrer les questions de manière à ce que les propositions apparaîssent dans un ordre aléatoire.

Exemple :
 
Combien le cheval possède t'il de pattes ? 
\begin{choice}[\random] 
   \baditem{deux pattes} 
   \baditem{quatres pattes} 
   \baditem{zéro patte} 
\end{choice} 
%%%% fin exemple %%%%

6.31  Comment modifier l'orientation des flottants ?

Le package 'rotating' disponible sur ftp://ftp.fdn.org/pub/CTAN/macros/latex/contrib/supported/rotating/ offre deux environnements sidewaysfigure pour les figures et sidewaystable pour les tableaux. Les figures ou tableaux sont alors insérés sur une page séparée.

Exemple :
 
\begin{sidewaystable} 
   \begin{tabular}{|c|c|} 
      \hline 
      Un & Deux \\ 
      Trois & Quatre \\ 
      \hline 
   \end{tabular} 
\end{sidewaystable} 
%%%% fin exemple %%%%
On peut également utiliser l'environnement sideways du package 'rotating'. De la même manière cet environnement génère une page séparée.

Exemple :
 
\begin{figure} 
  \begin{sideways} 
     \includegraphics{foobar.ps} 
  \end{sideways} 
\end{figure} 
%%%% fin exemple %%%%
Remarque :
il est fréquent que les visualiseurs de fichiers dvi ne sachent pas gérer les changements d'orientation à l'intérieur d'un document. Il faut pour voir le résultat définitif utiliser un visualiseur de postscript.

6.32  Comment faire référence plusieurs fois à la même note de bas de page ?

Il faut utiliser les commandes \footnotemark[] et \footnotetext[]{}. \footnotemark permet de gérer le compteur de notes et \footnotetext permet d'insérer le texte correspondant en bas de page.

Exemple :
 
bla blabla\footnotemark[1] bla bla bla blablabla\footnotemark[2] 
bla bla bla bla blabla\footnotemark[1] bla.

 

\footnotetext[1]{double bla} 
\footnotetext[2]{triple bla} 
%%%% fin exemple %%%%

ce qui donne :

 

bla blabla bla bla bla blablabla bla bla bla bla blabla bla.

 

Avec french, on peut utiliser la commande \refmark.

Exemple :
 
le vrai appel\footnote{C'est une note en bas de 
page\label{manote}} et le deuxième appel\refmark{manote} 
%%%% fin exemple %%%%
Remarque :
Si vous n'utilisez pas french, cette macro \refmark est décrite dans le cahier Gutenberg numéro 15, avril 1993, page 52. Ce cahier est accessible à l'URL : http://www.gutenberg.eu.org/publications/.

6.33  Comment éviter les orphelins en début ou fin de page ?

Il suffit d'ajouter les lignes :

%%%% debut macro %%%% 
\widowpenalty=10000 
\clubpenalty=10000 
\raggedbottom 
%%%% fin macro %%%%
dans le préambule du document.

Pour agir localement, on peut également utiliser la commande \enlargethispage (cf. question ).

6.34  Comment définir de nouveaux flottants ?

Le package 'float' de G. Williams, disponible sur ftp://ftp.fdn.org/pub/CTAN/macros/latex/contrib/supported/float/, permet de définir de nouveaux objets flottants. Il définit notamment des environnements permettant d'encadrer des objets ou de les séparer du reste du texte par des lignes horizontales.

Exemple :
 
\documentclass[11pt]{report} 
 \usepackage{float} 
 \usepackage{french} 
   
 \floatstyle{ruled} % pour que mes flottants soient séparés du 
                    % texte par des lignes. 
 \newfloat{important}{htbp}{loi}[section] 
         % important est le nom de mon nouvel environnement 
         % htbp sont les options de placement de mon flottant 
         % loi est l'extension du fichier qui sera utilise pour 
         % construire la liste de mes flottants 
         % section est le niveau duquel dependra la numerotation 
         % de mes flottants 
 \floatname{important}{Important} % titre du caption 
   
 \begin{document} 
   
 \listof{important}{Liste des textes importants.} % titre de la 
         % liste de mes flottants. 
   
 \chapter{Règles de bases.} 
   
 \section{Hommes célèbres.} 
   
    Parmi les citations des hommes célèbres dans le domaine que 
    nous étudions actuellement, il faut retenir celle de M. 
    Maxime rappelée dans le cadreref{imp-max}. 
   
    \begin{important} 
       M. Maxime a dit un jour: 
       \begin{quote} 
          Chacun doit se faire ses propres raisons. 
       \end{quote} 
       \caption{Adage de M. Maxime.} 
    \end{important} 
   
    Notez queldots 
 \end{document} 
 %%%% fin exemple %%%%

6.35  Comment utiliser la commande \caption hors d'un environnement flottant ?

Pour pouvoir attacher un titre de style table à un environnement non flottant (i.e. autre que figure ou table), il faut définir :

%%%% debut macro %%%% 
\makeatletter 
\def\@captype{table} 
\makeatother 
%%%% fin macro %%%%
Exemple :
 
Texte sur les couleurs.

 

\begin{itemize} 
   \item rouge 
   \item vert 
   \item bleu 
\end{itemize} 
\caption{Liste des couleurs primaires.}

 

Texte. 
%%%% fin exemple %%%%

La macro suivante propose une autre possibilité pour obtenir un titre de style table :

%%%% debut macro %%%% 
\makeatletter 
\def\captionof#1#2{{\def\@captype{#1}#2}} 
\makeatother 
%%%% fin macro %%%%
Exemple :
 
\begin{document} 
un texte

 

\captionof{table}{\caption{un titre ici}\label{foo}}

 

un autre texte 
\end{document} 
%%%% fin exemple %%%%

Le package 'capt-of' disponible sur ftp://ftp.fdn.org/pub/CTAN/macros/latex/contrib/other/misc/ définit la commande \captionof.

6.36  Comment encadrer un objet flottant ?

Il faut utiliser le package 'float' qui définit le style boxed. Ce package est disponible sur ftp://ftp.fdn.org/pub/CTAN/macros/latex/contrib/supported/float/. Voir la question pour un exemple plus complet.

Exemple :
 
Dans le préambule :

\floatstyle{ruled} 
\newfloat{nom-flottant}{placement}{ext-fic}[numerotation]
dans le corps du document :

\begin{nom-flottant}

 

\end{nom-flottant} 
%%%% fin exemple %%%%

Si le titre accompagnant le flottant peut être à l'extérieur du cadre désiré, on peut alors utiliser \fbox dans la déclaration du flottant.

Exemple :
 
\begin{table} 
   \begin{center} 
      \fbox{ 
      \begin{tabular}{cll} 
         &1 case & 2 cases \\ 
         &3 cases  & 4 cases \\\hline 
         Total: & 4 cases & 6 cases \\ 
      \end{tabular} 
      } 
      \end{center} 
   \caption{Titre extérieur.} 
\end{table} 
%%%% fin exemple %%%%
M. Loreti propose également le package 'bigbox' ci-dessous :

%%%% debut macro %%%% 
 % I like to put boxes around all of my figures and algorithms in 
 % reports, and finally came up with my 'bigbox' environment.  It 
 % makes a box as wide as the current text and sets the stuff 
 % inside with a narrower width. 
 % 
 % USAGE: 
 % 
 % \begin{figure}        (or {table}, or {center}, or ...) 
 %   \begin{bigbox} 
 %     \begin{tabbing} 
 %       ... 
 %     \end{tabbing} 
 %   \end{bigbox} 
 %   \caption{XXX} 
 % \end{figure} 
 % 
 %%% BIGBOX - environment 
 %%% 
 %%% TOM SHEFFLER 
 %%% Carnegie Mellon, Aug 1990 
 %%% 
 %%% Make an environment for boxing a figure and setting it in 
 %%% a narrower width. 
 \newdimen\boxfigwidth % width of figure box 
   
 \def\bigbox{\begingroup 
   % Figure out how wide to set the box in 
   \boxfigwidth=\hsize 
   \advance\boxfigwidth by -2\fboxrule 
   \advance\boxfigwidth by -2\fboxsep 
   \setbox4=\vbox\bgroup\hsize\boxfigwidth 
   % Make an invisible hrule so that the box is exactly this wide 
   \hrule height0pt width\boxfigwidth\smallskip% 
 % Some environments like TABBING and other LIST environments 
 % use this measure of line size - \LINEWIDTH=\HSIZE-\LEFTMARGIN-\RIGHTMARGIN? 
   \linewidth=\boxfigwidth 
 } 
 \def\endbigbox{\smallskip\egroup\fbox{\box4}\endgroup} 
 %%%% fin macro %%%%
Exemple :
 
\documentclass[11pt]{report} 
\usepackage{graphics} 
\usepackage{bigbox} 
\usepackage{french} 
\pagestyle{empty} 
\begin{document}

 

\begin{figure} 
   \begin{bigbox} 
      \includegraphics{../foobar.ps} 
   \end{bigbox} 
   \caption{Dessin.} 
\end{figure}

 

\end{document} 
%%%% fin exemple %%%%

6.37  Comment changer l'orientation d'un caption ?

La commande \rotcaption fournie avec le package 'rotating' permet de changer l'orientation d'un caption.

Exemple :
 
\begin{figure} 
   \centering 
   \begin{minipage}[c]{1in} 
      \includegraphics[angle=90,width=\linewidth]{poussin.eps} 
   \end{minipage} 
   \begin{minipage} 
      \rotcaption{Ma jolie figure coco} 
      \label{coco} 
   \end{minipage} 
\end{figure} 
%%%% fin exemple %%%%

6.38  Comment mettre en page un programme ?

Les packages 'program' (disponible sur ftp://ftp.fdn.org/pub/CTAN/macros/latex/contrib/supported/program/) et 'programs' permettent de mettre en reliefs des mots clés, d'utiliser des mathématiques dans des algorithmes, etc.

6.39  Comment inclure le source d'un programme ?

Les packages 'lgrind' et 'listings' répondent à la question (disponibles sur ftp://ftp.fdn.org/pub/CTAN/support/lgrind/ et ftp://ftp.fdn.org/pub/CTAN/macros/latex/contrib/supported/listings/). Ils reconnaissent tous les 2 à peu près 40 langages différents.

Le premier est composé d'un exécutable, 'lgrind' qui permet de transformer le source en question, par exemple, monfichier.c, en monfichier.tex, que l'on inclut directement dans son fichier LATEX, à l'aide d'une commande appropriée. L'inconvénient, est qu'évidemment, il y a un fichier .tex qui est généré en plus.

Notes :
 
Le deuxième ne nécessite pas de recompilation. On inclut directement le source, avec une commande adéquate. Avec ce package, on peut même définir son propre langage, le faire hériter d'un autre langage, avec des mots-clés spécifiques, etc.

À noter que, moyennant quelques paramètres à ajouter, il est tout à fait possible de faire en sorte que ces packages reconnaissent les fontes 8 bits. On peut lui faire comprendre par exemple, que lorsqu'il rencontre le caractère « ¹ », il le transforme en $\mathonesuperior$.

6.40  Comment obtenir un espace insécable ?

Le caractère est interprété par LATEX et permet d'obtenir un espace insécable.

Exemple :
 
Comme le montre l'exemple suivant: $a + 2 = 0$. 
%%%% fin exemple %%%%

6.41  Comment insérer une page blanche ?

Pour forcer LATEX à laisser une page blanche dans un document, il faut utiliser successivement les trois commandes :

Le principe est qu'il y ait quelque chose d'invisible sur la page blanche pour que LATEX la prenne en compte.

6.42  Comment supprimer l'indentation des paragraphes ?

\noindent au début du texte permet de ne pas indenter un paragraphe.

Pour systématiquement supprimer l'indentation du premier paragraphe d'une section par exemple, il faut redéfinir la commande \section. Il faut que le 4ème paramètre de \@startsection soit une distance négative, pour qu'il n'y ait pas de retrait au premier paragraphe du texte qui suit :

%%%% debut macro %%%% 
\makeatletter 
\renewcommand\section{\@startsection {section}{1}{\z@}% 
                           {-3.5ex \@plus -1ex \@minus -.2ex}% 
%%%%%%                       (4eme parametre) 
                           {2.3ex \@plus.2ex}% 
                           {\normalfont\Large\bfseries}} 
\makeatother 
%%%% fin macro %%%%

6.43  Comment modifier l'espacement entre caractères ?

Le package 'letterspace' permet de modifier l'espacement entre mots ou caractères.

Le package 'trackin' disponible sur ftp://ftp.fdn.org/pub/CTAN/macros/latex/contrib/other/tracking/ permet de jouer sur les espacements dans les mots ou les phrases pour les ajuster dans une longueur spécifiée.

6.44  Comment supprimer certaines coupures de mots ?

Pour agir sur un mot particulier, il faut utiliser la commande \hyphenation.

Exemple :
 
Visualisation des coupures :

\showhyphens{mousehole AlGaAs GaAs GaInP AlInP} 
Underfull \hbox (badness 10000) detected at line 0 
[] \OT1/cmr/m/n/10 mouse-hole Al-GaAs GaAs GaInP Al-InP
Interdiction de certaines coupures :

\hyphenation{AlGaAs GaAs GaInP AlInP} 
\showhyphens{mousehole AlGaAs GaAs GaInP AlInP} 
Underfull \hbox (badness 10000) detected at line 0 
[] \OT1/cmr/m/n/10 mouse-hole AlGaAs GaAs GaInP AlInP 
%%%% fin exemple %%%%
Moins propre mais tout aussi efficace, on peut inclure le mot à ne pas couper dans une hbox.

Exemple :
 
Même quand il est en bout de ligne ce mot très long n'est pas 
coupé CeMotTrèsLongNestPasCoupé.

 

Même quand il est en bout de ligne ce mot très long n'est pas 
coupé \hbox{CeMotTrèsLongNestPasCoupé}. 
%%%% fin exemple %%%%

Pour empêcher LATEX de couper les mots dans un paragraphe il suffit de l'encadrer par les commandes \begin{sloppypar} et \end{sloppypar}.

Pour empêcher LATEX de couper les mots dans tout un document, il faut utiliser la commande \sloppy dans le préambule du document. On peut également utiliser la déclaration :

\hyphenpenalty 10000.
De manière globale, on peut aussi déclarer :

\DeclareFontFamily{T1}{cmr}{\hyphenchar\font=-1}
Pour interdire la coupure de tous les mots commençant par une majuscule, il faut utiliser : \uchyph=0 dans le préambule du document.

On peut également interdire la coupure des mots d'une langue particulière dans un document multilingues en utilisant l'astuce suivante : il suffit de créer un fichier de motifs de césures vide, par exemple pour le russe

%%%% debut macro %%%% 
%%% ruhyph.tex %%% 
\patterns{} 
\endinput 
%%%%%%%%%%%%%%%%%%% 
%%%% fin macro %%%%
et dans le fichier language.dat ajouter la ligne russian ruhyph.tex

6.45  Comment mettre en forme des équations chimiques ?

Le package 'ppchtex' disponible sur ftp://ftp.fdn.org/pub/CTAN/macros/context/ permet d'écrire des formules chimiques.

Sur Mac ou PC, il existe MDL qui est une version freeware d'ISIS Draw qui permet de créer ses propres structures et de les sauver sous format eps. Pour plus de détails, consulter http://www.mdli.com/prod/ioffer.html.

Il existe le package 'chemsym' disponible sur ftp://ftp.fdn.org/pub/CTAN/macros/latex/contrib/supported/chemsym/ peut également être utile.

Le package 'XymTex' disponible sur ftp://ftp.fdn.org/pub/CTAN/macros/latex/contrib/other/xymtex/ ou ftp://ftp.fdn.org/pub/CTAN/macros/latex209/contrib/xymtex/ permet de définir des structures chimiques.

Remarques (F. Jacquet) :
 

6.46  Comment mettre en page un calendrier ?

Le package 'termcal' disponible sur ftp://ftp.fdn.org/pub/CTAN/macros/latex/contrib/supported/termcal/ permet de mettre en page un calendrier. Il permet de préciser quels jours doivent apparaître et d'insérer du texte soit régulièrement à certaines dates soit à des dates particulières.

Le package 'calendar ' disponible sur ftp://ftp.fdn.org/pub/CTAN/macros/plain/contrib/ propose un ensemble de macros pour mettre en page des calendriers.

Un autre package 'calendar' est également disponible sur ftp://ftp.fdn.org/pub/CTAN/macros/latex/contrib/supported/calendar/.

Il existe également le package 'yplan97' disponible sur ftp://ftp.fdn.org/pub/CTAN/macros/latex/contrib/other/yplan97/.

6.47  Comment forcer un caption sur plusieurs lignes ?

Pour forcer un retour à la ligne dans un caption, il faut utiliser

\caption[text1]{text2 \\\hspace{\linewidth} text3}.
Remarque :
text1 est le texte qui apparaîtra dans la liste des figures ou des tableaux.

6.48  Comment générer des barres de modification dans un document ?

Il suffit d'utiliser le package 'changebar' disponible sur ftp://ftp.fdn.org/pub/CTAN/macros/latex/contrib/supported/changebar/.

Exemple :
 
dans le préambule :

\usepackage[outerbars]{changebar} % permet de positionner les 
                                  % barres en marge externe 
\setcounter{changebargrey}{20} % permet de fixer le niveau de 
                               % gris des barres
et dans le document :

\begin{changebar} 
   texte.. 
\end{changebar}
ou

ancien texte ancien texte ancien texte \cbstart nouveau 
texte nouveau texte nouveau texte \cbend ancien texte ancien 
texte ancien texte ancien texte 
%%%% fin exemple %%%%

6.49  Comment encadrer du texte verbatim ?

Le package 'moreverb' propose un environnement encadré.

Exemple :
 
\begin{boxedverbatim} La commande \LaTeX permet 
d'appeler le logo 
   LaTeX. 
\end{boxedverbatim} 
%%%% fin exemple %%%%

6.50  Comment écrire du texte en forme de losange ou autre ?

Le package 'shapepar' disponible sur ftp://ftp.fdn.org/pub/CTAN/macros/latex/contrib/other/misc/ définit des environnements losange, coeur, etc.

Exemple :
 
\diamondpar{mon paragraphe en forme de losange.} 
%%%% fin exemple %%%%

6.51  Comment isoler les flottants par un trait horizontal ?

Pour que les flottants qui sont renvoyés en début (respectivement en fin) de page soient isolés du texte par un trait horizontal en dessous (respectivement au-dessus), il faut activer les options suivantes dans le préambule du document :

\let\topfigrule\hrule 
\let\botfigrule\hrule
Si leur comportement n'est pas satisfaisant, on peut le redéfinir :

\newcommand{\topfigrule}{% 
  \vspace*{3pt}\noindent\rule{\linewidth}{0.4pt}\vspace{-3.4pt}} 
\newcommand{\botfigrule}{% 
  \vspace*{3pt}\noindent\rule{\linewidth}{0.4pt}\vspace{-3.4pt}}

6.52  Comment insérer un espace vertical dans une page ?

Il suffit d'utiliser la commande \vspace et de préciser en argument la hauteur de l'espace voulu.

Exemple :
 
Texte avant.

 

\vspace{2cm}

 

Texte après. 
%%%% fin exemple %%%%

ce qui donne :

 

Texte avant.






Texte après.

 

Remarque :
la commande \vspace* force l'insertion d'un espace vertical même si ce dernier se situe sur un changement de page.

6.53  Comment insérer un espace horizontal dans un texte ?

De la même manière que pour les espaces verticaux, il existe la commande \hspace pour insérer un espace horizontal dans un texte.

Exemple :
 
blabla bla \hspace{3cm} bli bli bli 
%%%% fin exemple %%%%
ce qui donne :

 

blabla bla                         bli bli bli

 

Remarque :
la commande \hspace* force l'insertion d'un espace même en début ou en fin de ligne.

6.54  Comment définir un style de paragraphe ?

Pour agir de manière globale (sur tout le document), les paramètres de définition d'un paragraphe sont :

Exemple :
 
\setlength{\parindent}{1cm} 
\setlength{\parskip}{1ex plus 0.5ex minus 0.2ex} 
%%%% fin exemple %%%%
Pour forcer (supprimer) localement l'indentation d'un paragraphe, il existe la commande \(no)indent.

6.55  À quoi sert la commande \stretch ?

La commande \stretch{x} permet d'insérer un espace élastique qui va s'ajuster de manière à forcer l'occupation de toute une ligne (en mode horizontal) ou de toute une page (en mode vertical). Le paramètre x est appelé facteur d'élasticité, il intervient dès que plusieurs commandes \stretch sont appelées sur la même ligne.

Exemples :
 
bla\hspace{\stretch{1}} bla bla \hspace{\stretch{2}} bla bla  
bla 
un texte normal \vspace{\stretch{1}} et un texte qui finit de  
remplir la ligne courante et qui se poursuit tout en bas de la 
page sur la dernière ligne. 
%%%% fin exemple %%%%
ce qui donne :

 

bla                        bla bla                         bla bla bla un texte normal







et un texte qui finit de remplir la ligne courante et qui se poursuit tout en bas de la page sur la dernière ligne.

6.56  Comment justifier un paragraphe à gauche ou à droite ?

Les environnements flushleft et flushright permettent de justifier à gauche ou à droite une portion de texte.

Exemple :
 
\begin{flushleft} 
   Texte aligné à gauche. Texte aligné à gauche. Texte aligné à 
   gauche. Texte aligné à gauche. Texte aligné à gauche. Texte 
   aligné à gauche. Texte aligné à gauche. Texte aligné à 
   gauche. Texte aligné à gauche. Texte aligné à gauche. Texte 
   aligné à gauche. 
\end{flushleft} 
%%%% fin exemple %%%%
ce qui donne :

Texte aligné à gauche. Texte aligné à gauche. Texte aligné à gauche. Texte aligné à gauche. Texte aligné à gauche. Texte aligné à gauche. Texte aligné à gauche. Texte aligné à gauche. Texte aligné à gauche. Texte aligné à gauche. Texte aligné à gauche.

 

Il existe également les commandes \raggedright et \raggedleft.

Exemple :
 
{\raggedleft 
Colle à droite. Colle à droite. Colle à droite. Colle à droite. 
Colle à droite. Colle à droite. Colle à droite. 
 

Colle à droite. Colle à droite. Colle à droite. Colle à droite. Colle à 
droite. Colle à droite. } 
%%%% fin exemple %%%%

ce qui donne :

Colle à droite. Colle à droite. Colle à droite. Colle à droite. Colle à droite. Colle à droite. Colle à droite. Colle à droite. Colle à droite. Colle à droite. Colle à droite. Colle à droite. Colle à droite.

 

Le package 'ragged2e' disponible sur ftp://ftp.fdn.org/pub/CTAN/macros/latex/contrib/supported/ragged2e/ offre en plus des commandes (\Centering, \RaggedLeft, et \RaggedRight) et des environnements qui permettent de mieux gérer les césures.

6.57  Comment mettre en page des citations ?

Il existe deux environnements de gestion des citations :

Exemple :
 
\begin{quote} 
   L'environnement \texttt{quote} n'indente pas ces paragraphes 
   par contre l'espace vertical entre ces derniers est supérieur 
   à celui d'un texte standard.

 

   La preuve! 
\end{quote}

 

\begin{quotation} 
   L'environnement \texttt{quotation} indente la première ligne 
   de ses paragraphes et sépare ses paragraphes d'un espace 
   vertical standard.

 

   La preuve!

 

\end{quotation}

 

Dans les deux cas les marges droite et gauche sont plus 
importantes que celles d'un texte standard. 
%%%% fin exemple %%%%

ce qui donne :

L'environnement quote n'indente pas ces paragraphes par contre l'espace vertical entre ces derniers est supérieur à celui d'un texte standard. La preuve !
L'environnement quotation indente la première ligne de ses paragraphes et sépare ses paragraphe d'un espace vertical standard. La preuve !
Dans les deux cas les marges droite et gauche sont plus importante que celles d'un texte standard.

6.58  Comment insérer un texte sans que LATEX le mette en forme ?

L'environnement verbatim permet d'insérer un texte tel quel sans que LATEX ne le traite. Il permet notamment de faire apparaître des commandes LATEX.

Exemple :
 
\begin{verbatim} 
   La commande \LaTeX permet d'imprimer le logo LaTeX. 
\end{verbatim} 
%%%% fin exemple %%%%
ce qui donne :

La commande \LaTeX permet d'imprimer le logo LaTeX.

 

La commande \verb?texte? permet de faire la même chose localement. Le premier caractère ? permet de marquer le début du mode verbatim et le second en marque la fin. Il peut être remplacé par n'importe quel autre caractère sauf un espace ou *.

6.59  Comment visualiser des espaces ?

L'environnement verbatim* et la commande \verb* permettent de visualiser les espaces insérés dans un texte en les remplaçant par un caractère spécial en forme de u.

Exemple :
 
Je mange \verb*?   ? pomme. 
%%%% fin exemple %%%%
ce qui donne :

 

Je mange pomme.

 

LATEX propose la commande \textvisiblespace.

En TEX, c'est le caractère 32 de la fonte cmtt, qui est défini dans le source du TEXBook par :

\def\]{\leavevmode\hbox{\tt\char`\ }} % visible space
Vous pouvez donc définir:

\def\vs{\leavevmode\hbox{\tt\char`\ }} % visible space 
et \vs donnera ce fameux caractère.

6.60  Comment insérer du code LATEX dans un document LATEX ?

Le package 'example' offre un environnement example qui permet en ne tapant qu'une seule fois le code d'avoir côte à côte le code LATEX et son résultat après compilation.

Exemple :
 
\begin{example} 
   La commande \LaTeX permet d'appeler le logo LaTeX. 
\end{example} 
%%%% fin exemple %%%%
La solution la plus classique consiste à faire :

\hbox{\vtop{\hsize 0.5\hsize le permier texte}\vtop{\hsize 
0.5\hsize{\begin{verbatim} le même texte \end{verbatim}}}}

6.61  Comment écrire du texte en couleurs ?

Voir à ce sujet le Cahier GUTenberg numéro 16 (février 1994, entièrement consacré à ce problème) et l'article de M. Goossens et M. Jouhet dans Cahier GUTenberg 21 (juin 1995, pages 30-52), accessible à l'URL : http://www.gutenberg.eu.org/publications/.

Le package 'color' est disponible sur ftp://ftp.fdn.org/pub/CTAN/macros/latex/required/graphics/. Il permet de coloriser le texte ou le fond du document.

Exemple :
 
\textcolor{red}{Important} 
%%%% fin exemple %%%%
pstricks disponible sur ftp://ftp.fdn.org/pub/CTAN/graphics/pstricks/ est un ensemble de macros PostScript compatibles avec Plain TEX, LATEX, AmSTEX, and AmS-LATEX. Il comprend notamment des macros pour la colorisation, la gestion des graphiques, le dessin de camembert, d'arbres, etc.

Voir également la question : Comment griser le fond (background) d'un paragraphe ?

6.62  Comment construire un arbre ?

Le package 'qobitree', est disponible sur ftp://ftp.fdn.org/pub/CTAN/macros/latex/contrib/other/qobitree/.

Le package 'treesvr' est également disponible sur ftp://ftp.fdn.org/pub/CTAN/macros/latex/contrib/supported/treesvr/.

Exemple :
 
\begin{picture}(100,100) 
\setlength\unitlength{2mm} 
\branchlabels ABC         % 012 is the default 
\root(2,10)        0.     % root at absolute coordinate (2,10) 
                          % its (internally used) label is 0 
                          % the space before the 0 obligatory 
\branch2{16}       0:1,2. % node 0 has children 1 and 2 
                          % the text "16" is written above it 
                          % space optional, :,. obligatory 
\leaf{4}{$u_1$}    1.     % node 1 is a leaf 
                          % "4" written above, "$u_1$" at right 
\branch2{12}       2:3,7. % branch to node 3 goes up labeled A 
  \tbranch2{9}     3:4,5,6. 
    \leaf{4}{$u_3$}4. 
    \leaf{3}{$u_4$}5. 
    \leaf{2}{$u_5$}6. 
  \leaf{3}{$u_2$}  7. 
\end{picture} 
%%%% fin exemple %%%%
Le package 'treetex' disponible sur ftp://ftp.fdn.org/pub/CTAN/macros/latex209/contrib/trees/tree_tex/ permet de metttre en page des arbres sous LATEX.

Les packages 'epic', 'eepic', 'ecltree' permettent de créer des arbres (les trois packages sont nécessaires pour une utilisation avec Latex2e pour garder la compatibilité avec Latex2.09).

Exemple :
 
\documentstyle[epic,eepic,ecltree]{article} 
\begin{document}

 

\begin{bundle}{racine} 
\chunk{feuille 1} 
\chunk{feuille 2} 
\drawwith{\dottedline{3}} 
\chunk{feuille 3} 
\drawwith{\drawline} 
\chunk{feuille 4} 
\end{bundle}

 

\end{document} 
%%%% fin exemple %%%%

6.63  Comment mettre en page un poème ?

Il existe l'environnement verse. Il gère les marges, les retours à la ligne dans une strophe se font par \\ et les strophes sont séparées par des lignes blanches.

Exemple :
 
\begin{verse} 
   J'ai un poney gris, \\ 
   Qui galope à travers la prairie.

 

   Grignote, grignote dans ma main, \\ 
   La carotte rousse du jardin.

 

   Mes cousins ont un poney blanc, \\ 
   Qui parcourt chemins et champs. 
\end{verse} 
%%%% fin exemple %%%%

ce qui donne :

J'ai un poney gris,
Qui galope à travers la prairie. Grignote, grignote dans ma main,
La carotte rousse du jardin. Mes cousins ont un poney blanc,
Qui parcourt chemins et champs.

 

Ci-dessous un exemple de style de JHB Nijhof (nijhojhb@aston.ac.uk) :

Exemple :
 
\documentstyle[a4,12pt]{article} 
 

\def\testline{\par\noindent\hbox to 0pt{\hss*\hss}\hfill 
\hbox to 0pt{\hss*\hss}\hfill\hbox to 0pt{\hss*\hss}\par} 
\def\centerstar{\par\medskip\noindent\hbox to\hsize% 
{\hss*\hss}\par\medskip} 
 

\newbox\poembox 
\newbox\widebox 
\newdimen\centerx 
\newcount\linecount 
\newdimen\poemleftmargin 
\def\newpoem{\setbox0=\box\poembox \setbox0=\box\widebox 
\linecount=0} % empty boxes 
\newpoem % probably not necessary 
\def\poemline#1{\setbox0=\hbox{\strut #1}% 
 \setbox\poembox=\vbox{\unvbox\poembox\copy0}% 
 \setbox\widebox=\hbox{\unhbox\widebox\copy0}% 
 \advance \linecount 1} 
\def\setpoem{% you'll want a clearpage here 
\centerx=\wd\widebox \divide\centerx\linecount \divide\centerx 2% 
%now centerx is the x of the center of gravity 
\poemleftmargin=0.5\hsize \advance \poemleftmargin-\centerx 
\noindent\kern\poemleftmargin\box\poembox 
\par 
\newpoem} 
 

\begin{document} 
 

\testline % for viewing the margins/ middle 
\poemline{Rozen verwelken} 
\poemline{schepen vergaan} 
\poemline{maar onze liefde} 
\poemline{zal blijven bestaan} 
\setpoem 
 

\centerstar 
 

\poemline{Rozen verwelken} 
\poemline{schepen vergaan} 
\poemline{maar onze liefde zal blijven bestaan} 
\setpoem 
 

\centerstar 
 

\poemline{Rozen verwelken, schepen vergaan} 
\poemline{maar onze liefde zal blijven bestaan} 
\setpoem 
 

\centerstar 
 

\poemline{Rozen verwelken} 
\poemline{\qquad schepen vergaan} 
\poemline{maar onze liefde} 
\poemline{\qquad zal blijven bestaan} 
\setpoem 
 

\centerstar 
   

\end{document} 
%%%% fin exemple %%%%

6.64  Comment aligner des paragraphes ?

On peut utiliser la commande \parindent.

Exemple :
 
{\settowidth{\parindent} 
{Notes:\ }

 

\makebox[0pt][r] 
{Notes:\ }La première note pour dire que ...

 

La seconde pour préciser que ... 

%%%% fin exemple %%%%

ce qui donne :

Notes : 
Notes : 
La première note pour dire que ... La seconde pour préciser que ...

6.65  Comment construire une liste d'éléments ?

Il existe trois environnements de liste par défaut :

Exemples :
 
\begin{itemize} 
   \item un élément 
   \item un autre élément 
\end{itemize}

 

\begin{description} 
   \item[Genre] Le genre peut être féminin ou masculin 
   \item[Nombre] Le nombre peut être singulier ou pluriel 
\end{description}

 

\begin{enumerate} 
   \item premier élément 
   \item deuxième élément 
\end{enumerate} 
%%%% fin exemple %%%%

ce qui donne :

Genre
Le genre peut être féminin ou masculin
Nombre
Le nombre peut être singulier ou pluriel

  1. premier élément
  2. deuxième élément

6.66  Comment faire une note dans une marge ?

On peut utiliser la commande \marginpar{note marge}.

Exemple :
 
La valeur du paramètre temps est fixée à 
12minutes\marginpar{AC}. En tenant compte de cette hypothèse 
les résultats obtenus sont les suivants:

 

ldots 
%%%% fin exemple %%%%

ce qui donne 

 

La valeur du paramètre temps est fixée à 12 minutes
AC
. En tenant compte de cette hypothèse les résultats obtenus sont les suivants :  ...

 

Remarque :
Pour inverser les notes dans les marges (droite/gauche) il suffit de mettre dans le préambule : \reversemarginpar.

6.67  Comment faire une page de garde ?

Les commandes permettant de définir une page de garde sont :

La page de garde est ensuite générée par la commande \maketitle appelée dans le corps du document (généralement juste après la balise \begin{document}).

Exemple :
 
\documentclass{report} 
\usepackage{french}

 

\newlength{\larg} 
\setlength{\larg}{14.5cm}

 

\title{ 
{\rule{\larg}{1mm}}\vspace{7mm} 
\begin{tabular}{p{4cm} r} 
   & {\Huge {\bf {FAQ} \LaTeX{} française}} \\ 
   & \\ 
   & {\huge Pour débutants et confirmés} 
\end{tabular}\\ 
\vspace{2mm} 
{\rule{\larg}{1mm}} 
\vspace{2mm} \\ 
\begin{tabular}{p{11cm} r} 
   & {\large \bf Version 2.0} \\ 
   & {\large  \today} 
\end{tabular}\\ 
\vspace{5.5cm} 

\author{\begin{tabular}{p{13.7cm}} 
Marie-Paule Kluth 
\end{tabular}\\ 
\hline } 
\date{}

 

\begin{document} 
\maketitle

 

Voici mon document.

 

\end{document} 
%%%% fin exemple %%%%

6.68  Comment utiliser \thanks dans chaque titre de chapitre ?

Pour associer des remerciements dans chaque chapitre d'un thèse, par exemple, il faut définir sa propre macro à base de \footnote.

Exemple (R. Fairbairns) :
 
\documentclass{report} 
\usepackage{french}

 

\newcommand\thankschapter[2]{% 
   % arg 1 is chapter title 
   % arg 2 is `thanks' text 
   \edef\savefootnote{\thefootnote} 
   \renewcommand\thefootnote{\fnsymbol{footnote}} 
   \chapter[#1]{#1\footnote[1]{#2}} 
   \renewcommand\thefootnote{\arabic{footnote}} 
   \setcounter{footnote}{\savefootnote} 

\begin{document} 
\chapter{Préface.}{\textit{Un grand merci à M. Nigaudon pour 
avoir accepter de préfacer ce livre.\\}}

 

Ce livre traitant de \LaTeX,ldots

 

\end{document} 
%%%% fin exemple %%%%

6.69  Comment mettre un résumé et un abstract dans un document ?

Le package 'french' propose les environnements resume et abstract.

Exemple (textes sur la même page) :
 
\documentclass{article} 
\usepackage{french} 
\pagestyle{empty} 
\begin{document} 
\begin{resume} résumé en français \end{resume} 
    \english 
\begin{abstract} the same in english \end{abstract} 
   \french

 

   Cet articleldots 
\end{document} 
%%%% fin exemple %%%%

Remarque :
\resume n'est qu'un alias de \abstract.
Exemple (chaque résumé est sur une page séparée) :
 
\begin{english} 
   \begin{abstract} 
      english text 
   \end{abstract} 
\end{english}

 

\begin{french} 
   \begin{abstract} 
      Texte français 
   \end{abstract} 
\end{french} 
%%%% fin exemple %%%%

Le package 'babel' peut également être utilisé.

Exemple :
 
\usepackage[francais,english]{babel} 
... 
\selectlanguage{francais} 
resume : je parle français 
.... 
\selectlanguage{english} 
abstract : I speak english 
.... 
%%%% fin exemple %%%%
Autre solution qui ne permet toutefois pas d'avoir les deux textes sur la même page :

%%%% debut macro %%%% 
\renewcommand{\abstractname}{Résumé} 
\begin{abstract} résumé en français \end{abstract} 
\renewcommand{\abstractname}{Abstract} 
\begin{abstract} the same in english \end{abstract} 
%%%% fin macro %%%%

6.70  Comment définir de nouvelles couleurs ?

Il existe deux repères pour cela :

\definecolor{nouveau_nom}{cmyk}{w,x,y,z} 
\newcmykcolor{le_nom_de_la_couleur}{w x y z}
\definecolor{nouveau_nom}{rgb}{a,b,c}
ou :

\newrgbcolor{le_nom_de_la_couleur}{a b c}
\newhsbcolor{le_nom_de_la_couleur}{num1 num2 num3}

6.71  Comment placer des figures face à face en recto-verso ?

Dans un document recto-verso, pour s'assurer qu'une figure apparaisse sur la page de gauche et une autre sur la page de droite en face, il faut utiliser le package 'dpfloat' disponible sur: http://weber.u.washington.edu/~fox/tex/dpfloat.html.

6.72  Quelles sont les différences entre a4paper et letterpaper ?

L'option a4paper exécute :

  \setlength\paperheight {297mm} 
  \setlength\paperwidth  {210mm}
L'option par défaut est letterpaper qui fait

  \setlength\paperheight {11in} 
  \setlength\paperwidth  {8.5in}
Remarque (J. Kanze) :
Il faut remarquer que la différence n'est pas énorme : les défauts en mm sont 279x216. En fait, sur le petit test que j'ai fait, les coupures de lignes se trouvaient exactement aux mêmes endroits - en revanche, chaque page contenait bien quelques lignes de plus avec a4paper.

6.73  Comment couper une chaîne de caractères ?

Pour forcer la coupure d'une chaîne de caractères, il faut utiliser la commande \- à l'endroit où la chaîne doit être coupée.

Exemple :
 
Je décide que le dernier mot de cette ligne soit coupé en deux 
ici: do\-cumentation. N'est-ce pas beau? 
%%%% fin exemple %%%%
ce qui donne :

 

Je décide que le dernier mot de cette ligne soit coupé en deux ici:  documentation. N'est ce pas beau ?

 

Pour indiquer à LATEX comment couper certain mots, on peut utiliser la commande \hyphenation.

Exemple :
 
\hyphenation{do-cu-ment} 
Je décide que le dernier mot de cette ligne soit coupé en deux 
ici: documentation. N'est ce pas beau? 
%%%% fin exemple %%%%

6.74  Comment modifier l'espace entre une figure et sa légende ?

Les deux variables qui définissent respectivement l'espace avant une légende et l'espace après sont : \abovecaptionskip et \belowcaptionskip.

Exemple de redimensionnement :
 
\setlength\abovecaptionskip{0.25ex} 
%%%% fin exemple %%%%

Chapter 7
Gestion des tableaux

Les tableaux permettent de regrouper des information d'une manière synthétique et facilement compréhensive. LATEX permet de mettre en page de beau et grand tableau si l'on connait les commandes de mise en page.

7.1  Comment faire un tableau ?

En mode texte, il faut utiliser l'environnement tabular. Chaque colonne est ensuite décrite par r, l ou c pour obtenir une colonne de texte justifié à droite, à gauche ou centré. Chaque déclaration de colonne peut être délimitée par une séparation verticale : | ou rien. Une ligne horizontale entre deux lignes s'obtient par la commande \hline. Le contenu de chaque colonne est séparé de celui de la suivante par le symbole &. Le changement de ligne est obtenu par la commande \\.

Lorsque plusieurs colonnes adjacentes ont la même définition, on peut grouper leurs déclarations avec la commande *{nb_colonnes}{déclaration}

Exemple :
 
\begin{tabular}{|*{2}{c|}l r|} 
   \hline 
   une & deux & trois & quatre \\ 
   case centrée & encore centrée & à gauche & à droite \\ 
   \hline 
\end{tabular} 
%%%% fin exemple %%%%
Ce qui donne :

 

une deux trois quatre
case centrée encore centrée à gauche à droite

7.2  Comment gérer les tableaux de plus d'une page ?

Le package 'supertabular' permet de gérer automatiquement (ou presque) les coupures de tableaux. Il est disponible sur ftp://ftp.fdn.org/pub/CTAN/macros/latex/contrib/supported/supertabular/.

Il calcule la longueur du tableau à chaque \\ et vérifie si la fin de page est atteinte. Si oui, il coupe le tableau. Les commandes à utiliser sont :

Exemple :
 
\begin{center} 
    \tablefirsthead{\hline  \multicolumn{1}{|c}{Nombre} 
    & \multicolumn{1}{c}{Nombre$2$} 
    & Nombre$4$ 
    & \multicolumn{1}{c|}{Nombre!} \\ \hline} 
    \tablehead{\hline \multicolumn{4}{|l|}{\small\sl continued 
    from previous page}\\ 
    \hline \multicolumn{1}{|c}{ Nombre} 
    & \multicolumn{1}{c}{Nombre$2$} 
    & Nombre$4$ 
    & \multicolumn{1}{c|}{Nombre!} \\ \hline} 
    \tabletail{\hline\multicolumn{4}{|r|}{\small\sl Suite page 
    suivanteldots}\\\hline} \tablelasttail{\hline} 
    \bottomcaption{Fin.} 
    \par 
    \begin{supertabular}{| r@{\hspace{6.5mm}}| r@{\hspace{5.5mm}}| r | r|} 
      1   &     1  &        1  &           1    \\ 
      2   &     4  &       16  &           2    \\ 
      3   &     9  &       81  &           6    \\ 
      4   &    16  &      256  &          24    \\ 
      5   &    25  &      625  &         120    \\ 
      6   &    36  &     1296  &         720    \\ 
      7   &    49  &     2401  &        5040    \\ 
      8   &    64  &     4096  &       40320    \\ 
      9   &    81  &     6561  &      362880    \\ 
      10  &   100  &    10000  &     3628800    \\ 
      11  &   121  &    14641  &    39916800    \\ 
      12  &   144  &    20736  &   479001600    \\ 
      13  &   169  &    28561  &  6.22702080E+9 \\ 
      14  &   196  &    38416  &  8.71782912E+10\\ 
      15  &   225  &    50625  &  1.30767437E+12\\ 
      16  &   256  &    65536  &  2.09227899E+13\\ 
      17  &   289  &    83521  &  3.55687428E+14\\ 
      18  &   324  &   104976  &  6.40237370E+15\\ 
      19  &   361  &   130321  &  1.21645100E+17\\ 
      20  &   400  &   160000  &  2.43290200E+18\\ 
      \hline 
      1   &     1  &        1  &           1    \\ 
      2   &     4  &       16  &           2    \\ 
      3   &     9  &       81  &           6    \\ 
      4   &    16  &      256  &          24    \\ 
      5   &    25  &      625  &         120    \\ 
      6   &    36  &     1296  &         720    \\ 
      7   &    49  &     2401  &        5040    \\ 
      8   &    64  &     4096  &       40320    \\ 
      9   &    81  &     6561  &      362880    \\ 
      10  &   100  &    10000  &     3628800    \\ 
      11  &   121  &    14641  &    39916800    \\ 
      12  &   144  &    20736  &   479001600    \\ 
      13  &   169  &    28561  &  6.22702080E+9 \\ 
      14  &   196  &    38416  &  8.71782912E+10\\ 
      15  &   225  &    50625  &  1.30767437E+12\\ 
      16  &   256  &    65536  &  2.09227899E+13\\ 
      17  &   289  &    83521  &  3.55687428E+14\\ 
      18  &   324  &   104976  &  6.40237370E+15\\ 
      19  &   361  &   130321  &  1.21645100E+17\\ 
      20  &   400  &   160000  &  2.43290200E+18\\ 
      \hline 
      1   &     1  &        1  &           1    \\ 
      2   &     4  &       16  &           2    \\ 
      3   &     9  &       81  &           6    \\ 
      4   &    16  &      256  &          24    \\ 
      5   &    25  &      625  &         120    \\ 
   \end{supertabular} 
\end{center} 
%%%% fin exemple %%%%
ce qui donne :

| r@| r@| r | r| 1 1 1 1
2 4 16 2
3 9 81 6
4 16 256 24
5 25 625 120
6 36 1296 720
7 49 2401 5040
8 64 4096 40320
9 81 6561 362880
10 100 10000 3628800
11 121 14641 39916800
12 144 20736 479001600
13 169 28561 6.22702080E+9
14 196 38416 8.71782912E+10
15 225 50625 1.30767437E+12
16 256 65536 2.09227899E+13
17 289 83521 3.55687428E+14
18 324 104976 6.40237370E+15
19 361 130321 1.21645100E+17
20 400 160000 2.43290200E+18
1 1 1 1
2 4 16 2
3 9 81 6
4 16 256 24
5 25 625 120
6 36 1296 720
7 49 2401 5040
8 64 4096 40320
9 81 6561 362880
10 100 10000 3628800
11 121 14641 39916800
12 144 20736 479001600
13 169 28561 6.22702080E+9
14 196 38416 8.71782912E+10
15 225 50625 1.30767437E+12
16 256 65536 2.09227899E+13
17 289 83521 3.55687428E+14
18 324 104976 6.40237370E+15
19 361 130321 1.21645100E+17
20 400 160000 2.43290200E+18
1 1 1 1
2 4 16 2
3 9 81 6
4 16 256 24
5 25 625 120

 

Le package 'longtable', de D. Carlisle, disponible sur ftp://ftp.fdn.org/pub/CTAN/macros/latex/required/tools/, fonctionne de la même manière mais mieux et permet de définir une taille de tableau commune sur toutes les pages. Les commandes associées à ce package sont \endfirsthead, \endhead, \endfoot et \endlastfoot.

\begin{longtable} 
   {|p{0.2\linewidth}|p{0.2\linewidth}|p{0.2\linewidth}|} 
   \hline 
   Premiere colonne & Deuxieme & Troisieme \endfirsthead 
   \hline 
   Premiere & Deuxieme & Troisieme \\ 
   \multicolumn{3}{|p{0.6666\linewidth}|}{Suite ... } \\ 
   \endhead 
   \hline 
   \multicolumn{3}{|p{0.6666\linewidth}|}{Suite page suivante} 
   \\ \hline \endfoot \hline 
   \multicolumn{3}{|p{0.6666\linewidth}|}{C'est fini} \\ 
   \hline 
   \endlastfoot \hline 
   1   &     1  &        1  \\ 
   2   &     4  &       16  \\ 
   3   &     9  &       81  \\ 
   1   &     1  &        1  \\ 
   2   &     4  &       16  \\ 
   3   &     9  &       81  \\ 
   1   &     1  &        1  \\ 
   2   &     4  &       16  \\ 
   3   &     9  &       81  \\ 
   1   &     1  &        1  \\ 
   2   &     4  &       16  \\ 
   3   &     9  &       81  \\ 
 \end{longtable} 
ce qui donne: 

Premiere colonne Deuxieme Troisieme
Premiere Deuxieme Troisieme
Suite ...
Suite page suivante
C'est fini
1 1 1
2 4 16
3 9 81
1 1 1
2 4 16
3 9 81
1 1 1
2 4 16
3 9 81
1 1 1
2 4 16
3 9 81

 

au lieu de faire des calculs de largeurs de colonne, on peut aussi mettre un \setlongtables dans le préambule et déclarer ses tableaux comme à l'habitude en \begin{longtable}{|c|c|c|} et LATEX se charge du reste. Dans ce cas, l'ajustement des colonnes peut nécessiter plusieurs (jusqu'à trois) compilations enchaînées.

Le package 'ltxtable' de D. Carlisle, disponible sur ftp://ftp.fdn.org/pub/CTAN/macros/latex/contrib/supported/carlisle/ permet de profiter des fonctionnalités de tabularx et de longtable.

7.3  Comment modifier l'orientation d'un tableau ?

On peut utiliser la commande \rotatebox du package 'graphics' de D. Carlisle. Ce package est disponible sur ftp://ftp.fdn.org/pub/CTAN/macros/latex/required/graphics/.

Exemple :
 
\rotatebox{90}{ 
\begin{tabular}{|c|c|} 
   \hline 
    salut & coucou \\ 
   \hline 
\end{tabular} 

%%%% fin exemple %%%%
ce qui donne :

Le style 'lscape' de D. Carlisle marche aussi avec l'environnement longtable (du même).

De même l'environnement sidewaystable du package 'rotating' permet d'inclure des tableaux en mode landscape. Le sens de rotation peut être changé par l'option counterclockwise.

Exemple :
 
\documentclass{report} 
\usepackage[counterclockwise]{rotating} 
\usepackage{french} 
\pagestyle{empty} 
\begin{document} 
\rotatebox{90}{ 
\begin{tabular}{|c|c|} 
   \hline 
    vu & à voir \\ 
    de Latour & Van Gogh \\ 
   \hline 
\end{tabular} 

\end{document} 
%%%% fin exemple %%%%
Le package 'rotfloat' disponible sur ftp://ftp.fdn.org/pub/CTAN/macros/latex/contrib/supported/rotfloat/ permet de gérer la rotation des flottants.

7.4  Comment nommer un tableau ?

Pour pouvoir associer une légende (\caption) à un tableau, il suffit de l'encapsuler dans un environnement table.

Exemple :
 
\begin{table}[htbp] 
   \begin{center} 
      \begin{tabular}{|c|c|} 
         \hline 
         un & deux \\ 
         \hline 
         trois & quatre \\ 
         \hline 
      \end{tabular} 
      \caption{Nom du tableau. \label{table-nom}} 
   \end{center} 
\end{table} 
%%%% fin exemple %%%%
ce qui donne :

un deux
trois quatre
Table 7.1: Nom du tableau.

7.5  Comment modifier l'épaisseur des lignes d'un tableau ?

Pour obtenir des lignes horizontales fines ou épaisses dans un tableau, il faut utiliser les commandes

\setlength{\doublerulesep}{\arrayrulewidth} 
dans l'entête du document puis dans le tableau

\\\hline         pour une ligne fine 
\\\hline\hline   pour une ligne épaisse
Autre solution : la macro de A. Kessi (alain.kessi@psi.ch)

%%%% debut macro %%%% 
\makeatletter 
\def\hlinewd#1{% 
\noalign{\ifnum0=`}\fi\hrule \@height #1 % 
\futurelet\reserved@a\@xhline} 
\makeatother 
%%%% fin macro %%%%
Exemple :
 
\begin{tabular}{|l|r|}    \hline 
     premier &   1     \\ \hline 
     second  &   2     \\ \hlinewd{5pt} 
     total   &   3     \\ \hline 
\end{tabular} 
%%%% fin exemple %%%%
Même chose pour les lignes verticales.

%%%% debut macro %%%% 
  \begin{tabular}{@{\,\vrule width 5pt\,}c|c|} 
%%%% fin macro %%%%
Le package 'easytable' disponible sur ftp://ftp.fdn.org/pub/CTAN/macros/latex/contrib/supported/easy/ propose différents styles de lignes de séparation des cellules d'un tableau.

7.6  Comment griser des cellules d'un tableau ?

Le package 'colortab' disponible par ftp sur ftp://ftp.princeton.edu/pub/tvz/ ou sur ftp://ftp.fdn.org/pub/CTAN/graphics/pstricks/generic/ (accompagné de sa doc dans ftp://ftp.fdn.org/pub/CTAN/graphics/pstricks/origdoc/) permet de griser ou de mettre en couleur certaines parties de tableau.

Remarque :
colortab n'est pas actuellement maintenu par Van Zandt et l'extension longtable, avec laquelle il était compatible, ne fonctionne plus.
Il est possible également d'utiliser le package 'shade' disponible sur {macros/generic/}.

D. Carlisle propose également le package 'colortbl' sur ftp://ftp.fdn.org/pub/CTAN/macros/contrib/supported/carlisle/. Ce package permet non seulement de gérer de la couleur mais il est en outre compatible avec le package 'longtable'. Il nécessite les packages 'array' et 'color'.

Exemple :
 
\begin{tabular}{|>{\columncolor[gray]{.8}}l% 
|>{\color{white}\columncolor[gray]{.2}}r|} 
   \hline 
   UN & DEUX \\ 
   TROIS & QUATRE \\ 
   \hline 
\end{tabular} 
%%%% fin exemple %%%%
ce qui donne :

 

UN DEUX
TROIS QUATRE

7.7  Comment changer la fonte d'une colonne ?

Pour préciser une fonte spécifique dans une colonne d'un tableau, on peut utiliser le package 'array'. Ce dernier est disponible sur ftp://ftp.fdn.org/pub/CTAN/macros/latex/required/tools/.

Exemple :
 
\begin{tabular}{>{\bfseries}l >{\slshape}r c} 
     gras & penché & normal \\ 
\end{tabular} 
%%%% fin exemple %%%%
ce qui donne :

 

gras penché normal

7.8  Comment créer des notes de bas de page dans un tableau ?

Il suffit d'encapsuler le tableau dans un environnement minipage et pour supprimer le trait de séparation des notes de bas de page de déclarer : \

renewcommand{\footnoterule}{}.
Exemple :
 
\begin{minipage}[t]{5cm} 
   \renewcommand{\footnoterule}{} % permet de supprimer le 
                                  % trait de séparation 
   \begin{tabular}{|c|c|} 
      \hline 
      Air & Terre\footnote{Feu}\\  
      \hline 
   \end{tabular} 
\end{minipage} 
%%%% fin exemple %%%%
Remarque :
dans ce cas, la note apparaît juste après le tableau, dans l'environnement minipage.
On peut également utiliser :

\footnote[cptr]{texte.}
toujours dans un environnement minipage, et gérer soi-même ses compteurs (cptr est un entier alors que les marques qui apparaîssent dans le tableau sont les lettres de l'alphabet).

Remarque :
ici aussi la note apparaît en dessous du tableau.
Il existe également deux commandes spécifiques \footnotemark[] et \footnotetext[]{}. \footnotemark permet de gérer le compteur de notes et \footnotetext permet d'insérer le texte correspondant en bas de page.

Exemple :
 
\begin{tabular}{|c|c|} 
   \hline 
   donnee1\footnotemark[1] & donnee2\footnotemark[2] \\ 
   \hline 
\end{tabular} 
\footnotetext[1]{Note associ\'ee \`a la donn\'ee 1.} 
\footnotetext[2]{Note associ\'ee \`a la donn\'ee 2.} 
%%%% fin exemple %%%%
ce qui donnes :

 

donnee1 donnee2

Remarque :
ici les notes apparaîssent bien dans le bas de page.

7.9  Comment écrire un texte sur plusieurs colonnes ?

Il faut utiliser la commande :

\multicolumn{nb_colonnes}{alignement}{Texte}
Exemple :
 
\begin{tabular}[b]{|l|c|} 
   \hline 
   \multicolumn{2}{|c|}{Texte sur 2 colonnes} \\ 
   \hline \hline 
   donnée1 & donnée2 \\ 
   A & B \\ 
   \hline 
\end{tabular} 
%%%% fin exemple %%%%
ce qui donne :

 

Texte sur 2 colonnes
donnée1 donnée2
A B

Remarque :
dans le cas où le nombre de colonnes à couvrir est égal à 1, cette commande peut être utilisée pour modifier l'alignement (c, l, r, |, etc) d'une cellule.

7.10  Comment passer certaines cellules en reverse vidéo ?

Le package 'color' disponible sur ftp://ftp.fdn.org/pub/CTAN/macros/latex/required/graphics/ permet entre autres de le faire. En fait, il permet de définir des couleurs de texte et de fond de page.

Exemples :
 
\tabcolsep=30pt 
\begin{tabular}{|c|c|c|} 
\hline 
1&&\\ 
&\colorbox{black}{\strut{\color{white}Coucou}}&\\ 
&&2\\ 
\hline 
\end{tabular}
ce qui donne :

 

1
2

 

ou ()

 

\newlength\Coucoulength 
\settowidth\Coucoulength{Coucou Coucou Coucou}

 

\begin{tabular}{|c|c|c|} 
\hline 
1&Coucou Coucou Coucou &\\ 
&\colorbox{black}{\makebox[\Coucoulength][c]{\color{white}Coucou}} 
&\\ 
&&2\\ 
\hline 
\end{tabular} 
%%%% fin exemple %%%%

ce qui donne :

 

1Coucou Coucou Coucou
2

 

7.11  Comment fixer la largeur d'une colonne ?

p{largeur} dans les descripteurs de colonne permet de fixer la largeur d'une colonne. Dans ce cas par défaut le texte est justifié à gauche. Les commandes \centering et \raggedright permettent respectivement de le centrer ou de le justifier à droite.

Exemple :
 
\begin{center} 
   \begin{tabular}{|c|l|p{4cm}|r|} 
      \hline 
      centré & à gauche & largeur fixe & à droite \\ 
      bla bla bla & bla bla bla &  bla bla bla &  bla bla bla \\ 
      \hline 
   \end{tabular} 
\end{center} 
%%%% fin exemple %%%%
ce qui donne :

 

centré à gauche largeur fixe à droite
bla bla bla bla bla bla bla bla bla bla bla bla

 

Avec le package 'array', disponible sur ftp://ftp.fdn.org/pub/CTAN/macros/latex/required/tools/, il faut utiliser l'option m{largeur}.

Le package 'easytable' disponible sur ftp://ftp.fdn.org/pub/CTAN/macros/latex/contrib/supported/easy/ permet facilement d'écrire des tableaux dont les colonnes ont une largeur fixe.

7.12  Comment écrire un texte sur plusieurs lignes ?

Il existe le package 'multirow' disponible sur ftp://ftp.fdn.org/pub/CTAN/macros/latex209/contrib/misc/. Il permet d'écrire un texte à cheval sur deux lignes.

Exemple :
 
\begin{table}[htbp] 
   \begin{center} 
   \begin{tabular}{|c|c||c|c|} 
         \hline 
         \multirow{2}{0.5cm}{k} & \multirow{2}{0.5cm}{$p_G$} & 
         \multicolumn{2}{c|}{test} \\ 
         \cline{3-4} 
         & & DADWRD & RARWRD \\ 
         \hline 
         2 & $1$ & 90 n & 228 n \\ 
         3 & $p_d$ & 202 n & 449 n \\ 
         4 & $p_d2$ & 424 n & 891 n \\ 
         5 & $p_d3$ & 866 n  & 1774 n \\ 
         \hline 
      \end{tabular} 
      \caption{Nombre d'opération par cellule des tests 
      pseudo-aléatoires de fautes de type k-coupling. 
      \label{table-compar}} 
   \end{center} 
\end{table} 
%%%% fin exemple %%%%
ce qui donne :

 

test
DADWRD RARWRD
2 1 90 n 228 n
3 pd 202 n 449 n
4 pd2 424 n 891 n
5 pd3 866 n 1774 n
Table 7.2: Nombre d'opération par cellule des tests pseudo-aléatoires de fautes de type k-coupling.

 

7.13  Comment diviser une cellule par une diagonale ?

Il faut utiliser le package 'slashbox' disponible par ftp sur ftp://ftp.tohoku.ac.jp/pub/TeX/latex-styles/bear_collections/style-files/.

Exemple (de la doc) :
 
\begin{tabular}{|l||*{5}{c|}}\hline 
\backslashbox{Room}{Date} 
&\makebox[3em]{5/31}&\makebox[3em]{6/1}&\makebox[3em]{6/2} 
&\makebox[3em]{6/3}&\makebox[3em]{6/4}\\\hline\hline 
Meeting Room &&&&&\\\hline 
Auditorium &&&&&\\\hline 
Seminar Room &&&&&\\\hline 
\end{tabular} 
%%%% fin exemple %%%%
ce qui donne :

5/31


6/1

6/2

6/3

6/4
Meeting Room
Auditorium
Seminar Room

7.14  Comment définir une colonne en mode mathématique dans un tableau ?

Avec le package 'array' disponible sur ftp://ftp.fdn.org/pub/CTAN/macros/latex/required/tools/, il suffit de déclarer \begin{tabular}{>{$}c<{$}cc} pour avoir une colonne en mode mathématique et deux colonnes de texte.

7.15  Comment modifier le nombre de tableaux par page ?

cf. question .

7.16  Comment mettre deux tableaux côte à côte ?

cf. question .

7.17  Comment définir un séparateur de colonne ?

@{symbole} entre deux descripteurs de colonnes permet de définir "symbole" comme séparateur de colonnes. Cela remplace |.

Exemple :
 
\begin{tabular}{|l @{\textbf{ est }} l|} 
   \hline 
   Le lion & féroce. \\ 
   Le chien & fidèle. \\ 
   \hline 
\end{tabular} 
%%%% fin exemple %%%%
ce qui donne :

 

Le lion est féroce.
Le chien est fidèle.

7.18  Comment obtenir des lignes partielles dans un tableau ?

La commande \cline{ColonneDebut-ColonneFin} est faite pour ça.

Exemple :
 
\begin{center} 
   \begin{tabular}{|c|c|c||c|c|c|c|c|c|c||c|} 
      \hline \hline 
      a & b & c & d & e & f & g & h & i & j & k \\ 
      \cline{1-4} \cline{6-6} \cline{8-9} 
      1 & 2 & 3 & 4 & 5 & 6 & 7 & 8 & 9 & 10 & 11 \\ 
      \hline 
   \end{tabular} 
\end{center} 
%%%% fin exemple %%%%
ce qui donne :

 

a b c d e f g h i j k
1 2 3 4 5 6 7 8 9 10 11

7.19  Comment éviter que du texte de grande taille n'atteigne le cadre des cellules ?

Il faut pour cela utiliser la commande \strut qui simule un objet vertical invisible, après le changement de fonte.

Exemple :
 
\begin{tabular}{|l|} 
   \hline 
   {\large  HELLO} dfg \\ 
   \hline 
\end{tabular} 
\begin{tabular}{|l|} 
   \hline 
   {\large\strut  HELLO} dfg \\ 
   \hline 
\end{tabular} 
%%%% fin exemple %%%%
ce qui donne :

 

HELLO dfg
HELLO dfg

 

Il existe l'équivalent mathématique : \mathstrut.

 

On peut également agrandir la hauteur des lignes avec la commande \arraystretch :

\renewcommand{\arraystretch}{1.5}

7.20  Comment fixer la largeur d'un tableau ?

Le package 'tabularx' disponible sur ftp://ftp.fdn.org/pub/CTAN/macros/latex/required/tools/ permet de définir une largeur de tableau.

Le package 'easytable' disponible sur ftp://ftp.fdn.org/pub/CTAN/macros/latex/contrib/supported/easy/ permet facilement de fixer des largeur de colonnes ou de lignes.

7.21  Comment tracer des traits discontinus ?

Il faut utiliser les packages 'hvdashln' et 'array'.

Exemple :
 
dans le préambule du document :

\usepackage{hvdashln,array} 
\setlength{\hdashlinewidth}{.5pt} 
\setlength{\hdashlinegap}{2pt}
dans le texte :

\
\left[ \begin{array}{ccc@{}>{\vdashline}c} 
        a_1 & b_1 & c_1 & d_1 \\ 
        a_2 & b_2 & c_2 & d_2 \\ 
         a_3 & b_3 & c_3 & d_3 \\ 
        a_4 & b_4 & c_4 & d_4 
\end{array} \right] 
\
%%%% fin exemple %%%%

7.22  Comment fixer la taille et justifier une colonne ?

Le problème qui se pose lorsqu'on utilise :

\begin{tabular}{|>{\raggedright}p{0.33\textwidth}<{}| 
   >{\raggedleft}p{0.66\textwidth}<{}|} 
   A& \\ 
   B&C \\ 
\end{tabular}
vient du fait que \\ est redéfini par tabular et les commandes ragged. On ne peut donc plus utiliser \\ dans un texte.

Il existe différentes solutions pour pallier ce problème :

\makeatletter 
\newtoks\fintableau 
\let\fintableau\@arraycr 
\makeatother
Exemple :
 
\begin{tabular}{|r|r|>{\raggedleft}m{190pt}|} 
   \hline 
   col1 & col 2 & col3 \tabularnewline 
   une & deux & trois \\ quatre \tabularnewline 
   \hline 
\end{tabular} 
%%%% fin exemple %%%%
ce qui donne :

 

col1 col 2 col3 une deux trois
quatre

 

\newcommand\PreserveBackSlash[1]{\let\temp=\\#1\let\\=\temp}
puis en utilisant :

\begin{tabular}{|r|r|>{\PreserveBackSlash\raggedleft}m{190pt}|}

7.23  Comment faire une liste de tableaux ?

Il suffit d'inclure la commande \listoftables à l'endroit où l'on veut inclure cette liste. Sachant que cette liste fait référence aux pages où apparaissent les tableaux, il faut enchaîner au moins deux compilations LATEX pour que toutes les références soient exactes.

Chapter 8
Gestion des figures

La gestion des figures est un peu particulière. Les points suivants décrivent la manière et les trucs pratiques à connaître pour rendre un document graphiquement agréable.

8.1  Comment inclure une figure ?

Remarque :
A. K. Goel a écrit un long document concernant les problèmes de gestion de figures et d'images dans LATEX. Ce dernier est disponible par ftp anonyme sur ftp://math.uwaterloo.ca/pub/figsInLatex.ps.Z ou sur ftp://ftp.fdn.org/pub/CTAN/info/figsinltx.ps
De même, K. Reckdahl a écrit "Using EPS Graphics in Documents". Ce document est disponible sur ftp://ftp.fdn.org/pub/CTAN/info/epslatex.ps.

Sous , il faut utiliser l'un des packages :

Exemple :
 
\begin{figure}[htbp] 
   \begin{center} 
      \includegraphics{images/fig1.ps} 
   \end{center} 
   \caption{\footnotesize blah blah blah} 
\end{figure} 
%%%% fin exemple %%%%
Exemple :
 
\includegraphics[width=\linewidth, draft=true]{figure.eps} 
%%%% fin exemple %%%%
\epsfbox{nom-figure.(e)ps}
\begin{figure}[htbp] 
   \centerline{\epsfxsize=10cm \epsfbox{nom-figure.format}} 
 \caption{.  \label{fig-}} 
\end{figure}
Exemple :
 
\begin{figure}[p] 
   \centerline{\epsfig{file=nom_figure.eps, 
   width=largeur, 
   height=hauteur}} 
   \caption{Titre.} 
   \label{nom_label} 
\end{figure} 
%%%% fin exemple %%%%

8.2  Comment nommer une figure ?

C'est la commande \caption qui permet de faire cela. Par défaut le titre de la figure apparaîtra en dessous. Cf exemples ci-dessus. Pour que le titre apparaisse au dessus, il faut redéfinir les longueurs suivantes.

Exemple :
 
\setlength\abovecaptionskip{0pt} 
\setlength\belowcaptionskip{10pt}
puis appeler :
 
\begin{figure} 
\caption{... ... ...} 
\includegraphics{dessin.eps} 
\end{figure} 
%%%% fin exemple %%%%

8.3  Comment placer des figures côte à côte ?

Sous , il faut utiliser le package 'graphics' ou 'graphicx' disponibles sur ftp://ftp.fdn.org/pub/CTAN/macros/latex/required/graphics/.

Exemples :
 
\begin{figure} 
   \begin{minipage}[c]{.46\linewidth} 
      \includegraphics{figure1.format} 
   \end{minipage} \hfill 
   \begin{minipage}[c]{.46\linewidth} 
      \includegraphics{figure2.format} 
   \end{minipage} 
\end{figure}

 

\begin{figure} 
\includegraphics[width=5cm]{fig1.eps}\hfill 
\includegraphics[width=5cm]{fig2.eps} 
\caption{Titre commun}\label{fig:somefiglabel} 
\end{figure} 
%%%% fin exemple %%%%

Une autre solution consiste à mettre chaque figure dans une case d'un tableau.

Exemple (avec le package graphics ou graphicx) :
 
\begin{tabular}{cc} 
   \includegraphics{figure1.eps} & 
   \includegraphics{figure2.eps} \\ 
\end{tabular} 
%%%% fin exemple %%%%
LATEX2.09. Pour mettre des figures côte à côte, il suffit d'encapsuler leur appel dans des minipages.

Exemples :
 
\begin{minipage}[t]{.46\linewidth} 
   \center\epsfxsize= 5cm \epsfbox{fig1.eps} 
\end{minipage} % ne pas sauter de ligne 
\begin{minipage}[t]{.46\linewidth} 
   \center\epsfxsize= 5cm \epsfbox{fig2.eps} 
\end{minipage}
ou si l'on veut attacher des titres aux figures :

\begin{figure} 
 \begin{minipage}[b]{.46\linewidth} 
  \centering\epsfig{figure=fig1.ps,width=\linewidth} 
  \caption{premiere figure \label{fig1}} 
 \end{minipage} \hfill 
 \begin{minipage}[b]{.46\linewidth} 
  \centering\epsfig{figure=fig2.ps,width=\linewidth} 
  \caption{deuxieme figure \label{fig2}} 
 \end{minipage} 
\end{figure} 
%%%% fin exemple %%%%
Exemple :
 
\begin{figure}[ht] 
\begin{center} 
  \subfigure[I]{\epsfig{figure=st1.ps,width=6.58cm}}\quad 
  \subfigure[II]{\epsfig{figure=st2.ps,width=5.0cm}}\\ 
  \subfigure[III]{\epsfig{figure=st3.ps,width=5.0cm}} 
\end{center} 
\caption{Impermeable surface treatments} 
\label{fig:inf} 
\end{figure} 
%%%% fin exemple %%%%
Une autre possibilité pour obtenir un titre par figure est d'utiliser le package 'epslatex' disponible sur ftp://ftp.fdn.org/pub/CTAN/info/.

8.4  Comment modifier le nombre de figures par page ?

En fait, on ne peut agir que globalement sur le nombre de flottants autorisé par page. Il n'y a pas de sélection figure/tables/....

Il arrive fréquemment que lorsqu'un flottant dépasse 60% d'une page, LATEX préfère changer de page plutôt que d'utiliser les 40% de l'espace restant.

La commande \floatpagefraction permet de redéfinir l'espace minimum que peuvent occuper des flottants. Cela permet de limiter le "blanc" sur une page contenant des flottants.

Exemple :
 
\renewcommand{\floatpagefraction}{.9} 
utilisée avec la commande : 
\renewcommand{\textfraction}{.1} 
permet de dire que le texte peut n'occuper que 10% d'une 
page, et donc que des flottants peuvent occuper les 90% restant. 
%%%% fin exemple %%%%
Il y a d'autres paramètres intéressants :

\setcounter{totalnumber}{4}
qui détermine le nombre de flottants autorisés par page,

\renewcommand{\topfraction}{.8} 
et

\renewcommand{\bottomfraction}{.8}
qui indiquent la fraction maximum du haut ou du bas de la page que peuvent occuper des flottants.

Remarque :
Il est recommandé de ne jamais mettre 100% comme paramètre.

8.5  Comment superposer du texte sur des figures ?

PSFrag (disponible sur ftp://ftp.fdn.org/pub/CTAN/macros/latex/contrib/supported/psfrag/) donne cette possibilité. La solution consiste en fait à :

  1. faire le graphique .ps AVEC des textes et légendes MAIS approximatifs
  2. utiliser PSfrag pour qu'il remplace les textes approximatifs par des textes < < LATEX > >
  3. l'écriture < < par dessus > > (le remplacement en fait) est fait par PSfrag.
Pour plus de détails, consulter le document "Using EPS Graphics in documents" disponible sur ftp://ftp.fdn.org/pub/CTAN/info/epslatex.ps.

On peut également générer une courbe dans un fichier .eps, qui peux ensuite être inclu dans un environnement "picture", dans lequel il est ensuite possible d'ajouter du texte avec des \put. Cela demande pas mal de mises au point.

Exemple :
 
\setlength\unitlength{1cm} 
\begin{picture}(10,10) 
\put(0,0){\includegraphics{mondessin.eps}} 
\put(10,10){Mon commentaire \LaTeX{} avec des $maths$} 
\end{picture} 
%%%% fin exemple %%%%
Cette technique permet de conserver la puissance de LATEX et d'avoir une typographie homogène mais elle a l'inconvenient d'être assez lourde.

PSTricks est un ensemble de macros TEX, disponible sur ftp://ftp.fdn.org/pub/CTAN/graphics/pstricks/, qui permet également de faire cela. Il nécessite un gros investissement (il y a une centaine de pages de documentation). Cependant, si l'on se limite à des commandes simples (comme par exemple écrire un programme en C qui trace le dessin, avec simplement des points, des droites, et un peu de texte), la liste sommaire des commandes suffit (6 pages).

Xfig offre une autre solution. Après avoir inclu un fichier postscript généré par un autre programme, on peut rajouter du texte ou des commandes LATEX dessus. xfig -sp Sauvegarder comme "combined PS/LATEX"

Metapost est un langage graphique très proche de Metafont, mais qui génère du postscript. Il permet de produire des figures avec du texte et est bien interfacé avec TEX (Knuth l'utilise). Metapost est integré dans web2c 7.0. Metapost a déjà été porté sous MS-DOS et Mac (CMacTEX). Pour plus d'informations, vous pouvez consulter la page de D. Roegel : http://www.loria.fr/~roegel/metapost.html.

pstoedit permet également de visualiser des fichiers postscript (sans bitmaps) et d'ajouter du texte ou des figures par dessus.

8.6  Comment réaliser des captures d'écran ?

Un outil très utile pour effectuer des captures d'écran sous Unix est xv. xv est un éditeur d'images écrit par J. Bradley disponible sous unix. xv est capable de gérer différents formats d'image (encodage PS, GIF, JPEG, TIFF,...). Il permet de visualiser des images et aussi de réaliser des captures d'écran, soit partielles définies à la souris, soit par fenêtre X Window. Il suffit ensuite de sauvegarder la saisie d'écran de xv en format postscript et de l'appeler sous LATEX.

xv est accessible par ftp sur ftp://ftp.lip6.fr/.

Sur PC il y a pcxdump disponible sur http://micros.hensa.ac.uk/. Le package 'verbtext', disponible sur CTAN, permet ensuite d'appeler la saisie réalisée.

De même, le package 'scrdumps' et l'utilitaire scr2tex.exe sous DOS permettent d'inclure des saisies d'écran dans un document LATEX.

8.7  Comment tracer une courbe ?

Xgraphic est un outil de tracé de courbes simple d'utilisation (les options sont accessibles en interactif) mais limité aux courbes 2D. Il est disponible avec une doc française à http://blanche.polytechnique.fr/ et ftp://barbes.polytechnique.fr/pub/Xgraphic/.

xmgr marche également très bien.

GNUplot est disponible sous Unix, sous Dos, sous Windows et sous macOS (http://www.ee.gatech.edu/users/schooley/gnuplot.html). Il possède une sortie LATEX (eepic). Il permet de tracer des courbes (2D et 3D) à partir de valeurs ou d'une fonction. Les formats de sortie sont LATEX ou postscript.

En outre, sous Unix, il est possible de récupérer des fichiers GNUplot exportés par

        set terminal fig 
        set output "graph.fig"
puis de les modifier.

La FAQ GNUplot est disponible à http://www.uni-karlsruhe.de/~ig25/gnuplot-faq/.

Xy-pic compatible plain TEX, LATEX2.09 et permet de tracer des courbes, de réaliser des diagrammes commutatifs, des automates, et plein d'autres choses. Pour plus de renseignements, consulter :

  1. ftp://ftp.mpce.mq.edu.au/pub/maths/TeX/ ou
  2. http://www.mpce.mq.edu.au/~ross/Xy-pic.html ou
  3. http://www.brics.dk/~krisrose/Xy-pic.html
Le package 'xypic' est disponible sur ftp://ftp.fdn.org/pub/CTAN/macros/generic/diagrams/xypic/.

Sur PC grapher et surfer permettent également de tracer des courbes et des surfaces (ils sont indépendants de LATEX, mais une saisie d'écran est toujours possible).

Le package 'curves' disponible sur ftp://ftp.fdn.org/pub/CTAN/macros/latex/contrib/supported/curves/ permet de définir des courbes dans l'environnement picture.

Mathematica est également utilisable pour tracer des courbes 2D et 3D.

8.8  Comment est géré le positionnement des figures ?

Comme mentionné précédemment (), il existe différentes options de placement des figures. Les plus classiques sont

\begin{figure}[htbp] 
pour laisser à LATEX la possibilité de placer les figures suivant ses critères de beauté. (h) impose si possible le placement de la figure à l'appel de la macro ci-dessus. Sinon la figure sera placée en haut de la page suivante (t) ou en bas (b), voire sur une page seule (p).

En revanche, pour forcer (dans la mesure du possible) LATEX à placer une figure là où elle a été appelée, il faut utiliser le package 'float', de A. Lingnau, (\usepackage{float}) et l'option H (\begin{figure}[H]). Ce package permet de définir un tel placement par défaut via la commande \floatplacement{figure}{H}. Il est disponible sur ftp://ftp.fdn.org/pub/CTAN/macros/latex/contrib/supported/float/. Voir la question pour un exemple d'utilisation de ce package.

De même l'utilisation du caractère ! devant une option de placement permet de forcer à effectuer son placement au plus tôt (suivant l'option choisie).

Remarque (D.Barbier) :
si un flottant ne peut pas être placé avec les paramètres indiqués , celui-ci et tous les flottants qui viennent après seront mis à la fin du chapitre (en fait, ils seront insérés grâce à la commande \clearpage).
Le package 'placeins' de D. Arsenau permet de mettre des garde-fous dans le texte. Par exemple, lorsqu'on place une figure en dur avec une commande du style :

%%%% debut macro %%%% 
\makeatletter 
\newenvironment{figureH}{\begin{center} 
   \newcommand{\@captype}{figure}}{\end{center}} 
\makeatother 
%%%% fin macro %%%%
l'utilisation de \FloatBarrier juste avant, permet de vider le buffer de flottants actuellement stockés.

8.9  Comment placer une légende à côté d'une figure ?

Pour placer une légende à côté d'une figure, il faut utiliser l'environnement minipage.

Exemple (extrait du cahier GUTenberg 22 pour l'article sur esperluette) :
 
mettre dans le préambule :

%%%% debut macro %%%% 
\newlength\jataille 
\newcommand{\figgauche}[3]% 
{\jataille=\textwidth\advance\jataille by -#1 
\advance\jataille by -.5cm 
\begin{minipage}[c]{#1} 
   \includegraphics[width=#1]{#2} 
\end{minipage}\hfill 
\begin{minipage}[c]{\jataille} 
   \footnotesize #3 \normalsize 
\end{minipage}} 
%%%% fin macro %%%%
puis utiliser :

\figgauche{5cm}{totor.eps}{Titre.}
On peut également utiliser des parbox.

Exemple :
 
\begin{figure} 
\parbox{7cm}{...figure}\parbox{7cm}{\caption{---}} 
\end{figure} 
%%%% fin exemple %%%%

8.10  Comment insérer des figures dans multicol ?

On ne peut insérer que des figures de la largeur de la page, et pas de la lageur d'une colonne, donc des figure* (resp. table*). Un flottant n'apparaîtra pas sur la page où il est défini, mais, au mieux, sur la page d'après. Le positionnement [h] n'a donc pas de sens.

Exemple (avec le package 'graphicx') :
 
\begin{figure*} 
    \includegraphics[width=3cm]{totor.eps} 
\end{figure*} 
%%%% fin exemple %%%%
Remarque :
dans ce cas la gestion de \caption semble ne pas être correcte.

8.11  Comment faire apparaître toutes les figures en fin de document ?

Le package 'endfloat', disponible sur ftp://ftp.fdn.org/pub/CTAN/macros/latex/contrib/supported/endfloat/, permet de reporter toutes les figures en fin de document.

ATTENTION :
les classes AMS ne définissent pas les environnements figure* et table*. En outre, le package 'endfloat' a le mauvais goût de supposer qu'ils sont définis. Un remède est de faire :
            \documentclass[a4paper,11pt]{amsart} 
            \newenvironment{figure*}{\figure}{\endfigure} 
            \usepackage{endfloat}

8.12  Comment insérer des images Mathematica ?

8.13  Comment modifier la taille d'une bounding box ?

En format eps (encapsulated postscript), la `bounding box` permet de préciser la taille d'une image. La syntaxe est la suivante :

%%BoundingBox: 0 0 507 257
Les coordonnées correspondent dans l'ordre :

Les coordonnées verticales étant prises à partir du bas, et celles horizontales à partir de la gauche.

De plus ces coordonnées sont exprimées en points PostScript, c'est à dire en soixante-douzième de pouce, i.e.:

1 pt PS = 1/72 pouce = 2.54/72 cm.
Malheureusement, il arrive parfois que la taille de la bounding box soit supérieure à celle de la taille réelle du dessin qu'elle contient (il n'y a pas de mise à l'échelle). Le package 'boxedepsf', de L. Siebenmann, offre les commandes \Trim qui permettent de résoudre le probleme.

A. J. Carr a adapté ce package à . Son package s'appelle 'boxedeps'. Ce dernier est disponible sur ftp://ftp.fdn.org/pub/CTAN/macros/generic/boxed/ ou sur ftp://matups.math.u-psud.fr/pub/TeX/Graphics.dir/ArtIntegration.dir/boxedeps.dir/.

Exemple :
 
\TrimTop{15pct}\BoxedEPSF{toto} 
avec pct = pourcentage de la hauteur. 
%%%% fin exemple %%%%

8.14  Comment obtenir une figure avec un titre non numéroté ?

Il faut utiliser la commande \unnumberedcaption dont voici la définition :

%%%% debut macro %%%% 
\makeatletter    % <=== in a .sty file delete this 
  
\newcommand{\unnumberedcaption}% 
        {\@dblarg{\@unnumberedcaption\@captype}} 
  
\newcommand{\@unnumberedcaption}{}% undefined yet 
\long\def\@unnumberedcaption#1[#2]#3{\par 
  \addcontentsline{\csname ext@#1\endcsname}{#1}{% 
    % orig: \protect\numberline{\csname the#1\endcsname}% 
    %{\ignorespaces #2} 
    \protect\numberline{}{\ignorespaces #2}% 
    }% 
  \begingroup 
    \@parboxrestore 
    \normalsize 
    % orig: \@makecaption{\csname fnum@#1\endcsname}% 
    %{\ignorespaces #3}\par 
    \@makeunnumberedcaption{\ignorespaces #3}\par 
  \endgroup} 
  
% redefine \@makeunnumberedcaption (like \@makecaption) 
% for your own layout 
\newcommand{\@makeunnumberedcaption}[1]{% 
  \vskip\abovecaptionskip 
  \sbox\@tempboxa{#1}% 
  \ifdim \wd\@tempboxa >\hsize 
    #1\par 
  \else 
    \global \@minipagefalse 
    \hbox to\hsize{\hfil\box\@tempboxa\hfil}% 
  \fi 
  \vskip\belowcaptionskip} 
  
% for LaTeX 2.09 compatibility, define \above/belowcaptionskip: 
\@ifundefined{abovecaptionskip}{% 
  \newlength{\abovecaptionskip}% 
  \setlength{\abovecaptionskip}{10pt}% 
}{} 
\@ifundefined{belowcaptionskip}{% 
  \newlength{\belowcaptionskip}% 
  \setlength{\belowcaptionskip}{0pt}% 
}{} 
   

\makeatother    % <=== in a .sty file delete this 
%%%% fin macro %%%%

Remarque :
le package 'french' V3,49 inclut cette macro.

8.15  Comment redéfinir le style de caption ?

Par exemple pour changer la fonte de Figure : en gras, il faut utiliser :

%%%% debut macro %%%% 
\makeatletter 
\renewcommand{\fnum@figure}{\small\textbf{\figurenamethefigure}} 
\makeatother 
%%%% fin macro %%%%
Pour agir sur les espaces horizontaux de part et d'autre de la légende d'un flottant, on peut utiliser caption2.sty, et définir la longueur \captionmargin qui sera insérée à gauche et à droite de la légende.

8.16  Comment numéroter les figures en fonction des sections ?

On peut déjà remarquer que la numérotation se fait automatiquement en fonction des chapitres lorsque l'on utilise la classe de document 'book'. Par contre, ce n'est pas le cas lorsque l'on utilise la classe 'article'. Ainsi, pour obtenir des figures numérotées 1.1, 1.2, 1.3, ... dans la section 1, 2.1, 2.2, 2.3, ... dans la section 2, etc..., il suffit de redéfinir la commande \thefigure et de remettre le compteur de figures à zéro lorsque l'on change de section :

%%%% debut macro %%%%

\makeatletter

\renewcommand{\thefigure}{\ifnum \c@section>\z@ \thesection.\fi

\@arabic\c@figure}

\@addtoreset{figure}{section}

\makeatother

%%%% fin macro %%%%

Remarque
: pour renuméroter les sous-figures de la même manière (avec le package 'subfigure'), il suffit de rajouter la ligne \let\p@subfigure\thefigure dans le préambule.

8.17  Comment fondre une image dans du texte ?

Le package 'floatfig', de T. Kneser, disponible sur ftp://ftp.fdn.org/pub/CTAN/macros/latex/contrib/other/floatfig/, permet d'entourer une figure de texte de manière très efficace grâce à l'environnement floatingfigure. Ce package a été conçu pour LATEX2.09, pour des documents sans colonnes.

Exemple :
 
\begin{floatingfigure}[l]{4cm} 
   \includegraphics[width=4cm]{dessin} 
   \caption{Titre} 
\end{floatingfigure} 
%%%% fin exemple %%%%
Remarque :
le [l] du \begin{float...} sert à mettre la figure à gauche. [r] permet de l'avoir à droite, [p] la met à gauche sur une page de gauche et à droite sur une page de droite.
Le package 'floatflt', de T. Kneser et M. Dahlgren, disponible sur ftp://ftp.fdn.org/pub/CTAN/macros/latex/contrib/other/floatflt/, a été écrit pour . Il étend les possibilités de floatfig par de nombreuses options et est utilisable pour les figures et les tableaux.

Exemple :
 
\begin{floatingfigure}[options]{width of figure} 
   figure contents 
\end{floatingfigure} 
%%%% fin exemple %%%%
Le package 'picinpar' pour LATEX2.09 est disponible sur ftp://ftp.fdn.org/pub/CTAN/macros/latex209/contrib/picinpar/ et sur ftp://ftp.fdn.org/pub/CTAN/systems/msdos/4alltex/disk04/. Il permet de définir un nombre de lignes avant lequel la figure pourra être insérée dans le texte. La taille de la figure dépend de son contenu, sa position est variable, et peut s'étaler sur plusieurs paragraphes.

ATTENTION :
Ce package n'est pas compatible avec amstex.
Exemple :
 
\begin{window}[#lines before, l|r|c, picture contents, caption} 
   ... paragraph text ... 
\end{window} 
%%%% fin exemple %%%%
Mieux que le précédent, le package 'picins' disponible sur ftp://ftp.fdn.org/pub/CTAN/macros/latex209/contrib/picins/ permet d'inclure des figures dans des paragraphes.

Exemple :
 
\parpic(width,height)(x-off,y-off)[Options][Position]{Picture} 
    Paragraph text.... 
%%%% fin exemple %%%%
Le package 'wrapfig' permet de définir la hauteur de la figure, celle-ci ci peut apparaître à droite ou à gauche dans le texte ou encore dans une marge. Ce package n'est pas compatible avec les environnements de liste.

Exemple :
 
\begin{wrapfigure}[height in lines]{l|r}[overhang]{width} 
   {figure, caption etc.} 
\end{wrapfigure} 
%%%% fin exemple %%%%
Le package 'flow' met obligatoirement la figure dans une boîte avec un cadre et ne permet pas de définir de caption.

Exemple :
 
\flow[L|R]{paragraph text}{figure box} 
%%%% fin exemple %%%%
Le package 'window' de E. Schaluck, permet également d'intégrer une figure dans un paragraphe mais il n'est plus maintenu. Il a été écrit pour LATEX2.09 mais il est compatible .

Exemple :
 
\windowbox[toplines][inwindow: contents][ratio: l r] 
... paragraph text ... \par 
%%%% fin exemple %%%%
Voici les résultats d'un test comparatif effectué par P. van Oostrum :

 

A B C D E F G H I J K
picinipar + + + + + +
wrapfig + + + + + H - +
flow - - + +
floatfig + - - + + -
floatflt + + + + + - +
window - - + + + +
picins + - + + + +

Table 8.1: Résultat du test comparatif

 

avec :

A:
figure captions/counting/list of figures
B:
table captions/counting/list of tables
C:
Left/right possible (+ = both)
D:
Alternating left/right for twosided docs
E:
can be placed in the middle of text with twosided flowing
F:
possible to start after the beginning of paragraph
G:
can (more or less) float in the text
H:
auto detection of size of figure (H=height only)
I:
works with list environments
J:
works with twocolumn
K:
works with multicol
Les meilleurs packages semblent être : picins, floatflt et wrapfig.

Voici également une macro de C. Mercat :

%%%% debut macro %%%% 
\def\textdess#1#2 % #1={blabla} #2=\dessin{monbodessin} 
{\hbox{

 

\setbox2=\hbox{#2} %c'est le dessin

 

\count2=\wd2 %c'est la largeur du dessin. 
%\showthe\count2 
\multiply\count2 by-1 
\advance\count2 by\hsize %count2 vaut le reste de la page 
\advance\count2 by-2000000  %c'est pour la marge droite 
\count1=\count2 
\advance\count2 by-2000000  %c'est pour la marge du milieu 
\setbox1=\hbox to\count1sp{     %c'est le texte 
\vbox{\hsize=\count2sp  %c'est la largeur 
#1 
}\hfill} 
\count1=\ht1  %c'est la hauteur 
\advance\count1 \dp1

 

\count2=\ht2 
\advance\count0 \dp2

 

 

\ifnum\count2<\count1 \count2=\count1 \fi 
%c'est la + grde des 2 hauteurs 
\vtop to \count2sp {\vfill \box1 \vfill} 
\vtop to \count2sp {\vfill \box2 \vfill} 
}} 
%%%% fin macro %%%%

8.18  Comment réaliser des diagrammes en bâtons ?

Il existe le package 'bar', disponible sur ftp://ftp.fdn.org/pub/CTAN/macros/latex209/contrib/misc/, qui offre un environnement barenv.

8.19  Comment faire un organigramme ?

Il existe plusieurs contributions (toutes assez anciennes), disponibles sur CTAN, dédiées spécifiquement à la réalisation de différents types d'organigrammes :

  1. ftp://ftp.fdn.org/pub/CTAN/macros/latex209/contrib/nassflow/
  2. ftp://ftp.fdn.org/pub/CTAN/macros/latex209/contrib/rail/
  3. ftp://ftp.fdn.org/pub/CTAN/support/flow/
A priori, le dernier est le plus évolué et est basé sur un pré-processeur écrit en langage C, ce qui fait qu'il offre une interface souple et puissante.

8.20  Comment centrer une figure très large ?

Le package 'bigcenter' ci-dessous permet de centrer des figures très larges sans message d'erreur de type overful.

%%%% debut macro %%%% 
%%% ----------debut de bigcenter.sty--------------

 

%%% nouvel environnement bigcenter 
%%% pour centrer sur toute la page (sans overfull)

 

\newskip\@bigflushglue \@bigflushglue = -100pt plus 1fil

 

\def\bigcenter{\trivlist \bigcentering\item\relax} 
\def\bigcentering{\let\\\@centercr\rightskip\@bigflushglue% 
\leftskip\@bigflushglue 
\parindent\z@\parfillskip\z@skip} 
\def\endbigcenter{\endtrivlist}

 

%%% ----------fin de bigcenter.sty-------------- 
%%%% fin macro %%%%

8.21  Comment passer de ps à eps ?

S. Heiden :
Pour faire ce travail de transformation il faut interpréter TOUT le code PostScript d'un fichier PS (être une imprimante PS en gros), identifier la boîte englobante de ce qui est composé sur UNE page et insérer vers le début ou la fin du fichier PS original la fameuse ligne de la forme :
        %%BoundingBox: 72 246 522 597
qui renseigne les macros graphics, donc dvips, donc l'imprimante sur cette fenêtre de clipping.

Syntaxe :
 
%%BoundingBox: llx lly urx ury 
Qui sont deux points (lower left)(x,y) et (upper right)(x,y) définissant la fenêtre. Si on n'est pas, soi-meme, une imprimante PostScript on peut faire interpréter le code PS original par une imprimante qui produira une page. On peut alors mesurer la boîte englobante et la position de la figure dans la page (la portion de toner qui nous intéresse) et composer la ligne %%BoundingBox en s'exprimant en points d'1/72 de pouces et sachant que le système de coordonnées est :

En effet, une imprimante n'imprime jamais sur le bord réel d'une page. Il y a une fenêtre de clipping systématique matérielle plus petite que la page A4 et centrée. L'origine du système correspond au coin de la feuille ; le décalage du système de coordonnées arrive surtout sur les coordonnées Y qui dépendent de l'avancement du papier contrairement aux X, calés par les bords (qui n'a pas connu de vieille imprimante LaserWriter pouvant louper de 5 cm le bord inférieur d'une feuille A4 ?-).

Si on fait calculer et imprimer la bbox par l'imprimante (par le code de bbfig par ex.), ce problème ne se pose plus. Sans parler de ps2epsi qui est la meilleure solution mais alors il faut distinguer l'environnement de chaque système d'exploitation (Unix, Mac, PC) et les outils correspondants.

Remarque :
Si on a plus d'une figure, que l'on s'intéresse à la forêt amazonnienne où que l'on est un peu riche, on peut acheter Adobe Distiller ou autre pour faire ce genre de travail. Par ailleurs Ghostview peut jouer le rôle de l'imprimante si l'écran est correctement calibré (à vérifier avec xdpyinfo et une règle...). En gros il faut un outil qui sait composer du PostScript. Après, tout dépend de ce qu'il propose.
Suggestions :
 
Le document "Using EPS graphics in latex2e documents" de K. Reckdahl répond également à cette question cf section Converting PS files to EPS.

ps2epsi, qui est livré avec GhostScript permet de convertir du postscript en postscript encapsulé.

8.22  Comment changer l'orientation d'une figure ?

Le package 'rotfloat' disponible sur ftp://ftp.fdn.org/pub/CTAN/macros/latex/contrib/supported/rotfloat/ permet de gérer la rotation des flottants.

On peut utiliser la commande \rotatebox du package 'graphics' de D. Carlisle. Ce package est disponible sur ftp://ftp.fdn.org/pub/CTAN/macros/latex/required/graphics/.

Exemple :
 
\rotatebox{90}{ 
\includegraphics{dessin1.eps} 

%%%% fin exemple %%%%
La commande \includegraphics du package 'graphicx' peut prendre comme paramètre un angle de rotation.

Exemple :
 
\begin{figure} 
   \centering 
      \includegraphics[angle=90,width=\textwidth]{dessin.eps} 
\end{figure} 
%%%% fin exemple %%%%

8.23  Comment gérer des sous-figures sur plusieurs pages ?

Un seul environnement figure ne peut pas s'étendre sur plusieurs pages, il faut donc gérer le problème à la main en instanciant les numéros "à la main".

Exemple :
 
\documentclass{article} 
\usepackage{graphicx,subfigure} 
\begin{document}

 

  \begin{figure} 
    \centering 
    \subfigure[First Part]{% 
       \label{fig:graphics:a}% label for subfigure 
       \includegraphics[width=\textwidth]{box.eps}}% 
    \caption{Large Graphics}% 
    \label{fig:graphics}% label for figure 
  \end{figure}

 

  \addtocounter{figure}{-1} 
  \begin{figure} 
    \addtocounter{subfigure}{1} 
    \centering 
    \subfigure[Second Part]{% 
       \label{fig:graphics:b}% label for subfigure 
       \includegraphics[width=\textwidth]{box.eps}}% 
    \caption{Large Graphics (con't)}% 
  \end{figure}

 

\end{document} 
%%%% fin exemple %%%%

8.24  Comment générer une liste des figures d'un document ?

Il existe pour cela la commande \listoffigures. Elle collecte tous les numéros de figures ainsi que leur titre et le numéro de la page ou elles apparaissent.

Remarque :
une bonne mise à jour de cette liste nécessite au moins deux compilations successives.

8.25  Comment faire une figure sous LATEX ?

On distingue deux techniques majeures :

  1. soit on utilise un logiciel de dessin et on inclut une figure sous un format défini (en général postscript ou encapsulated postscript). Pour une liste de logiciels, voir la question .
  2. on utilise les commandes LATEX disponibles. L'environnement picture permet ainsi de composer des figures comprenant du texte, des segments de droites, des cercles et des figures géométriques.
Les commandes LATEX les plus courantes pour le dessin sont :

Exemple :
 
\begin{picture}(largeur, hauteur) 
   \put(0,0){\line(1,0){3}} 
   \put(0,3){\line(1,1){4}} 
\end{picture} 
%%%% fin exemple %%%%
Le package epic définit un ensemble de commandes plus complet que l'ensemble par défaut de LATEX. Il s'agit des commandes : \multiputlist, \matrixput, \grid, \dottedline, \dashline, \drawline, \jput, \picsquare et \putfile, ainsi que des environnements : dottedjoin, dashjoin et drawjoin.

Exemple :
 
\matrixput(0,0)(10,0){6}(0,10){3}{\circle{4}} 
\matrixput(2,0)(10,0){5}(0,10){3}{\line(1,0){6}} 
\matrixput(0,2)(10,0){6}(0,10){2}{\line(0,1){6}} 
%%%% fin exemple %%%%
PSTricks de T. Van Zandt propose un ensemble très complet de macros TEX qui permettent de jouer avec des dessins (dessin, couleur, rotation, superposition, ...). L'ensemble PSTricks est composé de plusieurs packages : pstricks, pst-plot, multido, pst-node, pst-coil, gradient, colortab, textpath, charpath et pst2eps. Une documentation d'une centaine de page est également fournie et nécessaire.

ATTENTION :
PSTricks fait appel à des macros postscript qui ne sont donc pas toujours visibles dans les previewer de .dvi.
Exemple :
 
\psellipse(.5,0)(1.5,1)

 

\parametricplot[plotstyle=dots,plotpoints=13]% 
        {-6}{6}{1.2 t exp 1.2 t neg exp}

 

\psshadowbox{\textbf Grest!!}

 

\pszigzag[coilarm=.5,linearc=.1{<->}(4,0) 
%%%% fin exemple %%%%

8.26  Comment mettre un commentaire à côté d'une figure ?

Pour mettre un commentaire à côté d'une figure et pour que le texte soit centré verticalement par rapport à la figure, il faut utiliser des \parbox.

Exemple :
 
\begin{tabular}{lc} 
$M=0{,}01$ & \parbox[c]{12cm}{\includegraphics[width=12cm]{fig6a.eps}} \\ 
$M=0{,}1$  & \parbox[c]{12cm}{\includegraphics[width=12cm]{fig6b.eps}} 
% etc... 
\end{tabular} 
%%%% fin exemple %%%%

Chapter 9
Inclusion de fichiers

Il est parfois intéressant de pouvoir inclure un fichier de texte complet à l'intérieur d'un document LATEX. Par exemple le code source d'un programme, ou encore un fichier de dessin au format LATEX.

9.1  Comment inclure des fichiers en mode verbatim ?

Le package 'verbatim', de R. Schopf, permet via la commande \verbatiminput, qui prend en argument un nom de fichier, d'inclure un fichier en mode verbatim. Ce package est disponible sur ftp://ftp.fdn.org/pub/CTAN/macros/latex/distribs/ ou ftp://ftp.fdn.org/pub/CTAN/macros/latex/required/tools/.

L'environnement alltt du package du même nom (package dû à J. Braams) permet la même prouesse tout en gardant active l'interprétation des commandes LATEX dont le nom commence par le caractère \ (le "backslash" reste actif). Il est disponible dans sur ftp://ftp.fdn.org/pub/CTAN/macros/latex/contrib/misc/ ou sur ftp://ftp.fdn.org/pub/CTAN/macros/latex/base/.

Exemple :
 
\begin{alltt} 
   Notez la différence subtile entre $f(x)$ et \(f(x)\) grâce à 
   l'utilisation du "backslash". 
\end{alltt} 
%%%% fin exemple %%%%
Les commandes \listinginput et \verbatimtabinput du package 'moreverb' (disponible sur ftp://ftp.fdn.org/pub/CTAN/macros/latex/contrib/other/misc/) permettent d'inclure des documents en mode verbatim avec ou sans numérotation des lignes du fichier inclu.

Exemple :
 

\small 
\listinginput[5]{10}{totor.c} 

Les paramètres [5] et {10} indiquent que la numérotation des 
lignes doit se faire de 5 en 5 en commençant à 10. 
%%%% fin exemple %%%%
Il existe aussi le package 'verbtext' disponible sur CTAN.

Le package 'fancyvrb' est également disponible sur CTAN.

Pour insérer du code lisp dans un document LATEX, il existe le package 'lispcode' disponible par ftp sur ftp://ki-server.informatik.uni-wuerzburg.de/pub/tex/.

On peut aussi essayer le package 'verbasef' (verbatim automatic segmentation of external files) disponible sur CTAN. Il utilise l'environnement figure.

Encore un autre, le package 'cprog' disponible sur ftp://ftp.fdn.org/pub/CTAN/macros/latex209/contrib/misc/ permet d'inclure des morceaux de code dans un document et de les gérer comme des flottants.

Le package 'sverb' de M. Wooding propose un environnement listing.

9.2  Comment gérer un document par parties ?

Pour travailler sur un gros document, il est plus agréable de le découper en plusieurs fichiers. Il y aura quoiqu'il en soit un fichier principal (celui qui comprend le préambule et les commandes \begin{document} et \end{document}).

\input{fichier} permet d'inclure le fichier "fichier" dans le fichier principal. Cette commande réalise une importation pure et simple. Elle est plutôt réservée à l'importation de fichiers de macros.

\include{chapitre} permet d'intégrer le fichier "chapitre.tex" dans le document principal en commençant une nouvelle page. Cette commande réinitialise la numérotation des titres. Utilisée avec la commande \includeonly{chapitre}, cette commande mise dans le préambule, permet de ne recompiler le fichier principal que sur les parties indiquées.

Remarque :
la commande \include ne permet pas d'intégrer un fichier contenant lui-même une commande \include.
Exemple :
 
\documentclass{report} 
\includeonly{chap1, chap3} 
\begin{document} 
\include{chap1} 
\include{chap2} 
\include{chap3} 
\end{document} 
%%%% fin exemple %%%%
Remarque :
avec le package 'french', pour que la numérotation des chapitres ne soit pas réinitialisée lorsqu'on change de partie, il faut ajouter la commande \noresetatpart en début de document après le \begin{document}

9.3  Comment isoler une partie d'un fichier ps ou dvi ?

Il faut utiliser un outil qui permet de découper en page :

9.4  Comment inclure un fichier PICT ?

On peut utiliser la commande \special{pictfile mondessin.pict} ou \put(0,0){\special{pict=Mondessin}}.

9.5  Comment spécifier un chemin pour les fichiers à inclure ?

Il suffit d'utiliser la commande \import{chemin}{fichier}. Elle évite de préciser le chemin où se trouvent les fichiers appelés par la commande \input à chaque appel.

%%%% debut macro %%%% 
% import.sty: allows input of a file from another directory: 

\import{path}{file}

 

\def\import{\begingroup 
  \protected@edef\@tempa{\endgroup 
    \noexpand\@import{\@ifundefined{input@path}{}{\input@path}}% 
    {\@ifundefined{Ginput@path}{}{\Ginput@path}}}% 
  \@tempa} 
\def\@import#1#2#3#4{% 
  \def\input@path{{#3}#1}\def\Ginput@path{{#3}#2}% 
  \input{#4}% 
  \def\input@path{#1}\ifx\input@path\@empty \let\input@path\@undefined \fi 
  \def\Ginput@path{#2}\ifx\Ginput@path\@empty \let\Ginput@path\@undefined  
\fi 

%%%% fin macro %%%%

Exemple :
 
\import{Chapitres/}{chapitre1} 
%%%% fin exemple %%%%
Pour les figures, la package 'graphicx' propose la commande \graphicspath à utiliser en tête de document.

Exemple :
 
\graphicspath{{Chapitre1/gfx/}{Chapitre2/gfx/}} 
%%%% fin exemple %%%%
On peut également modifier certaines variables d'environnement lorsqu'elles existent.

Exemple (UNIX tcsh) :
 
setenv TEXINPUTS /home/figures/:"$TEXINPUTS" 
%%%% fin exemple %%%%

Chapter 10
Hauts et bas de pages

Une jolie mise en page passe par la réalisation des entêtes et des pieds de pages. On peut y intégrer les numéros de pages, des informations concernant l'auteur, un renvoi du titre du chapitre ... bref tout ce qui donne un aspect professionnel à votre document.

10.1  Comment définir les hauts et bas de page ?

Remarque :
lorsque le haut ou bas de page défini est trop grand on voit apparaître des messages d'erreur du style "Overfull \vbox". Il faut alors redimensionner la longueur correspondante.
Exemple
(dans le préambule) :
           \addtolength{\headheight}{1.5pt} 
           %%%% fin exemple %%%%
Par défaut, LATEX offre la numérotation des pages en bas de page (style plain). Mais, il propose également 3 autres styles de mise en page. Il s'agit des styles :

  1. empty (hauts et bas de pages vides),
  2. headings(la numérotation des pages apparaît en haut ainsi que différentes informations suivant la classe de document), et
  3. myheadings (les commandes \markboth et \markright permettent de définir les informations qui devront apparaître dans le haut de page. \markboth{entête gauche}{entête droite} s'utilise pour un document recto-verso alors que \markright{entête} s'applique à toutes les pages d'un document en simple recto).
L'appel d'un style pour tout le document se fait via la commande \pagestyle{style}. La commande \thispagestyle{} permet d'appeler un style sur une page particulière.

Remarque :
malgré une déclaration globale de style de page, il se peut que des déclarations locales de style soient également nécessaires, puisque certaines commandes LATEX réinitialisent le style de la page sur laquelle elles apparaissent.
Le package 'fancyheadings', de P. van Oostrum, est disponible sur ftp://ftp.fdn.org/pub/CTAN/macros/latex209/contrib/fancyheadings/. Il est compatible LATEX2.09 et . Il permet de définir des entêtes et des pieds de page relativement facilement. La définition des pages spéciales se fait par : \thispagestyle{xxx} ou xxx peut être fancy (utilise les définitions ci-dessous sur une page en respectant le style plain pour les autres ), plain (style TEX) ou fancyplain(permet de redéfinir le style plain et donc de disposer de deux styles : fancy et plain). L'application d'un style à toutes les pages d'un document s'obtient par \pagestyle{nom_style}.

Les macros principales sont :

On distingue les styles fancy et plain par :

\lhead[\fancyplain{paire gauche plain}{paire gauche fancy}] 
{\fancyplain{paire gauche plain}{paire gauche fancy}}
Le style par défaut est alors plain.

Ce package définit quatre nouvelles longueurs :

  1. \headrulewidth
  2. \footrulewidth
  3. \plainheadrulewidth
  4. \plainfootrulewidth
Exemple :
 
\documentclass{article} 
\usepackage{fancyheadings} 
\pagestyle{fancy} 
\usepackage{graphicx} 
\renewcommand{\sectionmark}[1]{\markboth{#1}{}} 
\renewcommand{\subsectionmark}[1]{\markright{#1}} 
\rfoot{\leftmark\\\rightmark} 
\lhead{\includegraphics[width=0.5cm]{foobar.ps}}

 

\begin{document}

 

   \section{Première section.} 
      Intro. 
      \subsection{Sous-section.} 
         Texte. 
\end{document} 
%%%% fin exemple %%%%

Remarque :
sur certaines pages où le style de page est réinitialisé par certaines commandes (telles que \tableofcontents), il faut repréciser le style voulu via la commande \thispagestyle.
Exemple :
 
\addtocontents{toc}{\protect\thispagestyle{fancyplain}} 
%%%% fin exemple %%%%
Le package 'fancyhdr' est le successeur du package 'fancyheadings' pour . Il est disponible sur ftp://ftp.fdn.org/pub/CTAN/macros/latex/contrib/supported/fancyhdr/.

10.2  Comment obtenir une note de bas de page ?

La commande \footnote{texte} permet très facilement d'obtenir des notes de bas de page sans se préoccuper de leur mise en page.

Le package 'ftn' disponible sur CTAN dans ftp://ftp.fdn.org/pub/CTAN/macros/latex209/contrib/misc/ permet d'étendre la commande footnote à tous les environnements non flottants.

10.3  Comment mettre les notes de bas de page en fin de document ?

Le package 'endnotes', de J. Lavagnino, disponible sur CTAN permet de faire cela.

10.4  Comment réduire les rappels de titres dans un haut ou bas de page ?

Pour éviter que des titres trop longs n'apparaissent dans les entêtes ou les pieds de pages, il suffit de passer un titre plus court en option des commandes de structuration de document.

Remarque :
dans ce cas, ce sont les titres courts qui apparaîtront dans la table des matières.
Exemple :
 
\documentclass{article} 
\usepackage{fancyheadings} 
\pagestyle{fancy} 
\usepackage{graphicx} 
\renewcommand{\sectionmark}[1]{\markboth{#1}{}} 
\renewcommand{\subsectionmark}[1]{\markright{#1}} 
\rfoot{\leftmark\\\rightmark} 
\lhead{\includegraphics[width=0.5cm]{foobar.ps}}

 

\begin{document}

 

   \section[Titre résumé.]{Un titre trop long pour les rappels.} 
      Intro. 
      \subsection{Sous-section.} 
         Texte. 
\end{document} 
%%%% fin exemple %%%%

10.5  Comment référencer une note de bas de page ?

Il suffit simplement de définir un label à l'intérieur de la commande \footnote et de la référencer par \ref.

Exemple :
 
bla bla bla\footnote{Notons que ce bla l\`a est diff\'erent des 
pr\'ec\'edents\label{footnote-bla}}...

 

[...]

 

..., comme l'indique la noteref{footnote-bla}, ... 
%%%% fin exemple %%%%

10.6  Comment supprimer la numérotation des pages ?

Pour supprimer la numérotation des pages, il faut utiliser la commande \pagestyle{empty}. Si elle ne suffit pas, il faut en plus utiliser \thispagestyle{empty} sur les pages où la numérotation subsiste. Ce comportement se justifie par le fait que certaines commandes comme \tableofcontents ou \chapter réinitialisent le style de page sur laquelle ils apparaissent.

On peut également redéfinir le style plain à empty \let\ps@plain=\ps@empty.

Le package 'nopageno' de D. Carlisle, disponible sur ftp://ftp.fdn.org/pub/CTAN/macros/latex/contrib/supported/carlisle/, de supprimer la numérotation de toutes les pages même celles qui réinitialisent le style de page.

10.7  Comment numéroter les pages par rapport à la dernière ?

Pour pouvoir référencer les pages d'un document par rapport à la dernière page (e.g. page 54/345), il faut utiliser le package 'lastpage' (disponible sur ftp://ftp.fdn.org/pub/CTAN/macros/latex/contrib/other/lastpage/).

Exemple :
 
\documentclass{report} 
\usepackage{french} 
\usepackage{lastpage}

 

\makeatletter 
\renewcommand{\@evenfoot}% 
        {\hfil \upshape page {\thepage} de \pageref{LastPage}} 
\renewcommand{\@oddfoot}{\@evenfoot} 
\makeatother

 

\begin{document} 
Texte. 
\end{document} 
%%%% fin exemple %%%%

Si l'on ne dispose pas du package 'lastpage', on peut définir ce label "à la main" en ajoutant \label{LastPage} juste avant \end{document} ou encore en utilisant la commande \AtEndDocument{\label{LastPage}} dans le préambule du document.

10.8  Comment supprimer le trait de séparation des notes de bas de page ?

Il suffit de mettre dans le préambule :

\renewcommand{\footnoterule}{}

10.9  Comment modifier la numérotation des pages ?

Il faut modifier la commande \thepage.

Exemple (P. van Oostrum) :
 
\documentclass{report} 
\usepackage{french}

 

\makeatletter 
\renewcommand{\thepage}{\thechapter-\arabic{page}} 
% chapter-page numbering 
\@addtoreset{page}{chapter} 
% reset page number when chapter is stepped 
% The next magic makes the page counter be reset to one rather 
% than zero 
\def\@stpelt#1{\global\csname c@#1\endcsname 
               \expandafter\ifx \csname#1\endcsname \page 
                  \@ne 
               \else 
                  \z@ \fi} 
\makeatother 
\begin{document} 
\chapter{Introduction} 
Texte. 
\end{document} 
%%%% fin exemple %%%%

Il existe également le package ci-dessous de A. Kielhorn.

%%%% debut macro %%%% 
%% 
%% This is page-per-chapter-package 
%% version 2.0 
%% 
%% Don't use it with refrep! 
%% Refrep has these commands already implemented 
%% 
%% This version forces openright!! 
%% 
%% Index-commands should work in chapters and appedices, 
%% they will not work as expected in the preface when the 
%% pagenumbering is not arabic. (MakeIndex can't sort roman- 
%% numbers) 
%% 
%% Bugs: 
%%       The index is sortet according to the pagenumber 
%%       without looking at the chapternumber. 
%%       I don't think MakeIndex could handle that. 
%% 
  
\NeedsTeXFormat{LaTeX2e} 
\ProvidesPackage{pagepc}[1995/05/13] 
  
\@ifundefined{chapter} 
{\PackageError{pagepc}{% 
  You can't number your pages per chapter\MessageBreak when you 
  have no chapters 
  }{% 
  Use ``report'' or ``book'' instead. 
  } 
}% 
{} 
  
%% Reset the pagecounter to 1 at the start of a new chapter 
%% 
\let\ppchapter=\@chapter 
\def\@chapter{\if@pageperchapter\setcounter{page}{1}\fi 
                    \ppchapter} 
  
%% Force a pagebreak at the start of the appendix, otherwise 
%% the number of the page right before the appendix comes 
%% out wrong 
%% 
\let\ppappendix=\appendix 
\def\appendix{\if@pageperchapter\newpage\fi\ppappendix} 
  
\newif\if@pageperchapter \@pageperchapterfalse 
  
%% This command enables Page-per-Chapter, it is *not* on by 
%% default to allow roman pagenumbers in the preface 
%% (see sample-document) 
%% 
\newcommand{\pageperchapter} 
    {\@pageperchaptertrue 
     \@openrighttrue 
     % Remember old setting for chapter 0 = preface 
     \let\ppthepage=\thepage 
     % The new number needs more space 
     \renewcommand\@pnumwidth{2.55em} 
     % Here it comes :-) 
     \renewcommand\thepage{% 
       \ifnum \c@chapter = \z@ 
          \ppthepage 
       \else 
          \thechapter\ -- \arabic{page} 
       \fi 
       } 
     } 
  
%% This is a hack to make MakeIndex happy :-( 
%% You can't use the |-form of an indexentry because 
%% it is used to store the chapternumber. 
%% 
\def\@wrindex#1{% 
   \ifnum \c@chapter = \z@ 
      \protected@write\@indexfile{}% 
      {\string\indexentry{#1}{\arabic{page}}}% 
      % The above is wrong if pagenumbering!=arabic, 
      % but I think this is better than nothing. 
   \else 
       \protected@write\@indexfile{}% 
      {\string\indexentry{#1|ppc{\thechapter}}{\arabic{page}}}% 
   \fi 
 \endgroup 
 \@esphack 

  
%% This prints the pagenumber in the index 
%% 
\def\ppc#1#2{#1 -- #2} 
  
\endinput 
%%%% fin macro %%%%
Pour changer la valeur du compteur de page, il suffit d'utiliser :

\setcounter{page}{17}.

10.10  Comment supprimer les entêtes et bas de page de pages vierges ?

Lorsqu'on utilise l'option 'openright' pour faire débuter un nouveau chapitre sur une page de droite dans un document recto-verso, pour ne pas afficher les entêtes et bas de page sur une page de gauche restée vierge, on peut utiliser la commande suivante (cf. LATEX Companion) :

\newcommand{\clearemptydoublepage}{% 
        \newpage{\pagestyle{empty}\cleardoublepage}}
Il est alors nécessaire d'utiliser cette commande avant la commande \chapter.

10.11  Comment gérer des en-têtes avec des environnements verbatim multi-pages ?

Le problème consiste à inclure un fichier avec la commande \input dans un haut ou bas de page alors que le corps du document comprend des environnements verbatim qui s'étalent sur plus d'une page. Dans ce cas, LATEX au lieu d'interpréter la commande \input fait un copier-coller du contenu du fichier appelé par la commande \input (c'est comme s'il était en mode verbatim au moment de traiter le \input).

La solution consiste à utiliser une boîte via \savebox.

Remarque :
cela ne marche que si les informations à imprimer ne varient pas au fil des pages.
Exemple :
 
\newsavebox{\logo} 
\savebox{\logo}[2cm][c]{\input{logo-late.pstex_t}} 
%%%% fin exemple %%%%

10.12  Comment utiliser \footnote dans un titre ?

Il faut utiliser la commande \protect.

Exemple :
 
   \section{Titre de section\protect\footnote{Commentaire sur 
      ce titre.}.} 
%%%% fin exemple %%%%
ATTENTION : le problème dans ce cas est que la note apparaîtra aussi bien dans l'en-tête si l'en-tête rappelle le titre des sections que dans la table des matières. Pour supprimer ces apparitions il faut alors utiliser la même technique que pour les titres résumés :

\section[foo]{foo\protect\footnote{foooo}}
Il existe également le package 'stblftnt' de R. Fairbairns disponible sur ftp://ftp.fdn.org/pub/CTAN/macros/latex/contrib/other/misc/ qui gère le problème énoncé ci-dessus sans avoir à réécrire tout le titre.

10.13  Comment placer les notes de bas de page les unes à côté des autres ?

Il faut utiliser le package 'footnote', de R. Fairbairns, disponible sur ftp://ftp.fdn.org/pub/CTAN/macros/latex/contrib/supported/footmisc/ avec l'option para :

\usepackage[para]{footmisc}.

10.14  Comment réinitialiser le compteur de note de bas de page à chaque page ?

Il faut utiliser le package 'footnote', de R. Fairbairns, disponible sur ftp://ftp.fdn.org/pub/CTAN/macros/latex/contrib/supported/footnote/ avec l'option perpage :

\usepackage[perpage]{footnote
Le package 'footnpag' disponible sur ftp://ftp.fdn.org/pub/CTAN/macros/latex/contrib/supported/footnpag/ permet également de faire cela.

10.15  Comment modifier le style des notes de bas de page ?

Le package 'footmisc' disponible sur ftp://ftp.fdn.org/pub/CTAN/macros/latex/contrib/supported/footmisc/ permet de définir son propre style de note de bas de page.

Le package 'ftnright' de F. Mittelbach redéfinit le placement des notes de bas de page dans un document multi-colonnes. Lors de l'utilisation de ce package, les notes de bas de page appelées dans une page seront réunies et placées à la fin de la dernière colonne de cette page.

Remarque :
Lorsque plusieurs packages sont utilisés simultanément dans un même document, ftnright doit, si possible, être appelé en dernier.

10.16  Comment utiliser le mode verbatim dans une note de bas de page ?

Par défaut, le mode verbatim n'est pas accessible dans une note de bas de page (et de manière générale inutilisable dans un argument d'une autre commande).

\DeclareRobustCommand\espacement{{\fontencoding{OT1} 
\selectfont \texttt{\char32}}}
\tt sous LaTeX2.09 
\texttt sous LaTeX2e

10.17  Comment éviter qu'une note de bas de page s'étale sur plusieurs pages ?

La commande \samepage peut permettre de forcer LATEX à ne pas couper le contenu d'une note de bas de page.

Exemple sans la commande :
 
Texte texte texte (1). Texte texte texte (2). Texte texte texte 
(3). Texte texte texte (4). Texte texte texte (5). Un autre 
texte\footnote{Un long long long long long long long long long 
long long long long long long long long long long long long long 
long long long long long long long long long long long long long 
long long long long long commentaire sur cet autre texte.}. 
Texte texte texte (6). Texte texte texte (7). Texte texte texte 
(8). Texte texte texte (9). Texte texte texte (10). Texte texte 
texte (11). Texte texte texte (12). Texte texte texte (13). 
Texte texte texte (14). Texte texte texte (15). Texte texte 
texte (16). Texte texte texte (17). Texte texte texte (18). 
Texte texte texte (19). Texte texte texte (20). Texte texte 
texte (21). Texte texte texte (22). 
%%%% fin exemple %%%%
Exemple avec :
 
Texte texte texte (1). Texte texte texte (2). Texte texte texte 
(3). Texte texte texte (4). Texte texte texte (5). Un autre 
texte\samepage\footnote{Un long long long long long long long 
long long long long long long long long long long long long long 
long long long long long long long long long long long long long 
long long long long long long long commentaire sur cet autre 
texte.}. Texte texte texte (6). Texte texte texte (7). Texte 
texte texte (8). Texte texte texte (9). Texte texte texte (10). 
Texte texte texte (11). Texte texte texte (12). Texte texte 
texte (13). Texte texte texte (14). Texte texte texte (15). 
Texte texte texte (16). Texte texte texte (17). Texte texte 
texte (18). Texte texte texte (19). Texte texte texte (20). 
Texte texte texte (21). Texte texte texte (22). 
%%%% fin exemple %%%%
On peut également utiliser une \parbox mais cela peut modifier la structure de la page (empiètement du bas de page).

Exemple :
 
\footnote{\parbox[t]{0.94\linewidth}{This is a footnote ...} 
%%%% fin exemple %%%%
L'emploi de la commande \enlargethispage peut également suffire.

Une solution plus globale (et non pas au cas par cas comme les solutions précédentes) consiste à redéfinir la pénalité associée à \interfootnotelinepenalty. Cela permet de relâcher le critère de beauté associé au placement des bas de page.

Exemple :
 
Mettre dans le préambule \interfootnotelinepenalty=10000. 
%%%% fin exemple %%%%
Remarque :
\raggedbottom permet en plus à LATEX de ne pas s'inquiéter si le corps de la note n'apparaît pas sur la même page que son appel.

Chapter 11
LATEX, le français et les autres langues

Bien que prévu au départ pour la langue de Shakespeare, TEX/LATEX permet de rédiger facilement dansnotre belle langue française.

11.1  Comment franciser un document LATEX ?

'babel' version 3.6 disponible sur ftp://ftp.fdn.org/pub/CTAN/macros/latex/required/babel/, est un package de J. Braams.

babel permet de composer des documents multilingues. Son appel sous se fait par

\usepackage[langue_1, langue_2, ..., langue_n]{babel}.
Parmi les langues actuellement disponibles, on peut citer english, german, italian, french, français. Le changement de langue se fait via la commande

\selectlanguage.
Exemple
:
\selectlanguage{spanish} 
%%%% fin exemple %%%%
La langue par défaut est la dernière de la liste passée en paramètre à babel (langue_n dans l'exemple précédent).

Ce package est le plus courant sur le plan international mais il est souvent contesté en France. On lui préfère alors le package 'french'.

Le package 'french' de B. Gaulle est disponible par ftp sur ftp://ftp.gutenberg.eu.org/pub/gut/french/ ou sur ftp://ftp.loria.fr/pub/ctan/language/french/.

Lors de l'installation, respectez les instructions d'installation accompagnant french.sty (une FAQ est également disponible). Le fichier des motifs de césure est désormais unique (frhyph.tex), que l'on travaille avec le codage de fontes T1 ou OT1 (avec option MlTEX).

Vous pouvez également utiliser la série de commandes suivantes, gentiment envoyées par Denis Barbier:

---------- citation ----------  
make -f Makefile.unx ALLTEXPATH=/usr/share/texmf \  
install-hyphen install-config install-base install-index texhash 
 

Puis modifier le fichier /usr/share/texmf/web2c/fmtutil.cnf et ajouter la ligne ci-dessous:  
frlatex tex language.dat -mltex frlatex.ini 
Taper ensuite fmtutil --byfmt frlatex texlinks 
La commande pour compiler du LaTeX avec french est alors frlatex.  
------------- fin -------------

Remarque :
Dans un fichier source (.tex), il est conseillé de charger french après tous les autres packages.
french prend notamment en charge la mise en page, la traduction des balises LATEX visibles dans le document final (Chapitre, Table des matières, ...), ....

ATTENTION :
ces deux options (french et babel) ne sont pas toujours parfaitement compatibles (les dernières versions le sont french3,49 et babel 3.6).
AL: A vérifier...

  1. \usepackage[francais]{babel} et \usepackage[frenchb]{babel} font appel à l'option frenchb maintenue par D. Flipo.
  2. \usepackage[french]{babel} fait appel au package french de B. Gaulle à condition que french ait été installé (et plus particulièrement french.ldf)
  3. les versions de tous ces packages sont très importantes (babel, frenchb, french) pour une bonne coexistence. Une version 3.6 beta de Babel est sur CTAN mais il paraît qu'il y avait un pb avec french. Babel 3.5e et french V3.46 collaborent assez bien dans tous les sens.
  4. les fichiers de césure sont les mêmes pour babel et pour french.
ATTENTION :
dernières distributions de french (V4 et post) ne sont plus compatibles LATEX 2.09 !

11.2  Comment corriger les coupures de mots accentués ?

Une solution consiste à utiliser MlTEX (multilingual TEX). C'est le moteur TEX de M. Ferguson. Il permet en particulier de gérer les coupures de mots accentués. Certaines des idées utilisées dans ce moteur ont d'ailleurs été reprises par la suite dans TEX V3.

Suivant la fonte avec laquelle on travaille (i.e. suivant qu'elle dispose des caractères accentués ou non), il peut exister des problèmes d'interaction entre les lettres accentuées et les règles de coupure des mots. En effet, l'utilisation d'une fonte 7 bits impose que les caractères accentués soient fabriqués par la macro \accent de TEX qui inhibe toute coupure pour la suite du mot.

En l'utilisation de fontes respectant la norme T1 (codage de Cork + 8 bits) permet d'éviter ces problèmes. Il faut alors faire appel au package 'fontenc' :

\usepackage[T1]{fontenc}
Ce package nécessite que des fontes encodées T1 (suffisamment récentes) aient été installées (fontes dc par exemple).

Il faut en outre utiliser des modèles de coupure de mot encodées T1. Il existe pour cela deux fichiers disponibles sur CTAN. Il s'agit des fichiers fr8hyph.dc (ou mieux f8hyph, beaucoup plus récent) pour un codage 8 bits (fonte avec caractères accentués telle que dc) et fr7hyph (respectivement f7hyph) pour un codage 7 bits (accents TEX). L'association GUTenberg propose également de tels fichiers (cf. french paragraphe ).

Il peut y avoir également un problème de versions, l'algorithme de coupure des mots a changé entre les versions 2.9 et 3.0. Ainsi si vous utilisez Tex V.3.0 ou plus, il faut veiller à ce que les fichiers plain.tex et lplain.tex soient également en version 3.0 ou plus.

11.3  Comment utiliser les lettres accentuées ?

Les accents disponibles sous LATEX sont présentés à la question .

Pour éviter ces commandes barbares d'accentuation des caractères, on peut utiliser une fonte contenant toutes les lettres accentuées nécessaires (code 8 bits). Si besoin, il faut préciser un codage d'entrée via l'instruction :

\usepackage[codage d'entrée]{inputenc}
L'option est rendue nécessaire par le fait que les codes de caractères au-delà de 127 sont différents sous Dos, MacOS et Unix... Ainsi, le codage à preciser est latin1 pour un système Unix ou un PC sous Windows, applemac sous Mac adapté au français ou encore cp850 pour le code-page 850 sur PC (sous MS-DOS). Il existe également ansinew, cp438, latin2 selon les systèmes.

Exemple :
 
\usepackage[applemac]{inputenc} 
%%%% fin exemple %%%%
ATTENTION :
inputenc.sty « casse » les efforts faits par mlTEX lorsqu'on utilise les fontes codées OT1 (fontes CMR ou PostScript).
mapcodes de M. Piotrowski autorise également un grand nombre de codages de caractères (iso8859-1 (latin1), iso8859-2 (latin2), ibm850 ou 852, hproman8, etc).

11.4  Comment composer du texte en grec moderne ou classique ?

Il suffit d'utiliser l'option greek du package babel disponible sur CTAN. Son site primaire est ftp://obelix.ee.duth.gr/pub/TeX/. La version 1.0 fournie avec babel ne supporte pas les accents ni les marques de respirations par contre la toute nouvelle version 1.1 les supporte.

On trouvera un fichier de césure de mots dans ftp://obelix.ee.duth.gr/pub/TeX/hyphen.greek.

11.5  Comment composer du texte en hébreu moderne ou classique ?

Il suffit d'utiliser le package ArabTEX 3.05 disponible sur CTAN. Son site primaire est ftp://ftp.informatik.uni-stuttgart.de/pub/arabtex/. Ce package permet la vocalisation mais pas les marques de cantillation.

Exemple :
 
\documentclass{article} 
\usepackage{arabtex,hebtex} 
\setcode{standard} 
\sethebrew

 

\begin{document} 
Voici du texte en hébreu 
\begin{arabtext} 
yOm tOb 
\end{arabtex} 
Fin du texte en hébreux 
\end{document}

%%%% fin exemple %%%%

11.6  Comment composer du texte en arabe ?

Il suffit d'utiliser le package ArabTEX 3.05 disponible sur CTAN. Son site primaire est ftp://ftp.informatik.uni-stuttgart.de/pub/arabtex/.

11.7  Comment définir les césures de groupes de mots conjugués avec un trait d'union ?

(B. Gaulle) Par défaut, TEX coupe tout mot composé à l'endroit exact du trait d'union et uniquement à cet endroit. Mais cela peut se modifier. Dans ce but, j'ai  ajouté à french (V4,02) une commande \allowhyphens qui permet de laisser à TEX le choix des autres coupures dans le mot.

Remarque :
la commande \hyphenation ne permet pas d'intégrer un trait d'union car ce caractère sert à indiquer un point de coupure et uniquement cela.
Voici donc la solution :

\showhyphens{{définissez\allowhyphens-le}
donne alors comme points de coupure : dé-fi-nis-sez-le

Chapter 12
Mathématiques

Ce pour quoi LATEX a été écrit : les mathématiques, c'est un domaine où il excelle.

12.1  Comment passer en mode mathématique ?

Pour les mathématiques en ligne, il faut utiliser :

  1. $ et $ ou
  2. \( et \) ou
  3. \begin{math} et \end{math}
Exemple :
 
Résoudre l'équation: 
$ 3 + 4 x = 0 $ 
%%%% fin exemple %%%%
ce qui donne :

 

Résoudre l'équation : 3 + 4 x = 0

 

Pour les formules isolées, il faut utiliser les modes :

  1. $$ et $$ ou
  2. \[ et \] ou
  3. \begin{displaymath} et  \end{displaymath}.
Exemple :
 
Résoudre l'équation: 
$$ 
3 + 4 x = 0 
$$ 
%%%% fin exemple %%%%
ce qui donne :

 

Résoudre l'équation :
3 + 4 x = 0

 

Avec l'environnement \displaystyle, on peut quand même avoir une formule de style isolée dans le texte mais cela modifie localement les interlignes.

Exemple
:
Texte texte texte texte texte texte texte texte texte texte 
texte texte texte texte texte texte texte texte texte texte 
texte texte texte texte texte texte texte texte texte texte 
$\displaystyle \sum_{i=0}n u_i $  texte texte texte texte 
texte texte texte texte texte texte texte texte texte texte 
%%%% fin exemple %%%%
ce qui donne :

 

Texte texte texte texte texte texte texte texte texte texte texte texte texte texte texte texte texte texte texte texte texte texte texte texte texte texte texte texte texte texte åi=0n ui texte texte texte texte texte texte texte texte texte texte texte texte texte texte

 

De même, on peut utiliser

$\sum\limits{i=0}n u_i$.
ce qui donne :

 

åi=0n ui.

 

Pour écrire des mathématiques sous , les packages 'amsmath', disponible sur ftp://ftp.fdn.org/pub/CTAN/fonts/ams/amslatex/, et 'amssymb' sont incontournables. En LATEX2.09, on peut utiliser 'amstex', disponible sur ftp://ftp.fdn.org/pub/CTAN/macros/amstex/, 'amsby' et 'amsopn' mais il faut savoir que amstex est obsolète.

ATTENTION :
en mode mathématique, la gestion des espaces et des retours à la ligne est entièrement prise en charge par LATEX (cf. question 12.33). En outre, les lignes vides ne sont pas autorisées.

12.2  Qu'est que AMS-LATEX ?

AMS-TEX est un terme utilisé pour désigner un ensemble de fichiers distribués par l'American Mathematical Society (AMS). AMS-LATEX s'appuie sur TEX et . AMS-LATEX est principalement dû a F. Mittelbach et R. Schopf. C'est un outil complémentaire à LATEX pour écrire des mathématiques (il devient d'ailleurs rapidement indispensable).

Il existe une FAQ disponible sur ftp://ftp.fdn.org/pub/CTAN//macros/latex/amslatex/amslatex/math/amslatex.faq.

Les packages proposés sont 'amsmath' (complet), 'amstext' (pour écrire du texte en mode mathématique), 'amsbsy' (pour les symboles gras), 'amsopn' (pour la déclaration d'opérateurs), 'amsthm' (pour les environnements proof et theorem), 'amsintx' (pour étendre la syntaxe des sommes et intégrales), 'amscd' (pour les diagrammes commutatifs), 'amsxtra' et 'upref' (pour les références croisées).

Ces packages sont disponibles par ftp sur ftp://e-math.ams.org/pub/tex/amslatex/. Il faut également récupérer les fontes associées dans ftp://e-math.ams.org/pub/tex/amsfonts/, ftp://e-math.ams.org/pub/tex/amsltx11/ et ftp://e-math.ams.org/pub/tex/amstex/. Sur CTAN, les répertoires correspondants sont ftp://ftp.fdn.org/pub/CTAN/macros/latex/required/amslatex/, ftp://ftp.fdn.org/pub/CTAN/fonts/amsfonts/, ftp://ftp.fdn.org/pub/CTAN/macros/latex209/contrib/amslatex/ et ftp://ftp.fdn.org/pub/CTAN/macros/amstex/. Il existe également le site WEB http://www.ams.org/tex.

12.3  Comment écrire les symboles d'ensembles ?

Un peu d'histoire (T. Bouche) : au temps jadis, dans les livres, les symboles d'ensemble étaient imprimés en gras pour qu'on les voie bien. Problème, comment, lorsque l'on écrit sur un tableau noir, maintenir ce type de distinction ? En fait, au lieu d'écraser la craie avec force sur le tableau pour élargir un trait, on a décidé de le doubler. Ce qui est comique, c'est qu'en retour, les livres se sont mis à copier l'usage des profs, et ont distingué le "gras" (bold), du "gras de tableau" qui est une fonte à part (dessin normal à barre redoublée).

Les symboles mathématiques d'ensemble et bien d'autres sont disponibles par défaut dans les fontes AMS : famille msam (e.g., msam10 pour 10pt) et msbm. Ces fontes sont disponibles par ftp anonyme sur ftp://e-math.ams.org/pub/tex/amsfonts/ ou sur ftp://ftp.fdn.org/pub/CTAN/fonts/ams/amsfonts/sources/symbols/. Pour avoir accès aux symboles, il faut alors utiliser les packages 'amsfonts' et 'amssymb'.

Exemple :
 
soit on définit une commande spéciale (dans le préambule):

        \newcommand{\R}{\mathbb{R}}
et on utilise :

         $\R$
soit on tape directement :

        $\mathbb{R}$ (LaTeX2e) ou 
        ${\Bbb R}$ (LaTeX2.09) 
        %%%% fin exemple %%%%
Si vous ne disposez pas des packages ams vous pouvez au grand dam de certains puristes utiliser les commandes :

        \def\N{\mbox{I\hspace{-.15em}N}} 
        \def\Z{\mbox{Z\hspace{-.3em}Z}}
        \def\R\mbox{I\hspace{-.15em}R}}
        \def\C{\mbox{l\hspace{-.47em}C}}
Il faut savoir dans ce cas que leur utilisation est limitée :

C. Fiorio (fiorio@math.tu-berlin.de) propose également un certain nombre de macros () :

%%%% debut macro %%%% 

%%  Les ensembles de nombres 

\def\nbR{\ensuremath{\mathrm{I\!R}}} % IR 
\def\nbN{\ensuremath{\mathrm{I\!N}}} % IN 
\def\nbF{\ensuremath{\mathrm{I\!F}}} % IF 
\def\nbH{\ensuremath{\mathrm{I\!H}}} % IH 
\def\nbK{\ensuremath{\mathrm{I\!K}}} % IK 
\def\nbL{\ensuremath{\mathrm{I\!L}}} % IL 
\def\nbM{\ensuremath{\mathrm{I\!M}}} % IM 
\def\nbP{\ensuremath{\mathrm{I\!P}}} % IP 

\nbOne : 1I : symbol one 
\def\nbOne{{\mathchoice {\rm 1\mskip-4mu l} {\rm 1\mskip-4mu l} 
{\rm 1\mskip-4.5mu l} {\rm 1\mskip-5mu l}}} 

\nbC   :  Nombres Complexes 
\def\nbC{{\mathchoice {\setbox0=\hbox{$\displaystyle\rm C$}% 
\hbox{\hbox to0pt{\kern0.4\wd0\vrule height0.9\ht0\hss}\box0}} 
{\setbox0=\hbox{$\textstyle\rm C$}\hbox{\hbox 
to0pt{\kern0.4\wd0\vrule height0.9\ht0\hss}\box0}} 
{\setbox0=\hbox{$\scriptstyle\rm C$}\hbox{\hbox 
to0pt{\kern0.4\wd0\vrule height0.9\ht0\hss}\box0}} 
{\setbox0=\hbox{$\scriptscriptstyle\rm C$}\hbox{\hbox 
to0pt{\kern0.4\wd0\vrule height0.9\ht0\hss}\box0}}}} 

\nbQ   : Nombres Rationnels Q 
\def\nbQ{{\mathchoice {\setbox0=\hbox{$\displaystyle\rm 
Q$}\hbox{\raise 
0.15\ht0\hbox to0pt{\kern0.4\wd0\vrule height0.8\ht0\hss}\box0}} 
{\setbox0=\hbox{$\textstyle\rm Q$}\hbox{\raise 
0.15\ht0\hbox to0pt{\kern0.4\wd0\vrule height0.8\ht0\hss}\box0}} 
{\setbox0=\hbox{$\scriptstyle\rm Q$}\hbox{\raise 
0.15\ht0\hbox to0pt{\kern0.4\wd0\vrule height0.7\ht0\hss}\box0}} 
{\setbox0=\hbox{$\scriptscriptstyle\rm Q$}\hbox{\raise 
0.15\ht0\hbox to0pt{\kern0.4\wd0\vrule height0.7\ht0\hss}\box0}}}} 

\nbT   : T 
\def\nbT{{\mathchoice {\setbox0=\hbox{$\displaystyle\rm 
T$}\hbox{\hbox to0pt{\kern0.3\wd0\vrule height0.9\ht0\hss}\box0}} 
{\setbox0=\hbox{$\textstyle\rm T$}\hbox{\hbox 
to0pt{\kern0.3\wd0\vrule height0.9\ht0\hss}\box0}} 
{\setbox0=\hbox{$\scriptstyle\rm T$}\hbox{\hbox 
to0pt{\kern0.3\wd0\vrule height0.9\ht0\hss}\box0}} 
{\setbox0=\hbox{$\scriptscriptstyle\rm T$}\hbox{\hbox 
to0pt{\kern0.3\wd0\vrule height0.9\ht0\hss}\box0}}}} 

\nbS   : S 
\def\nbS{{\mathchoice 
{\setbox0=\hbox{$\displaystyle     \rm S$}\hbox{\raise0.5\ht0% 
\hbox to0pt{\kern0.35\wd0\vrule height0.45\ht0\hss}\hbox 
to0pt{\kern0.55\wd0\vrule height0.5\ht0\hss}\box0}} 
{\setbox0=\hbox{$\textstyle        \rm S$}\hbox{\raise0.5\ht0% 
\hbox to0pt{\kern0.35\wd0\vrule height0.45\ht0\hss}\hbox 
to0pt{\kern0.55\wd0\vrule height0.5\ht0\hss}\box0}} 
{\setbox0=\hbox{$\scriptstyle      \rm S$}\hbox{\raise0.5\ht0% 
\hboxto0pt{\kern0.35\wd0\vrule height0.45\ht0\hss}\raise0.05\ht0% 
\hbox to0pt{\kern0.5\wd0\vrule height0.45\ht0\hss}\box0}} 
{\setbox0=\hbox{$\scriptscriptstyle\rm S$}\hbox{\raise0.5\ht0% 
\hboxto0pt{\kern0.4\wd0\vrule height0.45\ht0\hss}\raise0.05\ht0% 
\hbox to0pt{\kern0.55\wd0\vrule height0.45\ht0\hss}\box0}}}} 

\nbZ   : Entiers Relatifs Z 
\def\nbZ{{\mathchoice {\hbox{$\sf\textstyle Z\kern-0.4em Z$}} 
{\hbox{$\sf\textstyle Z\kern-0.4em Z$}} 
{\hbox{$\sf\scriptstyle Z\kern-0.3em Z$}} 
{\hbox{$\sf\scriptscriptstyle Z\kern-0.2em Z$}}}} 
%%%% fin macro %%%%
Exemple :
 
\nbN $ pour les entiers naturels 
%%%% fin exemple %%%%
Autres alternatives :

Exemple :
 
\font\bbold=bbold12 
\newcommand{\R}{\mbox{\bbold R}} 
%%%% fin exemple %%%%
Le package 'bbm' disponible sur ftp://ftp.fdn.org/pub/CTAN/fonts/cm/bbm/ propose certaines variantes pour les fontes Computer Modern.

12.4  Comment numéroter les équations ?

Sous , les options de classe leqno et fleqn permettent d'indiquer si les numéros d'équations doivent apparaître à droite ou à gauche des équations.

Exemple, pour que les numéros apparaîssent à gauche :
 
\documentclass[leqno]{report} 
\begin{document} 
\begin{eqnarray} 
   x + 4 & =  & 0 \\ 
   8 - y & =  & 0 
\end{eqnarray} 
%%%% fin exemple %%%%
ce qui donne :


x + 4
=
0
(12.1)
8 - y
=
0
(12.2)

Pour supprimer la numérotation des équations, il suffit d'ajouter le caractère * aux noms des environnements d'équation.

Exemple
:
\begin{eqnarray*} 
   x + 4 & =  & 0 \\ 
   8 - y & =  & 0 
\end{eqnarray*} 
%%%% fin exemple %%%%
ce qui donne :


x + 4
=
0
8 - y
=
0

Pour supprimer la numérotation d'une ligne particulière dans un groupe d'équations, il suffit d'utiliser la commande \nonumber (avant les \\). Avec amsmath, il faut utiliser \notag.

Pour réinitialiser le compteur d'équations dans chaque section, il faut inclure :

%%%% debut macro %%%% 
\makeatletter 
\renewcommand\theequation{\thesection.\arabic{equation}} 
\@addtoreset{equation}{section} 
\makeatother 
%%%% fin macro %%%%
dans le préambule.

Remarque :
dans ce cas, il ne faut pas mettre d'équation dans un chapitre avant la première section sous peine d'avoir un numéro du style 3.0.1.
Pour modifier le style de numérotation, il faut modifier la commande \theequation.

Exemple 1 :
 
\documentclass{report} 
\usepackage{french} 
\pagestyle{empty} 
\renewcommand{\theequation}{\thesection \Alph{equation}} 
\begin{document} 
\chapter{Systèmes d'équations.} 
   \section{À une variable.} 
      \begin{eqnarray} 
         x + 4 & =  & 0 \\ 
          x & =  & 3 
      \end{eqnarray} 
   \section{À deux variables.} 
      \begin{eqnarray} 
         x + 4 & =  & 0 \\ 
         8 - y & =  & 0 
      \end{eqnarray} 
\end{document} 
%%%% fin exemple %%%%
Exemple 2 (pour avoir les numéros entre crochets) :
 
\makeatletter 
\renewcommand{\@eqnnum}{{\normalfont\normalcolor\theequation}} 
\renewcommand{\theequation}{[\arabic{equation}]} 
\makeatother

 

\begin{document} 
      \begin{eqnarray} 
         x + 4 & =  & 0 \\ 
         y - 2 & =  & 0 
      \end{eqnarray} 
%%%% fin exemple %%%%

Avec le package 'amsmath', disponible sur ftp://ftp.fdn.org/pub/CTAN/fonts/ams/amslatex/, la commande \numberwithin permet de numéroter les équations suivant le paragraphe auquel elles appartiennent. Il faut alors appeler :

\numberwithin{equation}{section} 
dans le préambule.

Les packages 'seceqn' et 'apeqnum' disponibles sur CTAN permettent pour le premier de numéroter les équations par section et pour le second de numéroter individuellement les équations dans les annexes.

Le package 'deleq' disponible sur ftp://ftp.fdn.org/pub/CTAN/macros/latex/contrib/supported/ ou sur ftp://ftp.fdn.org/pub/CTAN/macros/latex/contrib/other/deleq/ permet de définir un label par groupe d'équations (4) en plus des labels individuels de type (4a), (4b), etc.

La commande \tag{xxx} du package 'amsmath' sous permet de personnaliser la numérotation des équations.

Exemple :
pour permettre à deux équations d'avoir le même numéro :
\begin{equation} 
  x - y = 0 
  \label{equa} 
\end{equation} 
... 
\begin{equation} 
  x = y 
  \tag{\eqref{equa}} 
\end{equation} 
%%%% fin exemple %%%%
ce qui donne :


x - y = 0
(12.3)
...
x = y\tag\eqrefequa
(12.4)

Remarque :
\eqref remplace \ref pour les équations : la fonte utilisée est toujours la même (c'est plus beau).
Les environnements subequations (du package 'amsmath') et subeqnarray (du package 'subeqnarray') permettent de référencer différentes lignes d'un même groupe d'équations par des indices (3.a), (3.b), etc.

Sous subequations, une référence à une ligne particulière s'obtient alors par un \label placé sur cette ligne alors qu'une référence au système d'équation s'obtient par un \label placé immédiatement après \begin{subequations}.

Exemple :
 
\begin{subequations} \label{E:gp} 
  \begin{gather} 
    x_1 x_2 + x_12 x_22 + x_3 \label{E:gp1} \\ 
    x_1 x_3 + x_12 x_32 + x_2 \label{E:gp2} \\ 
    x_1 x_2 x_3 \label{E:gp3} 
  \end{gather} 
\end{subequations}

 

[...]

 

Voir le systèmeref{E-gp} et en particulier 
l'équationref{E-gp2}. 
%%%% fin exemple %%%%

ce qui donne :

x_1 x_2 + x_1^2 x_2^2 + x_3
x_1 x_3 + x_1^2 x_3^2 + x_2
x_1 x_2 x_3 [...] Voir le système  et en particulier l'équation .

Remarque :
c'est le \\ qui incrémente le compteur des sous-références.

12.5  Comment aligner des équations ?

Pour aligner des équations sur un signe de relation, il suffit sous LATEX d'appeler l'environnement eqnarray avec ou sans la commande \lefteqn ou si l'on dispose du package 'amsmath', disponible sur ftp://ftp.fdn.org/pub/CTAN/fonts/ams/amslatex/, d'utiliser un des environnements split, multline, align ou flalign.

Exemples :
 
\begin{align} 
   f(x) & = & x2 + 8x + 16 \\ 
   & = & (x+4)2 
\end{align}

 

\begin{eqnarray} 
   \lefteqn{x + 3y - 6z} \\ 
   & = & -4a + 5b -7b \\ 
   & = & -4a - 2b 
\end{eqnarray}

 

\begin{align} 
   \begin{split} 
      A &= B + C + D + E + F + G \\ 
      &\quad + H + I 
   \end{split} \\ 
   F &= G + H 
\end{align} 
%%%% fin exemple %%%%

ce qui donne :

f(x) = x^2 + 8x + 16
= (x+4)^2
x + 3y - 6z
(12.5)
=
-4a + 5b -7b
(12.6)
=
-4a - 2b
(12.7)
A = B + C + D + E + F + G
    + H + I
F = G + H

L'environnement equationarray du package 'eqnarray' associe les avantages des environnements eqnarray et array. Il n'est pas limité en nombre de colonnes.

Pour produire le système d'équations suivant :

         x  = y  + z 
         x1 = y1 + z1
on peut utiliser l'environnement alignat du package 'amsmath'. Toutefois, il faut noter que cet environnement est alors détourné de son utilisation normale. En effet, ce dernier est prévu pour aligner des objets différents sur des colonnes alternativement justifiées à droite puis à gauche. L'utilisation de colonnes vides permet alors de choisir la justification voulue. D'autre part l'utilisation de {} permet d'obtenir des espacements "normaux".

Exemple :
 
\begin{alignat}{2} 
   x & = y &&+ z\\ 
   x_1 & = y_1 &&+ z_1 
\end{alignat}

 

\begin{alignat}{5} 
    10a& ={}&  3x&& 3y& +{}& 18z&& 2w&\\ 
     6a& ={}& 17x&&   & +{}&  5z&& 19w& 
\end{alignat} 
%%%% fin exemple %%%%

ce qui donne :

 

2 x = y + z
x_1 = y_1 + z_1 5 10a = 3x 3y + 18z 2w
6a = 17x + 5z 19w

Le package 'mathenv' disponible sur CTAN propose un éventail d'environnements mathématiques pour aligner des entités en colonne.

12.6  Comment générer des vecteurs ?

Il suffit d'utiliser le package 'amsmath', disponible sur ftp://ftp.fdn.org/pub/CTAN/fonts/ams/amslatex/, et d'appeler la commande \overrightarrow.

Exemple :
 
Le vecteur $\overrightarrow{u}$ 
%%%% fin exemple %%%%
ce qui donne :

 

Le vecteur [( ®) || u]

 

On peut également définir ses propres flèches notamment pour régler leur hauteur par rapport aux lettres qu'elles surmontent. La solution présentée ci-dessous utilise les packages 'amsmath', 'amssymb' et 'xy' :

%%%% debut macro %%%% 
\newcommand{\xyflecheverladroite} 
{\mbox{\xymatrix{*{\hphantom{OM}}\ar[]+L;[]+R}}}

 

\newcommand{\ra}[1] 
{\mathchoice 
{\overset{\mbox{\xymatrix{*{\hphantom{\displaystyle #1}} 
\ar[]+L;[]+R}}}{\displaystyle #1}}% 
{\overset{\mbox{\xymatrix{*{\hphantom{\textstyle #1}} 
\ar[]+L;[]+R}}}{\textstyle #1}}% 
{\overset{\mbox{\xymatrix{*{\hphantom{\scriptstyle #1}} 
\ar[]+L;[]+R}}}{\scriptstyle #1}}% 
{\overset{\mbox{\xymatrix{*{\hphantom{\scriptscriptstyle #1}} 
\ar[]+L;[]+R}}}{\scriptscriptstyle #1}}% }

 

%Pour changer la distance de la flèche, on peut procéder ainsi. 
%\renewcommand{\ra}[1] 
%{\overset{\raisebox{-1pt}{\mbox{\xymatrix{*{\hphantom{#1}} 
%\ar[]+L;[]+R}}}}{#1}} 
%%%% fin macro %%%%

Exemples :
 
$$ \ra{OM} $$ 
$\ra{OM}$ 
$\ra{OM_i}$ 
$$ \ra{OM} _{\ra{OM}_{\ra{OM}}}$$ 
%%%% fin exemple %%%%
Le package 'vector', de N. Efford, est disponible sur ftp://ftp.fdn.org/pub/CTAN/macros/latex/contrib/supported/vector/. Il offre notamment un certain nombre de vecteurs utiles aux physiciens et des commandes automatiques d'énumération des coordonnées.

Exemple :
 
$(\irvec[4]x)$

 

$\bvec{y} = \left[ \cvec{y}{0}{3} \right]$ 
%%%% fin exemple %%%%

Le package 'easyvector' disponible sur ftp://ftp.fdn.org/pub/CTAN/macros/latex/contrib/supported/easy/ permet de définir des vecteurs suivant une syntaxe de type C.

12.7  Comment écrire du texte en mode mathématique ?

La commande \text{xxx} du package 'amsmath', disponible sur ftp://ftp.fdn.org/pub/CTAN/fonts/ams/amslatex/, permet d'inclure du texte dans une formule mathématique sans que les accents et les espacements mathématiques ne soient nécessaires.

La commande \textrm{xxx} du package 'amsmath' (ou amstext) permet en outre de prendre en compte la taille de la fonte courante.

Exemple :
 
La fonction: $ f_{\textrm{tracage}}  $ sert au tracé de courbes. 
%%%% fin exemple %%%%
Il est également possible d'utiliser la commande \mbox.

Remarque :
aucune de ces commandes ne gère les retours à la ligne en fin de ligne.

12.8  Comment ajuster la taille de délimiteurs ?

Pour ajuster la taille de délimiteurs tels que les parenthèses, les accolades, etc, il suffit d'utiliser les commandes \left et \right.

Remarque :
si l'on veut utiliser un seul délimiteur (sans son symétrique) il faut utiliser \right. en fermeture.
Exemples :
 
$\left( \frac{1}{2} \right)$ 
$\left \\mathrm{et} \right \}$ 
Résoudre le système: 
\
\left \
\begin{array}{ccc} 
   x + y & = & 0 \\ 
   x - y & = & 0 
\end{array} 
\right. 
\]

%%%% fin exemple %%%%

ce qui donne :

 

( [ 1/2] ) { et } Résoudre le système :
ì
í
î
x + y
=
0
x - y
=
0

Remarque :
Pour changer la taille des délimiteurs de façon relative à la taille des données qu'ils encadrent, c'est le paramètre \delimitershortfall qui est utilisé. Lorsqu'il vaut 0pt, les délimiteurs qu'il contrôle sont au plus aussi grands que le contenu qu'ils encadrent ; pour que les délimiteurs puissent être plus grands que leur contenu, il faut donner une valeur négative à \delimitershortfall.
Pour imposer son propre choix de taille de délimiteurs notamment pour des délimiteurs imbriqués, il existe les commandes \big, \Big, \bigg et \Bigg.

Exemples :
 
$\Big( (x+1) (x-1) \Big) {2}$ 
$\big\{\Big\{\bigg\{\Bigg\{$ 
%%%% fin exemple %%%%
ce qui donne :

 

( (x+1) (x-1) ) 2 {{{{

12.9  Comment changer de fonte en mode mathématique ?

Il faut utiliser les packages 'amsmath' ou 'amssymb' puis :

on peut aussi générer tout l'alphabet correspondant :

\DeclareSymbolFont{boldsymbols}{OMS}{cmsy}{b}{n}

\DeclareSymbolFontAlphabet{\mathbfcal}{boldsymbols}

puis utiliser la commande

\mathbfcal
Remarque :
La commande \mathcal ne marche qu'avec des majuscules.
\DeclareMathAlphabet\mbi{OML}{cmm}{b}{it}
\DeclareSymbolFont{mathbold}{OML}{cmm}{b}{it} 
\DeclareMathSymbol{\biGamma}{\mathord}{mathbold}{0}
\boldsymbol{\alpha}
\DeclareSymbolFont{mathbold}{OML}{cmm}{b}{it} 
\DeclareMathSymbol{\balpha}{\mathord}{mathbold}{11}
Remarque :
si cela ne marche pas, c'est que les caractères gras correspondants n'existent pas. Il faut alors soit utiliser la commande \pmb{...} soit utiliser un package supplémentaire tel que 'amsbsy'. Il existe par ailleurs des versions postscript de fontes AMS disponibles sur CTAN.
Exemple :
 
Une formule importante: 
\mathversion{bold} 
$$\sum_{i=0}n u_i + v_i$$

 

Une formule moins importante: 
\mathversion{normal} 
$$\sum_{i=0}n v_i$$ 
%%%% fin exemple %%%%

ce qui donne :

Une formule importante :
n
å
i=0 
ui + vi
Une formule moins importante :
n
å
i=0 
vi

Le package 'bm' disponible sur ftp://ftp.fdn.org/pub/CTAN/macros/latex/contrib/supported/bm/ permet de bien gérer les symboles mathématiques en gras.

12.10  Comment obtenir le L de la transformée de Laplace ?

Pour obtenir le L de la transformée de Laplace, il faut utiliser le package 'mathrsfs' puis la commande

\renewcommand{\L}{\mathscr{L}}.
Remarque :
ce package fait appel à des fontes particulières qu'il faut également installer.

12.11  Comment réaliser un tableau en mode mathématique ?

Il faut utiliser l'environnement array.

Exemple :
 
Résultats: 
\
\begin{array}{|c @=c|} 
   \hline 
   \alpha + \beta & 12 \\ 
   \hline 
   \sqrt{\gamma} & 36 \\ 
   \hline 
\end{array} 
\
%%%% fin exemple %%%%
ce qui donne :

 

Résultats :
a+ b
=12

Ö
 

g
 
=36

12.12  Comment obtenir d'autres symboles mathématiques ?

LATEX propose un grand nombre de commandes de symboles.

\pm ± \cap Ç \diamond à \oplus Å
\mp \cup È \bigtriangleup \bigtriangleup \ominus \ominus
\times × \uplus \uplus \bigtriangledown \bigtriangledown \otimes Ä
\div ¸ \sqcap \sqcap \triangleleft \triangleleft \oslash Æ
\ast * \sqcup \sqcup \triangleright \triangleright \odot \odot
\star * \vee Ú \wedge Ù \bigcirc \bigcirc
\dagger f \setminus \ \amalg \amalg \circ °
\ddagger f \cdot · \wr \wr \bullet ·
\lhd \lhd \rhd \rhd \unlhd \unlhd \unrhd \unrhd

 

\leq £ \geq ³ \equiv º \models \models
\prec \prec \succ \succ \sim ~ \perp ^
\preceq \preceq \succeq \succeq \simeq @ \mid |
\ll << \gg >> \asymp \asymp \parallel ||
\subset Ì \supset É \approx » \bowtie \bowtie
\subseteq Í \supseteq Ê \cong @ \Join \Join
\sqsubset \sqsubset \sqsupset \sqsupset \neq ¹ \smile \smile
\sqsubseteq \sqsubseteq \sqsupseteq \sqsupseteq \doteq \doteq \frown \frown
\in Î \ni ' \propto µ
\vdash \vdash \dashv \dashv

 

\leftarrow ¬ \longleftarrow ¬ \uparrow ­
\Leftarrow Ü \Longleftarrow Ü \Uparrow Ý
\rightarrow ® \longrightarrow ® \downarrow ¯
\Rightarrow Þ \Longrightarrow Þ \Downarrow ß
\leftrightarrow « \longleftrightarrow « \updownarrow \updownarrow
\Leftrightarrow Û \Longleftrightarrow Û \Updownarrow \Updownarrow
\mapsto ® \longmapsto ® \nearrow \nearrow
\hookleftarrow \hookleftarrow \hookrightarrow \hookrightarrow \searrow \searrow
\leftharpoonup \leftharpoonup \rightharpoonup \rightharpoonup \swarrow \swarrow
\leftharpoondown \leftharpoondown \rightharpoondown \rightharpoondown \nwarrow \nwarrow
\rightleftharpoons \rightleftharpoons \leadsto \leadsto

 

\sum å \bigcap Ç \bigodot \bigodot
\prod Õ \bigcup È \bigotimes Ä
\coprod \coprod \bigsqcup \bigsqcup \bigoplus Å
\int ò \bigvee Ú \biguplus \biguplus
\oint (ò) \bigwedge Ù

 

\aleph À \prime ¢ \forall " \infty ¥
\hbar (h/2p) \emptyset Æ \exists $ \Box [¯]
\imath i \nabla Ñ \neg Ø \Diamond \Diamond
\jmath j \surd Ö \flat \flat \triangle \triangle
\ell l \top T \natural \natural \clubsuit §
\wp à \bot ^ \sharp \sharp \diamondsuit ¨
\Re  \| | \backslash \ \heartsuit ©
\Im Á \angle Ð \partial \spadesuit ª
\mho \mho

 

Des symboles mathématiques supplémentaires (en plus de ceux offerts par amsmath) sont disponibles dans les packages 'stmaryrd' disponible sur CTAN dans /fonts/stmaryrd/, et 'yhmath' également disponible sur CTAN dans /macros/latex/contrib/supported/yhmath.

12.13   Comment définir une fonction ?

Il existe un certain nombre de fonctions définies par défaut.

Il s'agit des fonctions :

\arccos arccos \cos cos \csc csc \exp exp
\arcsin arcsin \cosh cosh \deg deg \gcd gcd
\arctan arctan \cot cot \det det \hom hom
\arg arg \coth coth \dim dim \inf inf
\ker ker \limsup limsup \min min \sinh sinh
\lg lg \ln ln \Pr Pr \sup sup
\lim lim \log log \sec sec \tan tan
\liminf liminf \max max \sin sin \tanh tanh

\newcommand{\fonction}{\mathop{fonction}}
Si on utilise en plus la commande \nolimits, on peut déterminer la gestion des indices et exposants de cette fonction.

Exemple :
 
\newcommand{\rad}{\mathop{\mathrm{rad}}} 
\newcommand{\radn}{\mathop{\mathrm{rad}}\nolimits} 
\begin{document} 
\
\alpha \rad_{t \rightarrow \infty} = 0 
\]

 

\
\alpha \radn_{t \rightarrow \infty} = 0 
\
\newcommand{\rad}{\mathop{\mathrm{rad}}} 
\newcommand{\radn}{\mathop{\mathrm{rad}}\nolimits}

 

\begin{document} 
\
\alpha \rad_{t \rightarrow \infty} = 0 
\]

 

\
\alpha \radn_{t \rightarrow \infty} = 0 
\
\end{document} 
%%%% fin exemple %%%%

Sous , le package 'amsopn' est disponible sur ftp://ftp.fdn.org/pub/CTAN/fonts/ams/amslatex/ et par ftp sur ftp://e-math.ams.org/pub/tex/amslatex/inputs/. Il permet de déclarer de nouvelles fonctions qui seront gérées comme une fonction mathématique (gestion de la fonte, disposition des indices et exposants, etc).  Pour cela il faut déclarer une commande du type : \DeclareMathOperator{\fonction}{fonction} dans le préambule du document. Pour que les indices et exposants de la nouvelle fonction puissent être géres comme ceux de la fonction \sum , il faut alors plutot utiliser \DeclareMathOperator*.

De même, le package 'amstex' (LATEX2.09) définit les commandes \operatorname et \operatornamewithlimits.

Exemple :
 
$\operatorname{rad}$ 
%%%% fin exemple %%%%

12.14  Comment définir une matrice ?

Il suffit de construire un tableau sans lignes avec l'environnement array.

Exemple :
 
\
\begin{array}{ccc} 
   x_{11} & \cdots & x_{1p} \\ 
   \vdots & \ddots & \vdots \\ 
   x_{n1} & \cdots & x_{np} 
\end{array} 
\
%%%% fin exemple %%%%
ce qui donne :


x11
¼
x1p
:
···
:
xn1
¼
xnp

Pour encadrer cette matrice avec des délimiteurs, cf. paragraphe .

Le package 'amsmath',disponible sur ftp://ftp.fdn.org/pub/CTAN/fonts/ams/amslatex/, permet de définir une matrice de manière plus rapide qu'avec l'environnement array. Les environnements disponibles sont :

Exemple :
 
\
\begin{pmatrix} 
         a & b \\ 
         c & d 
\end{pmatrix} 
\
%%%% fin exemple %%%%
 

12.15  Comment encadrer des formules ?

Le package 'amsmath', disponible sur ftp://ftp.fdn.org/pub/CTAN/fonts/ams/amslatex/, offre la commande \boxed.

Exemple :
 
$$\boxed{a=b}$$ 
%%%% fin exemple %%%%
ce qui donne :


\boxeda=b

 

Il existe également la commande \fbox.

Remarqu :
Avec les environnements multi-lignes de AMSTEX (comme multline ou split), il peut être utile de les encapsuler dans un math ou dans un displaymath.
Exemple :
 
\begin{equation} 
   \fbox{$ 
   \begin{array}{rcl} 
      x + y + z & = & 0 \\ 
      2x + 2y + 2z & = & 0 
   \end{array} 
   $} 
\end{equation} 
%%%% fin exemple %%%%
ce qui donne :


x + y + z
=
0
2x + 2y + 2z
=
0
(12.8)

 

Le package 'fancybox' disponible sur ftp://ftp.fdn.org/pub/CTAN/macros/latex/contrib/other/seminar/inputs/ peut également être utile.

Exemple :
 
\linethickness{3pt} 
\Ovalbox{ 
\begin{Beqnarray} 
ds2&\,=\,& \displaystyle{\frac{\epsilon{\prime 2}}{L2} 
\frac{12N}{(N+1)(N+2)}} 
\end{Beqnarray} 

%%%% fin exemple %%%%
ce qui donne :

 

Une dernière solution consiste à mettre la formule dans un tableau d'une seule cellule.

12.16  Comment ajuster la longueur d'une flèche par rapport à un texte ?

Le package 'amsmath',disponible sur ftp://ftp.fdn.org/pub/CTAN/fonts/ams/amslatex/, permet de générer des flèches dont la longueur dépend de la longueur du texte qui est placé au dessus ou en dessous (ou de la chaîne la plus longue lorsqu'il y a à la fois un texte au dessus et un autre en dessous).

Exemple :
 
$\xrightarrow[\text{au dessus}]{\text{en dessous}}$ 
%%%% fin exemple %%%%
ce qui donne :

\xrightarrow[au dessus]en dessous

12.17  Comment obtenir des indices ou exposants à gauche ?

Les commandes \sideset \overset et \underset du package 'amsmath' permettent de placer du texte à gauche, en haut ou en bas.

Exemple, mettre dans le préambule :
 
\newcommand{\transposee}[1]{{\vphantom{#1}}{\mathit t}{#1}}
puis, dans le corps du document :

\
\transposee{ 
\begin{pmatrix} 
   a & b \\ 
   c & d \\ 
\end{pmatrix} 

\
%%%% fin exemple %%%%
Remarque :
on peut également utiliser \sideset qui ne marche que pour les opérateurs :
\newcommand{\transposee}[1]{\sideset{{\mathit{t}}}{}{#1}}
A défaut, on peut utiliser

${}_{j}H$ ou  ${_jH}$
Le package 'chemsym' disponible sur ftp://ftp.fdn.org/pub/CTAN/macros/latex/contrib/supported/chemsym/ peut également être utile.

12.18  Comment tracer des diagrammes commutatifs ?

Xy-pic est un outil simple et puissant qui permet de réaliser de tels diagrammes (cf. paragraphe 8.6).

Exemple :
 
\documentclass[a4paper,12pt]{article} 
\usepackage[all]{xy} 
\begin{document} 
\[\xymatrix{ 
  A \ar[d] \ar[r] \ar@{=}[rd] & B \ar[d] \\ 
  C \ar[r] & D } 
\
\end{document} 
%%%% fin exemple %%%%
On peut également utiliser le package 'amscd' disponible sur ftp://ftp.fdn.org/pub/CTAN/macros/latex/required/amslatex/math/ ou plus généralement le package 'amsmath' et l'environnement CD mais son offre est plus limitée.

Exemple 1 (amscd) :
 
\
\begin{CD} 
   \mathcal{F} @>\otimes> > T\\ 
   @VdrVlfV @ViVjV\\ 
   \mathtt{f} @= t 
\end{CD} 
\]

 

\
\begin{CD} 
d @> > > e @> > > f \\ 
@AAA @. @AAA \\ 
a @> > > b @> > > c 
\end{CD} 
\
%%%% fin exemple %%%%

Exemple 2 (amsmath) :
 
\
\begin{array}{ccccc} 
d & \longrightarrow & e & \longrightarrow &f \\ 
\uparrow &&&& \uparrow \\ 
a & \longrightarrow & b &\longrightarrow& c\\ 
\end{array} 
\
%%%% fin exemple %%%%
ce qui donne :


d
®
e
®
f
­
­
a
®
b
®
c

 

Remarque :
dans la dernière version de amscd, la syntaxe @>Exp1>Exp2> utilisée pour les flèches extensibles a été abandonnée au profit des commandes \xleftarrow et \xrightarrow.
Pour plus de détails consultez :

Il existe le package 'cd' écrit pour LATEX2.09 par D. Hankerson. Ce package étend les capacités des fonction AMS-TEX. Les longueurs de texte s'ajustent avec la longueur des arguments.

Exemple (doc package) :
 
\documentstyle[12pt,cd]{article} 
\newcommand{\cdrl}{\cd\rightleftarrows} 
\newcommand{\cdlr}{\cd\leftrightarrows} 
\newcommand{\cdr}{\cd\rightarrow} 
\newcommand{\cdl}{\cd\leftarrow} 
\newcommand{\cdu}{\cd\uparrow} 
\newcommand{\cdd}{\cd\downarrow} 
\newcommand{\cdud}{\cd\updownarrows} 
\newcommand{\cddu}{\cd\downuparrows} 
\begin{document} 
\[\CD 
\cdrl {\gamma}{\delta} H \cdr {}{\Delta} K \\ 
\cd. \cdud {f}{g} \cd| {h}{k} \\ 
\cdr {}{} G' \cd= {\beta}{} H' \endCD 
\
\end{document} 
%%%% fin exemple %%%%
Le package 'catmac' définit un ensemble de macros pour tracer des diagrammes commutatifs. Il propose par défaut un ensemble de formes mais permet également via un certain nombre de macros prédéfinies de composer ses propres formes.

Exemple :
 
\begin{center} 
   \resetparms 
   \square[A`B`C`D;f`g`h`k] 
\end{center} 
%%%% fin exemple %%%%
ATTENTION :
ce package n'est pas compatible avec le package french qui rend le ; actif.

12.19  Comment ajuster la taille de certains opérateurs ?

Pour ajuster la taille de certains opérateurs, il faut utiliser le package 'exscale' disponible sur ftp://ftp.fdn.org/pub/CTAN/macros/latex/base/.

Le package 'amsmath', disponible sur ftp://ftp.fdn.org/pub/CTAN/fonts/ams/amslatex/, donne la même possibilité.

12.20  Comment mettre en page des algorithmes ?

Les packages 'algorithm' et 'algorithmic' disponibles sur ftp://ftp.fdn.org/pub/CTAN/macros/latex/contrib/supported/algorithms/, ont été spécifiquement conçus pour mettre en forme des algorithmes.

Il existe aussi le package 'alg' disponible sur ftp://ftp.fdn.org/pub/CTAN/macros/latex/contrib/other/alg/.

Le package 'newalg' est également disponible sur ftp://ftp.fdn.org/pub/CTAN/macros/latex/contrib/supported/newalg/. Il propose un environnement algorithm qui par défaut utilise le mode mathématique et l'environnement array pour les alignements. La commande \text est également disponible. Le package gère les instructions : if-then-else, for, while, repeat, switch et propose un certain nombre de macros telles que call, error, algkey, return, nil.

Exemple :
 
\begin{algorithm}{StrictSup}{x, y} 
   \begin{IF}{x > y} 
      \RETURN x 
   \ELSE 
      \ERROR{x leq y} 
  \end{IF} 
\end{algorithm} 
%%%% fin exemple %%%%

12.21  Comment mettre en page des formules longues ?

L. Lamport définit certaines conventions dans sa page WEB : http://www.research.digital.com/SRC/personal/Leslie_Lamport/latex/latex.html.

12.22  Comment créer des unités de mesure ?

Le problème du mode mathématique est qu'il change automatiquement de fonte et qu'il utilise une gestion particulière des espaces. Le mieux pour écrire des unités de mesures du type m/s sans avoir à taper systématiquement des barbarismes du type \rm\,m/s\mit ou \,\textrm{m}/\textrm{s} est de définir une commande spéciale dans le préambule du document.

Exemples :
 
\newcommand{\units}[2]{#1\textrm{\thinspace #2}}
cette commande nécessite le package 'amsmath' utilisation :

\units{10}{m/s}
ou plus rapide :

\newcommand{\ms}[1]{\units{#1}{m/s}}
utilisation :

\ms{10}

%%%% fin exemple %%%%

12.23  Comment écrire proprement a/b ?

Pour qu'à l'impression le numérateur 'a' soit légèrement décalé vers le haut et vers la gauche et que le dénominateur 'b' soit légèrement décalé vers le bas et vers la droite, les deux étant plus petits que la fonte courante, il existe la commande \sfrac du package tugboat disponible sur ftp://ftp.fdn.org/pub/CTAN/macros/latex/contrib/supported/tugboat/.

12.24  Comment créer des notes de bas de page dans une formule mathématique ?

On peut utiliser directement la commande \footnote{} ou les commandes spécifiques \footnotemark[] et \footnotetext[]{} si l'on veut fixer soit même les compteurs. \footnotemark permet de gérer le compteur de notes et \footnotetext permet d'insérer le texte correspondant en bas de page.

Exemple :
 
\
Place_{libre}\footnotemark[1] + 
Place_{occup\acute ee}\footnotemark[2] + 
Place_{d\acute efaillante} = 
Capacit\acute e 
\
\footnotetext[1]{La place libreldots} 
\footnotetext[2]{La place occup\' eeldots} 
%%%% fin exemple %%%%
ce qui donne :

 


Placelibre\footnotemark[1] +Placeoccup\acute ee\footnotemark[2] +Placed\acute efaillante = Capacit\acute e

12.25  Comment forcer le style d'un tableau à displaystyle ?

Le style par défaut dans un "array" est textstyle. Pour éviter d'avoir à écrire \displaystyle dans chaque cellule, on peut déclarer un tableau entièrement en mode displaystyle via la macro :

%%%% debut macro %%%% 
\newenvironment{disarray}% 
 {\everymath{\displaystyle\everymath{}}\array}% 
 {\endarray} 
%%%% fin macro %%%%

12.26  Comment réduire les espaces gérés par eqnarray ?

Il suffit d'inclure la macro suivante dans le préambule de votre document :

%%%% debut macro %%%% 
\makeatletter 
\newlength{\earraycolsep} 
\setlength{\earraycolsep}{2pt} 
\def\eqnarray{\stepcounter{equation}\let\@currentlabel% 
\theequation 
\global\@eqnswtrue\m@th 
\global\@eqcnt\z@\tabskip\@centering\let\\\@eqncr 
$$\halign to\displaywidth\bgroup\@eqnsel\hskip\@centering 
$\displaystyle\tabskip\z@{##}$&\global\@eqcnt\@ne 
\hskip 2\earraycolsep \hfil$\displaystyle{##}$\hfil 
&\global\@eqcnt\tw@ \hskip 2\earraycolsep 
$\displaystyle\tabskip\z@{##}$\hfil 
\tabskip\@centering&\llap{##}\tabskip\z@\cr} 
\makeatother 
%%%% fin macro %%%%

12.27  Comment réduire la taille des indices ?

Il faut utiliser la commande \DeclareMathSizes :

\DeclareMathSizes{10}{10}{6}{5} 
\DeclareMathSizes{11}{11}{7}{5} 
\DeclareMathSizes{12}{12}{7}{5}
Ces déclarations permettent de réduire la taille des indices des fontes 10pt, 11pt et 12pt.

Exemple :
 
\documentclass[12pt]{report} 
\usepackage{french} 
\pagestyle{empty} 
\DeclareMathSizes{12}{12}{7}{5} 
\begin{document} 
La capacité totale est définie par: 
\
C_{libre} + C_{occup\acute ee} = C_{totale} 
\
\end{document} 
%%%% fin exemple %%%%
On peut aussi utiliser les commandes scriptstyle ou scriptscriptstyle.

Exemple :
 
$\rho_{AB}$ 
$\rho_{\scriptstyle AB}$ 
$\rho_{\scriptscriptstyle AB}$ 
%%%% fin exemple %%%%
ce qui donne :

rAB r AB r\scriptscriptstyle AB

 

En utilisant les caractères romans, on peut appliquer la commande \small.

Exemple
:
$\rho_{\textrm{\small AB}}$ 
%%%% fin exemple %%%%
ce qui donne :

rAB

12.28  Comment aligner des indices ?

La commande \vphantom définit une boîte invisible dont la hauteur est celle de son argument.

Exemple :
 
\
f_{\vphantom{\widehat{ef}} d} = f_{\widehat{ef}} \mathrm{\ au \ 
lieu \ de:} f_d = f_{\widehat{ef}} 
\
%%%% fin exemple %%%%
ce qui donne :


f d = f[^ef]  au  lieu  de : fd = f[^ef]

12.29  Comment mettre en page des théorèmes ?

LATEX offre la commande

\newtheorem{nom}[compteur]{texte}[section]
où nom est une clé pour identifier le théorème, et texte le titre du théorème qui sera imprimé. compteur permet de référencer un autre théorème pour obtenir la même numérotation. section permet de préciser le niveau de numérotation voulu.

Exemple :
 
dans le préambule :

\newtheorem{th-imp}{Théorème important}[section] 
\newtheorem{th-u}{Théorème}[subsection]
dans le texte :

 

\section{Introduction.}

 

   \begin{th-imp} \label{label-th-AA} 
      BLA BLA 
   \end{th-imp}

 

   \subsection{Bases.}

 

      \begin{th-u} \label{label-th-aaa} 
         bla bla bla 
      \end{th-u}

 

      \begin{th-u} 
         bla ble 
      \end{th-u}

 

\section{Conclusion}

 

   Le théorème importantref{label-th-AA} est à apprendre par 
   c\oe{}ur, le théorèmeref{label-th-aaa} peut être utile. 
%%%% fin exemple %%%%

Les trois packages suivants permettent de définir son propre style.

  1. Le package 'amsmath', disponible sur ftp://ftp.fdn.org/pub/CTAN//fonts/ams/amslatex/, définit l'environnement proof en insérant un carré. Des exemples de théorèmes se trouvent dans le fichier thmtest.tex
  2. Le package 'theorem', disponible sur CTAN. Voir le fichier theorem.dvi
  3. Il existe également le package 'ntheorem' disponible sur CTAN ou sur http://www.informatik.uni-freiburg.de/~may/Ntheorem/ntheorem.html. Il peut générer une liste des théoremes de la même manière que 'listoffigures', est compatible avec amsmath, permet d'ajouter des symboles à la fin d'un théoreme 'carré, q.e.d.), permet la référence à d'autres théoremes. Voir le fichier ntheorem.dvi
Voici un tableau récapitulatif :

Package amsthm theorem ntheorem
Retour à la ligne après le titre Ö Ö Ö
Pas de numérotation d'un théoreme Ö Ö
Position du numéro du théoreme Ö Ö
Style de la numérotation Ö
Ponctuation après le titre (:,.,etc) Ö Ö
Fonte du titre ou du corps Ö Ö Ö
Indentation du titre Ö
Espace vertical autour du théorème Ö Ö Ö

Table 12.1: Tableau récapitulatif des fonctionnalités des packages

12.30  Comment changer la taille de la fonte ?

Voici une solution présentée par T. Bouche et S.A. Zaimi :

Exemple :
 
\documentclass{report} 
\def\mathtitre#1{ 
\font\tenrm=cmr10   scaled \magstep#1 
\font\sevenrm=cmr7  scaled \magstep#1 
\font\fiverm=cmr5   scaled \magstep#1 
\font\teni=cmmi10   scaled \magstep#1 
\font\seveni=cmmi7  scaled \magstep#1 
\font\fivei=cmmi5   scaled \magstep#1 
\font\tensy=cmsy10  scaled \magstep#1 
\font\sevensy=cmsy7 scaled \magstep#1 
\font\fivesy=cmsy5  scaled \magstep#1 
\font\tenex=cmex10  scaled \magstep#1 
\textfont0=\tenrm \scriptfont0=\sevenrm \scriptscriptfont0=\fiverm 
\textfont1=\teni  \scriptfont1=\seveni  \scriptscriptfont1=\fivei 
\textfont2=\tensy \scriptfont2=\sevensy \scriptscriptfont2=\fivesy 
\textfont3=\tenex \scriptfont3=\tenex \scriptscriptfont3=\tenex 
}

 

\begin{document} 
        {\mathtitre0 $$A+B=C{BA}$$} 
        {\mathtitre1 $$A+B=C{BA}$$} 
        {\mathtitre2 $$A+B=C{BA}$$} 
        {\mathtitre3 $$A+B=C{BA}$$} 
        {\mathtitre4 $$A+B=C{BA}$$} 
        {\mathtitre5 $$A+B=C{BA}$$} 
\end{document} 
%%%% fin exemple %%%%

ATTENTION :
il ne faut pas que le bloc en police \mathtitreXX se trouve à cheval sur deux pages... cela mettrait le numéro de page (et les en-têtes-pieds de page) dans ladite police...

12.31  A quoi sert la commande \displaystyle ?

Elle permet de forcer LATEX à gérer les indices et les exposants comme si il était en mode mathématique isolé. Cela peut alors induire une modification locale de l'interligne courant.

Exemple :
 
Elle permet de forcer \LaTeX à gérer les indices et les 
exposants comme si il était en mode mathématique isolé 
$\displaystyle{\sum_{i=0}n x_i}$. Cela peut alors induire une 
modification locale de l'interligne courant. Ce texte contient 
un exemple probant. Le changement d'interligne est bien visible. 
%%%% fin exemple %%%%

12.32  Comment aligner des données sur le point décimal ?

Le package 'dcolumn' disponible sur ftp://ftp.fdn.org/pub/CTAN/macros/latex/required/tools/ permet d'aligner les nombres d'un tableau par rapport à leur point décimal.

On peut également utiliser un tableau avec une virgule comme séparateur de colonnes.

Exemple :
 
\
\begin{array}{r @{,} l} 
   1&2 \\ 
   233&456 \\ 
   x&y \\ 
\end{array} 
\
%%%% fin exemple %%%%

12.33  Comment gérer les espaces en mode mathématique ?

Voici les différents espaces prédéfinis :

Exemple :
 
\
\begin{array}{l} 
   sin cos \\ 
   sin \! cos \\ 
   sin \, cos \\ 
   sin \  cos \\ 
   sin \quad cos \\ 
   sin \qquad cos \\ 
\end{array} 
\
%%%% fin exemple %%%%
ce qui donne :


sin cos
sin cos
sin  cos
sin  cos
sin     cos
sin        cos

12.34  Comment ecrire un interieur proprement ?

Pour écrire un intérieur suivant les habitudes mathématiques françaises, Yannis Haralambous propose un package, ymath, disponible sur le CTAN dans ftp://ftp.fdn.org/pub/CTAN/macros/latex/contrib/supported/yhmath. On y trouve notamment les commandes \ring{} et \windering{} pour définir un intérieur.

Exemple :
 
\[ > \ring{A}=\widering{\bigcup\limits_{x\in A} \{x\}} > \]

%%%% fin exemple %%%% 

On pourra utiliser aussi \wideparen pour étendre ce genre de notation à d'autre caractères que le rond de l'intérieur.

12.35  Comment obtenir des accolades horizontales ?

\overbrace et \underbrace.

Exemple :
 
\
\overbrace{f(x)}{=0} + \underbrace{g(y)}_{=0} = 0 
\
%%%% fin exemple %%%%
ce qui donne :


=0

f(x)

 
+

g(y)
=0 
= 0

12.36  Comment obtenir des points de suspensions ?

En mode math, il existe différents types de points de suspension suivant leur orientation :

Exemple :
 
\
\begin{array}{ccc} 
   x_{11} & \cdots & x_{1p} \\ 
   \vdots & \ddots & \vdots \\ 
   x_{n1} & \cdots & x_{np} 
\end{array} 
\
%%%% fin exemple %%%%
ce qui donne :


x11
¼
x1p
:
···
:
xn1
¼
xnp

12.37  Comment mettre en page un système d'équations ?

On peut utiliser les commandes \left délimiteur et \right classiques devant un tableau.

Exemple :
 
\
\left \
\begin{array}{c @{=} c} 
    x & \sin a \cos b \\ 
    y & \sin a \sin b \\ 
\end{array} 
\right. 
\
%%%% fin exemple %%%%
ce qui donne :


ì
ï
í
ï
î
x
=sina cosb
y
=sina sinb

 

On peut utiliser le package 'mathenv', permettant de numéroter le système ou chaque équation du système, le compteur pouvant alors être personnalisé.

Exemples :
 
% numérotation du système 
\begin{System} 
  2x + y = 6 \\ 
  3x + y = 7 
\end{System}

 

%numérotation de chaque équation du système 
\begin{EqSystem} 
  2x + y = 6 \\ 
  3x + y = 7 
\end{EqSystem} 
%%%% fin exemple %%%%

On peut utiliser le package 'amsmath', permettant de placer des alternatives conditionnelles.

Exemple :
 
\begin{equation*} 
  |x|= 
     \begin{cases} 
        -x & \text{si $x$ est impair} \\ 
        x  & \text{si $x$ est pair ou nul} 
     \end{cases} 
\end{equation*} 
%%%% fin exemple %%%%
 

Le package 'cases' disponible sur ftp://ftp.fdn.org/pub/CTAN/macros/latex/contrib/other/misc/ gère les systèmes d'équations.

Exemple :
 
\begin{numcases}{|x|=} 
    x, & for $x \geq 0$\\ 
    -x, & for $x < 0$ 
\end{numcases} 
%%%% fin exemple %%%%
Voici une adaptation de ce package par J. Laurens :

%%%% debut macro %%%% 
%  fichier xtdeqnra.sty             ver 1.0    Decembre 1997 

%  Copyright (C) 1997 by Jerome LAURENS 

 %  Ces macros peuvent etre librement transmises, sans modification aucune 
%  tant que cette notice est intacte. 
%  Base' sur le paquetage cases.sty de D.Arseneau 
%  et la commande \eqnarray de Leslie Lamport et de l'e'quipe LaTeX3. 

%  On rajoute un parame`tre optionnel a` l'environnement eqnarray qui 
%  est un delimiteur gauche 

%  Exemple : source 

%  \begin{eqnarray}[\{] 
%   x_1 & = & valeur_1 \\ 
%   x_2 & = & valeur_2 \\ 
%   x_3 & = & valeur_3 
%  \end{eqnarray} 

%  Re'sultat 

%               /  x_1 = valeur_1           (1) 
%               | 
%               <  x_2 = valeur_2           (2) 
%               | 
%               \  x_3 = valeur_3           (3) 

%  Pour utiliser ce paquetage, 
 %  inclure  "\usepackage{xtdeqnra}"  apre`s \documentclass. 

 %  On peut utiliser l'environnement subequations pour avoir (1a), (1b) et(1c) 
% - - - - - 

  
\ProvidesPackage{xtdeqnra}[1997/12/05 version 1.0 ] 
  
%  - sauvegarde de l'environnement eqnarray 
\let\x@eqnarray\eqnarray 
\let\endx@eqnarray\endeqnarray 

%  - environnement de remplacement 
\def\eqnarray{\@ifnextchar[% 
  {\let\endeqnarray\endy@eqnarray\y@eqnarray} 
  {\let\endeqnarray\endx@eqnarray\x@eqnarray}} 

%  - nouvel environnement avec un de'limiteur a` gauche 
\def\y@eqnarray[#1]% 
\gdef\@leftdelimiter{#1} 
  \displaymath 
  \setbox\tw@\vbox 
    \bgroup 
      \stepcounter{equation}% 
      \def\@currentlabel{\p@equation\theequation}% 
      \global\@eqnswtrue 
      \m@th 
      \everycr{}% 
      \tabskip\@centering 
      \let\\\@eqncr 
      \halign to\displaywidth 
        \bgroup 
          \hskip \@ne\arraycolsep 
          \hfil$\displaystyle 
          \tabskip\z@skip{##}$% 
          \@eqnsel 
          &\global\@eqcnt\@ne 
          \hskip \tw@\arraycolsep 
          \hfil${##}$\hfil 
          &\global\@eqcnt\tw@ 
          \hskip\tw@\arraycolsep 
          $\displaystyle{##}$\hfil 
          \unskip\hfil 
          \tabskip\@centering% \unskip removes space if no explanations 
                  &\global\@eqcnt\thr@@ 
          \hb@xt@\z@\bgroup\hss##\egroup 
          \tabskip\z@skip 
          \cr 

\def\endy@eqnarray{% 
          \@@eqncr 
        \egroup % end \halign, which does not contain brace 
      \global\advance\c@equation\m@ne 
      \unskip\unpenalty 
      \unskip\unpenalty 
      \setbox\z@\lastbox % grab last line 
      \nointerlineskip 
      \copy\z@ % then put it back 
      \setbox\z@\hbox{\unhbox\z@}% 
      \global\dimen@i\wd\z@ 
    \egroup% end \vbox (box\tw@, box\z@ is restored to LHS) 
  \hbox to\displaywidth{% 
    \m@th % assemble the whole equation 
    \hskip\@centering 
    \hbox to\dimen@i{% 
      $\displaystyle% 
      \dimen@\ht\tw@ 
      \advance\dimen@\dp\tw@ % get size of brace 
      \setbox\z@\hbox{$\mathord{\left\@leftdelimiter\vcenter 
to\dimen@{\vfil}\right.}$}% 
      \dimen@\wd\z@ 
      \kern-\dimen@ 
      \box\z@% 
      \n@space % make brace 
      $\hfil} 
    \hskip\@centering % finished first part (filled whole line) 
    \kern-\displaywidth$% 
    \vcenter{\box\tw@}$% overlay the alignment 
  }% end the \hbox 
  \enddisplaymath 
  \global\@ignoretrue 

  
\endinput 
  
% fin du fichier xtdeqnra.sty 
%%%% fin macro %%%%
Le package 'delarray' disponible sur ftp://ftp.fdn.org/pub/CTAN/macros/latex/required/tools/ peut également être utilisé dans ce but.

12.38  Comment mettre en page des diagrammes de Feynman ?

Le package 'feynmf' disponible sur ftp://ftp.fdn.org/pub/CTAN/macros/latex/contrib/supported/feynmf/ permet de gérer la mise en page de diagrammes de Feynman.

12.39  Comment obtenir une fraction ?

LATEX propose la commande \frac{arg1}{arg2} dont l'argument arg1 est le numérateur et arg2 dénominateur.

Exemple :
 
en formule: 
$$\frac{\alpha}{6 + \sqrt{2}}$$ 
ou dans le texte $\frac{\alpha}{6 + \sqrt{2}}$. 
%%%% fin exemple %%%%
ce qui donne :

en formule :
 a

6 + Ö2
ou dans le texte [(a)/(6 + Ö2)].

12.40  Comment obtenir une racine ?

Il existe la commande \sqrt[n]{arg}, qui représente la racine nième de arg. [n] est un paramètre optionnel.

Exemple :
 
$$ \sqrt[3]{\sqrt{\sqrt[\gamma]{x+y}}} $$ 
%%%% fin exemple %%%%
ce qui donne :


3 æ
Ö
 
 

  æ
Ö

g
Ö

x+y
 
 

12.41  Quels sont les délimiteurs disponibles ?

12.42  Comment appeler les lettres grecques ?

Il suffit de faire précéder leur nom en toutes lettres d'un backslash :

12.43  Quels sont les accents mathématique standards ?

Remarque :
pour placer un accent sur un i ou un j, il faut utiliser les commandes \imath et \jmath qui permettent de supprimer le point par défaut.
Il en existe également d'autres pour couvrir des ensembles de lettres :

12.44  Comment superposer deux symboles ?

Il existe pour cela la commande

\stackrel{dessus}{dessous}
Exemple :
 
\
\stackrel{\theta}{\Rightarrow} B 
\
%%%% fin exemple %%%%
ce qui donne :


A q
Þ
 
B

12.45  Comment définir un nouvel opérateur ou symbole ?

Il existe pour cela la commande \DeclareMathOperator du package 'amsmath', disponible sur ftp://ftp.fdn.org/pub/CTAN/fonts/ams/amslatex/. Elle s'utilise dans le préambule.

Exemple :
 
\DeclareMathOperator{\init}{init} 
\
\init f = 0 \mathrm{\ au \ lieu \ de:} init f = 0 
\
%%%% fin exemple %%%%
On peut également utiliser \newcommand, toujours dans le préambule du document.

Exemple :
 
\newcommand{\affecte}{\mathrel{:=}} 
$x \affecte 0$ 
$y \affecte 1$ 
%%%% fin exemple %%%%
On peut utiliser \def.

Exemple :
 
\makeatletter 
\def\log{\mathop{\operator@font log}\nolimits} 
\makeatother 
$\log 28 = 1.447158$ 
%%%% fin exemple %%%%
Pour définir un nouveau symbole de plusieurs caractères, il faut utiliser \mathit.

12.46  Comment obtenir des lettres grecques en gras ?

On peut utiliser le package 'amsmath', disponible sur ftp://ftp.fdn.org/pub/CTAN/fonts/ams/amslatex/.

Exemple :
 
... 
\usepackage{amsmath} 
... 
\begin{document} 
... 
$\boldsymbol{\alpha}$ 
%%%% fin exemple %%%%
On peut également utiliser le package 'bm' disponible sur ftp://ftp.fdn.org/pub/CTAN/macros/latex/contrib/supported/bm/.

Exemple :
 
... 
\usepackage{bm} 
... 
\begin{document} 
... 
$\bm{\alpha}$ 
%%%% fin exemple %%%%

12.47  Comment tracer un tableau de variations ?

Avec l'environnement array en mode mathématiques (cf. question ).

Exemple :
 

\begin{array}{|c|ccccr|} 
\hline 
x     & -\infty  &    & 0  &     & +\infty  \\ 
\hline 
f'(x) & 5         &  + & 0  & - &  -10 \\ 
\hline 
 &  & & 10 & & \\       % ligne des valeurs "max" 
f(x) &  &\nearrow   &  &\searrow  & \\   % flèches 
& -\infty & & & & -10 \\          % ligne des valeurs "min" 
\hline 
\end{array} 

%%%% fin exemple %%%%
ce qui donne :

x
-¥
0
+¥
f¢(x)
5
+
0
-
-10
10
f(x)
\nearrow
\searrow
-¥
-10

 

On peut faire de plus jolies flèches (redimensonnables) avec les commandes graphiques.

Pour cela, il faut choisir une unité de longueur avant par:

        \unitlength=1cm
et remplacer par exemple \nearrow par (flèches de 2cm de large et de haut)

Exemple :
 
\begin{minipage}{2cm} 
  \begin{picture}(2,2) 
     \put(0,0){\vector(1,1){2}} 
 \end{picture} 
\end{minipage} 
%%%% fin exemple %%%%
ce qui donne :


Picture 1

Chapter 13
Références Croisées

La gestion facile des réferences aux équations, aux tables et aux figures....

13.1  Quelles sont les commandes de base ?

LATEX permet de gérer automatiquement des références croisées grâce aux commandes suivantes :

L'utilisation de ces commandes nécessite alors plusieurs (deux à trois) compilations successives, jusqu'à ce que toutes les références croisées soient correctes.

ATTENTION :
pour les figures et les tables, la commande \label doit se situer juste après un \caption ou en faire partie pour que les références soient correctes.
ATTENTION :
lorsque l'on travaille avec une version inférieure à 3.6 du package babel et l'option french, sachant que cette dernière redéfinit certains caractères de ponctuation tels que :;?! pour gérer automatiquement les espaces, il ne faut pas les utiliser dans des noms de label : \label{section:Introduction} est interdit.
Exemple
:
Remplir la tableref{table-add}, pagepageref{table-add} 
\begin{table}[htbp] 
   \begin{center} 
      \begin{tabular}{c|c} 
         \hline 
          Question & Réponse  \\ 
         \hline 
         1 + 12 & \\ 
         45 + 76 & \\ 
      \end{tabular} 
      \caption{Additions. \label{table-add}} 
   \end{center} 
\end{table} 
%%%% fin exemple %%%%
ce qui donne :

Remplir la table 13.1, page pageref

Question Réponse
1 + 12
45 + 76
Table 13.1: Additions.

Le package 'prettyref' disponible sur ftp://ftp.fdn.org/pub/CTAN/macros/latex/contrib/supported/prettyref/ permet de définir ses propres styles de référence.

13.2  Comment obtenir un renvoi à une page ?

Il suffit d'utiliser la commande \pageref présentée ci-dessus.

Le package 'varioref', de F. Mittelbach, disponible sur ftp://ftp.fdn.org/pub/CTAN/macros/latex/required/tools/, permet de n'appeler une référence de page que si l'objet flottant auquel elle se rattache n'est pas présent sur la page courante. Pour cela, ce package définit la commande \vpageref qui ne fera apparaître le numéro de la page référencée que si ce n'est pas la page courante. Cette commande peut également introduire des expressions du type ``sur la page précédente'', ``sur cette même page'', etc. Dans ce cas, on précise la langue voulue en option lors de l'appel du package.

Exemple :
 
\documentclass{report}

\usepackage{french}

\usepackage[french]{varioref}

 

\begin{document} 
Remplir la tableref{table-add}vpageref{table-add} 
\begin{table}[htbp] 
   \begin{center} 
      \begin{tabular}{c|c} 
         \hline 
          Question & Réponse  \\ 
         \hline 
         1 + 12 & \\ 
         45 + 76 & \\ 
      \end{tabular} 
      \caption{Additions. \label{table-add}} 
   \end{center} 
\end{table} 
\end{document} 
%%%% fin exemple %%%%

Remarque :
Ce package est incompatible avec les styles de page sans numérotation (e.g. empty).

13.3  Comment obtenir des références croisées à partir de plusieurs sources ?

Le package 'xr', de D. Carlisle, permet de faire des références d'un document à l'autre. Il est disponible sur ftp://ftp.fdn.org/pub/CTAN/macros/latex/required/tools/. Le document contenant les labels est appelé dans le document contenant les références via la commande \externaldocument. La commande \externaldocument[KEY]{DOCUMENT} permet même de gérer les références de même nom.

Exemple :
 
        \documentclass[a4paper,12pt]{book} 
        \begin{document} 
           \section{Ma section.\label{ma-sect}} 
        \end{document}
        \documentclass[11pt]{report} 
        \usepackage{xr} 
        \externaldocument{essai} 
        \begin{document} 
        Pour plus de détails, voir le document \textit{essai}, 
        paragrapheref{ma-sect} 
        \end{document} 
       %%%% fin exemple %%%%

13.4  Comment définir des liens hypertexte sous LATEX ?

Le package 'hyperref' de S. Rahtz disponible sur ftp://ftp.fdn.org/pub/CTAN/macros/latex/contrib/supported/hyperref/ permet de définir des liens hypertexte pour PDF. Il définit pour cela des commandes de type \ref{}.

Voir également les packages 'hyper' et 'hyperlatex' disponibles sur ftp://ftp.fdn.org/pub/CTAN/macros/latex/contrib/supported/hyper/.

La distribution Y&Y TEX System supporte des liens en TEX debuggable dans le previewer qui seront automatiquement convertis en liens PDF pour Adobe.

13.5  Comment faire référence à ses propres compteurs ?

Lorsque l'on définit son propre environnement avec ses popres compteurs, pour que les commandes \label et \ref puissent les prendre en compte, il suffit d'utiliser \refstepcounter au lieu de \stepcounter pour incrémenter le compteur auquel on fait référence.

13.6  Comment faire référence au début d'un flottant et pas au titre?

Lorsque l'on utilise les techniques classiques de références, le \label fera référence au \caption puisque c'est là que le compteur est incrémenté. Mais le \caption est souvent en bas de figure (ou de tableau, ou de tout autre flottant). Ca ne pose pas de problème pour un document papier, mais pour de l'hypertexte, il serait plus intéressant d'utiliser la solution proposée par Michel Bovani :

Exemple :
 
\begin{figure} \refstepcounter{figure}\label{fig-numero13}\addtocounter{figure}{-1}

\includegraphics{Figure13.pdf}

\caption{C'est ma figure.}

\end{figure}

%%%% fin exemple %%%%

Chapter 14
Table des matières

Tout savoir pour donner un plan complet de son document.

14.1  Comment générer une table des matières ?

LATEX propose la commande \tableofcontents à appeler à l'endroit où l'on veut insérer la table des matières.

14.2  Comment générer plusieurs tables des matières ?

Le package 'french' propose la commande sommaire[n] qui permet d'inclure une table des matière de niveau de profondeur n.

Remarque :
dans ce cas \tableofcontents reste utilisable.
Le package 'shorttoc' ci-dessous, de J.P. Drucbert, définit la commande \shorttableofcontents{TITRE}{NIVEAU}.

Remarque :
dans ce cas \tableofcontents reste également utilisable.
%%%% debut macro %%%% 
%shorttoc.sty 
\ProvidesPackage{shorttoc}% 
[1995/01/04 v1.0 Short table of contents package (JPFD)] 
\def\@startshorttoc#1{\bgroup 
  \makeatletter 
  \@input{\jobname.#1}% 
  \global\@nobreakfalse \egroup} 
  \def\shorttableofcontents#1#2{\bgroup\c@tocdepth=#2\@restonecolfalse 
  \if@twocolumn\@restonecoltrue\onecolumn\fi 
  \@ifundefined{chapter}% 
  {\section*{{#1} 
        \@mkboth{\uppercase{#1}}{\uppercase{#1}}}}% 
  {\chapter*{{#1} 
        \@mkboth{\uppercase{#1}}{\uppercase{#1}}}}% 
  \@startshorttoc{toc}\if@restonecol\twocolumn\fi\egroup} 
%%%% fin macro %%%%
Le package 'minitoc', disponible sur ftp://ftp.fdn.org/pub/CTAN/macros/latex/contrib/supported/minitoc/, permet de construire une minitable des matières au début de chaque chapitre sous les classes 'book' et 'report'. Pour l'utiliser, il faut appeler les commandes \dominitoc avant la commande \tableofcontents habituelle. La commande \faketableofcontents permet de ne garder que les tables des matières locales et remplace alors la commande \tableofcontents. La commande \minitoc doit être appelée après chaque commande de début de chapitre \chapter dans lequel on veut inclure une table locale. A chaque appel de minitoc correspond un fichier .mtc<n> où n est le numéro du chapitre.

Le compteur minitocdepth permet de fixer la profondeur des tables des matières désirées.

Exemple :
 
\documentclass{report} 
\usepackage{minitoc} 
\usepackage{french} 
\setcounter{minitocdepth}{1} 
\begin{document}

 

\dominitoc 
\tableofcontents

 

\chapter{Introduction.} 
\minitoc

 

\section{Quelques rappels.} 
   Texte.

 

   \subsection{La gravité.}

 

      Texte sur la gravité.

 

   \subsection{Loi de Newton.}

 

      Deux corps s'attirent en raison inverse du carré 
      de leur distance et proportionnellement à leur 
      masse.

 

\section{Nouveaux théorèmes.}

 

   Encore des résultats.

 

\section{Approfondissement.}

 

   Toujours plus loin.

 

pour ne faire apparaître que les titres de sections. 
\end{document} 
%%%% fin exemple %%%%

Le bon fonctionnement des numérotations peut nécessiter jusqu'à trois compilations enchaînées.

ATTENTION :
minitoc sait faire:
Par contre, il ne fait pas les tables des matières par section (et en dessous) pour les classes book et report, ni les tables des matières par sous-section (et en dessous) pour la classe article.

14.3  Comment ajouter une entrée dans la table des matières ?

Pour ajouter un titre de paragraphe qui n'apparaît pas par défaut dans la table des matières, il faut utiliser la commande \addcontentsline{1}{2}{3} où :

Exemple :
 
\bibliography{sdf} 
\addcontentsline{toc}{chapter}{Bibliographie.} 
%%%% fin exemple %%%%

14.4  Comment changer le titre de la table des matières ?

On peut utiliser la commande :

\renewcommand{\contentsname}{Contenu du document.}
Remarque :
suivant les packages utilisés, il faut placer cette définition avant ou après le \begin{document}.

14.5  Comment changer la profondeur de la table des matières ?

Les sections sont numérotées si leur profondeur est inférieure à secnumdepth.

Elles apparaissent dans la table des matières si leur profondeur est inférieure à tocdepth.

Il suffit de changer les valeurs de ces deux compteurs. Il peut alors être nécessaire d'enchaîner deux compilations LATEX.

Exemple :
 
\documentclass[a4paper]{article} 
\usepackage{french} 
\pagestyle{empty} 
% Profondeur de \subsubsection = 3 
\setcounter{tocdepth}{3}     % Dans la table des matieres 
\setcounter{secnumdepth}{3}  % Avec un numero.

 

\begin{document} 
\tableofcontents 
\vspace{.5cm} 
\hrule

 

\part{Aaa.} 
\section{aaa.} 
   \subsection{bbb.} 
      \subsubsection{ccc.} 
\end{document} 
%%%% fin exemple %%%%

14.6  Comment gérer des chapitres de préface, d'introduction, et de conclusion non numérotés ?

La classe book définit les commandes \frontmatter, \mainmatter et \backmatter qui permettent de faire cela.

Exemple :
 
\documentclass[a4paper,12pt,openany]{book} 
\usepackage{french} 
\def\TEXTE{Bla bla bla...} 
\begin{document} 
\frontmatter 
%\begin{abstract}  Bla, bla... \end{abstract} 
\chapter{Préface} \TEXTE 
\chapter{Introduction} \TEXTE 
\mainmatter 
\chapter{Début} \TEXTE 
\chapter{Suite} \TEXTE 
\chapter{Fin}  \TEXTE 
\appendix 
\chapter{Résultats élémentaires} 
\chapter{Résultats compliqués} 
\backmatter 
\chapter{Conclusion}  \TEXTE 
\tableofcontents 
\end{document} 
%%%% fin exemple %%%%
L'autre solution consiste à définir ses propres commandes \introduction, \preface, etc, à partir des définitions de \chapter sans prendre en compte la numérotation.

On peut également utiliser les commandes étoilées et gérer soit même la table des matières et éventuellement les en-têtes et bas de page.

Exemple :
 
\chapter*{Intro} 
\addcontentsline{toc}{chapter}{Intro...} 
%%%% fin exemple %%%%

14.7  Comment enlever le numérotation des pages de tdm ?

Il suffit d'inclure les lignes suivantes dans le préambule du document :

%%%% debut macro %%%% 
\makeatletter 
\def\addcontentsline@toc#1#2#3{% 
   \addtocontents{#1}{\protect\thispagestyle{empty}}% 
   \addtocontents{#1}{\protect\contentsline{#2}{#3}{\thepage}}} 
\def\addcontentsline#1#2#3{% 
  \expandafter\@ifundefined{addcontentsline@#1}% 
  {\addtocontents{#1}{\protect\contentsline{#2}{#3}{\thepage}}} 
  {\csname addcontentsline@#1\endcsname{#1}{#2}{#3}}} 
\makeatother 
%%%% fin macro %%%%

Chapter 15
Bibliographie sous LATEX

La plupart des documents scientifiques utilisent les références bibliographiques. LATEX permet une gestion aisée et pratique.

15.1  Comment construire une bibliographie ?

Il existe deux techniques pour construire une bibliographie :

  1. la première consiste à l'intégrer dans le corps même du document (fichier.tex),
  2. la seconde consiste à la définir dans un fichier particulier (fichier.bib) puis à l'appeler dans le corps du document.
Technique on-line : Il suffit d'appeler l'environnement thebibliography et de définir des bibitem.

Exemple
:
\begin{thebibliography}{2} 
   \bibitem[label]{cle} Auteur, TITRE, editeur, annee 
   \bibitem[LAM94]{lam1} L. LAMPORT, {\it \LaTeX : A Document 
   preparation system, Addison-Wesley, 1994} 
\end{thebibliography} 
%%%% fin exemple %%%%
[label] définit le label qui apparaîtra dans la bibliographie et dans le texte après appel de la commande \cite{cle}.

BibTEX permet de construire simplement une bibliographie. La technique consiste alors à écrire un ou plusieurs fichiers.bib qui seront appelés dans le document source par la commande \bibliography{fichier1, fichier2,...}. BibTEX est disponible sur ftp://ftp.fdn.org/pub/CTAN/biblio/bibtex.

Le style de la bibliographie doit être défini par \bibliographystyle{xxx} avec xxx valant généralement plain ou alpha, mais il existe aussi d'autres styles. La commande \bibliographystyle doit être appelée après \begin{document}.

Le contenu d'un fichier.bib est un ensemble de déclarations du style :

@ARTICLE{cle, 
author = "liste-noms-auteur", 
title = "titre-article", 
journal = "nom-journal", 
year = "annee-parution", 

% OPTIONAL FIELDS 
% volume = "", 
% number = "", 
% pages =  "", 
% month =  "", 
% note =  "",
La syntaxe des différentes déclarations possibles peut se trouver dans toute bonne documentation LATEX ou dans les menus des éditeurs tels que emacs ou Alpha.

Losqu'on utilise BibTEX, il faut alors enchaîner plusieurs compilations pour que toutes les références soient exactes. Généralement, il faut exécuter une passe LATEX, une passe Bibtex et encore deux passes LATEX.

Pour pouvoir inclure dans la bibliographie des références à des documents qui ne sont pas cités explicitement par une commande \cite, il suffit d'inclure la commande \nocite{la-ref-non-citee} avant la commande \bibliography{fichier_biblio}. Pour faire apparaître toutes les références non citées sans toutes les énumérer, on peut utiliser \nocite{*}.

BIBDB, disponible sur CTAN, est un gestionnaire de bibliographie BIBTEX, facile à manipuler qui tourne sous DOS.

Bibview est un programme tournant sous X11 et permettant la saisie et la consultation de fichiers bibtex.

15.2  Comment gérer plusieurs bibliographies ?

Le package 'chapterbib' permet d'inclure une bibliographie par chapitre. Il est disponible sur ftp://ftp.fdn.org/pub/CTAN/macros/latex/contrib/supported/cite/. Les bibliographies réparties peuvent alors apparaître comme section de ce chapitre grâce à la commande :

\usepackage[sectionbib]{chapterbib}.
Le package 'bibunits', de A. Fernandez, permet de partager une bibliographie en plusieurs unités, chacune attenante à une section, ou à un chapitre d'un article ou d'un livre donné.

Exemple :
 
////////////// fichier rapport.tex ///////////// 
\documentclass[a4paper]{report} 
\usepackage{bibunits} 
\usepackage{french} 
\begin{document} 
\chapter{Algorithmes.} 
Présentation d'algorithmes. 
\nocite* 
\bibliographystyle{alpha} 
\bibliography{alg} 
\chapter{Géométrie.} 
Texte sur la géométrie. 
\nocite* 
\bibliographystyle{plain} 
\bibliography{geom} 
\end{document} 

////////////// fichier alg.bib ///////////// 
@UNPUBLISHED{algg, 
author = MPK, 
title = Algorithmes et rythmes, 
note = RAS, 
}

 

////////////// fichier geom.bib ///////////// 
@INBOOK{geomm, 
author = Me again, 
title = Principes de géométrie, 
chapter = Les droites, 
publisher = Home publishing, 
year = 1997, 

%%%% fin exemple %%%%

Le package 'bibtopics', de P. Basso, permet de contruire une bibliographie générale comportant des rubriques diverses (livres, articles, thèses, ...) ou des sujets divers (physique, mathématiques, ...). Il est disponible par ftp sur ftp://ftp.gutenberg.eu.org/pub/gut/contrib/.

Remarque
: pour les versions récentes de LATEX, il peut être nécessaire d'inclure dans le préambule du document les lignes :
                \makeatletter

                \let\if@openbib\iffalse

                \makeatother

Voici un petit fichier 'multibib.sty' de V. Henn, créé en recopiant les définitions LATEX des commandes relatives à la gestion des bibliographies, et en les rendant paramétrables.

Les commandes qui ont été définies sont :

%%%% debut macro %%%% 
%% A sauver dans  multibib.sty 
%%%%%% biblio generique %%%%%%%% 
  
\newcommand\newbibliographyGENE[1]{% 
\expandafter\expandafter\expandafter\newwrite\@nameuse{@#1aux} 
\immediate\expandafter\expandafter\openout\@nameuse{@#1aux}=#1.aux 
%\expandafter\expandafter\expandafter\show\@nameuse{@#1aux} 

  
\def\citeGENE#1{\@ifnextchar [{\@tempswatrue\@citexGENE{#1}}% 
{\@tempswafalse\@citexGENE{#1}[]}} 
  
\def\@citexGENE#1[#2]#3{\if@filesw\immediate\expandafter% 
\expandafter\expandafter\write\@nameuse{@#1aux}{\string% 
\citation{#3}}\fi 
 \let\@citea\@empty 
 \@cite{\@for\@citeb:=#3\do 
  {\@citea\def\@citea{,\penalty\@m\ }% 
   \def\@tempa##1##2\@nil{\edef\@citeb{\if##1\space##2% 
   \else##1##2\fi}}% 
   \expandafter\@tempa\@citeb\@nil 
   \@ifundefined{b@\@citeb}{{\reset@font\bf ?}\@warning 
    {Citation `\@citeb' on page \thepage \space undefined}}% 
   \hbox{\csname b@\@citeb\endcsname}}}{#2}} 
  
\def\bibliographyGENE#1#2{\if@filesw\immediate\expandafter% 
\expandafter\expandafter\write\@nameuse{@#1aux}{\string% 
\bibdata{#2}}\fi 
  \@input{#1.bbl}} 
  
\def\bibliographystyleGENE#1#2{\if@filesw\immediate\expandafter% 
\expandafter\expandafter\write\@nameuse{@#1aux} 
    {\string\bibstyle{#2}}\fi} 
  
\def\nociteGENE#1#2{\@bsphack 
  \if@filesw\immediate\write\@nameuse{@#1aux}{\string% 
  \citation{#2}}\fi 
  \@esphack} 
%%%% fin macro %%%% 
 
Exemple d'utilisation :
Les fichiers gene.bib et speci.bib contiennent respectivement une référence bibliographie de clé cle-gene et cle-speci.
\documentclass{report} 
\usepackage{multibib} 
\usepackage{french} 
\begin{document} 
\newbibliographyGENE{Toto} 
\newbibliographyGENE{Titi} 
\chapter{Introduction.} 
blabla bla \citeGENE{Toto}{cle-gene} et 
\citeGENE{Titi}{cle-speci} rebla rebla rebla 
\bibliographystyleGENE{Toto}{plain} 
\bibliographyGENE{Toto}{gene} 
\bibliographystyleGENE{Titi}{alpha} 
\bibliographyGENE{Titi}{speci} 
\end{document}
il faut alors compiler de la manière suivante :

latex fichier-source 
bibtex Toto 
bibtex Titi 
latex fichier-source 
latex fichier-source 
%%%% fin exemple %%%%

15.3  Comment changer de langue dans une bibliographie ?

Il existe des versions francisées des styles bibliographiques (fplain, falpha, fralpha, frealfullname...) qui ne sont pas parfaits mais qui peuvent être modifiés facilement. Voir la distribution GUTenberg.

Exemple :
 
\documentstyle{falpha} 
%%%% fin exemple %%%%
Sachant que les items d'une bibliographie sont des paragraphes, ils peuvent commencer par la spécification d'une langue. Si on utilise le package 'french', on dispose des commandes \french et \english et si l'on utilise le package 'babel', il faut utiliser la commande \selectlanguage{} (cf. paragraphe ).

Voir également le package 'frbib' pour les bibliographies françaises.

Le package 'mlbib' disponible sur ftp://ftp.fdn.org/pub/CTAN/macros/latex/contrib/supported/mlbib/ permet de gérer des bibliographies multilingues.

15.4  Comment renvoyer une référence en note de bas de page ?

Pour remplacer une référence insérée dans le texte par la commande \cite, par un appel à une note de bas de page contenant une référence succinte à un document (la référence détaillée apparaissant toujours dans la bibliographie), il faut redéfinir la commande \cite dans le préambule du document par :

%%%% debut macro %%%% 
\makeatletter 
\def\@cite#1#2{% 
  \footnote{#1\if@tempswa, #2\fi}} 
\makeatother 
%%%% fin macro %%%%
Les informations reportées dans la note de bas de page dépendent du style de bibliographie choisi.

Le package 'overcite', de D. Arseneau, est disponible sur ftp://ftp.fdn.org/pub/CTAN/macros/latex/contrib/supported/cite/. Il permet de faire référence à des documents par un exposant.

Le problème qui se pose alors est de distinguer les références, des notes de bas de page. Si le document contient moins de 10 notes de bas de page, on peut utiliser la commande \renewcommand{\thefootnote}{\fnsymbol{footnote}}. Sinon pour disposer de 10 notes de bas de page par page, il existe le package 'footmisc' de R. Fairbairns disponible sur ftp://ftp.fdn.org/pub/CTAN/macros/latex/contrib/supported/footmisc/.

Exemple :
 
\usepackage[perpage,symbol]{footmisc} 
%%%% fin exemple %%%%
Le package 'camel' disponible sur ftp://ftp.fdn.org/pub/CTAN/macros/latex/contrib/supported/camel/, propose différents styles de bibliographies renvoyant à des notes de bas de page.

E. Domenjoud a également écrit le package 'footbib', disponible sur CTAN, qui permet de faire cela. Eric.Domenjoud@loria.fr.

15.5  Comment faire référence à un document ?

Il suffit d'utiliser la commande \cite avec en paramètre la clé du document (cf. paragraphe ). La commande \cite peut prendre un texte en option.

Exemple :
 
Voir: \cite[chapitre 1]{cle}. 
%%%% fin exemple %%%%

15.6  Comment grouper des références multiples ?

Le package 'cite' de D. Arseneau est disponible sur ftp://ftp.fdn.org/pub/CTAN/macros/latex/contrib/supported/cite/. Il permet de trier et de grouper les références multiples. Par exemple, il permet automatiquement de générer [1-6] au lieu de lister [1, 2, 3, 4, 5, 6].

Il existe également les packages 'mcite' et 'rangecite' (pour LATEX2.09) disponibles sur ftp://ftp.fdn.org/pub/CTAN/macros/latex/contrib/supported/mcite/ et dans ftp://ftp.fdn.org/pub/CTAN/macros/latex209/contrib/misc/.

15.7  Comment changer le titre de la bibliographie ?

Suivant la classe de document que l'on utilise (article, book, report, ...), il faut redéfinir :

\renewcommand{\bibname}{R\'ef\'erences}
ou

\renewcommand{\refname}{R\'ef\'erences}
dans le préambule.

Remarque
: avec certains packages comme french, il faut placer cette nouvelle définition de commande après le \begin{document}.

15.8  Comment changer le style de la bibliographie ?

Il existe un certain nombre de style par défaut (alpha, plain, unsrt, abbrrv) que l'on peut appeler par la commande \bibliographystyle{xxx} placée après la commande \begin{document}.

Remarque
: Certains styles comme plain, acm ou apalike trient alphabétiquement les références d'une bibliographie suivant le premier auteur de chaque référence. D'autres comme alpha utilisent la première lettre du nom de chacun des auteurs multiples.
Il existe également le package 'custom-bib' disponible sur ftp://ftp.fdn.org/pub/CTAN/macros/latex/contrib/supported/custom-bib/ qui permet de définir ses propres styles sans avoir à se plonger dans le language BibTEX. Le programme est interactif.

De même le package 'natbib' est disponible sur ftp://ftp.fdn.org/pub/CTAN/macros/latex/contrib/supported/natbib. Il permet entre autres de classer la bibliographie de plusieurs façons différentes, notamment par ordre alphabétique pour les auteurs, par l'année de parution, etc.

Pour remplacer la numérotation [1] par 1., on peut utiliser la commande :

\makeatletter 
\renewcommand{\@biblabel}[1]{\quad #1.} 
\makeatother
Pour supprimer toute numérotation, il faut utiliser de la même manière :

\makeatletter 
\renewcommand{\@biblabel}[1]{} 
\makeatother
Le package 'easybib' disponible sur ftp://ftp.fdn.org/pub/CTAN/macros/latex/contrib/supported/easy/ permet par une syntaxe simple de définir ses propres styles bibliographiques.

15.9  Comment construire une bibliographie à partir de plusieurs fichiers .bib ?

Pour cela il existe le package 'biblist' de J. chrod, disponible sur ftp://ftp.fdn.org/pub/CTAN/macros/latex209/contrib/biblist/.

Exemple :
 
\documentclass[12pt]{article} 
\usepackage{biblist} 
\begin{document} 
\nocite{*} 
\bibliographystyle{plain} 
\bibliography{bibfile1,bibfile2,...} 
\end{document} 
%%%% fin exemple %%%%

15.10  Comment utiliser la commande \cite dans un \caption ?

On peut mettre la commande \cite dans un \caption à condition de la protéger : \protect\cite.

15.11  Comment référencer une thèse française ou un mémoire ?

Il faut définir de nouvelles références. Pour cela, copier le fichier .bst dans un autre fichier .bst avec un nom différent et rechercher PhD thesis puis remplacer par Thèse de doctorat.

Exemple :
 
FUNCTION {thesedoc} 
{ output.bibitem 
format.authors "author" output.check 
new.block 
format.btitle "title" output.check 
new.block 
"Th\`ese de Doctorat" format.thesis.type output.nonnull 
school "school" output.check 
address output 
format.date "year" output.check 
new.block 
note output 
fin.entry 

%%%% fin exemple %%%%
Idem pour les mémoires de DEA :

FUNCTION {memdea} 
{ output.bibitem 
  format.authors "author" output.check 
  new.block 
  format.title "title" output.check 
  new.block 
  "M\'emoire de D.E.A." format.thesis.type output.nonnull 
  school "school" output.check 
  address output 
  format.date "year" output.check 
  new.block 
  note output 
  fin.entry 
}

15.12  Comment supprimer la virgule supplémentaire dans une liste d'auteurs ?

Par défaut, dans une liste d'auteurs le ``et'' qui introduit le dernier auteur est précédé d'une virgule. Pour la supprimer, il faut modifier la fonction format.names dans un nouveau fichier .bst de la manière suivante :

%%%% debut macro %%%% 
FUNCTION {format.names} 
{ 's :=3D 
  #1 'nameptr :=3D 
  s num.names$ 'numnames :=3D 
  numnames 'namesleft :=3D 
    { namesleft #0 > } 
    { s nameptr "{ff}{vv}{ll}{, jj}" format.name$ 't :=3D 
      nameptr #1 > 
        { namesleft #1 > 
            { ", " * t * } 
            { namesleft #2 > 
                { "," * } 
                'skip$ 
              if$ 
              t "others" =3D 
                { " etal." * } 
                { " and " * t * } 
              if$ 
            } 
          if$ 
        } 
        't 
      if$ 
      nameptr #1 + 'nameptr :=3D 
      namesleft #1 - 'namesleft :=3D 
    } 
  while$ 

%%%% fin macro %%%%

15.13  Comment configurer la commande \cite ?

Le package 'cite' disponible sur ftp://ftp.fdn.org/pub/CTAN/macros/latex/contrib/supported/cite/ offre certaines possibilités.

Exemple :
 
\usepackage{cite} 
\renewcommand\citepunct{;\penalty999\ } 
%%%% fin exemple %%%%
Pour afficher une clé à la place du label [??] dans le cas où LATEX/BibTex ne trouve pas la clé spécifiée dans la bibliographie, on peut utiliser la macro suivante :

%%%% debut macro %%%% 
\def\@citex[#1]#2{% 
 \let\@citea\@empty 
 \@cite{\@for\@citeb:=#2\do 
  {\@citea\def\@citea{,\penalty\@m\ }% 
   \edef\@citeb{\expandafter\@firstofone\@citeb}% 
   \if@filesw\immediate\write\@auxout{\string\citation{\@citeb}} 
   \fi 
%%%% Ligne originale %%%% 
%     \@ifundefined{b@\@citeb}{\mbox{\reset@font\bfseries ?}% 
%%%% Ligne modifiée %%%% 
   \@ifundefined{b@\@citeb}{\mbox{\reset@font\bfseries \@citeb}% 
     \G@refundefinedtrue 
     \@latex@warning 
       {Citation `\@citeb' on page \thepage \space undefined}}% 
     {\hbox{\csname b@\@citeb\endcsname}}}}{#1}} 
%%%% fin macro %%%%

15.14  Comment construire une liste d'auteurs ?

Pour séparer les éléments d'une liste d'auteurs, il faut utiliser ``and''.

Exemple :
 
Dewitt, D.J. and Naughton, J. 
%%%% fin exemple %%%%

15.15  Comment spécifier un tri dans une bibliographie ?

Il existe l'outil BibTool disponible sur ftp://ftp.fdn.org/pub/CTAN/biblio/bibtex/utils/bibtool/. BibTool est un outil de manipulation de bases de données BibTEX et il permet en particulier de spécifier différents ordres de tri pour une bibliographie.

Exemples :
 
bibtool -s file.bib -o the_new_bibtex_file.bib
pour trier suivant les auteurs et les titres

bibtool -s -- 'sort.format={%d(year)}' file.bib -o the_new_bibtex_file.bib
pour trier par ordre croissant des années de parution

bibtool -s -- 'sort.format={%s($type)%N(author)}' gn-publ.bib
pour trier par classe de document et par auteur.

L'appel du fichier ainsi trié se fait alors par exemple par :

\documentclass{article} 
\begin{document} 
\nocite{*} 
\bibliographystyle{unsrt} 
\bibliography{the_new_bibtex_file} 
\end{document} 
%%%% fin exemple %%%%

15.16  Comment référencer les pages contenant des citations ?

Il faut utiliser le package 'backref' disponible sur CTAN.

15.17  Où trouver des styles de bibliographie ?

Le package 'biblist' de J. Schrod offre différents styles de bibliographie prédéfinis. Il est disponible sur ftp://ftp.fdn.org/pub/CTAN/macros/latex209/contrib/biblist/.

Le package 'apacite' est disponible sur ftp://ftp.fdn.org/pub/CTAN/biblio/bibtex/contrib/.

Le package 'bbtbase' disponible sur CTAN propose des styles bibliographiques de base.

De nombreux autres styles sont disponibles sur CTAN.

15.18  Comment faire des références croisées ?

Il faut utiliser le champ crossref de BibTEX.

Exemple :
 
@InProceedings{contejean96rta, 
  author =      "Evelyne Contejean and Claude March\'e", 
  title =       "{CiME: Completion Modulo $E$}", 
  crossref =    "rta96", 
  pages =       "416--419", 
  year =        1996, 
  note =        "System Description", 
  ftp =         "ftp://ftp.lri.fr/LRI/art/march/cime-rta96.ps.gz", 
  abstract =    "http://www.lri.fr/marche/cime-rta96.html" 

@Proceedings{rta96, 
  title         = "7th International Conference on ...", 
  booktitle     = "7th International Conference on ...", 
  editor        = "Harald Ganzinger", 
  publisher     = SV, 
  year          = 1996, 
  month         = jul, 
  address       = "New Brunswick, NJ, USA", 
  series        = LNCS, 
  volume        = 1103, 

%%%% fin exemple %%%%
ATTENTION :
l'ordre de déclaration @InProceedings/@Proceedings dans le fichier .bib est important.

15.19  Comment citer une URL ?

Le style harvard propose un champ URL. Il est disponible à :

http://www.arch.su.edu.au/~peterw/latex/harvard/

ou sur

ftp://ftp.fdn.org/pub/CTAN/macros/latex/contrib/supported/harvard/.

Le style bibliographique 'utphys', de J. Distler, disponible à http://xxx.lanl.gov/hypertex/bibstyles/ ajoute le champ eprint à toutes les entrées BibTEX classiques.

On peut également utiliser la macro @MISC et mettre la référence URL dans le champ note.

M. Moreau propose la solution suivante : J'ai recupéré le fichier falpha.bst auquel j'ai ajouté :

FUNCTION {format.url} 
{ url empty$ 
    {"empty"} 
    { url emphasize } 
  if$ 

FUNCTION {onTheNet} 
{ output.bibitem 
  format.authors output 
  format.title "title" output.check 
  new.block 
  institution "institution" output.check 
  format.date output 
  new.block 
  format.url "url" output.check 
  fin.entry 
}
Autrement dit, si on peut avoir des documents de type onTheNet avec pour entrée obligatoire : title, institution, url et entrée optionnelle : author, year, month, note. L'URL est indiquée en italique.

15.20  Comment définir des initiales à deux lettres ?

En français, les prénoms commencant par Ch, Ph, Th... ont pour initiales leurs deux premières lettres (Ex : Philippe -> Ph.). Pour forcer BibTEX à considérer ces groupes de lettres, il faut utiliser : {\relax Ph}ilippe. BibTEX prend alors tout le groupe {..} pour une lettre. La commande \relax est considérée comme une commande d'accent qui est supprimée lors du tri alphabétique. Elle est nécessaire car lorsqu'un groupe apparaît dans un nom, BibTEX s'attend à trouver une commande d'accent juste après l'accolade ouvrante.

15.21  Comment conserver les majuscules dans les titres ?

Il suffit de mettre les majuscules entres accolades.

15.22  Comment changer l'espace entre les item ?

Il faut copier la définition de l'environnement thebibliography qui se trouve dans le fichier de style de la classe utilisée (par exemple article.cls pour un article) dans un fichier de style (qui sera appelé via une commande \usepackage) ou dans le préambule de votre document (entre \makeatletter, \makeatother).

Il faut ensuite remplacer \newenvironment par \renewenvironment et modifier la définition en ajoutant \setlength{\itemsep}{0pt} après \list.

15.23  Comment réaliser des fiches de lecture ?

S. O. Genaud (genaud@galaad.u-strasbg.fr) a écrit un petit logiciel de gestion des références bibliographiques au format BibTEX, sous la forme d'une interface graphique. La version actuelle n'est peut être pas exempte de tout bug. Elle tourne sur Linux, SunOS et Solaris avec la librairie xview.

Pour récupérer l'exécutable ou avoir plus d'informations : http://icps.u-strasbg.fr/~genaud/FRM/

Le programme bibfrm permet de lire un fichier BibTEX ou de créer un fichier de références bibliographiques au format BibTEX et d'associer à chaque référence un résumé. L'interface graphique permet de savoir immédiatement quels sont les champs optionnels et obligatoires pour un type de document donné. Le logiciel permet de parcourir les différentes références du fichier, de faire une recherche sur une chaîne, de trier les références par nom d'auteur, année ou type de document. Pour chaque référence, un résumé peut être saisi dans l'éditeur de texte incorporé dans l'interface. Ce résumé peut être envoyé par mail à une liste de diffusion prédéfinie.

Le package 'abstbook' disponible sur ftp://ftp.fdn.org/pub/CTAN/macros/latex/contrib/other/misc/ permet de mettre en page des catalogues de résumés.

15.24  Comment utiliser la commande \cite dans un item ?

Pour utiliser la commande \cite dans l'item d'un environnement description, il suffit de l'encadrer dans des accolades.

Exemple :
 
\item[{\cite[\S3.1]{Author1}}] 
%%%% fin exemple %%%%

15.25  Comment générer l'expression et al automatiquement ?

Voici une fonction ``format.names'' (tirée de plain.bst) où V. Henn a rajouté quelques lignes pour mettre un et al pour les auteurs de numéro supérieurs à 4. N'importe quel style biblio utilise une fonction avec une syntaxe similaire, il n'y a qu'à insérer les nouvelles lignes dans la fonction format.names (juste avant la fin du while) du fichier biblio qui vous plaît et le tour est joué...

%%%% debut macro %%%% 
FUNCTION {format.names} 
{ 's := 
  #1 'nameptr := 
  s num.names$ 'numnames := 
  numnames 'namesleft := 
  { namesleft #0 > } 
  { s nameptr "{ff}{vv}{ll}{, jj}" format.name$ 't := 
    nameptr #1 > 
    { namesleft #1 > 
      { ", " * t * } 
      { numnames #2 > 
          { "," * } 
          'skip$ 
        if$ 
        t "others" = 
          { " etal." * } 
          { " and " * t * } 
        if$ 
      } 
      if$ 
    }   % fin du "si nameptr > 1" 
    't  % "si nameptr = 1" 
  if$ 
  nameptr #1 + 'nameptr := 
  namesleft #1 - 'namesleft := 
% ------------------------------ 
% si à ce stade on se retrouve avec des noms à placer et que 
% le pointeur nameptr est égal à 4 alors il faut 
% 1. ajouter un "et al." 
% 2. ne pas prendre en compte les auteurs suivants : nameleft:=0 
    nameptr #4 =  namesleft #0 >  and 
    {  " \emph{etal.}" * 
       #0 'namesleft := 
    } 
    'skip$ 
    if$ 
% fin des ajouts 
% ------------------------------ 
  } 
  while$ % tant qu'il reste des nom à placer 

%%%% fin macro %%%%

Chapter 16
Index

La création d'un index est aussi gèrée par LATEX. Quelques commandes utiles qui permettent de le faire rapidement.

16.1  Quelles sont les commandes de base ?

Pour permettre à LATEX (via un générateur d'index) de construire un index, il faut référencer dans le texte tous les mots que l'on souhaite y voir apparaître. La commande à utiliser pour cela est \index. Elle permet la construction d'un fichier .idx dans lequel sont répertoriées toutes les références indiquées dans le texte.

Les entrées de ce fichier sont ensuite triées dans un fichier nom_fichier.ind via la compilation :

        makeindex fichier.idx
qui sera inclut dans le source .tex par la commande

\printindex 
ou

\input{nom_fichier.ind}
Pour indiquer à LATEX qu'il doit construire un index, il faut également ajouter la commande \makeindex dans le préambule du document.

Remarque :
compte tenu du temps pris pour générer un index, il est conseillé de ne faire apparaître la commande \makeindex que lors des dernières compilations.
Si les commandes de génération d'index ne sont pas définies par défaut, ou dans un package autre que makeidx (tel que french de B. Gaulle par exemple), il faut utiliser le package 'makeidx' (cas notamment de la commande \printindex).

Remarque :
Pour éviter de répéter la commande \index dans un texte où elle devrait apparaître souvent, on peut utiliser les commandes \index{mot_a_indexer|(} au début du texte et \index{mot_a_indexer|)} à la fin.
Pour plus de détails, consulter ftp://ftp.fdn.org/pub/CTAN/indexing/makeindex/doc/makeindex.ps

16.2  Comment construire un index hiérarchique ?

Pour construire un index à plusieurs niveaux d'entrée, il faut utiliser les commandes suivantes :

\index{niveau1} 
comme précédemment puis, pour faire apparaître un sous-thème de ce niveau, on appellera :

\index{niveau1!niveau1.1}
Exemple :
 
\documentclass{report} 
\usepackage{makeidx} 
\usepackage{french} 
\pagestyle{empty} 
\makeindex 
\begin{document} 
\chapter{Sports.} 
Le sport\index{Sport} c'est fantastique! 
Mes sports préférés sont: 
\begin{itemize} 
   \item l'équitation\index{Sport!Equitation} et en particulier 
   les disciplines de dressage\index{Sport!Equitation!Dressage} 
   et de complet\index{Sport!Equitation!Complet}: 
   \item l'escalade\index{Sport!Escalade} et surtout les 
   sorties en falaise; 
   \item le judo\index{Sport!Judo}. 
\end{itemize} 
\printindex 
\end{document} 
%%%% fin exemple %%%%
Ces fonctions sont disponibles dans le package 'index' ou (exclusif) 'french'.

16.3  Quels sont les générateurs d'index ?

makeindex, pour LATEX sous Unix, Macintosh, MS-DOS ou OS/2 est disponible sur ftp://ftp.fdn.org/pub/CTAN/indexing/makeindex/, ftp://ftp.fdn.org/pub/CTAN/systems/mac/macmakeindex.sit et dans les ditributions emTEX et gTEX.

texindex, fondé sur sed pour LATEX sous Unix, est disponible sur ftp://ftp.fdn.org/pub/CTAN/support/texindex/.

idxtex, pour LATEX sous VMS, est disponible sur ftp://ftp.fdn.org/pub/CTAN/indexing/glo+idxtex/.

texix pour TEX sur CMS et Macintosh.

16.4  Comment changer le style de certains mots indexés ?

Il faut utiliser la commande :

\index{mot_a_indexer@mot_reporte}
Exemple :
 
\documentclass{report} 
\usepackage{makeidx} 
\usepackage{french} 
\pagestyle{empty} 
\makeindex 
\begin{document} 
\chapter{Sports.} 
Le sport\index{Sport@\textbf{Sport}} c'est fantastique! 
Mes sports préférés sont: 
\begin{itemize} 
   \item l'équitation\index{Sport@\textbf{Sport}!Equitation} et 
   en particulier les disciplines de 
   dressage\index{Sport@\textbf{Sport}!Equitation!Dressage} et de 
   complet\index{Sport@\textbf{Sport}!Equitation!Complet}: 
   \item l'escalade\index{Sport@\textbf{Sport}!Escalade} et 
   surtout les sorties en falaise; 
   \item le judo\index{Sport@\textbf{Sport}!Judo}. 
\end{itemize} 
\printindex 
\end{document} 
%%%% fin exemple %%%%

16.5  Comment changer le style des pages de référence ?

Pour mettre en relief certains numéros de référence par rapport à d'autres, on peut utiliser les macros suivantes :

        \newcommand{\idb}[1]{\textbf{#1}}
        \newcommand{\idi}[1]{\it{#1}}
\index{mot_a_indexer|idb}

16.6  Comment rappeler certains mots dans un haut de page ?

Le package 'fancyhdr' () disponible sur ftp://ftp.fdn.org/pub/CTAN/macros/latex/contrib/supported/fancyhdr/ permet entre autres de faire apparaître le premier mot de la page en haut à gauche de la page et le dernier en haut à droite.

16.7  Comment générer plusieurs index ?

Il faut utiliser le package 'index' (et, si nécessaire, charger le package 'french' après).

Il existe également le package 'multind'.

Remarque :
french (3.45) fonctionne bien avec ``multind'', par contre ``multind'' n'implémente pas toutes les commandes d'index (notamment, le séparateur ``!'' ne fonctionne pas).
AL: toujours vrai ?

16.8  Qu'est ce que IdXTEX ?

IdXTEX programme écrit en C par R. Aurbach facilite la génération d'index. Il est disponible sur ftp://ftp.fdn.org/pub/CTAN/indexing/glo+idxtex/.

16.9  Qu'est ce que xindy ?

xindy est un système d'indexation. Il est disponible sur ftp://ftp.fdn.org/pub/CTAN/support/xindy/. Il est capable de gérer différentes langues suivant différentes lettres (notamment les lettres accentuées) et règles de tri. Il est paramétrable en fonction de repères de localisation dans le document qui ne correspondent pas forcément à un numéro de page ou de paragraphe prédéfini.

Pour plus de détails, consulter : http://www.iti.informatik.th-darmstadt.de/xindy/.

Chapter 17
Glossaire

Le glossaire ressemble à l'index. Il est souvent necessaire de le générer à part du document original.

17.1  Quelles sont les commandes de base ?

La création d'un glossaire avec le package 'french' s'apparente à la création d'un index. La commande d'insertion d'un mot dans le glossaire est \glossary, la commande de construction du fichier .glo est \makeglossary. L'insertion du glossaire dans le document source se fait par \glossaire et \input{fichier.glo}.

Exemple :
 
\documentclass{report} 
\usepackage{french} 
\makeglossary 
\begin{document} 
 Construisons un glossaire avec les abréviations suivantes : 
PS\glossary{[PS :] Pur Sang}, PSA\glossary{[PSA :] Pur Sang 
Arabe}, SF\glossary{[SF :] Selle Français}, PP\glossary{[PP :] 
Paso Péruvien}\ldots 
\glossaire 
\input{fichier.glo} 
\end{document} 
%%%% fin exemple %%%%
Pour trier un glossaire avec makeindex, il faut utiliser la commande :

(UNIX) makeindex -s style -o fichier.gls fichier.glo
L'appel du glossaire se fait alors par la commande

\printglossary.
Remarque : un exemple de style gglo.ist est disponible sur ftp://ftp.fdn.org/pub/CTAN/macros/latex/contrib/supported/nomencl/ ou dans http://planck.plmsc.psu.edu/~boris/.

17.2  Quels sont les générateurs de glossaire ?

Le programme ``GloTEX'' de R. Aurbach, utilise une base de données pour créer un glossaire. Il est donc possible d'utiliser la même BD pour plusieurs documents, et ainsi d'avoir des glossaires cohérents (les mêmes termes auront la même définition). GloTEX est disponible sur ftp://ftp.fdn.org/pub/CTAN/indexing/glo+idxtex/.

L'outil ``GlossTEX'', de V. Yavuz, disponible sur ftp://ftp.fdn.org/pub/CTAN/support/glosstex/ est un générateur de glossaire, de liste d'acronymes ou d'autres listes. Il s'appuie sur LATEX et MakeIndex. Il combine les fonctionnalités de GloTEX et nomencl.

La construction du glossaire nécessite les compilations suivantes :

Exemple :
 
\documentclass{article} 
\usepackage{glosstex} 
\begin{document} 
This document is typeset using \LaTeX\glosstex{LaTeX}. 
   \section*{Glossary} 
      \printglosstex 
\end{document} 
%%%% fin exemple %%%%

Chapter 18
Styles prédéfinis

Pour créer des documents avec un style déjà defini (une thèse, un rapport technique) est assez aisé avec LATEX. De nombreux contributeurs ont écrit des paquets qui présente toutes les caractéristiques et environnements nécéssaires au style choisi.

18.1  Où trouver un style de thèse ?

Le package 'thesis' disponible sur ftp://ftp.fdn.org/pub/CTAN/macros/latex/contrib/supported/thesis/ est une classe de thèse basée sur la classe report.

Le serveur loria présente quelques exemples de styles de thèses : http://tex.loria.fr/.

La classe 'thloria' de Denis Roegel (roegel@loria.fr) est disponible en http://www.loria.fr/~roegel/TeX/TL.html.

Sa documentation peut aussi être trouvée en http://www.loria.fr/tex/guide.html.

Un autre exemple est disponible à l'université de Californie ou sur ftp://ftp.fdn.org/pub/CTAN/macros/latex209/contrib/ucthesis/.

'utthesis' est le package de l'université du Texas. Il est disponible sur ftp://ftp.fdn.org/pub/CTAN/macros/latex/contrib/supported/utthesis/.

Il existe également le package 'uwthesis' disponible sur ftp://ftp.fdn.org/pub/CTAN/macros/latex/contrib/supported/uwthesis/ de l'université de Washington.

Une classe de thèse est également disponible par ftp sur ftp://cat.rpi.edu/pub/tibbetts/thesis.zip.

18.2  Comment faire son CV en LATEX ?

Il existe deux packages : 'resume' et 'vita' disponibles sur CTAN. 'vita' se trouve sur ftp://ftp.fdn.org/pub/CTAN/macros/latex/contrib/other/vita/.

Le package 'ESIEEcv' écrit par B. Bayart offre un certain nombre d'environnements qui facilitent la mise en page type d'un CV. Il est disponible sur ftp://ftp.fdn.org/pub/CTAN/macros/latex/contrib/supported/ESIEEcv/.

Exemple :
 
\begin{rubrique}{Titre de la rubrique.} 
   \begin{sousrubrique} 
      \Date{deb-fin} 
      \Duree{longue} 
      \Lieu{Ca c'est passe ici} 
      \Titre{Ce que j'ai fait} 
      \Descr{Quelques details} 
      \Apport{Ce que cela m'a apporte} 
      \Apport{et ca aussi} 
   \end{sousrubrique} 
   \begin{sousrubrique} 
      \Competence{Parlote} 
      \Descr{Bonne maitrise} 
   \end{sousrubrique} 
\end{rubrique} 
%%%% fin exemple %%%%
Remarque :
la largeur de la première colonne peut être modifiée par :
\setlength{\largeurcolonne}{2.5cm}
Voici quelques macros qui peuvent être utiles :

Dans le préambule :

%%%% debut macro %%%% 
% Definition des catégories de rubriques. 
\newcommand{\categorie}[1]{\vspace*{1cm}\noindent% 
        {\large\textsl{#1}\par}\vspace*{2pt}\hrule\vspace*{.5cm}} 
% On definit une colonne pour les dates 
\newlength{\duree} 
\settowidth{\duree}{\textbf{1985-1987}} 
% La seconde colonnes doit occuper le reste de la page 
\setlength{\duree}{-\duree} 
\addtolength{\duree}{\textwidth} 
% Definition des rubriques. 
\newcommand{\rubrique}[2]{\noindent{\textbf{#1\ \ }}% 
   \parbox[t]{\duree}{#2}} 
%%%% fin macro %%%%
Utilisation :

\categorie{Formation:} 
\rubrique{1990--1992}{Étudiant à l'école Machin. Cette école 
propose un programmeldots. 
Major de promo.} 
\rubrique{1992--1993}{Spécialisation dans le domaineldots.} 
\categorie{Expérience:} 
\rubrique{1987-1988}{Stage ouvrier dans l'usine de carton de 
La Ville aux Cartons.}
On trouvera d'autres macros, de JM Lasgouttes à l'URL suivante : http://www-rocq.inria.fr/~lasgoutt/lyx/cv-1.5.tar.gz.

Ces macros sont distribuées avec LYX, et donc parfaitement intégrées à LYX.

18.3  Où trouver un format de publication ?

Des classes et packages de format de publication dans les revues scientifiques telles que IEEE, IFAC, SIAM, SIGGRAPH, etc, sont disponibles sur CTAN.

Pour plus de renseignements sur les documents IEEE, consulter la page http://www.ieee.org/pubs/authors.html qui donne des références de style pour LATEX2.09 et .

La classe paper, disponible sur ftp://ftp.fdn.org/pub/CTAN/macros/latex/contrib/supported/paper/, est dérivée de la classe article. Elle est mieux adaptée pour la publication, notamment en termes de présentation de la page, le choix des fontes. Elle définit un certain nombre de macros utiles.

De même, la classe elsart est disponible sur CTAN. Cette classe est en outre très bien documentée.

* On trouve également une classe pour journaux d'Academic Press, que l'on peut trouver sur http://www.academicpress.com/www/journal/latex_a.htm.

* Kluwer propose une classe sur http://www.wkap.nl/kaphtml.htm/STYLEFILES

* Annual Reviews (uniquement pour Annual Review of Nuclear and Particle Science) fait de même sur

http://www.AnnualReviews.org/ibbin/ibGate.exe?LOADPAGE=\%2fARI\%2fauthors\%2fauthors.htm

* American Chemical Society, ne fournit, quant à elle, qu'un "Guidelines" sur http://pubs.acs.org/instruct/texguide.html

18.4  Où trouver un style de manuel de référence ?

Il existe la classe 'refman' disponible sur ftp://ftp.fdn.org/pub/CTAN/macros/latex/contrib/supported/refman/.

18.5  Où trouver un style de poster ?

Il existe le package 'poster' disponible sur ftp://ftp.fdn.org/pub/CTAN/macros/generic/poster/. Il permet entre autres d'imprimer un texte s'étalant sur plusieurs pages avec des parties communes de manière à pouvoir recoller les morceaux après.

G. Kettl propose également un style de poster accompagné d'une documentation sur http://rphibm1.physik.uni-regensburg.de/cluster/software/tex/a0poster.html.

18.6  Comment mettre en forme une lettre interne ?

Le package 'newsletr' disponible sur ftp://ftp.fdn.org/pub/CTAN/macros/plain/contrib/newsletr/ offre un ensemble de macros pour mettre en page des "newsletter". Il est destiné à TEX (et pas LATEX). Le résultat est très satisfaisant (multicolonages, cadres simples, etc).

18.7  Comment créer son propre style ?

Lorsque l'on veut modifier un style prédéfini pour l'adapter à ses besoins, il est conseillé de créer son propre fichier .cls dans lequel seront réunies les nouvelles commandes de mise en page.

Ci-dessous un exemple de V. Henn pour définir son propre style d'article :

%%%% debut macro %%%% 
%%% A sauver dans MonArticle.cls  
%%%  
%%%  Format de style adapté de article.cls 
%%% 
%%%  V.H. le 13 avril 1995 
%%% 
\NeedsTeXFormat{LaTeX2e} 
\ProvidesClass{MonArticle}[1995/06/20 Article personnel] 
%%%  Chargement de la classe article, avec transfert d'options 
\PassOptionsToClass{a4paper}{article} % format a4paper par défaut 
\DeclareOption*{\PassOptionsToClass{\CurrentOption}{article}} 
\ProcessOptions 
\LoadClass{article} 
%%%  Chargement des Packages les plus courants 
\RequirePackage{frbib} 
\RequirePackage{general} 
\RequirePackage{fuzzy} 
\RequirePackage{bigcenter} 
\RequirePackage{traffic} 
\RequirePackage[dvips]{epsfig} 
\RequirePackage{epic} 
\RequirePackage{french} 
%%%  Destination de l'article (proposé pour une conf, une revue...) 
\def\Destination#1{\ifx{#1}{}\gdef\@Destination{}% 
\else\gdef\@Destination{#1}\fi} 
%%%   Destination vide par défaut 
\Destination{} 
%%%   Auteur par défaut, pour eviter de recopier a chaque fois 
\author{Vincent \fsc{Henn}\thanks{Laboratoire d'ingénierie 
circulation--transport, (Unité mixte \lsc{INRETS}--\lsc{ENTPE}), 
109, Avenue Salvador Allende, Case 24, 
F--69675 \fsc{Bron} Cedex, {\tt henn@inrets.fr}.}} 
%%%% fin macro %%%%
Utilisation :

\documentclass[a4paper,11pt]{MonArticle} 
\title{Exemple d'article} 
\Destination{IEEE} 
\begin{document} 
\maketitle 
\chapter{Introduction} 
Bla bla 
\end{document}

Chapter 19
Création de transparents

La création de transparents est aussi possible, bien sur le résultat est prévu pour l'impression.

19.1  Quels sont les packages et styles existants ?

La classe 'seminar', de T. van Zandt, offre de nombreuses possibilités et peu de modifications sont nécessaires pour pouvoir l'utiliser avec des documents initialement écrits pour ``slide''. seminar est disponible sur ftp://ftp.fdn.org/pub/CTAN/macros/latex/contrib/other/seminar/inputs/.

Remarque
: A l'origine, seminar.sty était un package ; aujourd'hui, il a évolué en classe.
La lecture du cahier GUTenberg no 16 disponible en ligne à l'URL http://www.gutenberg.eu.org/publications/publis.html peut être utile. Voir également la page http://tug.cs.umb.edu/applications/ ou http://www.tug.org/applications/Seminar/ qui présente notamment une FAQ.

Le package 'cours' permet également de réaliser des transparents. Il permet, à partir d'un fichier source de créer des transparents et de gérer un syllabus. Cela fonctionne très bien sous LATEX2.09 et (disponible sur les serveurs CTAN).

La classe 'foiltex', de J. Hafner, est pas mal du tout. Elle permet de définir des entêtes et des pieds de page. Elle a été adaptée à et est disponible sur ftp://ftp.fdn.org/pub/CTAN/macros/latex/contrib/supported/foiltex/.

La classe 'slides' est la classe de document fournie avec LATEX en remplacement de SliTEX, la solution proposée avec LATEX2.09 et disparue avec lui.

Exemple :
 
\documentclass[a4paper]{slides} 
\begin{document} 
\begin{slide} 
Sous cet environnement slides, le texte est : 
\begin{itemize} 
   \item en gros caractères, 
   \item d'une fonte sans serif, 
   \item centré verticalement. 
\end{itemize} 
\end{slide} 
\end{document} 
%%%% fin exemple %%%%

19.2  Comment définir un contour pour des transparents ?

Avec le package 'seminar', disponible sur ftp://ftp.fdn.org/pub/CTAN/macros/latex/contrib/other/seminar/.

Exemple :
 
\slideframe[]{oval} 
%%%% fin exemple %%%%
Il existe également le package 'fancybox', disponible sur ftp://ftp.fdn.org/pub/CTAN/macros/latex/contrib/others/seminar/inputs/, qui définit des fonctions telles que \shadowbox, \doublebox, \ovalbox,...

On peut également utiliser une commande \special (cf. paragraphe ).

Exemple :
Cet exemple nécessite dvips et travaille sur un format a4. Il permet d'obtenir un fond jaune clair avec un cadre noir à 1 cm du bord de la feuille et de définir deux zones de texte en vert.
%%%% debut macro %%%% 
\special{ 
 !userdict begin /bop-hook {gsave 
 1 1 0.5 setrgbcolor clippath fill 
 0 setlinecap 0 setlinejoin 2 setlinewidth 0 setgray 
 /Times-Bold findfont 10 scalefont setfont newpath 
 (Maurizio Loreti, 1996-02-25) 
 dup stringwidth pop neg 510 add 25 moveto 
 currentpoint 3 -1 roll 0 1 0 setrgbcolor show 0 setgray 
 moveto -6 3 rmoveto 28 28 28 814 15 arcto 
 28 814 567 814 15 arcto 79 814 lineto 6 -3 rmoveto 
 0 1 0 setrgbcolor 
 (CMS week at CERN) 
 show 0 setgray 6 3 rmoveto 567 814 567 28 15 arcto 
 567 28 516 28 15 arcto 516 28 lineto stroke grestore} 
 def end 

%%%% fin macro %%%% 
%%%% fin exemple %%%%

19.3  Comment inclure des commentaires dans les transparents ?

Dans l'environnement slide, l'environnement note permet de taper du texte qui sera imprimé sur une page séparée sans être intégré dans un transparent. Cela permet à l'auteur de taper ses notes personnelles sans qu'elles apparaissent dans un transparent.

Le package 'xcomment' disponible sur ftp://ftp.fdn.org/pub/CTAN/macros/latex/contrib/other/seminar/src/ permet de gérer l'inclusion ou non de certains commentaires.

19.4  Comment modifier l'interligne sous seminar ?

Pour modifier l'espacement entre lignes à l'intérieur des paragraphes (sous seminar), il faut utiliser le paramètre \slidestretch (cf. page 9 de la documentation de Seminar).

Exemple :
 
\documentclass[portrait]{seminar} 
\begin{document} 
\begin{slide*} 
   Ceci est le texte de la d\'eclaration que j'ai \`a vous 
   communiquer. Ceci est le texte de la d\'eclaration que j'ai 
   \`a vous communiquer. Ceci est le texte de la d\'eclaration 
   que j'ai \`a vous communiquer. 
\end{slide*} 
\def\slidestretch{0.5} 
\begin{slide*} 
   Ceci est le texte de la d\'eclaration que j'ai \`a vous 
   communiquer. Ceci est le texte de la d\'eclaration que j'ai 
   \`a vous communiquer. Ceci est le texte de la d\'eclaration 
   que j'ai \`a vous communiquer. 
\end{slide*} 
\end{document} 
%%%% fin exemple %%%%

19.5  Comment définir des en-têtes et pieds de pages ?

Le package 'seminar' permet de définir des entêtes et des pieds de pages.

Exemple :
 
\documentclass{seminar} 
\usepackage{french,enumerate} 
\newpagestyle{filets}% 
  {\hrulefill}% en haut 
  {\hrulefill\tiny\theslide}% en bas 
\slidepagestyle{filets} 
\begin{document} 
\begin{slide} 
   Voici les résultats de notre société pour les 5 derniers 
   trimestres : 
   \begin{enumerate}[Q1] 
      \item 120 MF 
      \item 123 MF 
      \item 145 MF 
      \item 143 MF 
      \item 193 MF 
   \end{enumerate} 
\end{slide} 
\end{document} 
%%%% fin exemple %%%%

19.6  Comment modifier la taille du cadre d'un transparent ?

Sous seminar, pour un changement local, il suffit de préciser les dimensions voules dans le \begin{slide} ou \begin{slide*}.

Pour un changement global, il faut modifier les variables \slidewidth et \slideheight.

Exemple :
 
\documentclass[a4,portrait]{seminar} 
\usepackage{semcolor} 
\begin{document} 
% Cadre de la taille par defaut (en A4) 
\begin{slide*} 
  Texte ... 
\end{slide*} 
% Changement global de la taille du cadre 
\addtolength{\slidewidth}{1cm} 
\addtolength{\slideheight}{1cm} 
\begin{slide*} 
  Texte... 
\end{slide*} 
% Changement local de la taille du cadre 
% [hauteur,largeur] en mode portrait 
% [largeur,hauteur] en mode paysage 
\begin{slide*}[15cm,10cm] 
  Texte 
\end{slide*} 
% On revient a la taille précédente 
\begin{slide*} 
  Texte... 
\end{slide*} 
\end{document} 
%%%% fin exemple %%%%

19.7  Comment empêcher les figures de flotter ?

On peut forcer l'option par défaut à H :

\usepackage{float} 
\newfloat{figure}{H}{lof} %Don't let them float by default 
\newfloat{table}{H}{lot} 
\floatname{figure}{\figurename} 
\floatname{table}{\tablename}

19.8  Comment gérer la couleur avec seminar ?

L'exemple suivant montre comment paramétrer la couleur du fond.

Exemple :
 
\documentclass[a4,portrait]{seminar} 
\usepackage{semcolor} 
\newcommand{\SlideColours}[1]{% 
\slideframe[\psset{fillcolor=#1,fillstyle=solid}]{scplain}} 
\begin{document} 
\SlideColours{blue} 
        \begin{slide*} 
           Mon transparent avec un fond bleu. 
        \end{slide*} 
\end{document} 
%%%% fin exemple %%%%
Pour utiliser d'autres couleurs que celles prédéfinies (black, blue, cyan, darkgray, gray, green, lightgray, magenta, red, white, yellow), il faut le faire via l'une des commandes de PSTricks définissant les couleurs (dans Seminar, par défaut la couleur est en effet gérée via PSTricks). PSTricks est disponible sur ftp://ftp.fdn.org/pub/CTAN/graphics/pstricks/.

Néanmoins, aujourd'hui il est conseillé d'utiliser plutôt l'extension standard 'color' de LATEX pour ce faire.

Exemple :
 
\documentclass[a4,portrait]{seminar} 
% Pour utiliser l'extension standard "color" avec Seminar 
\usepackage[dvips]{pstcol} 
\usepackage{semcolor} 
\newcommand{\SlideColours}[1]{% 
\slideframe[\psset{fillcolor=#1,fillstyle=solid}]{scplain}} 
\definecolor{Bleu}{rgb}{0.,0.,1.} 
\definecolor{Rose}{rgb}{1.,0.75,0.8} 
\begin{document} 
\SlideColours{Bleu} 
        \begin{slide*} 
           Mon transparent avec un fond bleu. 
        \end{slide*} 
\SlideColours{Rose} 
        \begin{slide*} 
           Mon transparent avec un fond rose. 
        \end{slide*} 
\end{document} 
%%%% fin exemple %%%%
On peut également composer un fond dégradé.

Exemple :
 
\documentclass[a4,portrait]{seminar} 
% To use the standard "color" package with Seminar 
\usepackage[dvips]{pstcol} 
\usepackage{semcolor} 
\usepackage{gradient} 
\definecolor{Gold}{rgb}{1.,0.84,0.} 
\slideframe[\psset{fillstyle=gradient,gradbegin=Gold, 
        gradend=yellow, gradmidpoint=0.5}]{scplain} 
\begin{document} 
        \begin{slide*} 
            Sympa le dégradé ! 
        \end{slide*} 
\end{document} 
%%%% fin exemple %%%%

19.9  Comment imprimer des transparents en miroir ?

Pour imprimer en miroir un document (pour faire de la photocomposition), il existe un prologue PostScript pour DVIPS que voici :

%%%% debut macro %%%% 
%% A sauver dans miroir.pro 
% Pour imprimer des films transparents en miroir 
% Pascal PERICHON - Distribution PC-TeXMF 2.6 - (c) 1997 
userdict begin 
/bop-hook{ 
   210 2.834 mul 
   0 translate 
   -1 1 scale 
}def end 
%%%% fin macro %%%%
Il s'utilise de la façon suivante :

       dvips -hmiroir.pro
Remarque :
N'oubliez pas de mettre ce fichier dans la path HEADERS de dvips (ou dans un chemin spécifié dans config.ps pour l'inclusion des en-têtes).
Le package 'mirr' disponible sur ftp://ftp.fdn.org/pub/CTAN/macros/generic/TeX-PS/ permet d'obtenir des fichiers miroir.

19.10  Comment inclure une image de fond ?

Le package 'seminar' permet d'inclure une image de fond.

Exemple :
 
\documentclass{seminar} 
\usepackage{semcolor} 
\usepackage{fancybox} 
\usepackage[dvips]{graphicx} 
\begin{document} 
\newslideframe{IMAGE}% 
{\boxput{\rput(1,0){\includegraphics[scale=0.4]{image.eps}}}{#1}} 
\slideframe*{IMAGE} 
\begin{slide} 
  My first word. 
\end{slide} 
\begin{slide} 
  My last word. 
\end{slide} 
\end{document} 
%%%% fin exemple %%%%
On peut également utiliser l'environnement picture.

Exemple :
 
\setlength{\unitlength}{1in} 
\begin{picture}(0,0) 
   \put(-2.2,-6){\includegraphics[width=\textwidth]{bilder/sigel.eps}} 
\end{picture} 
%%%% fin exemple %%%%

19.11  Comment imprimer plusieurs transparents par page ?

Pour cela, il existe le package '2up'.

Chapter 20
Lettres, mailing et faxs

Ecrire sa correspondance avec LATEX permet d'obtenir de jolis documents très professionnels.

20.1  Comment structurer une lettre ?

Il existe pour cela la classe letter. Celle-ci définit les commandes :

De manière générale, le positionnement des différents champs dépend de la langue utilisée dans la lettre.

Remarque :
lorsque plusieurs environnements letter sont utilisés dans un même source, ils produiront chacun une lettre. Toutes ces lettres auront la même adresse d'expéditeur et la même signature.
Exemple :
 
\documentclass[11pt]{letter} 
\name{expediteur} 
\address{adresse \\ de l'expediteur} 
\signature{signature \\ de l'expediteur} 
\begin{document} 
\begin{letter}{le destinataire} 
   \opening{Cher destinataire,} 
   texte -- texte -- texte -- texte -- texte -- texte -- texte 
   -- texte -- texte -- texte -- texte -- texte -- texte -- 
   texte -- texte -- texte -- texte -- texte -- texte -- texte 
   -- texte -- texte -- texte -- texte -- texte -- texte -- 
   texte -- texte -- texte -- texte -- texte -- texte -- texte 
   -- texte -- texte -- 
   \closing{Formule de politesse} 
   \ps{PS : j'ai oublie...} 
   \cc{Monsieur Truc.} 
   \encl{pieces jointes} 
\end{letter} 
\end{document} 
%%%% fin exemple %%%%
ATTENTION :
si l'on ajoute le package 'french' la disposition des champs utilisés change puisque les lettres anglo-saxones ne respectent pas la même typographie que les françaises.
La classe 'lettre' de D. Megevand est un outil très bien fait et adaptable à toutes les situations. Elle est disponible par ftp sur ftp://obsftp.unige.ch/pub/tex/macros/.

Exemple (D.Mégevand) :
 
\documentclass[11pt]{lettre} 
\usepackage{french} 
\begin{document} 
\begin{letter}{Mme Marie-Paule \textsc{Kluth}\\ 
   Alcatel Alsthom Recherche\\ 
   Route de Nozay\\ 
   \textbf{F-91460 Marcoussis}} 
\signature{Denis Mégevand} 
\def\concname{Objet :} 
\conc{FAQ de \TeX} 
\vref{A VOUS} 
\nref{DE MOI} 
\opening{Chère Madame,} 
Je vous remercie de citer ma classe lettre dans le \S20.1 de 
votre FAQ sur \TeX. Il est exact que l'on peut faire beaucoup de 
choses avec cette classe dans le domaine de la correspondance, 
notamment des lettres et des télécopies (fax). On peut également 
obtenir une page d'étiquettes avec les adresses des destinataires 
des lettres contenues dans le document.

Tous les paramètres de mise-en-page, ainsi que les chaînes  
"Concerne", "Cc", "PS", etc. peuvent être facilement modifiés.

 

Cependant, le pointeur vers 
html(???)://obswww.unige.ch/pub/tex/macros ne conduit nulle part. 
Le pointeur ftp est correct. Pour la doc, il existe une version 
en HTML sur http://obswww.unige.ch/Manuels/letdoc2.31.html. 
 

Au \S20.3, vous parlez de classe 'lettre', ce qui est correct, 
mais l'exemple donné montre un \verb+\documentstyle+, et les 
commandes \verb+\signature+, \verb+\vref+ et \verb+\nref+ ne sont 
pas à l'endroit specifié dans la doc. Cet exemple ne compilera 
pas, ou plutôt ne donnera pas ce qui est attendu. Voici comme est 
structurée cette lettre : 
\begin{verbatim} 
\documentclass[11pt]{lettre} 
[...] 
\end{verbatim} 
\closing{Meilleures salutations} 
\cc{À personne !} 
\end{letter} 
\end{document} 
%%%% fin exemple %%%%

La classe 'fribrief' disponible sur ftp://ftp.fdn.org/pub/CTAN/macros/latex/contrib/supported/fribrief/ permet également de mettre en page des lettres.

La classe 'akletter' disponible sur ftp://ftp.fdn.org/pub/CTAN/macros/latex/contrib/supported/akletter/ est une extension de la classe lettre classique. Elle permet notamment de définir sa propre entête et de gérer la position de l'adresse pour des enveloppes à fenêtre.

20.2  Comment préparer un mailing ?

Le package 'mailing', disponible sur ftp://ftp.fdn.org/pub/CTAN/macros/latex/contrib/supported/mailing/ permet de créer plusieurs documents similaires avec des valeurs définies dans des bases externes (fichiers ou autres). Il ne fonctionne qu'avec la classe 'letter'.

Il existe également le package 'envlab', de B. Veytsman (boris@plmsc.psu.edu), disponible sur ftp://ftp.fdn.org/pub/CTAN/macros/latex/contrib/supoorted/envlab/. Il est aujourd'hui aux standards américains mais doit être étendu à d'autres pays.

Le package 'labels' est également disponible sur ftp://ftp.fdn.org/pub/CTAN/macros/latex/contrib/supported/labels/.

Exemple :
 
\documentclass[12pt]{book} 
\usepackage{labels} 
\LabelCols=3 
\LabelRows=11 
\LeftBorder=8mm 
\RightBorder=4mm 
\TopBorder=2mm 
\BottomBorder=4mm 
%\LabelInfotrue 
\begin{document} 
%\footnotesize\sf 
\numberoflabels=3 
\addresslabel[\small\sf] 
{Me, Myself \& I\\ 
SomePlace\\ 
SomeCompany\\ 
SomeStreet\\ 
SomeTown, SomeZip} 
\end{document} 
%%%% fin exemple %%%%
Le package 'adrlist' disponible sur ftp://ftp.fdn.org/pub/CTAN/macros/latex/contrib/other/adrlist/ permet de gérer des listes d'adresses.

Le package 'envbig' disponible sur ftp://ftp.fdn.org/pub/CTAN/macros/latex/contrib/other/envbig/ permet d'imprimer des adresses sur des enveloppes.

Le package 'formlett' disponible sur ftp://ftp.fdn.org/pub/CTAN/macros/generic/formlett.sty permet de gérer des mailings.

20.3  Comment faire des références dans une lettre ?

Dans la classe 'lettre' de D. Megevand, il existe les commandes \nref, pour les références de l'expéditeur et \vref pour celles du destinataire.

Exemple :
 
\documentclass[11pt]{lettre} 
\usepackage{french} 
\begin{document} 
\begin{letter}{Mme Marie-Paule \textsc{Kluth}} 
\signature{MPK} 
\vref{A VOUS} 
\nref{DE MOI} 
 \opening{Cher ami,} 
   Texte.... 
   \closing{Sincèrement.} 
   \encl{Pièces jointes :} 
   \cc{copie àTotor} 
\end{letter} 
\end{document} 
%%%% fin exemple %%%%
Le package 'french' offre également les commandes \yourref et \ourref.

Exemple :
 
\documentclass[a4]{letter} 
\usepackage{french} 
\signature{ma signature} 
\begin{document} 
\yourref{mon texte} 
\ourref{mon autre texte} 
\begin{letter}{le destinataire} 
   \opening{Cher Monsieur,} 
   Texte... 
   \closing{Salutations} 
   \encl{pieces jointes} 
   \cc{copie a M. Totor} 
\end{letter} 
\end{document} 
%%%% fin exemple %%%%

20.4  Comment mettre en page un fax ?

Le package 'fax' disponible sur ftp://ftp.fdn.org/pub/CTAN/macros/latex/contrib/supported/fax/ propose un ensemble de commandes pour préparer un fax.

20.5  Comment positionner une adresse pour une enveloppe à fenêtre ?

La classe scrlettr.cls du package koma-script permet de faire cela.

20.6  Comment suppprimer la date sur une lettre ?

Sous la classe lettre disponible sur ftp://ftp.fdn.org/pub/CTAN/macros/, il existe la commande \nodate.

20.7  Comment inclure une figure dans une lettre ?

Il faut utiliser le package float et déclarer le type figure :

\usepackage{float} 
\newfloat{figure}{htbp}{lof}

Chapter 21
Symboles et logos

Les symboles et Logos sont le plus souvent contenus dans des fontes spéciales. Pour un dessin plus particulier, il est encore possible de l'écrire à la main par l'intermédaire d'une macro.

21.1  Où trouver des symboles électroniques ?

Le package 'epic' disponible sur CTAN permet d'accéder aux symboles électroniques.

Il y a également le package 'circ' disponible sur ftp://ftp.fdn.org/pub/CTAN/macros/generic/diagrams/circ/ qui fait ça.

AL: incompatible avec french ou pas alors ?

Le package 'circuit_macros' disponible sur ftp://ftp.fdn.org/pub/CTAN/graphics/circuit_macros.tar.gz permet de définir des circuts électriques à partir d'éléments de base tels que des amplis, des transistors, et portes logiques ...

21.2  Comment dessiner des circuits électroniques ?

Certains packages sont disponibles sur ftp://ftp.fdn.org/pub/CTAN/graphics/lcircuit/, ftp://ftp.fdn.org/pub/CTAN/graphics/circuit_macros/ et ftp://ftp.fdn.org/pub/CTAN/macros/generic/diagrams/circ/.

Il existe également des librairies xfig disponibles sur ftp://epb1.lbl.gov/xfig/.

21.3  Quelles sont les polices de symboles sous LATEX ?

Il y en a plusieurs que l'on peut trouver sur ftp://ftp.fdn.org/pub/CTAN/fonts/.

dancers Des petits bonshommes stylisés (dessins d'enfants) qui se trémoussent dans tous les sens...
dingbat Symboles Zapf dingbats
bbding Symboles Zapf dingbats
goblin On dirait des personnages (étranges) stylisés...
hands Des grosses mains pointant vers la gauche ou la droite
iching Yi-King. Avec des symboles chinois bizarres...
karta Symboles bizarres (voir aussi niceframes.sty)
knot noeuds
wasy2 Symboles

Table 21.1: Liste de polices de symboles disponible

21.4  Comment obtenir les symboles mâle et femelle ?

Les polices ``astrosym'', ``cmastro'', ``wasy'' ou ``wasy2'' définissent ces caractères.

Voir également le package 'genealogy' disponible sur ftp://ftp.fdn.org/pub/CTAN/fonts/genealogy/.

21.5  Comment obtenir le symbole degré ?

En fait le symbole ``degré'' ne devrait pas être confondu avec un < < petit o en exposant > >. C'est un caractère définit :

En ,

\DeclareTextSymbol{\degre}{T1}{6} 
\DeclareTextSymbol{\degre}{OT1}{23}
définiront la commande \degre, que vous travailliez en codage OT1 (défaut) ou T1 sélectionné par \usepackage[T1]{fontenc}

Le package 'french' définit de cette manière la commande \degre.

Le package 'babel' définit lui \degre et \degres.

Le package 'textcomp', de S. Rahtz, disponible sur ftp://ftp.fdn.org/pub/CTAN/fonts/psfonts/ts1/, basé sur les fontes DC 1.2 ou plus récentes, disponibles sur ftp://ftp.fdn.org/pub/CTAN/fonts/dc/, définit les commandes \textdegree et \textcentigrade.

Le package 'inputenc' disponible sur ftp://ftp.fdn.org/pub/CTAN/macros/latex/base/ fournit les commandes \textdegree et \mathdegree.

La solution qui consiste à mettre dans le préambule () :

\newcommand{\deg}{\ensuremath{circ}} 
ou

\newcommand{\deg}{\ensuremath{circ}\xspace} 
en utilisant le package 'xspace' ou encore

\newcommand{\deg}{\(\mathsurround=0pt{}circ\)}
puis utiliser \deg en mode texte ou mathématique, est donc à éviter.

21.6  Où trouver des symboles astronomiques ?

Il existe le package 'astro' disponible sur ftp://ftp.fdn.org/pub/CTAN/fonts/astro/.

21.7  Où trouver une police de symboles phonétiques ?

Il existe le package 'phonetic' disponible sur ftp://ftp.fdn.org/pub/CTAN/fonts/phonetic/.

La fonte ipa est disponible sur ftp://ftp.fdn.org/pub/CTAN/macros/latex/contrib/other/ipa/.

La fonte tipa de F. Rei est également disponible dans ftp://ftp.fdn.org/pub/CTAN/fonts/tipa/.

Il suffit d'utiliser le package "pifont", de S. Rhatz et W. Schmidt, disponible sur ftp://ftp.fdn.org/pub/CTAN/macros/latex/required/psnfss puis de faire \Pisymbol{psy}{210} ou encore \Pisymbol{psy}{226}.

21.8  Où trouver des opérateurs de logique floue ?

Il existe le package 'logic' disponible sur ftp://ftp.fdn.org/pub/CTAN/fonts/logic/.

V. Henn (henn@inrets.fr) propose ses propres définitions (). Sauvegarder les commandes suivantes dans un fichier 'fuzzy.sty'. Ce package nécessite les packages 'amsfonts' et 'xspace'.

ATTENTION :
Le fichier ci-dessous est perso, ce qui signifie :

  1. pas d'exhaustivité
  2. pas de garantie de solution optimale (en terme de 'pureté teX')
  3. pas de garantie de non bug
  4. ces opérateurs ne correspondent pas à une norme pour la notation : ce sont des choix perso (qui correspondent plus ou moins à l'usage)
%%%% debut macro %%%% 
%%% 
%%%  Format de style permettant d'utiliser directement un certain 
%%%  nombre  d'operateurs propres à la logique floue. Ces 
%%%  opérateurs sont généralement surlignés d'un tilde... 
%%% 
%%%    V.H., le 12 avril 1995 
%%% 
\RequirePackage{amsfonts} 
\RequirePackage{xspace} 
\message{Opérateurs flous} 
\newcommand{\fmin}{\mathop{\flou{\min }}} 
\newcommand{\fmax}{\mathop{\flou{\max }}} 
\newcommand{\V}{\mathop{\mathrm V\kern 0pt}} 
\newcommand{\ET}{\mathrel{\mathrm{ET}}} 
\newcommand{\OU}{\mathrel{\mathrm{OU}}} 
\newcommand{\Sim}{\mathop{\mathrm S \kern 0pt}} 
\newcommand{\hauteur}{\mathop{\mathrm h\kern 0pt}} 
\newcommand{\card}[1]{\| #1 \|} 
\newcommand{\flou}[1]{\ensuremath{\widetilde{#1}}} 
\newcommand{\R}{\ensuremath{\Bbb{R}}} 
\newcommand{\cpp}{\ensuremath{\hbox{C}{++}}\xspace} 
\newcommand{\Poss}{\mathop{\Pi}} 
%\newcommand{\Nec}{\mathop{{\cal{N}}}} 
\newcommand{\Nec}{\mathop{\mathrm N\kern 0pt}} 
\newcommand{\poss}{\operatoname{Poss}} 
\newcommand{\nec}{\operatoname{Néc}} 
\newcommand{\serie}[3]{% 
%  #1 -> le nom de la variable 
%  #2 -> l'indice de début 
%  #3 -> l'indice de fin 
\ensuremath{{#1}_{#2},\ldots,{#1}_{#3}}} 
\newcommand{\DP}{\fsc{Dubois} et \fsc{Prade}\xspace} 
%%%% fin macro %%%%

21.9  Comment obtenir le symbole de paragraphe ?

Il suffit de taper \S.

21.10  Comment obtenir le caractère 'registered' ?

La commande \textregistered est disponible lorsqu'on utilise une fonte qui le permet (TS1 ou 8r, p. ex. DC).

Le package 'textcomp', de S. Rahtz, disponible sur ftp://ftp.fdn.org/pub/CTAN/fonts/psfonts/ts1/ définit également ce caractère.

Voici d'autres définitions :

%%%% debut macro %%%% 
\def\Registered{\raisebox{1ex}{\kern-.1em\setbox\@tempboxa\hbox{% 
\footnotesize$\bigcirc$}\hbox 
to 0pt{\hbox 
to\wd\@tempboxa{\hss\tiny\textrm{R}\hss}\hss}\box\@tempboxa\kern-.1 
em}} 
%%%% fin macro %%%%
de Ulick Stafford (ulick.stafford@nd.edu)

%%%% debut macro %%%% 
\def\registered{{\ooalign {\hfil\raise .05ex\hbox{\scriptsize 
R}\hfil\crcr\mathhexbox20D}}} 
%%%% fin macro %%%%
de Maurice Dohmen (m.dohmen@cs.tudelft.nl)

%%%% debut macro %%%% 
\def\REgistered{{\ooalign 
{\hfil\raise.09ex\hbox{\tiny \sf R}\hfil\crcr\mathhexbox20D}}} 
%%%% fin macro %%%%
On peut également utiliser le package 'amssymb', de J. Knappen  (KNAPPEN@VKPMZD.kph.Uni-Mainz.DE) qui propose la commande \circledR.

Exemple :
 
\documentclass{article} 
\usepackage{amssymb} 
\usepackage{autredefs} 
\begin{document} 
\Registered 
\registered 
\REgistered 
\circledR 
\end{document} 
%%%% fin exemple %%%%
De Boris A. Veytsman <BAV2@psuvm.psu.edu> :

%%%% debut macro %%%% 
\newcommand{\reg}{\textsuperscript{\textcircled{\textsc r}}} 
%%%% fin macro %%%%

21.11  Où trouver le symbole trade-mark ?

Il suffit d'utiliser le package ``pifont'' puis de faire

\Pisymbol{psy}{212} 
ou encore

\Pisymbol{psy}{228}.
Avec des fontes TS1 (tc livrées avec dc ou ec, ou sinon ps) :

\usepackage{textcomp} 
\texttrademark

21.12  Comment obtenir un underscore ?

Il suffit de taper \_.

21.13  Où trouver le symbole radioactif ?

Il est disponible dans le package 'karta' disponible sur CTAN.

21.14  Comment obtenir le logo LATEX ?

Il suffit d'appeler la commande \LATEX.

21.15  Comment obtenir le logo ?

Le nouveau logo s'obtient par \LATEXe.

21.16  Où trouver des chiffres entourés ?

Le package 'go' disponible sur ftp://ftp.fdn.org/pub/CTAN/fonts/go/ propose des chiffres blancs sur fond noir.

Le package 'pifont' propose également des chiffres entourés.

A défaut, on peut construire ses propres symboles à l'aide du package 'overlay' ci-dessous.

%%%% debut macro %%%% 
%  Dans TTN  Vol3 Num 2 1994, Jeremy Gibbons propose une macro 
\overlay qui permet de créer des symboles en superposant 
% d'autres symboles. Le symbole ainsi construit peut changer de 
% taille en fonction du style (display, script...) 

%%% overlay.sty to overlay two symbols, respecting styles 
\def\loverlay#1#2{\mathpalette\@overlay{{#1}{#2}{}{\hfil}}} 
\def\overlay#1#2{\mathpalette\@overlay{{#1}{#2}{\hfil}{\hfil}}} 
\def\roverlay#1#2{\mathpalette\@overlay{{#1}{#2}{\hfil}{}}} 
        %  calls to \@overlay look like 
        %       \overlay\textstyle{{x}{y}{\hfil}{\hfil}} 
\def\@overlay#1#2{\@@overlay#1#2} 
        % strip brackets from 2nd arg, to get 
        %       \@@overlay\textstyle{x}{y}{\hfil}{\hfil} 
\def\@@overlay#1#2#3#4#5{{% 
        \def\overlaystyle{#1}% 
        \setbox0=\hbox{\m@th$\overlaystyle#2$}% 
        \setbox1=\hbox{\m@th$\overlaystyle#3$}% 
        \ifdim \wd0<\wd1 \setbox2=\box1 \setbox1=\box0% 
        \setbox0=\box2\fi % \box0 is now the wider box 
        \rlap{\hbox to \wd0{#4\box1\relax#5}}\box0}} 
%%% fin du style overlay 
%%%% fin macro %%%%
Utilisation :

$\overlay{symbole1}{symbole2}$
Exemple (V. Henn) :
 
\overlay{\lower.6ex\hbox{$\overlaystyle\smile$}} 
        {\raise.6ex\hbox{$\overlaystyle 
        {\roverlay{\circ}{\cdot}}{\roverlay{\circ}{\cdot}}$}} 

%%%% fin exemple %%%%

21.17  Comment obtenir le symbole numéro ?

Le package 'french' propose la commande \numero.

21.18  Comment obtenir les symboles pourcent et pourmille ?

Le symbole pourcent s'obtient symplement par la commande \%.

Le symbole pourmille est définit dans le package 'textcomp' via la commande \textperthousand.

ATTENTION :
ce package nécessite la disponibilité de fontes ec.
A défaut, on peut construire soit même le symbole :

%%%% debut macro %%%% 
\newcommand{\promille}{% 
  \relax\ifmmode\promillezeichen 
        \else\leavevmode\(\mathsurround=0pt\promillezeichen\)\fi} 
\newcommand{\promillezeichen}{% 
  \kern-.05em% 
  \raise.5ex\hbox{\the\scriptfont0 0}% 
  \kern-.15em/\kern-.15em% 
  \lower.25ex\hbox{\the\scriptfont0 00}} 
%%%% fin macro %%%%
Autre solution :

%%%% debut macro %%%% 
\newcommand{\promille}{% 
\def\pourmille{\hbox{$\,0\!/_{00}$}} 
%%%% fin macro %%%%

21.19  Comment obtenir un e dans l'o ?

Il faut appeler la commande \oe{}. Les accolades permettent d'isoler la commande dans un mot.

21.20  Quels sont les symboles réservés dans LATEX ?

Il s'agit des symboles : # $ % & _ \ { }.

Pour les imprimer il faut donc utiliser des commandes à savoir : \# \$ \% \& \_ \{} \{ \}. Pour le caractère tilde voir la question , et pour le backslash, il faut appeler \backslash en mode mathématique.

21.21  Comment obtenir des lettres cursives ?

Il suffit d'utiliser la fonte caligraphique.

Exemple :
 
$\cal{ABCDEFGHIJKLMNOPQRSTUVWXYZ}$ 
%%%% fin exemple %%%%
ce qui donne :

ABCDEFGHIJKLMNOPQRSTUVWXYZ

21.22  Comment obtenir le logo AMS-(La)TEX ?

Le logo AMS-TEX s'obtiens en appelant \AmS-\TEX et le logo AMS-LATEX en appelant \AmS-\LATEX.

21.23  Quels sont les symboles par défaut de LATEX ?

Voici une liste des symboles les plus courants :

21.24  Quels sont les différents tirets ?

Il existe effectivement différents tirets sous LATEX :

Exemple
:
Marie-Paule 
voir page 19--32 
entre deux thèmes --- par exemple. 
%%%% fin exemple %%%%
ce qui donne :

Marie-Paule
voir page 19-32
entre deux thèmes - par exemple.

21.25  Comment obtenir le symbole arobasse ?

Il existe différentes solutions qui doivent être sélectionnées en fonction de l'endroit où elles sont appelées : dans le texte, dans un titre dans une note de bas de page, ....

%%%% debut macro %%%% 
\def\at{\string@} 
\edef\at{\string@} 
\def\at{\char`@} 
\chardef\at=`\
%%%% fin macro %%%%

21.26  Comment obtenir un backslash ?

Il existe différentes techniques :

on peut également définir une commande :

21.27  Comment faire un carré plain de fin de démonstration ?

Le package 'amssymb' propose la commande \blacksquare.

Voici deux macros proposées par S. Cirilli:

%%%% debut macro %%%% 
\def\sqw{\hbox{\rlap{\leavevmode\raise.3ex\hbox{$\sqcap$}}$% 
\sqcup$}} 
\def\sqb{\hbox{\hskip5pt\vrule width4pt height6pt depth1.5pt% 
\hskip1pt}} 
% Rectangle noir: 
\def\qed{\ifmmode\hbox{\hfill\sqb}\else{\ifhmode\unskip\fi% 
\nobreak\hfil 
\penalty50\hskip1em\null\nobreak\hfil\sqb 
\parfillskip=0pt\finalhyphendemerits=0\endgraf}\fi} 
% Rectangle blanc: 
\def\cqfd{\ifmmode\sqw\else{\ifhmode\unskip\fi\nobreak\hfil 
\penalty50\hskip1em\null\nobreak\hfil\sqw 
\parfillskip=0pt\finalhyphendemerits=0\endgraf}\fi} 
%%%% fin macro %%%%

Chapter 22
Musique

Même les musiciens ne sont pas oubliés par notre formatteur de texte préféré. Différents paquets sont disponibles pour les Mozart en herbes.

22.1  Comment écrire de la musique sous LATEX ?

Le package 'MuTEX', écrit par A. Steinbach et A. Schofer, permet d'écrire de la musique. Il est disponible par ftp anonyme sur ftp://ftp.cs.ruu.nl/pub/TEX/MuTeX.tar.Z.

Un autre package encore plus performant est 'MusiXTEX' de D. Taupin (taupin@frups51.bitnet ou taupin@lps.u-psud.fr), R. Mitchell et A. Egler. Il permet d'écrire des partitions pour orchestres ou de la musique polyphonique. Il est disponible par ftp anonyme sur ftp://rsovax.ups.circe.fr/musictex/ ou sur ftp://hprib.lps.u-psud.fr/pub/musixtex/, ou encore sur ftp://ftp.fdn.org/pub/CTAN/macros/musixtex/taupin/ ou ftp://ftp.fdn.org/pub/CTAN/macros/musixtex/egler/. La lecture du cahier GUTenberg 21 apporte beaucoup d'informations à ce sujet. Voir http://www.ens.fr/gut/cahiers.html ou http://www.gutenberg.eu.org/publications/node7.html.

A voir également http://icking-music-archive.sunsite.dk/index.html le site de W. Icking avec une mine d'informations, en particulier des pointeurs vers des partitions saisies en musixtex, etc.,

Il existe une FAQ disponible sur ftp://ftp.fdn.org/pub/CTAN/macros/musictex/FAQ/.

Remarque :
le programme abc2mtex d'aide à l'utilisation de MusiXteX peut également être utile, voir http://www.gre.ac.uk/~c.walshaw/abc/

22.2  Comment convertir du midifile en MusicTEX  ?

Music TEX, de M. Beigbeder et J.J. Girardot permet la conversion de midifiles en Music TEX.

midi2tex permet également de traduire des fichiers de données midi. Il est disponible sur ftp://ftp.fdn.org/pub/CTAN/support/midi2tex/.

22.3  Existe-t'il une liste de discussion de musique ?

l existe une liste de discussion consacrée à l'écriture de musique en TEX. Pour s'y inscrire, il faut envoyer un mail ``subscribe'' à mutex-request@gmd.de.

Les articles postés sur cette liste sont archivés dans http://www.gmd.de/Mail/mutex-archive/.

Consulter également : http://www.gmd.de/Misc/Music/ ou ftp://ftp.gmd.de/music/.

22.4  Comment éditer un livre de chants ?

Il existe le package 'songbook' disponible sur ftp://ftp.fdn.org/pub/CTAN/macros/latex/contrib/supported/songbook/. Voir la page http://www.cyberus.ca/~crath/Misc/Songbook/index.html pour quelques exemples.

22.5  Comment mettre en page un programme de concert ?

Le package 'concprog' disponible sur ftp://ftp.fdn.org/pub/CTAN/macros/latex/contrib/supported/concprog/ propose un certain nombre de macros pour préparer un programme de concert.

Chapter 23
Conversion de fichiers

La portabilité, voici le maître mot de l'informatique moderne. Pour être portable, il est souvent nécéssaire de convertir les fichiers.

23.1  Comment générer un fichier .ps à partir d'un .dvi ?

dvips, de T. Rokicki, a l'avantage de savoir gérer les ``virtual fonts''. Il est disponible par ftp anonyme à :

dvitops, pour unix, msdos, vms et primos, de J. Clark, est disponible sur ftp://ftp.fdn.org/pub/CTAN/dviware/. dvitops ne gère pas les fontes virtuelles.

dvipsk, de K. Berry, est disponible sur ftp://ftp.fdn.org/pub/CTAN/dviware/.

23.2  Qu'est ce que le ``Literate Programming'' ?

Le ``Literate Programming'' est une approche de la programmation qui met en valeur le fait qu'un programme doit pouvoir être lu aussi bien par une personne que par un compilateur. Les outils associés (disponibles dans les archives CTAN) vont ainsi permettre de générer des documents combinant du code source et du texte (souvent au format TEX). Ensuite, la documentation et le code source seront automatiquement extraits de ce document unique. Pour plus d'information sur ces techniques, consulter : http://www.ius.cs.cmu.edu/help/Programming/literate.html. Il existe également une FAQ comp.programming.literate disponible sur ftp://ftp.fdn.org/pub/CTAN/help/comp.programming.literate_FAQ.

Remarque :
À l'origine, D.E. Knuth a basé TEX sur un système de ``programmation littéraire'' nommé Web. Le code source en question était du Pascal. Aujourd'hui où l'on ne trouve quasiment plus de compilateurs Pascal, il a fallu écrire des outils de passage au C. D'où l'outil Web2C actuellement utilisé. La plupart des distributions Unix utilisent Web2c maintenu actuellement par Karl Berry.

23.3  Comment convertir du LATEX en word ?

TEXport, de K-Talk permet de convertir vos fichier TEX et LATEX en documents WordPerfect ou Microsoft Word. Il traite les styles de fontes, les notes de bas de page, les caractères grecs, les mathématiques, les tableaux simples, les accents européens, ainsi que les commandes \def et \renewcommand.

Ce logiciel est payant (et cher). Pour plus de détails, voir : http://www.ktalk.com/.

23.4  Comment convertir du word en LATEX ?

Il y a l'outil ``Publishing Companion'' de K-Talk (voir http://www.ktalk.com/).

Wd2latex, pour MS-DOS, est disponible sur ftp://ftp.fdn.org/pub/CTAN/dviware/wd2latex/. Cet utilitaire paraît somme toute assez rudimentaire.

Word2x disponible sur ftp://ftp.fdn.org/pub/CTAN/tools/word2x/ permet de convertir du Word 6.0 en texte ou en LATEX.

Voir également texcnven dans http://www.kfa-juelich.de/isr/1/texconv/pctotex.html.

23.5  Comment convertir du scribe en LATEX ?

s2latex, de V. Jacobson, fait partie de la ditribution GUTenberg pour Unix. Celle-ci est disponible par ftp sur ftp://ftp.gutenberg.eu.org/pub/gut/distribunix/.

23.6  Comment convertir du WordPerfect en LATEX ?

wp2latex, de R.C. Houtepen, est disponible sur ftp://wuarchive.wustl.edu/mirrors/msdos/tex/ et sur ftp://ftp.fdn.org/pub/CTAN/support/wp2latex/. wp2latex a été écrit pour PC en Turbo Pascal. Il travaille à partir d'un document WordPerfect5.0. On note quelques limitations concernant les indices, la table des matières, les marges et les graphiques.

wp2latex, de G. Geers (glenn@qed.physics.su.oz.au), a été écrit en C. Il devrait être étendu à WordPerfect 5.1. La version la plus récente est disponible par ftp sur ftp://suphys.physics.su.oz.au/wp2latex/ ou sur ftp://ftp.fdn.org/pub/CTAN/support/wp2latex/glenn/.

texperf est disponible sur ftp://ftp.fdn.org/pub/CTAN/support/texperf/.

23.7  Comment convertir du LATEX en RTF ?

Le format RTF (Rich Text Format) correspond à un format ASCII contenant des informations de formatage lisibles (entre autres) par Word pour DOS, Word pour Windows et Word pour Macintosh. Pour plus d'informations sur ce format, voir :

TEX2RTF, est disponible sur ftp://ftp.fdn.org/pub/CTAN/support/tex2rtf/ ou par ftp sur ftp://ftp.aiai.ed.ac.uk/pub/packages/tex2rtf/. Il tourne sous SPARC Open Look, SPARC Motif et Windows3.1. TEX2RTF ne gère pas les formules mathématiques ni les tableaux compliqués.

LATEX2rtf, de F. Dorner et A. Granzer, est disponible sur ftp://ftp.fdn.org/pub/CTAN/support/latex2rtf/. Il s'appuie sur LATEX 2.09. Une version est également disponible sur ftp://ftp.lps.u-psud.fr/pub/ltx2rtf/ltx2rtf.zip.

LATEX2rtf produit du RTF avec la simple commande (sous DOS):

        ltx2rtf srcltx.tex
qui produit

        srcltx.rtf
ATTENTION :
 

  1. Ça ne traite pas toutes les maths mais ça traduit du texte LATEX pour être relu en Word.
  2. L'exécutable 386 est fourni mais il y a un makefile qui compile le tout pour UNIX
  3. Les lettres accentuées au codage 850 ne sont pas traduites correctement, utiliser le codage 7bits (\'e, \i, etc).
  4. Ça utilise la numérotation automatique pour les sections, les chapitres, etc. Mais les nombreux bugs que j'ai découverts dans Word 6.0 rendent la présentation un peu aléatoire.
  5. Ça traite les < < et les > > pour les guillemets.
  6. Ça traite (sauf erreur) les commandes de changements de polices de du genre \bfseries, \ttfamily, \sffamily, \slshape et les tailles de polices.

23.8  Comment convertir du RTF en (La)TEX ?

23.9  Comment convertir du Excel en LATEX ?

Le package 'Tabular', de A. Gjestland, est disponible par ftp sur ftp://ftp.ssh.no/pub/tabular/. Il existe deux versions Tabular4.xla pour Excel4 et Tabular5.xla pour Excel5. Une documentation est également disponible : docu-tab.ps.Z. Vous pouvez aussi consulter la page WEB http://www.ssh.no/~ag/tabular/docu-tab.html. Pour tous renseignements, contacter l'auteur par e-mail : ag@hsh.no.

Il existe également l'utilitaire Excel2LATEX disponible sur le WEB à http://www.informatik.uni-trier.de/CIP/marder/xl2latex.zip.

23.10  Comment convertir du HTML en LATEX ?

Remarque :
HTML étant à la base du SGML, une autre solution (non testée) serait d'utiliser un traducteur SGML->LATEX. Le système 'linuxdoc-sgml' devrait pouvoir faire ça si on lui donne la DTD de la version d'HTML concernée et les règles de traduction. Pour plus de détails consulter le document disponible sur ftp://ftp.loria.fr/pub/linux/sunsite/utils/text/linuxdoc-sgml-*.tar.gz

23.11  Comment convertir du LATEX en HTML ?

Remarque :
IDVI est un outil qui permet de présenter des documents WWW à partir de sources LATEX. IDVI est une implémentation Java d'un viewer dvi qui permet d'afficher des pages LATEX sur le WEB et non plus un document HTML. Pour plus de détails, consulter : http://www.geom.umn.edu/java/idvi/
Remarque :
le package 'html' permet d'insérer dans le source LATEX des commandes traitées de manière spéciale par le convertisseur latex2html.
ATTENTION :
cet utilitaire nécessite l'utilisation de et de emacs.

23.12  Comment convertir un fichier dvi en ascii ?

dvi2tty est disponible sur ftp://ftp.fdn.org/pub/CTAN/dviware/dvi2tty/.

dvispell de la distribution EmTEX permet également de convertir un fichier dvi en fichier texte.

23.13  Comment convertir du WEB en LATEX ?

SchemeWEB, de J. Ramsdell, est disponible sur ftp://ftp.fdn.org/pub/CTAN/web/schemeweb/.

23.14  Comment convertir du TEX en Framemaker ?

Framemaker dispose d'un filtre permettant d'importer des fichiers TEX.

23.15  Comment enlever les balises LATEX d'un document ?

detex, disponible sur ftp://ftp.fdn.org/pub/CTAN/support/detex/, permet d'enlever toutes les commandes et séquences de contrôle LATEX d'un document. Il existe sous Unix, Doc et Mac.

ATTENTION :
detex n'enlève que les macros, et pas leurs arguments, ce qui donne des résultats pas toujours très propres avec des documents LATEX.

23.16  Comment convertir du SGML en (La)TEX ?

Remarque :
SGML = Standard Generic Markup Language. SGML est une norme de balisage de documents structurés
Pour plus de détails, consulter :

sgmlspm, de D. Megginson, (Perl) est disponible à http://www.connection.co.uk/bob/sgml/SGMLSpm/sgmlspl/sgmlspl.html.

stil, de J. Schrod et C. Detig, (Common Lisp) est disponible par ftp sur ftp://ftp.th-darmstadt.de/pub/text/sgml/stil/.

Ces deux outils s'appuient sur le parser nsgmls de J. Clark qui traduit du SGML en ESIS.

linuxdoc-sgml s'appuie aussi sur sgmls de J.Clark.

sgml2tex, de P. Flynn, sur PC est disponible sur ftp://ftp.fdn.org/pub/CTAN/support/sgml2tex/.

23.17  Comment convertir du WinWord en LATEX ?

WINW2LTX, de A. Cottrell, est disponible sur ftp://ftp.fdn.org/pub/CTAN/support/winw2ltx/english/ww2/ pour WinWord2, et ftp://ftp.fdn.org/pub/CTAN/support/winw2ltx/english/ww6/ pour WinWord6. Il ne traite ni les équations, ni les graphiques.

23.18  Comment convertir un fichier 8 bits en fichier 7 bits ?

B. Raichle a fourni un tel convertisseur parfaitement portable qui se présente sous forme d'un programme ``.tex'' appelé convert.tex. Celui-ci est disponible sur ftp://ftp.fdn.org/pub/CTAN/language/typingtex/.

Il existe également une version modifiée par L. Siebenmann dans /Convert-RaichleHacked.dir/ (interface modifiée, ajout de quelques tables de transformation dont mac2dek.tbl ou pc2dek pour la conversion du Macintosh ou du PC vers la syntaxe classique de Knuth \'e, \c{c} etc.).

L'outil Tower of Babel (avec option TEX activée), permet également de réaliser de telles conversions. Il est disponible par ftp sur ftp://ftp.lip6.fr/pub/mac/info-mac/text/tower-of-babel-13.hqx.

recode de F. Pinard est un programme unix (GNU) qui convertit tous types de fichiers texte. De nombreux formats sont supportés (ASCIIs, EBCDIC, CDC, LATEX, etc).

Exemple :
 
recode latin1:applemac <fichier>
pour une conversion 7 bits vers 8 bits

dans emacs, le package iso-cvt.el, peut convertir automatiquement un fichier codé 7 bits en fichier codé 8 bits (et réciproquement) lors de la lecture et de l'écriture. Pour l'utiliser, ajouter (require iso-cvt) dans .emacs. Le fichier sera alors en 7 bits sur le disque.

De même, il existe le package x-symbol pour (X)emacs. Pour plus de détails, voir http://brahms.fmi.uni-passau.de/~wedler/x-symbol.html.

Il existe également les utilitaires kb7to8/8to7 distribués avec 'french'.

23.19  Comment convertir un fichier ChiWriter en TEX ?

chi2tex est disponible sur ftp://ftp.fdn.org/pub/CTAN/support/chi2tex/. Il y a plusieurs versions du convertisseur, selon la version de ChiWriter à convertir. A partir des versions 2 ou 3 cela ne marchait que pour des fichiers très simples (pas plus d'un niveau d'exposant et d'indice, etc). Il est probable que le convertisseur de la version 4 marche beaucoup mieux : il fait une analyse syntaxique des formules, alors que les versions 2 et 3 étaient purement graphiques.

23.20  Où trouver une FAQ de convertisseurs (La)TEX/Traitement de texte ?

La FAQ de W. Hennings est disponible sur http://www.kfa-juelich.de/isr/1/texconv.html ou sur ftp://ftp.fdn.org/pub/CTAN/help/wp-conv/.

Elle est également postée toutes les semaines dans comp.text.tex. AL: Ceci ne semble plus être le cas...

Voir également la page http://www.wotsit.demon.co.uk/text.htm ou la page http://wotsit.simware.com/.

23.21  Comment convertir une image en police metafont ?

C'est le programme bm2font qui permet de faire cela. Il est disponible sur CTAN. La documentation est aussi sur le LATEX Navigator (http://tex.loria.fr/).

Pour plus de détails sur metafont il existe la liste metafont@ens.fr. Pour s'inscrire, envoyez à

              listserv@ens.fr
le message :

              SUBSCRIBE metafont nom de forme libre

23.22  Comment convertir un fichier postscript en ascii ?

pstotext est un outil Unix freeware qui permet d'extraire du texte ascii d'un fichier postscript. Il utilise Ghostscript. pstotext est disponible à http://www.research.digital.com/SRC/virtualpaper/pstotext.html.

Sous Windows et OS/2 pstotext est accessible via l'outil GSview de R Lang (version 2.0 minimum). Voir http://www.cs.wisc.edu/~ghost/gsview/.

Pour DOS, pstotext est disponible à http://www.cs.ruu.nl/~piet/software.html.

23.23  Comment convertir un fichier pdf en ascii ?

L'outil pstotext présenté à la question précédente permet également d'extraire du texte ascii d'un fichier pdf.

23.24  Comment convertir du LATEX en PDF ?

Remarque :
il vaut nettement mieux demander à LATEX de travailler avec des polices PostScript, si l'on compte transformer le document en PDF par la suite. Si vous utilisez des polices à résolution fixe, le texte aura une apparence  on ne peut plus laide sur votre écran.

23.25  Comment définir son propre format de sortie ?

ltx2x disponible sur ftp://ftp.fdn.org/pub/CTAN/support/ltx2x/ permet de remplacer des commandes LATEX par des commandes définies par l'utilisateur.

Chapter 24
Préviewers et viewers

Les viewers et previewers présentés ci-dessous peuvent permettre entre autres de visualiser des documents .dvi et .ps. Toutefois, ils ne sont pas attachés à LATEX.

24.1  Où trouver un previewer ?

Où trouver un viewer ?

Ghostscript, Ghostview et GSview pour PC, Mac et station de travail sont disponibles sur http://www.cs.wisc.edu/~ghost/index.html.

viewps de T. Kiffe est inclu dans la distribution CMacTEX disponible sur CTAN.

TrueTEX est un viewer pour PC sous windows.

gv est une interface basée sur Ghostview et améliorée. Les actions disponibles sont nombreuses et il est possible de lire un document sur écran. On dispose aussi de l'anti-aliasing et de zooms impressionnants sur les parties de la page que l'on veut. gv peut même afficher des fichiers PDF si on dispose de la version de GhostScript adéquate.

Pour plus de détails consulter : http://wwwthep.physik.uni-mainz.de/~plass/gv/ ou un site FTP : ftp://thep.physik.uni-mainz.de/pub/gv/.

gv tourne sur les versions d'Unix les plus courantes, il faut néanmoins disposer du widget Xaw3d.

24.2  Comment visualiser des fichiers non postscript sous xdvi ?

D. Barbier propose le script suivant pour inclure des fichier MPS générés pas Metapost.

La démarche retenue est de pomper dans le .ps toutes les définitions qui manquent à ce pauvre xdvi. Par exemple, pour voir les dessins dans le fichier essai.tex, je fais :

    makepro essai.tex
Ce qui génère essai.ps (s'il est vieux) et produit un fichier essai.pro. Il ne reste plus qu'à mettre \special{header=essai.pro} avant le \begin{document} et le tour est joué. Quand on modifie le source (on rajoute des dessins), il suffit de relancer makepro, et de recompiler le document.

Quelques remarques :

Comme je ne connais pas grand chose en PostScript, toutes les remarques seront les bienvenues. Par exemple, les paramètres dans fshow et fstore sont mis au pif. Quelles sont les bonnes valeurs ? Comment faire pour que Psfrag passe à la moulinette ? Et bien d'autres choses.

%%%% debut macro %%%% 
 #!/bin/sh 
 # D. Barbier 31/10/97  barbier@cmapx.polytechnique.fr 
 # Commande : makepro nom_de_fichier 
 # Usage    : voir ci-dessous (ou taper makepro -h) 
 usage="Usage: `basename $0` nom[.tex]\n\ 
 Cree un preambule \"nom\".pro qui permet l affichage 
 des fichiers non postscript (inclus par \"nom\") avec 
 xdvi (version 20 et plus).\n\ 
 Les applications sont notamment l inclusion de fichiers 
 generes par Metapost." 
 

 case "$#:$1" in 
    1:-h*|1:--h*) echo "$usage"; exit 0;; 
    1:*) ;; 
    *) echo "$usage"; exit 1;; 
 esac

 

 if test -r "$1" ; then 
    filetex=$1 
    file=`echo "$filetex" | sed -e 's/\.[.]*$//'` 
 else 
    file=$1 
    filetex=$file.tex 
    if test ! -r "$filetex" ; then 
        echo "Erreur: le fichier $file n existe pas" 
        exit 1 
    fi 
 fi 
 filedvi=$file.dvi 
 fileps=$file.ps 
 filepro=$file.pro 
 rm -f $filepro 

 # On teste si la commande \special{header=...} 
 # est dans le fichier 
 # Autant ne faire qu'une compilation avant de 
 # voir le resultat 
 ligne="\\\\special{header=$filepro}" 
 if test -z "`grep $ligne $filetex`" ; then 
    echo "Rajouter la comande $ligne avant \\\\begin{document}" 
    exit 1 
 fi

 

 echo "%!" > $filepro

 

 # On ne compile le .tex que si necessaire. Il y a 
 # plusieurs possibilites pour voir la date de modification 
 # des fichiers : 
 # 1) utiliser newer fourni avec certaines distributions de TeX 
 #    (je n'ai pas reussi a le faire marcher). 
 # 2) test (si l'option -nt est supportee) 
 # 3) creer un makefile. 
 # J'ai choisi cette derniere solution 

 echo "$filedvi: $filetex\n\tlatex $filetex\n\ 
 $fileps: $filedvi\n\tdvips -o $fileps $filedvi\n" > .tmp$$

 

 make -f .tmp$$ $fileps

 

 rm -f .tmp$$

 

 awk ' 
 # Liste des preambules qu on veut utiliser 
 BEGIN {header["tex.pro"]=1;header["texps.pro"]=1} 
 /%%BeginProcSet:/ { if ( header[$2] == 1 ) {getline 
 while ( $1 != "%%EndProcSet" ) {print;getline}}} 
 /%%EndProlog/ {exit 0} 
 ' $fileps | 
 # On supprime la creation des dictionnaires. 
 sed -e 's%/TeXDict [[:digit:]]* dict \([[:alpha:]]*\)%%' \ 
    -e 's%/SDict [[:digit:]]* dict \([[:alpha:]]*\)%%' \ 
      > > $filepro 
 

 # Ce sont les commandes qui font l affichage. 
 # En remplacant les commandes de finclude.pro par celles-ci, 
 # ca a l air de marcher

 

 cat < < EOF > > $filepro 
 /fstore{TeXDict begin /VResolution 120 def /Resolution 120 
 def end 
 dup dict exch{dup 4 2 roll put}repeat def}bind def 
 /fshow{gsave 72 TeXDict /Resolution get div -72 TeXDict 
 /VResolution get div scale 1 
 5 div dup scale get cvx exec show grestore}bind def 
 EOF

 

 # Et la, ce sont les commandes de fontes 
 awk ' 
 /TeXDict begin ([[:digit:]]* ){5}/ {i++} 
 /TeXDict begin ([[:digit:]]* ){5}/,/end/ {if (i==1) print} 
 /%%EndProlog/ {exit 0} 
 ' $fileps > > $filepro 
 

 echo "Fichier $filepro genere"

 

 exit 0 
%%%% fin macro %%%%

Chapter 25
Logiciels de dessins

Afin de générer des fichiers graphiques que vous insérerez dans votre document TEX, différents logiciels sont disponibles.

25.1  Où trouver un logiciel de dessin ?

Remarque :
Il y a un excellent éditeur vectoriel GRATUIT sous windows, nommé MayuraDraw (anciennement PageDraw) et que l'on peut trouver sous http://www.wix.com/PageDraw/. Avec Ps2ai, cet éditeur permet d'importer des fichiers ps (presque) quelconques, de les manipuler et de les exporter au format pdf, eps, wmf, ai, ps, bmp et tif.
Remarque (R. Roussillon) :
Ce package mériterait d'être plus connu que ce qu'il est actuellement.
Exemple :
 
\polyline{(0,0),(1,2),(3,-1)} 
\arc[t]{(1,0),(1,1),(0.5,1)} 
%%%% fin exemple %%%%

25.2  Comment inclure des formules Latex dans Xfig ?

Il suffit de mettre le flag special au texte, de taper son texte LATEX en incluant des commandes, d'exporter son fichier (toto.fig par exemple) en deux parties : Combined PS/LATEX (PS part) Combined PS/LATEX (LATEX part) et de faire \input{toto.pstex_t}, en ayant bien sûr chargé au préalable le package 'epsfig' par la commande \usepackage{epsfig} placée dans le préambule.

On peut utiliser également tgif qui permet d'insérer du LATEX dans les figures. Son fonctionnement est différent : le source est compilé dans tgif même, et le résultat dvi est converti en Postscript et apparaît dans la figure. Le positionnement dans la figure de l'élément compilé est donc plus aisé, mais il est assez contraignant de lancer LATEX pour compiler chaque élément.

25.3  Comment gérer différents formats de figures ?

Chapter 26
Correcteur

L'orthographe est encore un gros problème dans les documents dactylographiés. Il n'est pas rares de vois des coquilles dans les meilleurs bouquins. La correction des mots s'avère donc utile.

26.1  Où trouver un correcteur orthographique ?

26.2  Où trouver un vérificateur de syntaxe LATEX ?

Utilisation :
 
 \documentclass[syntonly]{...}
Quelques exemples de services :

Cet outil est configurable. Il supporte LATEX209 et .

Le package 'refcheck' disponible sur ftp://ftp.fdn.org/pub/CTAN/macros/latex/contrib/supported/refcheck/ permet de vérifier les références d'un document.

Chapter 27
Assocations

Qui utilise LATEX, quels sont les organismes qui distribuent ce logiciel. Un point sur les contacts disponibles dans l'univers de la typographie numérique.

27.1  Qu'est ce que l'association GUTenberg ?

L'association GUTenberg (loi 1901) a pour objectifs de regrouper les utilisateurs francophones de TEX, de favoriser les échanges techniques permettant d'augmenter les possibilités d'impression et de publications scientifiques et d'offrir à ses adhérents un certain nombre de services dont des distributions TEX et LATEX francisées. Le serveur d'archives de GUTenberg est herbergé sur le serveur ftp anonyme : ftp://ftp.gutenberg.eu.org/pub/gut/ ou sur le WEB à http://www.gutenberg.eu.org.

GUTenberg publie la Lettre GUTenberg ainsi que les cahiers GUTenberg.

Remarque :
Le cahier 23 de GUTenberg est également une FAQ LATEX.
ATTENTION :
la FAQ que vous êtes en train de lire est indépendante de l'association GUTenberg.

27.2  Qu'est ce que l'association TUG ?

TUG (TEX Users Group) est une organisation internationale dont un tiers des membres est européen. TUGboat est la lettre de ce groupe. Pour plus de renseignements, contacter tug@tug.org par mail ou consulter le site WEB http://www.tug.org/.

27.3  Qu'est ce que l'association AsTEX ?

L'association AsTEX a comme objectif principal d'essayer de faire du travail utile au plus grand nombre, dans le domaine des logiciels scientifiques, et d'essayer de faire ce travail aussi bien que les éditeurs privés, mais dans un esprit de service public.

Cela passe par l'écriture d'utilitaires d'installation et de configuration automatisés (pour que le débutant en TEX/LATEX n'ait pas à lire 1000 pages de docs disparates, en anglais de surcroît, avant de pouvoir imprimer ``Bonjour''), par l'écriture de docs raisonnablement bien rédigées et agréables à consulter, par des distributions sur disquettes bien présentées. Cela passe également par la traduction de docs originales dans un français correct, etc.

Chapter 28
Fontes

Les fontes permettent de changer l'apparence des lettres de votre document. Bien que de nombreuses de fontes soient installées sur votre système par la distribution de base de LATEX, il peut être utiled'en ajouter.

28.1  Que signifient les sigles T1, mf, fd etc. ?

Contribution de P. Pichaureau (ppichaur@grannus.u-strasbg.fr) :

Voici une mise au point rapide, histoire que vous compreniez de quoi ça parle.

En 1990, lors d'une réunion d'utilisateurs de TEX, à Cork, il a été décidé de développer une table d'encodage particulière pour les fontes de TEX. Cette table contient des signes diacritiques et un certains nombre de symboles qui permettent de composer des textes dans un bon nombre de langues européennes.

Ce codage s'appelle T1 (parfois Cork encoding), et l'ancien codage de TEX s'appelle OT1 (Old T1). Les autres codages (U, etc.) concernent des polices particulières et/ou des polices qui ne  respectent ni T1 ni OT1.

Les fontes « standard » de TEX qui respectent ce codage s'appellent fontes EC (pour european coding, il me semble). Les fontes DC étaient une pré-version des fontes EC. La phase de mise au point des fontes EC ayant duré un certain nombre d'années, on s'y perd un peu.

Les fontes TC (text companion) contiennent un certain nombre de caractères textuels utilisés en mode mathématique. À l'origine, Knuth prenait ces caractères dans les polices textes normales, mais cela pose des problèmes si vous essayez d'utiliser d'autres polices de texte. C'est pour cela qu'on préfère maintenant les mettre à part.

Ça c'est pour les problèmes de codage.

Pour les pk, mf, etc. je refuse de rentrer dans les détails, mais voici un rapide aperçu de la question :

NFSS, c'est la manière dont sélectionne une fonte. LATEX ne le faisait pas assez proprement, alors on a fait le ménage. Un fichier .fd dit à quels sont les fichiers .mf à utiliser pour telle police, dans telle taille, dans telle famille, avec telle variation. C'est avec ce fichier que vous dites à de prendre la version sans serif dans tel fichier, la taille 9pt dans tel autre, et le gras dans ce troisième fichier.

En tout état de cause, tfm et fd sont indispensables à la bonne marche de latex2e. Les mf sont indispensables à la creation des tfm et despk, et les pk sont indispensables à la visualisation et à l'impression.

28.2  Quels sont les attributs d'une fonte ?

Une fonte possède :

28.3  Comment utiliser une fonte ?

Pour déclarer une fonte par défaut dans un document, il existe les commandes \familydefault, \encodingdefault, \seriesdefault et \shapedefault.

Exemple :
 
\documentclass{report} 
\usepackage{french} 
\pagestyle{empty} 
\renewcommand{\familydefault}{cmtt} 
\begin{document} 
\begin{verbatim} 
\renewcommand{\familydefault}{cmtt} 
\end{verbatim} 
dans le préambule d'un document permet de sélectionner la fonte 
computer moderne de type machine à écrire pour tout le corps du 
document. 
\end{document} 
%%%% fin exemple %%%%
En , un certain nombre de packages permettent de faire appel à une fonte particulière.

Exemple :
 
\documentclass{report} 
\usepackage{french} 
\pagestyle{empty} 
\usepackage{times} 
\begin{document} 
\begin{verbatim} 
\usepackage{times} 
\end{verbatim} 
dans le préambule d'un document permet de sélectionner la fonte 
times pour tout le corps du document. 
\end{document} 
%%%% fin exemple %%%%
A un niveau plus bas, dans la création d'un style par exemple, la sélection d'une fonte peut se faire de la manière suivante :

\fontfamily{ccr}\fontencoding{T1}\fontseries{c}\fontshape{sl}% 
\fontsize{9}{11pt}\selectfont
ou encore :

\fontsize{14}{16pt}\usefont{OT1}{cmdh}{bc}{it}
Pour définir une commande de changement de fonte, on peut utiliser \DeclareFixedFont.

Exemple :
 
\DeclareFixedFont{\petitefonte}{\encodingdefault}%  
\familydefault}{\seriesdefault}{\shapedefault}{6pt}  
\newcommand{\petit}{\petitefonte} 
%%%% fin exemple %%%%

28.4  Comment changer la forme d'une fonte ?

En , pour un changement ponctuel de fonte, un certain nombre de commandes sont disponibles par défaut :

Exemple :
 
Un \textbf{bel} arbre. 
Un \textbf{\textit{très bel}} arbre. 
%%%% fin exemple %%%%
ce qui donne :

Un bel arbre. Un très bel arbre.

 

Pour changer la fonte de tout un paragraphe, on utilisera plutôt les commandes \rmfamily, \sffamily, \ttfamily, \bfseries, \mdseries, \itshape, \slshape, \upshape et \scshape.

Remarque :
si on utilise ces commandes sur un seul mot ou sur un groupe de mot dans un paragraphe, alors l'espace suivant une telle déclaration ne sera pas géré (il vaut mieux dans ce cas utiliser les commandes \textxx).
Exemple :
 
{\scshape BlaBla }
ou

\begin{itshape} 
        blabla 
\end{itshape} 
%%%% fin exemple %%%%
ce qui donne :

BLABLA

ou

blabla

28.5  Comment changer la taille d'une fonte ?

Il existe les commandes \tiny, \scriptsize, \footnotesize, \small, \normalsize, \large, \Large, \LARGE, \huge, \Huge, classées dans l'ordre croissant de taille. Ces commandes sont prédéfinies en fonction de la classe de votre document.

Remarque :
changement de taille de la fonte entraîne automatiquement un changement de l'interligne.
Plus globalement, vous pouvez utiliser la commande \fontsize (cf. ).

28.6  Comment modifier la fonte des numéros de paragraphe ?

Pour modifier la fonte des numéros de paragraphe, il faut redéfinir seccntformat.

Exemple () :
 
\makeatletter 
\renewcommand\@seccntformat[1]{\texttt{\@nameuse{the#1}\quad}} 
\makeatother 
%%%% fin exemple %%%%

28.7  Comment modifier la fonte du mode verbatim ?

Le package 'verbatim' disponible sur ftp://ftp.fdn.org/pub/CTAN/macros/latex/required/tools/ permet de faire cela. Il est alors conseillé d'utiliser des fontes aux normes T1.

L'environnement verbatimcmd du package 'moreverb', disponible sur ftp://ftp.fdn.org/pub/CTAN/macros/latex/contrib/other/misc/, permet de garder les caractères backslash (\) et les accolades ({, et }) actifs. On peut donc entre autres opérer des changements de fonte.

Pour changer la taille de la fonte du mode verbatim, il faut l'encapsuler dans des commandes de modification de taille.

Exemple :
 
Texte avant. 
\begin{small} 
\begin{verbatim} 
  Texte... 
\end{verbatim} 
\end{small} 
Texte après. 
%%%% fin exemple %%%%

28.8  Comment réaliser des changements de fontes relatifs ?

Le package 'relsize' disponible sur ftp://ftp.fdn.org/pub/CTAN/macros/latex/contrib/other/misc/ permet de faire ce genre de choses. Les commandes offertes sont du type :

Exemple :
 
Un \relsize{1}texte \relsize{2}de \relsize{3}toutes 
\relsize{-1}les \relsize{-4}tailles. 
%%%% fin exemple %%%%
avec un argument, on trouve :

\textsmaller{text}, \textlarger{text}, \mathsmaller{A}, \mathlarger{B} 
%%%% fin exemple %%%%
Le package 'scalefnt' de D. Carlisle disponible sur ftp://ftp.fdn.org/pub/CTAN/macros/latex/contrib/supported/carlisle/ permet d'augmenter ou de diminuer suivant un facteur proportionnel, la taille de la fonte courante.

Exemple :
 
\scalefont{2} double la taille de la fonte courante 
\scalefont{.75} réduit de trois quarts la taille de la fonte 
courante. 
%%%% fin exemple %%%%

28.9  Comment mettre en évidence une portion de texte ?

Utiliser la commande \emph. Cette commande est définie dans la classe de votre document, et sert spécifiquement à mettre en évidence un mot, une expression ou toute une phrase. La plupart du temps, elle se contente de mettre en italique votre texte.

28.10  Où trouver des fontes ?

Sur CTAN bien sûr, dans ftp://ftp.fdn.org/pub/CTAN/fonts/.

28.11  Comment suivre le chargement des fontes ?

Le package 'tracefnt' permet de suivre le chargement des fontes lors de la compilation d'un document. Ce package définit plusieurs options :

28.12  Pourquoi MakeTEXPK est lancé alors que la fonte existe ?

P. Terray :

C'est un problème de mise à jour de la base lié à kpathsea. Normalement, MakeTEXPK met à jour la base ``ls-R'' de la TDS, à chaque ajout de police. Si cette base n'est pas autorisée pour tout le monde, ou si la variable TEXMF est mal réglée, dvips ou xdvi ne peuvent pas vérifier que cette police existe. Du coup, ils lancent MakeTEXPK pour la fabriquer. Et MakeTEXPK sachant par ailleurs où mettre la police, il vérifie qu'elle existe, et c'est pour ça qu'il donne le message comme quoi elle existe déjà.

Les solutions (UNIX) :

ATTENTION :
il faut avoir les droits de gestionnaire LATEX (ou root) pour exécuter texhash.

28.13  Comment utiliser \textsc dans un titre en conservant le gras ?

Remarque :
les sc grasses n'existent pas dans les fontes de Knuth. En revanche, elles existent dans les fontes EC.
Depuis les dernières modifications dans les fontes EC, la simple utilisation de ces fontes suffit à activer les petites capitales grasses.

28.14  Comment utiliser des fontes TrueType?

Une source d'information à ce sujet: http://www.radamir.com/tex/ttf-tex.htm

Chapter 29
Divers

Toutes les questions qui n'ont pas pu être classées dans les chapitres précédents sont regroupées ici. Il s'agit le plus souvent de petits trucs très utiles que tout bon LATEX nicien devrait connaître.

29.1  A quoi sert la commande \par ?

Elle permet de marquer explicitement la fin d'un paragraphe pour permettre à LATEX de le mettre en page. L'utilité de cette commande se justifie par le fait que LATEX met en page un texte par paragraphe. Elle est, le plus souvent, utilisée dans la définition de macros.

29.2  Comment commenter une partie d'un source LATEX ?

Exemple:
 
Un paragraphe qui intéresse tout le monde et qui est donc 
visible par tout le monde. 
\begin{comment} 
   Un paragraphe qui n'intéresse que moi et qui n'est visible 
   que dans le source de mon document. 
\end{comment} 
Un autre paragraphe qui intéresse tout le monde. 
%%%% fin exemple %%%%

29.3  Comment utiliser LATEX sur des petites machines ?

La capacité de TEX est limitée de manière interne. Cela signifie que ce sont des variables de TEX qui fixent le nombre de variables (et la taille mémoire) maximal que TEX s'autorisera à utiliser. Si vous utilisez plus de variables que prévu, TEX s'arrêtera sur le message d'erreur :

   sorry, TeX capacity exceeded.
Tout ceci est destiné à < < préserver > > votre machine, c'est à dire à empêcher TEX d'utiliser la totalité de la mémoire de la machine, ce qui conduit la plupart du temps à un plantage.

Sur une station, aucun problème, mais sur un micro-ordinateur, les configurations peuvent-être nombreuses. C'est pourquoi le TEX fournit en < < standard > > sur les macs, par exemple, des variables fixées assez bas (on suppose que le Mac a une toute petite mémoire comparée à une station). Mais il existe également un BigTEX, dont la seule différence avec le TEX normal est qu'il est autorisé à utiliser plus de mémoire (les variables en questions ont des valeurs plus importantes).

Du coup, si votre micro a une configuration plutôt musclée, vous pouvez compiler des documents plus gros, plus complexes, etc...

Heureusement Kiffe a changé de stratégie et dans les futures versions des outils les paramètres internes seront configurables (en passant par ResEdit) et donc on décidera soi-même la < < bigness > > de son TEX (cf. distributions emTEX et AsTEX). Pour changer les paramètres de compilation (pool size, stack, font mem, main mem etc.) cf. fichier emtex/doc/english/tex.doc, chapitre .

La version 7.0 de web2c est configurable par le fichier texmf.cnf.

Remarque : pour un gros document, il peut être utile de le découper en chapitres à inclure par la commande \include (cf. paragraphe ).

29.4  Comment visualiser des paramètres ?

\message{Valeur de /baselinestretch = \the\baselinestretch}
Exemple :
 
\documentclass{report} 
\usepackage{french} 
\pagestyle{empty} 
\begin{document} 
Texte. 
\showthe\baselineskip 
Texte. 
\end{document} 
%%%% fin exemple %%%%
Remarque :
ce package n'est pas compatible avec les styles multicolonnes ni avec l'option leqno.

29.5  Comment visualiser des compteurs ?

Pour visualiser la valeur d'un compteur, il suffit de demander

\immediate\write{16}{\the\nomducompteur} 
ou

\message{\the\nomducompteur}

29.6  A quoi servent \makeatletter et \makeatother ?

Le caractère ``@'' est spécial pour LATEX, et ne peut pas être utilisé directement comme partie d'un nom de macro dans un document LATEX. Pour utiliser certaines macros internes de LATEX (celles qui contiennent un ``@''), directement dans un document (au lieu d'utiliser un fichier .sty), il faut les encapsuler dans une macro dont le nom ne contient pas de ``@''. En outre, cette macro doit impérativement être déclarée dans le préambule du document, et être encadrée par les commandes \makeatletter et \makeatother.

En fait ces commandes permettent d'indiquer à LATEX que l'on veut effectivement travailler sur ses commandes internes  en le forçant à considérer le ``@'' (at) comme une lettre (makeatletter).

ATTENTION :
les commandes internes de LATEX sont à manipuler avec précaution.

29.7  Comment numéroter les lignes d'un document ?

Le package 'lineno' disponible sur ftp://ftp.fdn.org/pub/CTAN/macros/latex/contrib/supported/lineno/ permet de numéroter les lignes d'un document.

Il existe également le package 'numline' de M. Jaegermann et J. Fortune, pour du texte essentiellement. Ce package est disponible sur ftp://ftp.fdn.org/pub/CTAN/macros/latex/contrib/supported/numline/.

Pour numéroter les lignes d'un document inclu en mode verbatim, il faut utiliser la commande \listinginput du package 'moreverb' disponible sur ftp://ftp.fdn.org/pub/CTAN/macros/latex/contrib/supported/moreverb/.

Le package 'vruler' disponible sur ftp://ftp.fdn.org/pub/CTAN/macros/latex/contrib/supported/vrsion/ permet également de numéroter les lignes d'un document.

29.8  A quoi sert la commande \special ?

La commande \special permet à TEX d'envoyer des instructions particulières (non TEX) à un driver sans les interpréter. Les instructions ainsi passées sont généralement dépendantes du driver qui lui, saura les interpréter. L'utilisateur n'a en général pas à se soucier de cette commande (\special), elle est gérée par des packages tels que 'psfig' pour LATEX2.09 ou 'graphics' et 'graphicx' pour .

Remarque :
\includegraphics est standard dans et indépendant de la plateforme alors que le format de \special est dramatiquement dépendant de la plateforme de travail. Seul, le \includegraphics préserve l'aspect universel d'un document source et donc l'esprit universel de TEX.

29.9  Comment réaliser des calculs avec les variables LATEX ?

Pour pouvoir utiliser des expressions du type \truc * \machin dans une macro, il faut charger le package 'calc' disponible sur ftp://ftp.fdn.org/pub/CTAN/macros/latex/contrib/supported/calc/.

Exemple :
 
\setcounter{x}{3*\real{1.3}} 
%%%% fin exemple %%%%
Une autre solution consiste à utiliser les commandes \addtolength\dim\dim, \divide \dim by \dim, etc.

Le package 'realcalc' disponible sur ftp://ftp.fdn.org/pub/CTAN/macros/generic/realcalc/ permet de faire des calculs sur des réels.

Plus difficile à utiliser, il y a aussi le package 'trig'.

29.10  Où trouver une fonte 9pt ?

La classe de document 'amsart' disponible sur CTAN offre l'option 9pt.

De même les classes 'extarticle' et 'extreport' sont disponibles à http://www.informatik.uni-freiburg.de/~may/extsizes.html.

size9.clo de H. Steffani est disponible à http://www.tu-chemnitz.de/~hfst/size9.clo.

29.11  Comment automatiser les compilations LATEX ?

29.12  Comment obtenir des cadres gris ?

Le package 'shade' disponible sur ftp://ftp.fdn.org/pub/CTAN/macro/generic/ permet de tracer des cadres gris.

Le package 'psboxit', disponible sur ftp://ftp.fdn.org/pub/CTAN/macros/latex2.09/contrib/misc/, permet d'obtenir du gris ou de la couleur.

29.13  Comment obtenir certaines abréviations ?

Pour obtenir des abréviations dont certaines lettres doivent être rehaussées et de taille réduite, on peut utiliser les indices mathématiques. Mais, ce n'est pas une technique recommandable.

Exemple :
 
M${\mathrm{lle}}$ 
%%%% fin exemple %%%%
Le package 'babel' propose la commande \ieme pour les quantièmes.

Le package 'french' de B. Gaulle, définit la commande \fup pour écrire une chaîne de caractères en exposant, ainsi que les commandes \ier \iers \iere \iere \ieme \iemes pour les quantièmes.

Exemple :
 
M\fup{lle} 
%%%% fin exemple %%%%
Remarque :
pour que la commande \fup suive les changements de taille de fonte, il faut utiliser l'une des extensions prévues à cet effet, à savoir les extensions ``smaller'' et ``relsize'' (\usepackage{smaller,french} par exemple). Une copie de l'extension smaller est distribuée depuis 94 avec la distribution des fichiers du style french (sous le nom mysmall.sty). En revanche, la surélévation ne tient pas compte de l'inclinaison de la police.
Le package 'smaller' de D. Taupin est disponible par ftp sur ftp://ftp.lps.u-psud.fr/pub/latex/contrib/smaller.sty. Il propose également une commande \fup qui définit smaller et l@rger pour être compris par french.sty. Cette commande gère l'inclinaison des \sl et des \it.

ATTENTION :
le fup de B. Gaulle étant installé au \begin{document}, pour utiliser mon \fup modifié avec french.sty, il faut après le \begin{document} écrire:
            \makeatletter 
            \let\fup\f@up 
            \makeatother
On peut également définir une nouvelle commande \abbr dans le préambule du document :

\newcommand{\abbr}[1]{\raisebox{1ex}{\footnotesize #1}}
Cette solution a l'avantage d'éviter d'utiliser le mode mathématique improprement.

Il existe aussi la commande \textsuperscript qui permet d'obtenir un exposant sans passer en mode mathématique.

29.14  Comment gérer les espaces après une macro ?

Pour forcer LATEX à introduire un espace après une macro, il suffit de faire suivre son appel d'un backslash (\).

Exemple :
 
texte \oe\ texte \oe, texte 
texte \oe texte \oe, texte 
%%%% fin exemple %%%%
ce qui donne :

texte  texte , texte
texte texte , texte

 

Pour que LATEX gère lui même les espaces, il suffit d'utiliser systématiquement les caractères {}.

Exemple :
 
texte \oe{} texte \oe{}, texte \oe{}uf 
%%%% fin exemple %%%%
ce qui donne :

texte texte , texte uf

 

On peut également définir une macro TEX \keepspace :

%%%% debut macro %%%% 
\def\keepspace{\ifnum\catcode`\ =10 
  \let\next\keepspacebis \else \let\next\relax \fi 
  \next} 
\def \keepspacebis{\obeyspaces 
  \afterassignment\keepspaceaux\let\next= } 
{\obeyspaces% 
\gdef\keepspaceaux{% 
\ifx \next\space\let\next\ignorespaces\fi% 
\catcode`\  =10\relax\next}} 
%%%% 
%%%% fin macro %%%%
puis l'utiliser dans une autre définition  de macro.

Exemple :
 
\newcommand{\macro}{\textit{blablabla}\keepspace} 
%%%% fin exemple %%%%
Le package 'xspace' intègre ces définitions qui lui permette de gérer automatiquement l'espace après une macro suivant le contexte.

Exemple :
 
\documentclass{report} 
\usepackage{french,xspace} 
\pagestyle{empty} 
\newcommand{\macro}{aaaaa\xspace} 
\begin{document} 
\macro bbbb \macro, bbbb 
\end{document} 
%%%% fin exemple %%%%
Remarque :
à noter que le package xspace est conforme aux règles de typographie anglo-saxones donc la gestion des caractères de ponctuation composés tels que : où ; ne sera pas correcte en français. La solution consiste alors à utiliser le blanc insécable qui de toutes façons est recommandé dans ce cas : \macro: blabla. On peut également se créer son propre package 'fxspace'.

29.15  Où trouver les notices d'utilisation des packages ?

Généralement la notice d'utilisation d'un package est incluse dans le package lui même (fichier .dtx). Il suffit alors de compiler ce fichier. Il existe également le fichier doc disponible sur ftp://ftp.fdn.org/pub/CTAN/macros/latex/base/ qui décrit comment exploiter ces notices.

Si on lance ``latex package.dtx'' il sera produit le fichier package.sty ou package.cls correspondant et le fichier package.dvi de documentation de l'extension.

29.16  Comment obtenir des listes d'objets flottants ?

Certaines sont disponibles par défaut, il s'agit des listes de figures et de tables, il suffit alors d'appeler les commandes \listoffigures et \listoftables. D'autres sont liées à certains packages. Par exemple, le package 'algorithm' de P. Williams offre la commande \listofalgorithms.

Remarque :
la mise à jour de ces listes nécessite généralement plusieurs compilations LATEX.
Exemple :
 
\documentclass{report} 
\usepackage{algorithm,algorithmic} 
\begin{document} 
\listofalgorithms 
\chapter{toto} 
\begin{algorithm}[h!tp] 
   \begin{algorithmic}[2] 
      \REQUIRE $T_1$ et $T_2$ doivent \etre des tableaux de 
      valeurs additionnables, et doivent \etre de la m\eme 
      longueur $n$. \ENSURE $\forall 0\leq k< n \;\
      Result[k]=T_1[k]+T_2[k]$. 
      \FOR{$i\rightarrow 0$ to $n$} 
      \STATE $Result[i]\rightarrow T_1[i]+T_2[i]$ 
      \ENDFOR 
   \end{algorithmic} 
   \caption{Somme terme \`a terme de deux tableaux3} 
   \label{algo-ex-3} 
\end{algorithm} 
[recopie en trois exemplaires] 
\end{document} 
%%%% fin exemple %%%%

29.17  Comment connaître les versions des fichiers utilisés lors d'une compilation ?

Il suffit d'utiliser la commande \listfiles.

Exemple :
 
\documentclass{report} 
\listfiles 
\usepackage{french} 
\usepackage{graphics} 
\begin{document} 
   texte  ... 
\end{document} 
%%%% fin exemple %%%%

29.18  Pourquoi certaines commandes sont elles indépendantes ?

Les commandes LATEX de type verbatim sont particulières du fait qu'elles changent les codes de catégorie de leurs arguments. De ce fait, elles ne sont plus compatibles avec d'autres commandes et ne doivent pas être appelées par ces dernières. Certains caractères spéciaux doivent alors être obtenus par \texttt.

Exemple :
 
\texttt{\char`\\
%%%% fin exemple %%%%
On peut également utiliser l'environement lrbox de pour sauver le texte qui doit apparaître en verbatim dans une boîte et ensuite utiliser cette boîte.

29.19  Comment installer un package ?

Lorsqu'il s'agit d'un package conçu pour et utilisant les procédures et outils définis dans ce cadre, c'est simple : un package est fourni avec deux (ou plus, pour les packages plus complexes) fichiers, package.ins et package.dtx

Il faut commencer par exécuter ``latex package.ins'' pour créer le(s) fichier(s) de commandes lui(eux)-même(s) (typiquement package.sty) puis ``latex package.dtx'' pour générer la documentation, avec si besoin construction d'un index :

makeindex -s gind.ist -o package.ind package.idx
et d'un glossaire :

makeindex -s gglo.ist -o package.gls package.glo.
Ces fichiers doivent ensuite être déclarés dans une variable d'environnement afin d'être rendus visibles.

Exemple (UNIX) :
 
TEXINPUTS=.:/usr/local/TEX/texmf/tex:/users/ADMIN/NG/LATEX: 
/users/home/kluth/Section-equit/Modele 
%%%% fin exemple %%%%
Remarques :
 
Néanmoins, il est clair que ce mode d'organisation et de distribution n'est que le mode encouragé par l'équipe de développement de LATEX et n'est en rien une obligation. Un bon nombre de packages disponibles ne le suit d'ailleurs pas (!), pour diverses raisons (packages qui se veulent utilisables aussi bien avec Plain TEX qu'avec LATEX, < < vieux > > packages écrits pour LATEX209, packages écrits par des personnes qui veulent faire < < comme ça leur plait > >, etc.)... Dans ces différents cas, il faut < < aller y voir de plus près > > pour comprendre l'organisation choisie.

C'est justement le cas pour slashbox (!) : un seul fichier .sty est fourni. Il faut l'éditer pour extraire le fichier de documentation qui se trouve après la ligne 80 ``\endinput''.

Pour WINDOWS :
 

  1. copier le fichier package.ins dans un répertoire temporaire et le compiler avec LATEX ;
  2. déplacer le fichier package.sty dans un répertoire visible par LATEX, soit commun à tous les utilisateurs, soit local, soit personnel : - pour fpTEX, ce sont respectivement les répertoires (déclarés à l'installation de fpTEX) ./texmf/tex/latex/lerepertoiredemonpackage (à créer si besoin), ./texmf.local/tex/latex/lerepertoiredemonpackage (à créer si besoin), HOME/texmf/tex/latex/lerepertoiredemonpackage (à créer si besoin) ; - pour MikTEX, ce sont les mêmes répertoires mais le répertoire local est ./localtexmf/ et que le HOME ne semble pas être prévu : [FMF : je ne suis pas sûre de ce dernier point]
  3. reconstruire la base de données des packages (rebuild ls-R filenames databases pour fpTEX, refresh filename database pour MikTEX).

29.20  Comment générer des codes barres ?

29.21  Comment gérer des abbréviations ?

Le package 'abbrevs' disponible sur ftp://ftp.fdn.org/pub/CTAN/macros/latex/contrib/supported/monster/ permet automatiquement de remplacer des abbréviations par leur formule développée notamment lors d'un premier appel.

Le package 'acronym' disponible sur ftp://ftp.fdn.org/pub/CTAN/macros/latex/contrib/supported/acronym/ permet de s'assurer que tous les acronymes utilisés dans un document apparaissent au moins une fois sous une forme développée.

29.22  Comment imprimer un fichier postscript sur une imprimante non postscript ?

alladin est un excellent programme qui permet d'imprimer du postscript sur à peu près n'importe quelle imprimante. Cet outil est disponible par ftp sur ftp://ftp.lip6.fr/pub/ghost/aladdin/.

Remarque :
on ne peut pas dire que ce soit d'une très grande facilité d'installation, mais une fois que ce mauvais moment est passé, on ne peut plus s'en passer :-)

29.23  Comment surcharger une commande déjà existante ?

Pour pouvoir ajouter des définitions sur une commande prédéfinie sans la réécrire complètement, il faut utiliser la commande \let.

Exemple :
 
\let\standardsection=\section 
\def\section{\newpage\standardsection} 
%%%% fin exemple %%%%
Il existe aussi la commande expandafter.

Exemples :
 
\toks0 \expandafter{\section} 
\edef\section{\noexpand\newpage\the\toks0 } 
\expandafter\def\expandafter\section\expandafter{\expandafter% 
\newpage\section} 
%%%% fin exemple %%%%
Le package 'babel' propose également la commande \addto.

29.24  Comment reporter l'exécution d'une commande à la fin d'une page ?

Il existe pour cela le package 'afterpage' disponible sur ftp://ftp.fdn.org/pub/CTAN/macros/latex/required/tools/.

Exemple :
 
\afterpage{\clearpage} 
%%%% fin exemple %%%%

29.25  Comment définir des scripts interactifs ?

Le package 'dialogl' disponible sur ftp://ftp.fdn.org/pub/CTAN/macros/latex/contrib/supported/dialogl/ permet de définir des scripts LATEX interactifs.

29.26  Comment identifier une version provisoire ?

Le package 'draftcopy' disponible sur ftp://ftp.fdn.org/pub/CTAN/macros/latex/contrib/supported/draftcopy/ permet par exemple d'écrire le mot DRAFT en grisé en diagonale sur toutes ou certaines pages d'un document. Il permet également de faire moins voyant.

Exemple :
 
\documentclass{report} 
\usepackage{french} 
\usepackage{draftcopy} 
\begin{document} 
Voici une version provisoire de mon texte. Pourriez-vous la 
relire et me faire part de vos commentaires avant la fin de 
la semaine? 
Merci d'avance. 
\paragraph{Proposition.} 
Texte provisoire -- Texte provisoire -- Texte provisoire -- 
Texte provisoire -- Texte provisoire -- Texte provisoire -- 
Texte provisoire -- Texte provisoire -- Texte provisoire -- 
Texte provisoire -- Texte provisoire -- Texte provisoire -- 
Texte provisoire -- Texte provisoire -- Texte provisoire -- 
Texte provisoire -- Texte provisoire -- Texte provisoire -- 
Texte provisoire -- Texte provisoire -- Texte provisoire -- 
Texte provisoire -- Texte provisoire -- Texte provisoire -- 
Texte provisoire -- Texte provisoire -- Texte provisoire 
\end{document} 
%%%% fin exemple %%%%
Le package 'drafthead' également disponible sur ftp://ftp.fdn.org/pub/CTAN/macros/latex209/contrib/misc/ permet lui de n'agir que sur les en-têtes des documents.

On peut également utiliser la macro :

%%%% debut macro %%%% 
\special{ 
! userdict begin /bop-hook{ 
stroke 
gsave 240 100 translate 
65 rotate /Times-Roman findfont 220 scalefont setfont 
0 0 moveto 0.9 setgray (DRAFT) show grestore 
}def end} 
dans le préambule de son document. 
%%%% fin macro %%%%
Le package 'prelim2e' disponible sur ftp://ftp.fdn.org/pub/CTAN/macros/latex/contrib/supported/prelim2e/ offre également un marquage des versions préliminaires d'un document.

29.27  Comment obtenir des caractères barrés ?

Le package 'ulem' disponible sur ftp://ftp.fdn.org/pub/CTAN/macros/latex/contrib/other/misc/ permet de barrer des caractères.

Exemple :
 
\sout{je barre tout} 
%%%% fin exemple %%%%

29.28  A quoi sert la commande \ensuremath ?

La commande \ensuremath assure que son argument sera imprimé en mode mathématique quel que soit le mode courant.

Exemple :
 
\documentclass{report} 
\usepackage{french} 
\pagestyle{empty} 
\newcommand{\mc}{\ensuremath{(\alpha, \beta)}} 
\begin{document} 
Le couple \mc\ définit par $\mc = x+y, x-y$, ... 
\end{document} 
%%%% fin exemple %%%%

29.29  A quoi servent les commandes \(re)newcommand ?

La commande \newcommand permet à l'utilisateur de définir ces propres commandes. La commande \renewcommand permet de redéfinir des commandes LATEX déjà existantes. Elle s'utilise dans le préambule du document :

        \newcommand{nom_commande}[nb_arguments]{définition}
nb_arguments définit le nombre d'arguments de la nouvelle commande LATEX. Il est compris entre 0 et 9. Ils sont référencés par #no_argument dans la définition.

Exemple :
 
\documentclass{report} 
\usepackage{french} 
\pagestyle{empty} 
\newcommand{\be}{\begin{enumerate}} 
\newcommand{\ee}{\end{enumerate}} 
\newcommand{\bold}[1]{\textbf{#1}} 
\begin{document} 
\bold{Faites votre choix:} 
\be 
\item premier choix: A 
\item deuxième choix: B 
\item troisième choix: C 
\ee 
\end{document} 
%%%% fin exemple %%%%
Remarques :
La commande \providecommand permet de ne prendre en compte la nouvelle commande que si elle n'existe pas déjà.
Pour définir un argument optionnel, on utilise la syntaxe :

                \newcommand{nom}[nb_arg][defaut]{definition}
defaut permet de préciser une valeur par défaut de la variable qui sera utilisée en l'absence d'argument. L'argument optionnel ne peut être que #1.

29.30  Où trouver des hirondelles ?

On peut trouver la package 'cropmark' sur ftp://ftp.fdn.org/pub/CTAN/macros/generic/.

29.31  Comment tracer des lignes ?

On peut mettre la définition suivante dans le préambule du document :

\def\ligne#1{\leaders\hrule height #1\linethickness \hfill}
puis utiliser :

\ligne{5}
dans le texte.

Il existe également la commande \rule qui prend en premier argument, la longueur horizontale du trait et, en deuxième, son épaisseur. Elle possède également un argument optionnel qui permet de positionner la ligne traçée par rapport à la ligne d'écriture.

Exemple :
 
\rule[0.5ex]{\textwidth}{0.1mm} 
%%%% fin exemple %%%%
On peut aussi utiliser les commandes \hrule pour des lignes horizontales et \vrule pour des lignes verticales.

29.32  Comment imprimer le caractère ?

fournit la commande \textasciitilde sous fonte T1. Sous OT1, il faut définir :

\ProvideTextCommand{\textasciitilde}{OT1}{\{}}
Il est possible de passer par le mode verbatim via la commande :

\verb$$.
On peut aussi définir sa propre commande :

\def\mytilde{\raisebox{-.8ex}{\}\hspace{-0.15em}}
\{} permet également d'obtenir un tilde.

Une autre solution consiste à utiliser $\sim$.

Autre solution, utiliser : \char`\

Remarque :
la première solution est de loin la meilleure.

29.33  Comment visualiser tous les caractères d'une fonte ?

Le plus simple consiste à utiliser :

latex nfssfont
nfssfont.tex est un fichier disponible sur CTAN.

Il faut ensuite donner le nom de la fonte à examiner par exemple cmr10 puis construire la table des caractères en utilisant la commande \table. Cela permet de construire un fichier .dvi contenant les codes de tous les caractères.

L'appel d'un symbole se fait alors via la commande \symbol{x} où x est le code associé au caractère voulu (c'est un entier compris entre 0 et 255).

On peut également utiliser \char''<octal> ou \char<hexa> ou encore \char<decimal> mais ses commandes sont moins robustes.

29.34  A quoi sert % dans les macros ?

Le % en fin de ligne d'une macro permet d'indiquer à LATEX que la définition qu'il est en train de lire continue à la ligne suivante. De manière générale, les espaces après une macro sont ignorés, mais pas après les accolades. Ce caractère permet de s'assurer que des espaces non voulus ne seront pas pris en compte. Ainsi, il est prudent de mettre un % après chaque accolade qui se trouve en fin de ligne.

29.35  Comment inclure l'heure dans un document ?

La macro décrite ci-dessous, à inclure dans le préambule de votre document, définit la commande \timenow :

%%%% debut macro %%%% 
\makeatletter 
\def\timenow{\@tempcnta\time 
  \@tempcntb\@tempcnta 
  \divide\@tempcntb60 
  \ifnum10>\@tempcntb0\fi\number\@tempcntb 
  \multiply\@tempcntb60 
  \advance\@tempcnta-\@tempcntb 
  :\ifnum10>\@tempcnta0\fi\number\@tempcnta} 
\makeatother 
%%%% fin macro %%%%
Le package 'time' disponible sur ftp://ftp.fdn.org/pub/CTAN/macros/generic/time.sty permet d'inclure l'heure courante dans un document.

En ajoutant la macro suivante de H. Hanche-Olsen dans le préambule de son document, on peut via la commande

%%%% debut macro %%%% 
%\isodayandtime obtenir la date et l'heure courante : 
\begingroup 
\count0=\time \divide\count0by60 % Hour 
\count2=\count0 \multiply\count2by-60 \advance\count2by\time 
% Min 
\def\2#1{\ifnum#1<10 0\fi\the#1} 
\xdef\isodayandtime{\the\year-\2\month-\2\day\space\2{\count0}:% 
\2{\count2}} 
\endgroup 
%%%% fin macro %%%%
Autre exemple d'utilisation (qui nécessite dvips) :

%%%% debut macro %%%% 
\special{!userdict begin /bop-hook 
 {gsave -90 rotate -780 560 moveto 1.0 0.7 0.7 setrgbcolor 
 % 0.8 setgray 
 106 45 {dup mul exch dup mul add 1.0 exch sub} setscreen 
 /Times-Roman findfont 25 scalefont setfont (DRAFT 
 \isodayandtime)  show grestore} def} 
%%%% fin macro %%%%
On peut également utiliser la macro \time qui donne l'heure en minutes. Pour afficher l'heure en heures et minutes, il faut les calculer puis les afficher avec la macro \the.

Exemple 1 :
 
\documentclass{article} 
\usepackage{french} 
\pagestyle{empty} 
\begin{document} 
Cette compilation a démarré à \the\time\ minutes après minuit 
le \today. 
\end{document} 
%%%% fin exemple %%%%
Exemple 2 :
 
\usepackage{calc} 
\newcounter{hours}\newcounter{minutes} 
\newcommand{\printtime}{% 
  \setcounter{hours}{\time/60}% 
  \setcounter{minutes}{\time-\value{hours}*60}% 
  \thehours :\theminutes} 
%%%% fin exemple %%%%
Il existe également le package 'heure' de T. Bouche donné ci-dessous :

%%%% debut macro %%%% 
\NeedsTeXFormat{LaTeX2e} 
\ProvidesPackage{heure} 
[1996/6/11v0.1 affiche le jour et l'heure de production d'un 
document] 
\newif\if@heure@fin\@heure@finfalse 
\newif\if@heure@final\@heure@finalfalse 
\DeclareOption{fin}{\@heure@fintrue} 
\DeclareOption{final}{\@heure@finaltrue} 
\DeclareOption{draft}{} 
\ProcessOptions 
%%% définition de hours prise dans testfont 
\newcount\\newcount\
\def\hours{\n=\time \divide\n 60 
  \m=-\\multiply\m 60 \advance\\time 
  \twodigits\n\ :\ \twodigits\m} 
\def\twodigits#1{\ifnum #1<10 0\fi \number#1} 
%%%%%%% Tant que ça n'est pas définitif. 
\if@heure@final\else 
\AtBeginDocument{\begin{center}% 
  {\bfseries\itshape\large Version du \today\ à \hours }% 
\end{center}} 
\fi 
\if@heure@fin 
\AtEndDocument{% 
  \begin{flushright}% 
    \small \itshape (Le  \today\ à \hours )\hspace*{3em}% 
  \end{flushright}} 
\fi 
\endinput 
%%%% fin macro %%%%
Utilisation : Il suffit de passer fin, final ou draft en option de classe.

Exemple 1 :
 
\documentclass[11pt,draft]{article} 
\usepackage{heure} 
\begin{document} 
Voici le résumé de notre expérience avec le Brome: 
Bla bla bla. 
\end{document} 
%%%% fin exemple %%%%
Exemple 2 :
 
\documentclass[11pt,fin]{article} 
\usepackage{heure} 
\begin{document} 
Voici le résumé de notre expérience avec le Brome: 
Bla bla bla. 
\end{document} 
%%%% fin exemple %%%%
Exemple 3 :
 
\documentclass[11pt,final]{article} 
\usepackage{heure} 
\begin{document} 
Voici le résumé de notre expérience avec le Brome: 
Bla bla bla. 
\end{document} 
%%%% fin exemple %%%%

29.36  Comment compter le nombre de mots d'un fichier ?

A priori, il n'y a pas de solution simple à ce problème :

ATTENTION :
aucune de ces méthodes n'est idéale comme l'indique l'essai comparatif de T. Bouche :
                detex -i grfguide.tex | wc -w   donne 4420 
                dvitty grfguide.dvi |wc -w      donne 4524 
                wc -w grfguide.tex              donne 4539 
                ps2ascii grfguide.ps |wc -w     donne 5066

29.37  Comment rendre inactif un caractère ?

La commande \string permet de désactiver un caractère rendu actif par une macro.

Exemple :
 
En français le : est rendu actif pour que LaTeX gère lui-même l'espace à mettre avant.  
En revanche, dans une URL ou une adresse mail cet espace est superflu. On utilise donc : 
mailto\string:moi.meme@chez.mon.site 
%%%% fin exemple %%%%

29.38  Comment utiliser le mode verbatim dans une commande ?

fournit l'environnement lrbox.

Exemple :
 
\documentclass{article} 
\usepackage{alltt} 
\pagestyle{empty} 
% --------------- 
\newsavebox{\inverbatim} 
\begin{lrbox}{\inverbatim}% 
\noindent\begin{minipage}{\linewidth} 
\begin{alltt} 
                           1 
 -8 + 7 Sqrt[2] ArcTanh[-------] 
                        Sqrt[2] 
\end{alltt} 
\end{minipage}% 
\end{lrbox}% box 
% ------------------------------------ 
\begin{document} 
\noindent 
L'expression : \usebox{\inverbatim} démontrée au paragraphe 
précédent prend ici tout son sens. 
En effet l'expression \usebox{\inverbatim} permet\ldots 
\end{document} 
%%%% fin exemple %%%%

29.39  Comment redéfinir la commande \year ?

Pour ne faire apparaître que deux chiffres au lieu de quatre, il faut utiliser :

\newcommand{\ignoretwo}[2]{} 
\newcommand{\shortyear}{\expandafter\ignoretwo\the\year}
dans le préambule du document.

29.40  Qu'est ce que la magnification ?

La commande magnification permet de faire un zoom d'un document.

Exemple :
 
\mag=1440 
\documentclass{...} 
%%%% fin exemple %%%%
On peut également travailler sur le fichier postscript via dvips avec l'option -x nombre où nombre vaut par exemple 1200 pour un agrandissement de 20%.

De la même manière, le package 'scale' disponible sur ftp://ftp.fdn.org/pub/CTAN/macros/latex/contrib/supported/scale/ permet de modifier l'échelle d'un document avant impression.

29.41  Comment sont gérés les postscripts dans LATEX ?

P. Perichon :

TEX/LATEX + DVIPS procèdent en 2 temps :

Toutefois, certains visualiseurs DVI, par exemple XDVI sous UNIX pour ne pas le citer, détectent quelque chose du genre :

     PSfile="toto.eps" llx=0 lly=-1 urx=57 ury=29 rwi=4252
dans le fichier DVI, et appellent gentiment GostScript/GhostView qui vont préparer une image bitmap que le visualiseur DVI s'empressera de montrer (quand ça marche : avec psfig.sty pas de problème, mais avec l'extension graphicx...). Mais d'autres visualiseurs, comme DVIWIN, ne savent pas que faire d'un fichier PS.

DVI signifie DeVice Independent : ceci est un format de document indépendant du système d'impression. Donc non lié à un langage de description de page utilisé en interne par une quelconque imprimante (PostScript, PCL, HPGL, Windows GDI, ...). Il suffit après d'utiliser un convertisseur qui transforme le DVI en quelque chose que comprend votre imprimante (PostScript, HPGL, PCL, ...). Cette méthode permet d'imprimer sur n'importe quel type d'imprimante (il suffit d'avoir le bon convertisseur). Mais hélas en DVI, il n'a point été prévu d'y mettre une image (le problème n'est en effet pas simple). D'où l'utilisation de fichier EPS.

29.42  Pourquoi LATEX n'accepte-t'il pas tous les formats d'image ?

P. Perichon :

Le problème est le suivant : un fichier DVI + image TIFF, GIF, TGA,... comment faire digérer tout ça par votre imprimante ???? Le fichier DVI peut se faire convertir en un langage d'impression compréhensible par votre imprimante... mais vos formats d'images... faudrait-il en plus prévoir un système de conversion de TIFF, GIF, TGA, ... vers le langage d'impression de votre imprimante (Alladin avec GhostScript/GhostView propose quelques filtres, mais bon). A part quelques langages de description de pages comme PostScript et un ou deux autres qui pratiquent une compatibilité ascendante quant aux versions du langage qu'ils utilisent, les autres fabricants d'imprimantes changent de langage quasiment à chaque modèle ou série d'imprimante (impossible de tenir à jour autant de convertisseurs).

Pour tout cela le langage PostScript (PS et EPS) nous rend grandement service en nous simplifiant la vie (et pour d'autres raisons aussi). C'est peut-être un peu plus lourd à manipuler, mais plus simple pour tout le monde de convertir vos images en EPS. De plus, si vous ne disposez pas d'imprimante PostScript, GhostScript/GhostView se chargera d'interpréter le fichier PS (texte + images), fabriquera une bitmap à la bonne résolution et pourra l'envoyer à votre imprimante via votre pilote d'impression. Tout cela automatisé dans une bonne distribution est transparent à l'utilisateur.

Pour plus de détails, consulter le grfguide disponible sur ftp://ftp.fdn.org/pub/CTAN/macros/latex/packages/graphics/ et le document epslatex disponible sur ftp://ftp.fdn.org/pub/CTAN/info/.

29.43  A quoi sert la commande \(re)newenvironment ?

Les commandes \newenvironment et \renewenvironment permettent de définir des actions qui seront appliquées entre les commandes \begin{mon_environnement} et \end{mon_environnement}. Ces commandes doivent être appelées dans le préambule du document suivant la syntaxe :

        \newenvironment{nom}[nb_arg]{avant}{après}
avant et après sont les actions qui auront lieu à l'entrée et à la sortie du nouvel environnement.

Exemple :
 
\documentclass{report} 
\usepackage{french} 
\newenvironment{Relief}[1] 
{\mbox{\Large{\uppercase{#1}}}} 
\begin{document} 
\begin{Relief} 
   {Il}était une foisldots 
\end{Relief} 
\begin{Relief} 
   {U}n jour peut êtreldots 
\end{Relief} 
%%%% fin exemple %%%%

29.44  Comment récupérer le nom du fichier compilé ?

Pour récupérer le nom du fichier compilé et l'afficher lors d'une compilation, il faut utiliser la commande \jobname.

Exemple :
 
Le fichier source est \texttt{\jobname.tex}. 
%%%% fin exemple %%%%

29.45  Comment gérer des conditions de traitement dans un style ?

Le package 'ifthen' disponible sur ftp://ftp.fdn.org/pub/CTAN/macros/latex/base/ permet d'implémenter des traitements conditionnels tels que \ifthenelse et \whiledo.

29.46  A quoi servent les commandes \setlength et \addtolength ?

Ces deux commandes permettent de modifier la longueur de certains paramètres.

Exemples :
 
\setlength{nom_parametre}{longueur} 
\addtolength{nom_parametre}{longueur} 
%%%% fin exemple %%%%

29.47  Quelles sont les unités de mesure de TEX ?

TEX comprend six unités de mesure :

29.48  A quoi sert la commande \mbox ?

La commande \mbox permet à LATEX de considérer son argument comme une seule entité. Elle peut ainsi empêcher la coupure d'un mot ou d'un groupe de mots.

Exemple :
 
Mon numéro de téléphone est lembox{01 69 63 12 68}. 
%%%% fin exemple %%%%
ce qui donne :

Mon numéro de téléphone est le 01 69 63 12 68.

29.49  Comment obtenir des points de suspension ?

La commande \ldots permet d'obtenir trois points de suspension espacés correctement.

29.50  Comment désactiver une ligature ?

Pour désactiver une ligature, il faut introduire {} ou une \mbox{} entre les lettres ligaturées.

Exemple :
 
comparez effacer, ef{}facer et ef\mbox{}facer 
%%%% fin exemple %%%%
ce qui donne :

comparez effacer, effacer et effacer

29.51  Comment gérer les versions d'un document ?

Pour les versions provisoires, voir la question .

Il existe le package 'vrsion' disponible sur ftp://ftp.fdn.org/pub/CTAN/macros/latex/contrib/supported/vrsion/ qui permet via la commande \version d'inclure un numéro de version dans un document LATEX. Ce numéro est incrémenté à chaque compilation.

On peut également se reporter à la question pour inclure le jour et l'heure de compilation dans un document.

29.52  Comment changer certains titres ?

Il suffit de renommer les variables suivantes :

   \def\refname{R\'ef\'erences}% 
   \def\abstractname{R\'esum\'e}% 
   \def\bibname{Bibliographie}% 
   \def\prefacename{Pr\'eface}% 
   \def\chaptername{Chapitre}% 
   \def\appendixname{Annexe}% 
   \def\contentsname{Table des mati\`eres}% 
   \def\listfigurename{Table des figures}% 
   \def\listtablename{Liste des tableaux}% 
   \def\indexname{Index}% 
   \def\figurename{{\scshape Fig.}}% 
   \def\tablename{{\scshape Tab.}}% 
   \def\partname{\protect\@Fpt partie}% 
   \def\@Fpt{{\ifcase\value{part}\or Premi\`ere\or Deuxi\`eme\or 
   Troisi\`eme\or Quatri\`eme\or Cinqui\`eme\or Sixi\`eme\or 
   Septi\`eme\or Huiti\`eme\or Neuvi\`eme\or Dixi\`eme\or 
   Onzi\`eme\or  Douzi\`eme\or Treizi\`eme\or Quatorzi\`eme\or 
   Quinzi\`eme\or Seizi\`eme\or Dix-septi\`eme\or 
   Dix-huiti\`eme\or Dix-neuvi\`eme\or Vingti\`eme\fi}}% 
   \space\def\thepart{}}% 
   \def\pagename{page}% 
   \def\seename{{\emph{voir}}}% 
   \def\alsoname{{\emph{voir aussi}}}% 
   \def\enclname{P.J. }% 
   \def\ccname{Copie \`a }% 
   \def\headtoname{}% 
   \def\proofname{D\'emonstration}% for AMS-LaTeX
Remarque : lorsque certains changements de noms sont déjà gérés par un package (french par exemple) il faut placer les définitions ci-dessus après le \begin{document}.

29.53  Comment insérer un code source dans un document ?

29.54  Comment tracer une ligne horizontale ?

Pour obtenir une ligne centrée par rapport à la mi-hauteur au dessus de la ligne d'écriture, on peut utiliser ceci :

\hbox{\raisebox{0.4em}{\vrule depth 0pt height 0.4pt width 1cm}Toto}
de qui donne :

Toto

29.55  Comment générer un espace invisible de taille donnée ?

Il existe pour cela la commande \phantom. Celle-ci génère un espace invisible de la même longueur que son argument.

Exemple :
 
Complète les mots qui manquent dans le texte suivant: 
Le petit chaperon \phantom{rouge} se promèneldots. 
%%%% fin exemple %%%%
ce qui donne :

Complète les mots qui manquent dans le texte suivant :
Le petit chaperon            se promène ....

29.56  Qu'est ce qu'une correction italique ?

Pour que LATEX puisse gérer correctement le passage d'une fonte italique à une fonte droite, il peut être utile d'utiliser la commande \/. Cela permet d'augmenter légèrement l'espace qui sépare la dernière lettre en italique, de la première lettre droite.

Exemple :
 
Un \textit{cheval}\/ file au galop. 
%%%% fin exemple %%%%
ce qui donne :

Un cheval file au galop.

29.57  Quels sont les accents accessibles sous LATEX ?

Les accents accessibles sous LATEX sont les suivants :

29.58  Comment écrire dans un fichier pendant une compilation ?

On dispose pour cela des commandes \write et \newwrite.

Exemple :
 
\newwrite\test 
\openout\test=toto 
\write\test{Je m'appelle Paul} 
\write\test{\noexpand\thesection} 
\closeout\test 
\bye 
%%%% fin exemple %%%%
Remarque :
\noexpand permet d'inclure des commandes LATEX sans qu'elles soient interprêtées.
Le package 'french' propose la commande \originaloutput[fichier]{texte} pour écrire le ``texte'', tel qu'il est fourni, dans le ``fichier''. Dans ce cas, les caracteres actifs de french sont automatiquement desactivés. C'est donc l'equivalent de \immediate\write\fichier{texte}.

Le package 'sverb' disponible sur CTAN propose l'environnement verbwrite.

Exemple :
 
\begin{verbwrite}{fichier.tmp} 
   Ce texte va être sauvé dans le fichier fichier.tmp. 
\end{verbwrite} 
%%%% fin exemple %%%%

29.59  Comment gérer des compteurs ?

Un certain nombre de compteurs sont prédéfinis dans LATEX, il s'agit :

Pour définir son propre compteur, il faut utiliser la commande

\newcounter{nv_cptr}[ref_cptr]. 
Lorsque qu'un compteur déjà existant est passé en argument (ref_cptr), le nouveau compteur nv_cptr sera réinitialisé à chaque incrément via

\stepcounter 
ou

\refstepcounter 
du compteur ref_cptr.

Par défaut, la valeur d'un nouveau compteur est 0. La commande

\setcounter{cptr}{val} 
permet de lui donner une autre valeur. On peut également incrémenter la valeur d'un compteur via la commande

\addtocounter{cptr}{val} 
La valeur d'un compteur peut être récupérée, dans un calcul par exemple, grâce à la commande

\value{cptr}
\stepcounter{cptr} permet d'incrémenter le compteur cptr et de réinitialiser tous les compteurs liés par référence (argument ref_cptr de la commande \newcounter). La commande \refstepcounter{cptr} permet en outre de mettre à jour la valeur courante du compteur qui pourrait être appelé par un \ref.

\thecptr, où cptr est le nom d'un compteur permet d'afficher sa valeur.

Exemple :
 
\newcounter{section} 
\newcounter{subsection}[section] 
%%%% fin exemple %%%%

29.60  Quels sont les différents styles de compteurs ?

Il en existe six prédéfinis :

Exemple :
 
\renewcommand{\thesection}{\Roman{section}} 
\renewcommand{\thesubsection}{\thesection.\roman{subsection}} 
%%%% fin exemple %%%%

29.61  Comment programmer un traitement itératif ?

Le package 'multido' propose une commande \multido qui est une boucle TEX. Sa syntaxe est la suivante :

        \multido{variables}{nbiteration}{code}
Le code sera ainsi répété nbiteration fois. Les déclarations de variables sont séparées par des virgules. Un déclaration prend la forme :

        variable = valeurinitiale + increment
Elle est utile notamment pour le dessin de figures.

Exemple (doc package) :
 
\setlength{\unitlength}{1cm} 
\small 
\begin{picture}(8,1)(0,-.5) 
   \put(0,0){\vector(1,0){8}} 
   \multido{\i=0+1, \n=0+0.25}{8}{% 
   \put(\i,-.1){\line(0,1){.2}} 
   \put(\i,-.2){\makebox(0,0)[t]{\n}}} 
\end{picture} 
%%%% fin exemple %%%%

29.62  A quoi servent les commandes savebox ?

Il existe les commandes :

Exemple :
 
\newsavebox{\maboite} 
\sbox{\maboite}{\textbf{Alcatel Alsthom Recherche}} 
\usebox{\maboite} se situe à Marcoussis. Je travaille à 
\usebox{\maboite} depuisldots... 
%%%% fin exemple %%%%
ce qui donne :

Alcatel Alsthom Recherche
se situe à Marcoussis. Je travaille à Alcatel Alsthom Recherche depuis ldots

29.63  Comment résoudre certaines incompatibilités de packages ?

Il arrive parfois lorsque plusieurs packages sont utilisés simultanément que des macros différentes portent le même nom, ou que certaines macros perturbent le fonctionnement d'autres macros. Dans certains cas le problème peut se résoudre en changeant l'ordre d'appel des packages.

Exemple (J.P. Drucbert) :
 
Le paquetage psboxit définit des commandes \psboxit qui engendrent \special{ps: ...}. Le problème vient de ce deux-points. Si vous chargez psboxit.sty AVANT babel (option french), pas de problème, le deux-points est un caractère ordinaire (other) lors de la déclaration de la macro \psboxit et donc le \special écrira bien un ``:``.

Par contre si psboxit.sty est chargé APRES babel (option french), le deux-points est alors un caractère actif au moment de la déclaration de la macro \psboxit, et le \special n'écrira pas un deux-points, mais un espace insécable (en fait une macro interne de babel) et un :, d'où les erreurs. C'est un cas classique de catcodes pas très bien gérés (en fait psboxit.sty devrait sauver le catcode de : , donner la catégorie other au caractère :, définir \psboxit, puis restaurer le catcode). %%%% fin exemple %%%%

29.64  Qu'est ce que Lollipop ?

C'est un jeu de macro de V. Eijkhout, destiné à faciliter l'écriture de macros TEX. Il est disponible sur ftp://ftp.fdn.org/pub/CTAN/macros/lollipop/.

29.65  J'ai un problème avec babel !

En tentant d'utiliser babel, j'obtiens l'erreur suivante:

! Undefined control sequence. 
\add@accent ...l \mathchardef \accent@spacefactor 
                                                  \spacefactor }\accent #1 #... 
l.3 \begin{document}
Le problème est connu et a été corrigé par babel le 19 août 1999. Si vous ne pouvez pas mettre à jour votre paquetage de babel, utilisez le cache-bogue suivant, à mettre dans le préambule (donation de Javier Bezos):

\makeatletter 
\let\accent@spacefactor\relax 
\makeatother
Vous pouvez également ajouter la ligne suivante dans vos entêtes: \usepackage{t1enc}

En effet, le problème ne se manifeste pas en codage T1.

29.66  Qu'est ce que cette FAQ ?

Chapter 30
Remerciement et Contributions

Je [mpk] remercie vivement les personnes qui ont contribué de près ou de loin à l'élaboration de cette FAQ. Je remercie également les personnes qui me soutiennent et m'encouragent.

Je [bb] tiens à remercier les personnes qui m'ont envoyé des patchs ou des correctifs sur cette faq, et qui ont à vrai dire beaucoup plus travaillé que moi.

La liste des contributions ci-après se veut aussi complète que possible. Si vous avez été oublié, n'hésitez pas à me contacter.

Contributions [ depuis le 15/07/99 ]

Bibliography

[1]
P. Abrahams, K. Berry, and K. Hargreaves. TEX for the impatient. Addison Wesley, 1990.

[2]
b. Bringhurst. The Elements of Typographic Style.

[3]
M. Baudoin. Apprends LATEX.

[4]
B. Bayart. Joli manuel pour .

[5]
G. Blanchard. Pour une sémiologie de la typographie. Rencontres de Lure.

[6]
A. Borde. TEX by example: A Beginner's Guide. Academic Press, 1992.

[7]
Dreyfus and Richaudeau. La chose imprimée. Retz, 1985.

[8]
V. Eijkhout. TEX by Topic.

[9]
V. Gramet et J.P. Regourd. Objectif LATEX. Masson, 1995.

[10]
M. Goossens. , un apercu.

[11]
M. Goossens, F. Mittelbach, and A. Samarin. The LATEX companion. Addison Wesley, 1994.

[12]
M. Goossens, S. Rahtz, and F. Mittelbach. The LATEX Graphics Companion.

[13]
G. Gratzer. Math into LATEX : An Introduction to LATEX and AMS-LATEX. Birkhauser, Boston, 1996.

[14]
M. Herrb. The not so short introduction to .

[15]
D. E. Knuth. The TEXbook. Addison Wesley, 1984.

[16]
D. E. Knuth. TEX: The Program. Addison Wesley, 1986.

[17]
D.E. Knuth. Metafont. Addison Wesley Longman.

[18]
H. Kopla and P.W. Daly. A Guide to , document preparation for beginners and advanced users. Addison Wesley, 1995.

[19]
T. Lachand-Robert. La maîtrise de TEX et LATEX. Masson, 1995.

[20]
L. Lamport. LATEX reference manual. Addison Wesley.

[21]
L. Lamport. LATEX, a Document Preparation System. Addison Wesley, 1994.

[22]
L. Lamport. LATEX, a Document Preparation System. Addison Wesley, 1ère édition.

[23]
H. E. Meier. Le développement des caractères. Syntax Press, Suisse.

[24]
Imprimerie Nationale. Lexiques des règles typographiques en usage à l'imprimerie nationale. 3ème édition edition, 1990.

[25]
T. Oetiker. The not so short introduction to (Version Fran caise).

[26]
Y. Perousseaux. Manuel de typographie élémentaire. 1995.

[27]
D. Salomon. The advanced TEX book. Springer Verlag, 1995.

[28]
S. Sawyer and S. Krantz. LATEX, a Document Preparation System. CRC Press, 1995.

[29]
N. Schwarz. Introduction to TEX. Addison Wesley, 1989.

[30]
R. Seroul. Le petit livre de TEX. Interéditions, 1989.

[31]
C. Simian. LATEX, Manuel utilisateur simplifié. CNRS.

[32]
W. Snow. TEX for the beginner. Addison Wesley, 1992.

[33]
J. Warbrick. Essential LATEX.




File translated from TEX by TTH, version 3.01.
On 19 Nov 2001, 18:13.